Está en la página 1de 1012
The Pearson Guide to OBJECTIVE PHYSICS for the ll T-JEE = Short-Cut Methods = Important Formulae -* Solved Problems « Ten Model Test Papers ere reremeenieny: /* Exercise for Self-Practice Ravi Raj Dudeja @ os ‘The aim of this publication is to supply information taken from sources belived to be valid and reliable. This is not an attempt to render any type of professional advice or analysis, nor is it to be treated as such. While much care has been taken to ensure the veracity and accuracy of the information presented within, neither the publisher nor its authors bear any responsibility for eny damage arising from inadvertent omissions, negligence or inaccuracies (typographical or factual) that may have fourd theit way into this book. Copyright © 2009 Dorling Kindersley (India) Pvt. Ltd Copyright © 2007 Dorling Kindersley (India) Pvt. Lid This book is sold subject to the condition that it shall not, by way of trade or otherwise, be lent, resold, hired out, ‘or otherwise circulated without the publisher's prior written consent in any form of binding or cover other than that in which it is published and without a similar condition including this condition being imposed on the subsequent purchaser and without limiting the rights under copyright reserved above. no part of this publication may be reproduced. stored in or introduced into a retrieval system, or transmitted in any form or by any means (electronic, mechanical, photocopying, recording or otherwise), without the prior written permission of both the copyright owner and the above- mentioned publisher of this book ISBN 978-81-317-2677-8 First Impression Published by Dorling Kindersley (India) Pvt. Ltd, licencees of Pearson Education in South Asia, Head Office: 482 FIE, Patparganj, Delhi 110 092, India Registered Office: 14 Local Shopping Centre, Panchsheel Park, New Delhi 110017, India, Typeset by Rechna Graphics, Chandigarh Printed in India at Baba Barkha Nath Printers Preface 1o the Second Edition uw 5 ewan’ Nake 6. Work, Power and Energy ‘L_Conservation of Momentum &._Rotational Motion 9._ Gravitation 10. Interetomic Forces and Elasticity L1,_Hydrodynamics and Properties of Fluids Self Test Papers 351 \ Assertion Reasoning Type Questions BESRBERR ee we 8 EIR 12._Simple Harmonie Motion 13. Wave Motion and Waves in String 14__Sound Waves Self Test Papers Assertion Reasoning Tipe Questions BREE \ 15. Kinetic Theory of Gases, Calorimetery and Specific Heat of Gases 16. Thermodynamics ' 12._Heat Transfer Processes Self Test Papers A SBR BE a SS xertion Reasoning Type Questions 18. Electrostatics 19. _Gauss’s Law 20. Capacitors Self Test Papers Assertion Reasoning Type Questions 21. Electricity 24, Permanent Magnets 25. Magnetic Properties of Substance 26, Electromagnetic Induction 27. Alternating Current 28, Electromagnetic Waves Self Test Papers Assertion Reasoning Type Questions . SECTION 6 - Optics 29. Ray Opties and Optical Instruments 30,_Wave Opti 32, Photoelectric Effect and Dual Nature of Matter Seif Test Papers Assertion Reasoning Type Questions SECTION 7- Modern Physics. 33__Bohr’s Theory and Atomic Physics 34. X- Rays: i Sicee 36__ Semiconductors 37. Special Theory of Relativity ‘38,_Principles of Communications Self Test Papers Assertion Reasoning Type Questions 669 RI 64 780 EREBRES BEBE 1089 1129 Contents v SECTION 8 ~ Model Test Papers Model Test Paper 1 1135 Model Test Paper 2 1143 Model Test Paper 3 1149 Model Test Paper 4 1156 Model Test Paper § 1165 Model Test Paper 6 1175 Model Test Paper 7 1183 Model Test Paper 8 1192 ‘Model Test Paper 9 1202 ‘Model Test Paper 10 1212 Maich-the-following Type Questions 1221 Appendix 1. Useful Mathematical Relations 1259 Append 2. The Greek Alphabet 1261 Appendix 3. Periodic Table of Elements 1262 Appendix 4. Unit Conversion Factors 1263 Appendix 5. Numerical Constants 1264 Appendix 6. Exponential and Hyperbolic Functions 1265 Solved Paper 2006 1256 Solved Paper 2007 Lu Solved Paper 2008 1275 ek Copyrighted material PREFACE In order to succeed in the Indian Institute of Technology-Joint Entrance Examination, one needs to master both the fundamentals of physics and their applications. To geta good rank, aspirants have to answer questions ranging in difficulty levels inthe shortest period of time possible. Keeping this in mind, one has to understand concepts and know the fastest process of solving the questions. Taking these factors into consideration, this book spans the entire gamut of difficulty levels, many of which have been taken from previous years” examit The unique features of this book are as follows: ‘Terms, concepts and definitions that form Brief Review, to enable students to understand the subject matter in a Short Cuts and Points to Note provide ready-to-work formulae and the shortest possible approach to solving problems. ‘Caution is an sttempt to make students aware of mistakes commonly committed ‘Solved Problems cover a large number of topics as well as questions for practice. At the end of each section, there are Questions from Competitive Exams and Self Tesis together with Explanations and Answers for self assessment. Students are advised to solve these questions before they refer to solutions, At the end of the book, ten Model Test Papers are provided with answers and explanations. Self tests and model test papers will help the student to assess the amount of work required to understand concepts before actually taking the test With best wishes for a great success, Ravi Raj Dudeja PREFACE TO THE SECOND EDITION ‘The basic outlook of this new edition of The Pearson Guide to Objective Physics for the IIT-JEE remains the same— assisting aspirants in honing their problem-solving skills besides providing cutting-edge material for the examination, In. this edition, the question papers of 2006, 2007 and 2008 have been added for the students to practice. Wish you all the best. Mechanics Curren 1. Curren 2. Curren 3. (Cumrren 4 Curre 5. [Cuseren G. [Cusrren 7. ICurerer 8. [Cuseren 9. (Curren 10. (Curren 11. Units, Dimensions and Errors Vectors Motion in One and Two Dimensions Circular Motion Newton’s Law of Motion Work, Power and Energy Conservation of Momentum Rotational Motion Gravitation Interatomic Forces and Elasticity Hydrodynamics and Properties of Fluid Self Test Papers Assertion Reasoning Type Questions BRIEF REVIEW Physics is the branch of science in which we deal wi observations, measurement and description of natu phenomenon related to matter and energy. These natu phenomena can be classified as mechani ‘matter, sound, thermodynamics, light electricity, atom physics, nuclear physics, particle physics, semiconductor superconductors and s0 on. properties Physical Quantities The quantities by means of whi Units, Dimensions and Errors Jifferentdirestions are call For ex moment of inertia. anisotropic media even density efractive index, dielectric constant, electric ith -onductivity, stress, strain and so on become tensor. al A physical quantity which has only one component ral led a scalar or a tensor of zero rank. If it has more than one component but less than or equal to nie four, itis called a vector or tensor of rank 1. Ifthe rs, components are greater than four, itis termed as tensor of a higher rank. ich 6, Conversion factors Some physical quantit wedescrbethe laws of physics arecalled physical quantities convert into another when multiplied by a factor. Physical quantities may be divided into six categories 1, Constantor ratio Such quantities have only magnitude, for example, refractive index, dielectric constant, specific gravity ete. Such quantities have no units. 2, Scalars They have magnitude and unit. Some of them may have direction also but vector laws are not applied. Examples are charge. mass. distance. speed, current etc. 3. Vectors They possess magnitude, unit and direction. They alse follow triangle law of addition. For example, velocity, force, momentum, torque, displacement etc. 4. Phasors Possess magnitude (amplitude) and hase. They follow triangle law. SHM, waves, AC voltage and AC current are phasors. 5. Tensors Physical quantities which have no specified direction but have different values in For example, in a wave y= y, sin (cot ke) k is a conversion factor when & is multiplied by displacement or path difference it generates phase or phase difference. In frequency modulation KE, f. = 5 , kconverts voltage into angle and is termed as a conversion factor. Many other conversion factors can be thought of. In general, a physical quantity ~ magnitude * unit. If the unit changes the magnitude will also change. We apply Physical quantities may be divided into fundamental and derived quantities Fundamental quantities The quantities which do not depend upon any other quantity are called fundamental or absolute or basic quantities. intially, only these fundamental uantities—length, mass and time were considered. With the development of science, four more physical quantities were added. These are temperature, electric current, luminous intensity and amount of a substance, 4 Pearson Guide 10 Objective Physics Derived quantities The physical quantities derived from fundamental quantities are called derived quantities like velocity, acceleration, force, momentum etc. Units The fixed and definite quantity taken asstandard of reference with which other quantities of the same kind are ‘measured is defined as a unit. Fundamental units The units of fundamenta! quantities are called fundamental units. For example, units of length, ‘mass and time or those of fundamental physical quantities are called fundamental units. Table 1.1 listsall fundamental and supplementary units (SI) and their symbols, Derived units Units of derived physical quantities are called derived units. For example, units of velocity, density, force, momentum, volume etc. Initially thee systems of units, namely, CGS, FPS and ‘MKS based on these fundamental quantities Length, Mass and Time came into existence. In 1970, in a world conference a consensus evolved and a standard international system of units was developed. It is more popularly known as SI system, In addition to seven fundamental units, two supplementary units were also included, namely, plane angle ‘rangle (unit radian abbreviated asrad)and solid angle (unit stredian abbreviated str). SI units since 1978 are observed throughout the world Practical units Apart trom fundamental units, ‘supplementary units and derived units, we come across some practical units like Light yeer, horse power, energy unit (unit =3.6» 10°), Curie (Ci), Ratherford (R) ete, While writing unit, the following convention is adopted: (@) Unit named after a person starts with a capital letter, For example, Newton is writtenas N, Curie asCi, Rutherford R. (b) Fundamental units are written with small letters, for example, metre as m and kilogram as kg ete (©) The symbols are not expressed in plural form. For ‘example 50 metres will be written as 50m. (@)_ Punctuation mark such as fullstop are not used after the symbol of unit. For example, | Millilitre = I mlor fee (not m.L orc.) Physical SL unidimensional unit symbol quantity symibol Length metre i m Mass Kilogram = M ky Time second = T s ‘Temperature Kelvin. «KK K Electric current. Ampere. A A Luminous Candela ca intensity Amount of mole mol mol substance Supplementary nits “Angle radian rad Solidangle stredian str Table. 1.1. Fundamental Physical Quantities Standards of length The most common unit is metre (i), Foot isalso used sometimes. In 1889 the standard metre ‘was defined as the distance between two fixed marks engraved on a platinum-iridium bar preserved at constant temperature of 73.16 K and constant pressure of I bar in the international bureau of weights and measures at Serv Paris in France. All other meters are calibrated with it to an accuracy of 0.Ippm. In 1960 the standard metre was defined in terms of wavelength of Kr-86 and is called atomic standard of length. Im is the distance covered by 1650763.73 wavelength of ‘orangered lightof Kr-6 in vacuum. An accuracy 1:10° parts can be obtained with it. Tn 1983 metre was defined asthe length of path travelled by light in vacuumin 3y;shrag th second, Some other importa 1A= 10m, Le-ray unit(1 XU)= 10m units of length are I yard = 3 foot= 0.9144 m, 1" (inch) =2.4.em J astronomical unit] AU)= 1.49 % 10!'m, [light year (I ly)=9.46 10!"m, 1 Parsee (I pe)= 3.08 x 10!m=3.26ly Standard of Mass Originally Ike mass was defined as the mass of ire (10° ce) of water at 4°C, now-a-days standard kgis the mass of platinum-iridium eylinderstored ina special vault in the International Bureau of Standards at Serves (France), The accuracy of this standard is 1 in 10* parts To measure atomic masses the unit amu (or u) is used, th of mass of {2 atom \ =167%10 ke 2° 6.023x10' In FPS system pound (Ib) isthe unit of mass, sometimes slug isalso used. Lb = 0.45 12737 kg, Ishug =32.2 Ib = 1459 kg. Note In astrophysics, we sometimes come across cchandrashekhar limit, 1 chandrashekhar limit =1.4 times mass of the sun = 2.8 X 10" kg. Chandrashekhar showed that ifthe mass of an object becomes 1.4 times the mass of the sun, under gravitational collapse it tums out tobe white dwarf, Standards of time solar or lunar motion, Initially, twas defined on the basis of year I mean solar day = = 365.25 day L year jen _. and T= 355955246060 But because of tidal frietion the length of day is increasing ata rate of7 us per year. Therefore, in 1965 atomic standard was defined. According to this standard Is is the interval of 91192631770 vibrations of radiation corresponding to the transition between two specific hyperfine levels in "Cs (cesium) clock which will go wrong by Is in 3000 years. Hydrogen maser gives @ promise of producing error of 1s in 33,000,000 years. Note that (J) time can never flow back, i.e. negative time does notexis and (i/)ata given insant of time a particle cannot be present in more than one position in space Table [.2 describes prefixes used for multiple and submuitiples of metric quantities. Dimensions and dimensional formulae: All physical {quantities can be expressed in terms of seven fundamental ‘quantities. The powers to which these fundamental physical quantities be raised are termed as dimensions. For example, force ~ MLT°, i., force has dimensions of mass, length and time as 1, | and -2 and [MLT" is dimensional formula for force. Dimensional formulae for work, torque resistance ae [M*LT*], [ML°T"] and [ML?T 4 *) respectively ato" deca dar Dd femio f 10% — heeto oh 10 pico. op 10% kilo k 10) nano on 10% megs M lo" mito! om 107 Giga G 10° milli... m_— 10° terra T 10" centi c= 107 peta P 10" decid 10" xa E 10" Table. 12 Prefixes used before units Applications of dimensional formulae (a) To check the correctness of a given physical relation It is based on the principle of homogeneity, that is, the dimensions on two sides bbe same for a given relation. For example, F> 6anry dimensions of LHS F = MLT? RUS 6xmrv= [MEP] [LYLT"]= MET? because LHS and RHS have identical dimensions, therefore, the relation is dimensionally correct. We the cim incorrect. (b) Todetivea relation — We illustrate with an example, The amount of liquid flowing per second through a tube of radius r depends upon radius of the tube, coefficient of viscosity and pressure gradient, Derive the expression, ns on two sides differ, the relation is Units, Dimensions and Errors 5 # wpe) gorr(t) WD _ gga Daan’ 2 oo Gane BY where & isa dimensionless constant ‘Then [NPLIT*] = [MET (1) [MET] Leeepen wes Comparing powers of M, L and Ton both sides 0 =ate " 3 =-ath2e ~a-2e ‘The value of is determined experimentally. aw we Renee ip * ar Here k= 28 (© To convert the value of a physical quantity from one system of units to another system of units We illustrate with an example. Let us. convert 11 (SI) t0 erg (CGS) system energy & = MET? “(ele (eel Limitations of Dimensional analysis 1, Thedmersional analysis cannot be applied to desive arelation involving sum of products or product of sums form like s=u!+Jat®. it ean derive & “ ctterm, Its =e + Li relation ofa single product term. 1¢ » = at +-£ tobe derived using dimensional analysis then s — ut and 5 will be derived separately and then added from your knowledge. 2, The dimensional analysis cannot be applied to derive the relation involving more than three unknowns. However, we can join two variables to ‘make one, for example, we used pressure gradient (4) and not p and 1 separately. Note that we ean 8 Pearson Guide toDbjective Physics check the correctness of a relation involving any umber of variable or terns, 3. Lftwo quantities havesame dimensions then euch a relation involving these quantities cannot be derived. However, we can check the comectness of the relation. 4, Numerical constants, trignometric ratios and other dimensionless ratio cannot be derived. 5. Fora given dimension physical quantity is not ‘unique, For example, Torque and energy have the same dimensional formula [ML'T-*). ‘Significant Figures These give the accuracy with which ‘physical quantity is expressed. The number of digits which ate known reliably or about which we have confidence in measurement, plusthe first digitthatis uncertein, ae termed as significant figures. For instance, length of a table is 137.2 ‘em. This has 4 significant figures and 2 (after decimal) is uncertain. It is worth mentioning thet significant figures of hysical quantity depends upon the least count of the instrument with which it is being measured. Rules for defermining significant figures 1. Allthe nonzero digits are significant, for example 187.25 has 5 significant digits, 2. All the zeros occurring between two nonzero digits are significant. For instance, 105.003 has 6 significant digits. 3. Thezeros occurring between the decimal point and the non-zero digits are not significant provided integral part is zero, i.e, in 0.0023 there are only ‘wo significant point figures. 4. All_zeros to the right of a non-zero digit in number written without decimal are not significant, For ‘example, 32500 has only 3 significant figures. Exception This rule does not work when we record the values on actual measurement basis. For example, distance between two places is 1210m has 4 significant digits 5. All zeros occurring to the right of non-zero digit in ‘a number written with a decimal point are significant. For example, 2.3200 kas 5 significant figures. Note: The number of significant figures does not vary with the choice of units. For example, length of a rod is 76cm. If ‘we represent it as 0.76m or 0.00076km significant figures remain. In exponent form. exponent figure does not contribute. to significant figure. Thus 7.24 x 10’has only 3 significant digits. Rules for Rounding off 1, Ifthe digit be dropped islessthan 5 the preceding dligitbe kept unchanged. For example, 3.92, maybe expressed 3.9. 2. If the digit to be cropped is greater that 5, the preceding digit be increased by 1. For example, 5.87 be rounded off as 5.9. 3. If the digit to be dropped is 5 followed by digits ‘other than zero, then the preceding digit be increased by I. For example, 14.454 berounded offs 14.5 0 the first decimal place. Resolution, Accuracy and Precision of an Instrument Resolution Stands for east count or the minimum reading which instrument can read. ‘Accuracy An instrument is said to be accurate if the physical quantity measured by it resembles very closely to its true value. Precision An instrument is said to have high degree of precision if the measured value remains unchanged, howsoever, large number of times it may have been repeated. Errors in Measurement Deviation between measured ‘and actual (or mean) value of a physical quantity is called ‘error. For example, if a, is measured value of a physical ‘quantity and its true value iso, then error = Aa = |ay —4,| Errors my be divided into two types: systematic and random errors. Systematic Error Errors arising due to the system of ‘measurement or the errors made due to parts involved in the system of measurement are called systematic errors. Since the system involves instrument, observer and environment, therefore, systematic error is of three types, namely, instrumentation error, personal error (error made by observer due to carelessness or eye defect) and environmental error. Instrumental error equal to least count of instrument is ‘unavoidable and hence is always accounted for. Random error (or statistical error) The error which creeps in due to 2 large number of events or large quantity are termed es random errors, Consider an example. The probability of tossing a coin is 1/2, If we toss a coin 1000 times or 1000 coins are tossed simultaneously then we will hardly ever get 500 heads and 500 tails. Thus we find even random errors eannot be removed. Methods to express error from its mean value (of ell observations) is called absolute error. Thus As, =[x;~x,| where x, is mean value and x, is the ith component of the observation. Le, ratio of mean Bu absolute error to the true value is called relative error. © Percentageerror equals to relative error «100 Propagation or combination of errors 1, When x = a + b then maximum possible % error 100 2. When x= a—b then maximum possible % error = A 109 M800 x a-b 3. When x= ab or x= a/b then maximum possible % sro go- (00-42) . @ 4. When x= 22 then maximum possible % error ye Least count of vernier callipers also called vernier constant (VC) VC = (IMSD ~ 1VSD) (value of IMSD). If n VSD coincide with (n— 1) MSD then VC = (-2) (value of IMSD) Least count of screw gauge or spherometer . pitch ‘number of divisions on cirular scale number of divisions moved on linear scale \d pitch = = ee ‘number of rotations given 1, Remember all possible formulae connecting the physical quantity and see whose dimensions are known to you. Use that formula, for example c-24 ca2, 2-2 d ©. 2c if you use cn Z AE wine then the dimension of C is easily calculated. 2. Remember all the rules to find significant digits mentioned in Brief Review. 3, % 10. Radius of curvature using spherometer R =— Units, Dimensions and Errors 7 Remember the least count of instrument, sometimes itisnot given in problem; remember the rules to find the propagation of error and its calculation, ‘Ax _Aa+Ab @ Ifx=a+b then == a+b Ax (© x= a-biten == EE Ax (© Ifx=ab orx=alb then arb! @ Wee nda soccer ott or in general Ax = x (relative error) ‘Note that as we find maximum possible error, therefore, error due to each variable is added. Dimensional analysis always works to check the correctness of a relation dimensionally. In other ceases, it has its limitations as mentioned in Brief Review. If in a vernier callipers m VSD coincide with (n—1) MSD then vemier constant or its least count n-1 is We=(t (value of MSD) or = (value of MSD). Least count of screw gauge or spherometer pitch number of divisions on cireular scale and __number of divisions moved on linear scale piteh ‘number of rotations given = Linear distance moved in one rotation. Random error ~ Vn_where n ~ number of events or n= number of quantities. P oh 2 2 = Pearson Guide to Objective Physics ‘Not recalling the relation whose other dimensions arcknown in order to find the dimensional formula of a given physical quantity Assume we have to find dimensions of resistance wewrits R= — To write dimensions of Rwe need T to know dimensions of If we use power P= /°R or MET . Then R= [ML°7 47] can be ‘Not remembering rules to find significant figures. ‘Must remember these nules with exceptions given intext, Assuming that inx=a— or *=2 while finding 6 error, the error part of b should be subtracted from error part ofa. 5. 6. Since we always list maximum possible error, therefore, errors of each part must be added. Assuming only scalars and vectors 10 be physical quantities. Physical quantities are even constants or ratios like refractive index. specific gravity ete, Phasors which have amplitude and phase and follow triangle law ‘of addition SHM, waves, AC potential and current etc. are phasors. Moment of inertia, strain, stress, refractive index of anisotropic medium are tensors. ‘Assuming error is made by observer only due to hissher carelessness, ‘This type of error is personal error. Instrumental and environmental errors also occur. Random or statistical errors are also present. Assuming accuracy or precision of an instrument to be synonyms. Accuracy means the measured quantity by an instrumentis very closeto true value of the physical quantity. The more close the measured quantity to ‘rue value, the more accurate the instrument is If we repeat the measurement large number of times and each time we get the same reading then instrument is said to possess a high degree of precision. SOLVED PROBLEMS 2 Which of the following does not have the same dimension? (IIT Screening 2005) (a) Electric flax, electric field electric dipole moment (b) pressure, stress, Young's modulus (0) Electromotive force, potential difference, electric voltage (d)_ Heat, potential energy, work done. (@) Electric ux = Electric field x Area, dipole charge * length ‘The ratio of the dimension of Planck’s constant and that of moment of inertia is the dimension of ‘moment (a) time (b) frequency (©) angular momentum (@) velocity [CBSE PMT 2005} Outof the following pair, which one does not have the same dimensions? [AIERE 2005], (@) angular momentum and Planck’s constant (b) impulse and momentum (©) moment of inertia and moment of force (@) workand torque. (©) MOI [= Mi? = [ML2], moment of force =la=r T*, (©) MLT*, MET" (a) ML°T, MLT* @a fe MLT", Mur 7. Which of the following is not a unit of Young's modulus? (CET Karnatka 2005] (Nm (0) Nm (6) éyneem* (4) mega pascal ®) 8. If Mis macs of the earth and 2 its radius, the ratio of the gravitational acceleration and the gravitational constants {CET Karnatks 2005) oh, oO () MR? @% 3 = OM fam, Emer G Mea 9. A hypothetical experiment conducted to find Young's. cosOT't modulus ¥=SS 575 where + is torque and fis length then find x {CBSE PMT 2005 Mains} T'(MET*) ME y=ac'r? = — Y=Mr'r~ F outer 3 B 10. Find the dimensions of} (amar? (yer omer MT? 3 & ow density = SEMEN Gc) f= energy density = MET? _ yyips 77 Units, Dimensions and Errors 9 11. While printing a book a printermace certainmistakesin the following relation. Find the corectrelation, @) y=Asinod (b) y=Asin(@x+6) © y=Asin(or+9) A @ y= Zsinor+8 © 12, The velocity of surface waves depends upon surface tension, coefficient of viscosity and density. The relation is £ £ fa) pn (b) ” np 2 oF OF () (MET) = [MP2] [ML (MLA} ‘ s onsolvinga=1,b=—1e=0, y= 13. Which of the following pair has different dimensions? (@) Electric Pressure, energy density (b) Intensitye, Eze (©) Reynolds number and time constant (@ work, Torque © 14, A chocolate cookie isa circular disk of diameter 8.50 + 0.02 em ané thickness 0.050 + 0.005 em. The everege volume in em*is (@)2.83 +03 (11.35 4:12 (0238 £027 (931 £1.12 a _2dr at (ove wy Hated e dy =2,83{ 2222 4 008 @5 050 = 0.296 em ‘Thus V = 283 =0.3cm* 15, The fastest commercial aitineservice is 1450 mith, Find the speed in kmh and ms" @) 1938kmr', 618.3 ms" 10 Pearson Guide to Objective Physics (>) 2030 kmh, 623.1 ms* (©) 2334kmh",647.5ms* (@ None (€) 1450 1.61~2334mb-' 1 mile~ 61m 2334-5 ms! = 6475 ms! 16. Twocapecitors C,=5.2uF + O.uF and joined (i) In series (ii) In parallel Find the net capacitance in these two cases. (@)28%, 1.23% (0)3.6%,1.31% (©)34%,1.3% (3.9%, 1.15% (4) Inparallele=c,+e, and Bex 100 = AatAcs e ate, = 12.2HF are _ 0.2100 10 = 4 ae ate Inseries c= and Ol, o4 = (2424) x 100+ 115=3.9% ($3 3} 100+ 1.15=3.9% 17. CV" stands for (a) electric flux (b) electric pressure (c)electric field density (d) capacitance (@C=01V 18. A spherometer has 20 threads per em. Its circular scale has 100 divisions. Find the least count of spherometer. (a)5 um (b) 50m (03pm (@05um ® pitch Least count = PC) __ ‘number of division on circular scale GF oset08 F705 10%em 19. Vernier scale of Vernier callipers has 50 divisions which coincide with 49 main scale divisions. Find the Vernier constant. Given: there are 20 main scale divisions cm’ (@) 100 um (6) 1000 um (©) 10um (@)ynone of these BEM 6) C= Z5x(vahe of | MSD) Eo =30 39 ~0.001em |. The legs of a spherometer are 5 cm apart. There are 10 division cmon linear scale and circular scale has 100 divisions. The height h of ¢ convex mirror measured is 2 MSD + 37 circular scale divisions. Find radius of curvature of convex mirror. (@) 20.003 em (b) 18.408 em (©) 17.399 cm (4) 17.983 om Ho 0 em Least covat = 20 BENE 0) Least coms He h=2 (0.1) +37 (10°) =0.237 cm Ph 3 = Gey 10118 = 18.408 en * 6h 2 21. A student measured the length of a pendulum 1.351. and time for30 vibrations is 2 minutes 10 sec using his wrist watch. Find the percent error in g committed. (a) 1.72% (b) 1.813% (©)1.63% (1.513% (©) Percent erorin g = Ae 100 = (82+ 247 100 = AEs = (428 221) .100 =0.73+ 154= 1.613% 30 ote 22, Ifforce F= varies with distance r, Then write r the dimensions of K and b. Exponential must be dimensionless, Therefore ‘shall have dimension L"! and K= Fr=(MLT?) (2) = MDT? 23. What is the order of hair on your head? le 10 (10° @10 @0 24, What is the dimensional formula for resistivity? How does resistivity of Ge varies with temperature? MET?A?, Since Ge is a Samm »- 7 semiconductor, its resistivity falls with rise in ‘temperature. 25. Given x= 7" . The percent error in measurement of 4, band pis 1%, 0.5% and 0.75% respectively. Ifn=2, m =2 and r=4 then percent error in.x is @o (06% (©)5.25% (075% do, Foo (Stems =2x1+2(0,5)+4(0.75)=6% 26. Ina system of unitsif force F, acceleration A and time T are taken as fundamental units, then the dimensional formula of energy is [BHU 2005] (FAT (Far? (Fur (@) FAT (&) Energy = ML*7? =(MLT3} (LT?) (TF = Mist p2eabe a =1atb= abel, a-2b+e=-2 and c=2 27. Ina quartz oscillator L, Cand Rare analog of (@) compliance, mass, viscous damping (b) mass, viscous damping, compliance (© mass, compliance, viscous damping (@) viscous damping, compliance, mass © 28. Choose the correct statement. (@)_1 second = 10* shakes (b) 1 year has less number of seconds than number of shakes ina second (© _ 1 year has more seconds than number of shakes in a second @ A century has more minutes than number of shakes in a second. @.0) 29, ‘The dimensions ML'*T* may correspond to (@) work (© pressure © and () 30. Choose correct statement/s (®) A dimensionally correct equation may be correct (8) A dimensionally correct equation may beincorrect (b) linear momentum (@) energy density Units, Dimensions and Errors 11 (©) adimensionelly incorrect equation may be comreet (@ a dimensionally incorrect equation may be incorrect (a), (b), @) 31. A unitless quantity (@) never has nonzero dimensions (b) always has nonzero dimensions (©) may have a nonzero dimension (@) does not exist. @ 32. If force, length and time are fundamental quantities, then find the dimensions of (a) density (b) pressure (@)F=MLT? density = ML: (b) pressare = FA = FL? 33. Which of the following are dimensionally correct? FIAT @ = 2Toes8 © fe pre Pp amet = mst On on Ory (@), (b) and (c) are dimensionally correct, a& 1 «ae (@)isdimensionally correct () dimensionally incorrect (©) such mathematical relations cannot be tested (d) cannot say {Solution RO) Read the following passage and answer the questions given attheend. When numbers having uncertainties or errors are used to compute other numbers, these will be uncertain. It is especially important to understand this when a number obtained from measurements isto be compared with a value obtained from theoretical prediction. Assume a student wants to verify the value of the ratio of circumference to diameter of. circle. The correct value of ten digits is 3,14159 2654, He draws a circle and measures its diameter and circumference to its nearest millimeter obtaining the values 135 mm and 424 mm, respectively. Using acalculatorhe finds F=3.140740741. Note thatat least last six digits in hisresult are meaningless. 1. Why does measured value not match with calculated value? 412. Pearson Guide to Objective Physics duc to instrumental error due to personal error due to random error ® ® © (@)_ none of these (@ When an instruments used tomeasure, it has its least count. Therefore, we can measure the value to nearest correct value end rot absolute value 2. A micrometer is made using a bar having 20 threads per cm. On its circular scale it has 100 divisions. A student reads 5 divisions after 1 em mark on linear scale and 37 division of circular scale, coinciding with mainscale for the diameter of a cylinder. Write the diameter. (@) 1.2537 em (b) 1.2685 om (6) 1.2537 £ 0.0005 em (4) 1.2685 + 0.0005 om (d) d= pitch * (number of divisions on linear scale) + least count (number of circular scale divisions coinciding with linear scale) 1 20x100 1 err X37 = 1.2685 d= 1.2685 + 0000Sem 3. What is the value of z in the passage measured up to 4 digits by the student @3.140 (3141 (3.1407 (aia ((b)3.141 (ass) Read the following passageand answer the questions given attheend, ‘Though dimensional analysis is very useful in deducing certain relations, itcannot ead us too far. First of all, we need to know the quantities on which a physical quantity depends. Even then the method works only if the dependence is product type. For example, the distance travelled by a uniformly accelerated particle depends on the initial velocity u, acceleration a and the time ¢. But the method of dimensions cannot lead us to correct expression for x as it involves sum of two terms. Moreover, numerical constants having no dimensions cannot be deduced. | Also, the method can be employed if there are as many equations available as there are unknowns. In mechanical quantities we have 3 base quantities L, Mand T and hence 3 ‘exponents. Ifa physical relation depends on more than three bbase quantities it cannot be deduced. Similar constraints are present in electrical or other nonmechanical quantities. 1, List the base quantities used in electrical measurements. @3 w4 @> @7 | (ML, Pand For Qare fourbase quantities. 2. Listthe limitations focedin dimensional analysis EE @ Sum of products or product of sums camnot be determined, ice., expression should involve only ‘one term (product term) ‘Numerical value of consiantslike 7, ete. cannot be determined. (ii) Method ceases to work in mechanics if physical ‘quantities are more than 3. i), av _ prt 3. To derive “G,—"gay What method should be used? GEMM Since there are 4 variables and two of same dimensions, say r and /, we cannot derive directly. Instead we club £ as single quantity called pressure ‘gradient. 4, The uncertainty in measurement of the length of a window is L + /. The width measured is Ww. Find the uncertainty in measurement of area. Assume I and wesmall. But 0) cows 20H bw is Om (@ Li+ a or Ad =1W+ WL oie 1. Curie is the unit of (a) decay constant (b)activity (ophalt-ife (@average life ‘STunit of water equivalentofcalorimeteris (@)Kg (Ke Oke Ke" 3. The magnetic moment of electron is (a) 9.27* 10Joule/Tesla (0) 9.27 * 10 Tesla/Joule (©) 927107 Joule/Tesla (@)_ 927*10*Testa/Toule 4, One electrostatic unit (est) of charge is equivalent to {@) 33% 10Coulomb (b) 33% 10*Coulomb (©) 33% 10"Coulomb (@ 33% 10" Coulomb 5, The value of solar constant in SI system is— (@) 1340 watvm? (6) 1340 wat? (1340 mivatt (0) 1340 wattim 6. The correct statement about Poisson’s ratio is— (a) lis unit is Nim? () Itis dimensionless, (0) Its unit is Newton (@) Its dimensions are MLT The SI unit of gravitational potential is— (a)Joule/Kg (©)oulexkg, (Kg/loule (A) JoulexKe? 8. One watt-houris equivalent to (23.6 10° Joule (0)3.6« 10 Joule (63 10'Joule (63 * 10" Joule 9. An air bubble inside water oscillates due to some explosion with period 7.If Tor" E then deermine the values of a, b and c. Here P, dand E are the static pressure, density and total energy of explosion of water, respectively. The dimensions of intensity of energy are (a)MLT* (MET? (OML’ (Mir 1. The MKS anit of the quantity x 14/21is— (@)Naw/Radian (®) NinvRadian (Radian/Nim (@)NimvRadian? 12. Ocrsted in the unit of — (@) Intensity of magnetization (b) Magnetic moment (©) Magnetic induction (@ Magnetic ux 13, Lux isthe nit of — (@)Luminous flux (©) Luminous intensity (©)Dersity of illumination (4) Luminous efficiency Units, Dimensions and Errors 13 14, MAL°7° 0 are the dimensions of — (®) coefficient ofthermal conductivity () coefficient of viscosity (©) modules of rigidity (@)_ thermal resistance 15, Ampere-houris the unit of — (@)power (b) energy (c) quantity ofelectricity (d) strength of current 16. Which of the following is not tke unit of time? (2) leap year (©)lanarmonth (6) solar day (@)parallactic second 17. The dimension of the ratio of angular momentum and Jinear momentum is— (Le (Org OL (@) MLT 18, The SI unit of form factor is— (@ Ampere (b) Vout (©) watt (@) none of the above 19. One micron is equivalent to— (@) 04m (&) 10%m (©) 10'm @10'm 20, The velocity of a body falling under gravity is directly proportional to g*h’. If g and h are the acceleration due to gravity and height covered by the body, respectively, then determine the values of a and 6. 1 1 1 () yand 5 (by and -> 1 1 tt @ yan -5 @-; a5 21. The velocity ofa particle depends upon time according to the relation v= or + Ur 7The dimensions of e and ywill be— ()LT2,L1T" (b)L,L77,L72 ()ET,LLT? ITAL TL 22, iff he formula X= 3¥Z?, the dimensions of X and Z are those of the capacity and magnetic induction, then the dimensions of Y are @Mmeversat )eLeTst (MATA @ Meiers 23. The mass of electron in MeV is— (@) L02Mevic? (b)0.51 Mevic? (@)51 Mevic? (@) 102Mewic? 24, The ratio of nuclear magneton and Bohr magneton is— mm, Onn, omm, (@2mym,, 14 Pearson Guide to Objective Physics 25. The value of Faraday numberin SI unit is— (©) 9.65 Coulomb/Ke/equivalent (©) 9.65 « 107 Coulombikg/equivatent (©) 9.65 107 Coulombikg/equivalent (@ 9.65 Coulombikg/equivalent 26. The velocity of ripples on water surface depends upon the wavelength A, density of water d and acceleration duc to gravity g. Which of the following relations is correct among these quantities? (@) Pogh (by V2aeigh (©) Vakgd (d) Pagad 27. The fundamental unit of the quantity of matter is @ke ()mol. (gm (@mecer 28, In an experiment to determine acceieration duc to. | gravity by simple pendulum, a student commits 1% positive error in the measurement of length and 3% negative error in the measurement of time period. The percentage error in the value of g will be— @m% (b) 10% (©) 4% (a) 3% 29. Which of the following pairs is not matched? (©) Coefficient of self-induction-henry (b) Magnetic flux-weber © Electricflux-voltmeter. (@ Electric capacity-farad-meter. 30. Ifthe units of ML are doubled, then the unit of kinetic. | energy will become (@) times (b) 16 times (c)4times: (d)2 times 31. The SI unit of (1/2 VEC ) is equivalent to that of (@) Timeperiod (©) Frequency (© Wave tength (@) Weve number 32, Light year is theunitot (@) distance (©) time (© speed (@ mass 33. Theratio of MKS units and CGS units of coefficient of viscosity is— MKS nMKs =10 INKS 9, © nocs © ncas=°* MKS _ ni © cas =! © ncGs 34, The dimension of the expression Jf /MB is @L wr en or 3S. The Slunit of mobility of charges (1) is (a) Coulomb/s/Kg (0) Coulomb/Kg/s (€)CoulombiKy/s* (@) Coulombs/kg 36, The dimensions of the coefficient of viscosity are (@)MLT" ()MLT (MT @MuT 37. Ifthe error in the measurement of radius of a sphere is 1%, then the error in the measurement of volume will be (@)8% 05% 3% @ 1% ‘38. The ratio of the atomic radias to nuclear radius is (a) 10" ) 10" 10 @ 107 39. One fermi is equivalent to (@) 10" meter (©) 10 meter (©) 10 meter (@) 10 meter 40, Debye is the unit of (a) Magnetic dipole moment (b) Electric dipole moment (©) density (@_ RMS velocity 41, ‘The units of the temperature coefficient of resistance are @aK )K! (aK @@K 42. Match the following (@) Energy @)ML"T? (©) Torque QMLT* (©) Planks Constant @ MULT? (@ Angular Momentum (8) MLT? 43. How many wave lengths of Kr are contained in one meter? (@) 155316413 (0)652189.63 (@z4n27123 (6) 1650763.73 444, A physical quantity is represented by the relation Y= MLIT-. Ifthe percentage errors in the measurement of M,L and Tare a%, 8% and y% respectively, then the total error will be— (8) (ca— f+ 70)% (b) (aa ~ Bo~y6)% (©)(0a+ f+ 1% (@ (cat fo-w)% 45. A science student takes 100 observations in an ‘experiment, Second time he takes 500 observations in the same experiment. By doing so the possitle error becomes (a) Stimes (b) 1/Stimes (©) unchanged (d) none of these 46. Ifthe error in the measurement of radius ofa sphere is. 196 then the error in the measurement of volume will be (a) 11% — (b) 3% (5% (8% 47. Match the following Giteydiay = alee (b) Power (Qur (©) Blectric Pressure (ey Mer* (4) Surface Tension (yar? 48. Light year is the unit of (a) speed (b)mass (©) distance (@time 49. Debye is the unit of (@) density (b} rms velocity (©) electric dipolemoment (@) magnetic dipole moment 50. The unit of surface energy per unit area may be expressed as (a) Nm? (b) New (Nm @Nm Read the following passage and answer the questions given attheend. Physical quantitics are often divided into fundamental quantities and derived quantities. Such a division is arbitrary. A given quantity may be regarded as fundamental in one set of operations and derived in another, Derived quantities are those whose defining operations are based on other physical quantities. Examples of derived quantities are velocity, zceleration, force, work, momertum torque, volume, density cic. Fundamental quantities are not defined in terms of other physical quantitics. The number of quantitics regarded as fundamental is the minimum number to give a consistent and unambiguous description of all the quantities of physics. The fundamental quantities operational definition involves two steps—standard and the establishment of procedures for comparing the standard to the quantity to be measured. ‘An ideal standard shall have two principle characteristics. (i) Itis accessible, and 1.1m defined as the distance covered by 1650763.13 wavelengths of red-orange light of Kr“ (an accuracy 1:10” can be obtained) is ii) itis invariable, Units, Dimensions and Errors 15 {a)accessible (b) invariant (©)both (a) and (b) (@) inconsistant © 2. Chandershekher unit is the unit of (a)time (b) length (©)light intensity (a) mass @ 3. The time defined by '* Cs clock needs correction of 1s in every (a)3 days (®) years (©) 300 years (@) 3000 years @ 4, Hydrogen maser if used as standard of time gives a promise of producing error of | S in every (a) 33 years (6) 33 * 10° years (©) 33 «10° years (4) 33 * 10° years @ Read the following passage and answer the questions given atthe end. Absolute error isthe deviation from true value ofa measured value or deviation of ith value from its mean value (of all observations). Thus Ar, = |, x,| isabsolute error where x, is mean value and x, is the ith component of the observation. Relative error isthe ratio of mean absolute error to the true Value of physical quantiy ic. “or A® i relative eror. ae Error ispropagated. Inan instrument least count is considered as the maximum deviation. 1, A screw gauge measures more accurately than vernier callipers. implies (a) screw gauge will read precisely (b) screw gauge will read up to more decimal places (©) the errors of measurement in screw gauge is less ‘than that of vemiercallipers (@)__the error of measurement in screw gauge is atleast less by an order of magnitude, @..(4) 2. A student took 1D0 readings of a project under investigation. His friend took 400 readings. In which cease number of errors are less? (a) who took 100 readings (b} who 100k 400 readings 16 Pearson Guide to Objective Physics (©) both have errors equal in mumber (@)_ insufficient data to reply Em © 3. A serew gauge has 100 divisions on circular scale and 10 division in 1 em ona linear scale. Then least count is (a) 1mm (b) 0.1 mm () 107 mm (@) 10° mm Em © PASSAGE3 fe ‘Read the following passage and answer the questions given atthoend, Measurement can never be made with absolute precision, physical quantities obtained from experimental observations always have some uncertainty. A distance measured with an ordinary ruler or meter stick is usually precise only to the nearest millimeter, while a precision micrometer caliper ean measure distances dependably to 0.01 mm or even less. The precision of anumber is offen indicated by following it with the symbol + and a second number indicating the maximum Deak MC UPS aes 1 2 (3) 3. @) 4 8s () 2 () 10.) " 15) 16. (a) 7 ©) 18 22. (0) 23.) 24. (@) 25, 23. (¢) 30. (a) 3.) 32 36. (a) 7 ©) 33. @) 38, 142. {(@ 0), (0 >), (© Rd > R 43. 47. (P.O), 9R),(¢ P,Q, 98) 48 likely error. Ifthe diameter ofa steel rod is given as 56.47 + (0.02 mm, this means thatthe true value is unlikely to be less than $6.45 mm or greater than 56,49 mm. 1. The MOI of a body is J, its mass Mand Radius R. If radius is increased by 0.1%, find the increase in MOI. (a) 0.5% (0) 0.1% (©0.2% (A) none of these © 2. Which is more accurate if the following are the least counts of certain instruments? (a) 0.01 cm (6) 0.01 mm (0.01 m (@) 0001 km ©) a 1 3 “p Dee eeerine ne eee 1 i ‘ 5%. The erroris measurment of Fis (a) 1.5% (0) 2.0% (c) 2.5% (0) 3% ©) (e) 5 (a) 8. (b) 7 (a) (a) 2 ©) 8 (6) « ie) » ©) 2. () a) (b) 26. (a) 27, (b) 28. (a) (a) 33. (a) 34. (b) 35. (b) © a @ a.) (¢) 44. (c) 45. (b) 46. (b) ©) a. @ 50. (b) BRIEF REVIEW Vector The physical quantities which have magnitude and direction and are added according to the triangle law of addition are called vectors, or, directed segment which follow triangle law of addition ae called vectors Properties of Vectors In addition to magnitude and unit (@) it has specified direction, (b) it obeys triangle law of addition, (c) their addition is commutative i.e. AtB=B+A (a) Their addition is associative (4+B)+& ~ H+ (B+) Representationof Vectors in two forms: polar and cartesian. Vectors may be represented LAs § Paar pinssnon or weer PolarForm inthisform 04 and Gis angle as shown in Fig 2.1 (7, 8) where rismagnitude In this f m dmajtaj+a,k where and a, are coefficients and i, j, & are unit Vectors along +.y and zdirections, respectively, as illustrated in Fig, 22 Unit vector representation in rectangular coordinate system Types of Vector three types: In general vectors may be divided into 1. Proper Vectors 2. Axial Vectors 3. Inertial or Pseudo Vectors. ProperVectors Displacement, force, momentum etc. are Proper Vectors. ‘AxialVectors The vectors which zct along axis of rotation are called axial vectors. For example, angular velocity, torque, angular momentum, angular acceleration are axial vectors. Pseudo or Inertial Vectors The vectors used to makea ‘non inertial frame of reference into inertial frame of reference are called pseudo or inertial. Vectors may further be subdivided as @NullVector Ithas zero magnitude and indeterminate direction, UnitVector Magnitude ofunit vectors 1 Itspecifies 34 denmpmnvemnavennca= 4 4 i.e, vector divided by its magnitude represents unit vector. (ii) Like Vector or Parallel Vectors If two vectors have the same direction but different magnitude then they are said to be parallel or like vectors. Fig 2.3(a) shows like vectors. ey ss Like vectors () Unlike Vectors Two vectors having opposite directions and unequal magnitudes are called unlike vectors of parallel vectors in opposite sense. If their magnitudes are equal, they are called opposite vectors Fig 2.3(b) shows unlike vectors. ly ___ GREZIOME Unlike vectors (Equal Vectors Two parallel vectors having equal ‘magnitudes are called equal vectors. (i) Co-initial Vectors point, they are known as eo-initial vectors. In [f vectors have a common initial 24, OA ,OBand OC are co-initial vectors. ac Costa vectors (sil) Co-finear Vectors Like, unlike, equal, opposite vectors may be grouped as co-linear vectors if they are either in the same line or parallel. (viii) Co-planarVectors are termed as coplanar. Vectors lying in the same plane Resolution of a Vector Resolving 2 vector into its components is called resolution of a veetor. Using triangle lawonecan write in Fig.25 A= 4, +4, or = 4,1+4,5 Acos6i+ Asin j or a and Laws of additi¢ (a) Triangle Law represented compietely (in magnitude and direction) by two. sides of a triangle taken in order, then their resultant is represented by third side of the tangle tsken in opposite directions. In the fig 2.6 (a) OP+PG = 09 or A+B=R (@ @) Fig. Triangle law illustration If three vectors acting on # body may completely be repesented by three sides of a triangle taken in order then thesystem isin equilibrium. In Fig. 2.6(b) OP + PG+OR=4+B+(-R) or R-R ParallelogramLaw _ Iftwo vectors acting ona body may be represented completely by two adjacent sides of @ parallelogram, then their resultant is represented by a diagonal passing through the common point. In Fig 2.7 from equal vector PL = 00 = B from Alaw 4+3=R [GEREEEEE Perattctogram law of vector illustration V+ B +24B5ind Bsin6 A+ BeoO Note: 4-BSR&A+ Bie. R,,=A~B when 0 180° and R_,, = A+ Bwhen 8=0? 0¢ 8S 180°, Remember 8cannot exceed 180" ‘Note: Minimum number of coplanar vectors whose sumcan bbe zero (or required for equilibrium) =2 (if vectors are equal and opposite) =3 if vectors are unequal or not opposite, minimum ‘number of roncoplanar vectors whose sum can be zero=4 ‘Note: Subtraction of a vector is equivalent to addition of a negative vector. Multiplication of Vectors Two types of multiplication is defined (a) dot product or salar product, (b) Crossproduct fo vector product, Dot product or Scalar Product tan If the product of wo vectorsis a scalar, then this rule is applied AB ABcos0 BA ,ie., scalar product is commutative A{B+C)=AB+46, i.e, scalar product follows distributive law. Application of Dot Product 1. When the product of two vectors is a scalar. For Fs ,P= FX coment? example magnetic flux ¢= JB.ds etc 2. Tofindan angle between two vectors 8= cos: Vectors 19 I the dot product of two non zero vectors is zero, then tiey are perpendicular to one another. Find the component of a vector along a given direction. For instance, the component of 4 along B is Acos@= AB 7 Cross product or Vector product This product is used when the product of two vectors is a vector, ic., AxB ~ ABsind where fisa unit vector perpendicular to both 4 and B. Apply right-handed serew rule to find the dircotion of or 4 xB . Vector product is noncommutative ic. Ax ## Bx A (magnitude will be equal but direction will be opposite). Vector product is distributive, ie, Ax (B+E)= dx Bie Axe Rules: (i) 4 Ix] =hoxk (i) ix F-8 --Jxi, ix - 7 --’x], kxi = j--ixé Application of Vector Product 1. Cross product is used in rotational motion cr product ‘of two vectors is a vector, For example, Torque = sad PF , Poynting veetor B= Exii= 7, Angularmomentum £ (x8) 2. If the vector product of two nonzero vectors is zero, then they are parallel. It can be used to find angle 8 ) le |4>844[2R, = 4+ 8) 19, 3144 7+2k , B61 ~ 743k . Finda vector parallel to. Awhose magnitude is equal to that of B 9 (Blirsie2i) om (sii) © {Blais 28) (d) none ta (sisaj-+28\ views |= = 'al 9+16+4 - fE(irsie2i) 20, 8,2 are ree coplanar yectrs. Find te vetorsum a=8]-j.b--3) +2 ay (ay V5 ,297° (b) ¥3.63° (©) 3.297" (d) 3.63" BMT (a) R =a +h +2=7-27 [|= YBandiane =~2 01 9-297" 21. A block of mass m is connected to three springs, each of springconstantk asshown in Fig, 2.18. The block is pulled by x in the direstion of C. Find resultant spr constant, (ah (b)24 3k (c)3« a> (6) F= = (kx + bcos 60+ kc0s60) 2h eh, = 2k Fig. 2.18 22. A particle moves in the x y plane under the action of, atorce F such thatthe value ofits linear momentum p) at any instant is p = 2 (cosri + sint j). The angle 6 between F and p is (a) 60° (yas {e) 30° (a) 90° BEM «) (-sinsi+ cost) Fp = 4{-sintiscose]),(costi«sine} @ =90° 23. Consider a collection of large number of panicles, each moving with a speed v. The direction of velocity is randomly distributed in the collection. The magnitude of the relative velocity between a pair of particles averaged over all the pairs inthe collection (ayy mw oF or EN) 7, Wael = Vie? ay cos eo = 2v sin 2 (average the steamer the wind appears to blow at 18 km/h due north. Find the velocity of the wind. 26 Pearson Guideto Objective Physics (a) 5/5 ms" 30° North of east (b) Sms"! 60° North of east (c) 5Y§ ms 60° North of east (@) Sms '30° Northof east BRIM ©) = 1.1 (PaitY,3) -107 =59 or (¥i4¥,]) =5) +107 or t Wl = sy tan0= 5 or @ =30° ie. wind isblowingat 5/5 ms' 30° North of east 25, The position vector of a particle i 7 = alvosau i + sino 7}. The velocity of the particle (@) parallel to position veetor (b directed towards origin (©) directed away from origin (@) perpendicular to position vector. a0- sina + cosa 7 and a SEM ©) + = 26. A force 61+3)+% Newton displaces a particle from A(03,2) toB(S,1,6). Find the work done. (103 (6/225 (32) @au © d= Si-27+4k W =F = (01+37+8).(97-27 44k 24 27, Wind is blowing NE with 18 3 km hand steamer is heading due west with 18 km fr!. Inwhich direction is the flag on the mast fluttering? (a) North West (b) North (©) South West (@) South. OF =F saat F oup—5+51+5) =5 7. The flag will luterina direction opposite 10 the direction of motion. 28. The resultant of two forces equal in magnitude is equal 10 either of two vectors in magnitude, Find the angle between the forces (a) 60° (b) 45° © 90 (@) 120° ERM (0) = FPF om cose QO 120° 29, Aman goes 100 mNorth then 100 m Eastand then 20 m North and then 100.3 m South West, Find the sisplacement, (@) 20m West (b)20mEast (©)20m North (4) 20mSouth (c)d= 100 j+ 1007 +207 +(- 1007-1007) =207 TYPICAL PROBLEMS 30. Ariverflows3 km h''anda man iscapable of swimming, 2 km r!. He wishes to cross it in minimum time. In which direction will he swim? 2 ©) cos" | (2 (@) sin | 5 i(2 (2 (tan | 5 @eot'| 5 (a) Let us assume he swims atan angle @ with the perpendicular as shown. If river is Jm wide time taken to cross it —2sing horizontal distance covered along x direction during this period 1 =(-28ina 7355 forrtobemin & =o, for to bemin or ‘[}ecsoumne age 4] or 2 Vectors 27 Dose Oyo, 2 o Tend ¥ ® Fig. 219 ' ) i 31. A pilot is to flag an aircraft with velocity v due east " ‘Wind is blowing due south with 2 velocity . Find the o © time for a round joumey A to B and back (4 and B are Fig. 2.24 I distance away), “4 34 il ws angle rucca maneciea Wor (a0 (b) z n x OF OF SIEM ©) See Fig. 222 Fig, 2.22 Fig. 2.20 35, Two identical pendulums are tied from the same rigid support. One is tied horizontally. The other isreleased ‘when they are making the same angle @ with the vertical, 32. When mass mis rotated in a plane about a fixed point, its angular momentum is directed along (a) the radius (b) tangent to the orbit (©) the axis of rotation (4) 45°to the axis of rotation rave (by cote or (a) see*@ BSN (©) because angular momentam isan axial vector. (9) 7, cose = mg or 33. A pendulum hangs from the ceiling of a jeep moving it with a speed v alonga circle of radius f. Find the angle er with the vertical made by the pendulum, T, =mg cosé @o To T, ~ cos (@) none of these =secto 28 Pearson Guideto Objective Physics sew A me me Se Fig. 2.23 36. Sixteen beads ina stringare placed ona smooth incline as shown in equilibrium. The number of beads lying along the incline ate (at ie (2) none of these BRIMED ©) Lot reads be hanging vertically. Then (16 ont (os 1 sing = 5 n= ny mg sing =n mg. lo~ beads lie alon; the plane. cscs Fig, 2.24 37. Three particles A. B and C are situated atthe vertcles ofan equilateral triangle of side . Each of the particle strts moving with a constant velocity sich that is always directed towards B,B towards Cand Ctowards 4A. Find the time when they mect. 1 og wD On () We look into itasa problem of relative velocity find v,, in the direction of B. (@) none Fig, 2.25 38, Two partivals are thrown horizontally in opposite directions from the same point from a height ft with velocities 4 ms" and 3 ms". Find the separation between them when their velocities are perpendicular, (0.15 (b)025 5 (60.355 [d) none of these gt jand V,=3j—gtj are the For velocities 10 be oF velocities at any instant perpendicular toi 7 <0orr= J = 10.35)= 2.45 m, =0.35s separation =(u, + 6] wihanae 39. A ball is thrown with avelo eleration 6] +2 7. The velocity ofthe ball after 5 seeondsis fa) 307 +107 (0)307 +167 (©) 10} +28} (d) none of these BEIM (6) using v ¥,=6(5)=30 ms! r= 6+25)=16ms! v=307 +167 40. Ray 40 in medium I emerges as OB in medium UI thea refractive index ofmedium Hl with respect to medium is uta (a) 40.0P OB.OP ©) Fig. 2.26 ) 1. Vector Laws (a) vary if scale iy changed (b) vary ifrotation of axes is performed (c) vary if translation of coordinates is performed (d) are invariant under translation and rotation of the coordinates, @ 42. Block 4 is placed on 8, whose massis greater than that of A. Frieion is present between the blocks while surface below B is smooth. Force Fas shown increasing linearly with time, is applied at 0, The acceleration a and a, of A and B, respectively, are plotted against time / Choose the eorrect representation. 43. In Fig. 2.28 (a) mass of both the blocks are equal. v, and ¥, ate instantaneous speed of 4 and B. Then (a) B will never loose contact with the ground BRUM (a), a) P+ Pt= WP + Pa differentiating (i) 0 Vectors 29 44, The product of two vectors 4 and § may be (248 (oysaB (c)< dB (d) zero SSE (0). (0), (a) «> 4B = ABeose and jax = ABSing 48. X= 7B and ¥~ S.B then always (b) Amay not be equal to C (c) dand@ are parallel (4) Gand € are antiparallel (my 40. i + 8 = © VeuorsA and Bitrotated by @ in the same sense to form i’ and * then (a+ ek (b) a” +B +8 = {e| () A.B = 4B i BEET ©).(01.(0 47, J and B are wo vectors such that 4 + B = @ ina iven coordinate sustem, The axes are rotated by @. ‘Then in new coordinate system @4+b=e WA+d re (©) AB wold system)= 4 x B (new system) (e) Zand 8 (interchange in new system) Em) 48. A point moves according to the law x = at, y= atl — ert) where @ and ex are positive constants and cis time, Find the moment at which angle between = velocity vector and acceleration vector is a 30 Pearson Guide to Objective Physics 1 | -[a~2aae] 21 -2atP=1+(1-2at? or 1 -2arp*= Tore Read the following passage and answer the questions given atthe end. The advantage of the method of breaking up vectors into its components rather than adding direetly with the use of suitable trigonometric relations, is that we always deal with Tight-angled triangles and simplify the calculations, In adding vectors by the analytical method, the choice of coordinate axes determines how simple the process will be. Sometimes the components of the vectors with respect to a particular set of axes are known to begin with, so that choice of axes is obvious. Atother times the choice of axes can greatly simplify the job of resolution of the vectors into ‘components. For example, the axes can be oriented so that at least one of the vectors lies parallel o an axis. 1. Using the approach mentioned in the lines ‘The advantage ...... Simplify the calculations.” Solve the problem shownin Fig. 229 R = Oi cosd5i + Od sinds j + ABI BCcos607 + BCsin60 7 6146) 447-2742 j +6+248)7 + s+ (6423) Fig. 2.28 2. Using the procedure listed in the paragraph, find the and # in Fig.2.30(a) resultant of vectors. Fig. 2.30 (a) Fig. 2.30 (b) See Fig2.30(b)R =87 +6cos30i +6sin30 7 R =(8+3N5)i +37; (Rl 3 Bes 1 tanB = S73 gh = Taro With respect to 8ms vector. 3, How 4-B can be accomplished? Apply 4+ B ie. Adding —# to 4 || - (re saab coxt80-0) Read the following passageand answer the questions given attheend, ‘Three kinds of multiplication operation for vectors can be defined (i) multiplication of a vector by a scalar, (ii) ‘multiplication of two vectors in such a way as to yield a scalar, and (ii multiplication of two vectors in such a way as to yield another vector. The multipli tion of a vector by a sealarsimply means E—4A where kis a scalar. Such product only changes the magnitude of the vector. The directions will be the same ifk ispositive and direction will be opposite if kis negative, Dividing the vector by a scalar would only mean X = ie, multiplication by reciprocal of k. When we multiply a vector quantity by another vector ‘quantity, we must distinguish between the scalar (or dot) product and the vector (or cross) product, The scalar product of two vectors J and writenas 4.8 =ABcos@ where is mle between the vectors Z and B The vector product of two vector implies ¥ = Ax B= ABSing j, where j, is a unit vector perpendicular to both A and B The direction of 7 or is given by the direction of right-handed secew or right band thurnb rule, 1. The flow of electromagnetic energy can be accomplished by (2) dot product (6) product of vector with a sealar (€) by division of one vector with another Ire BZ, [Exé] 2, Choose correct statement on the basis of the paragraph. (@) Dotproduct means multiplying the magnitude of vector 4 by the projection of B on A. (b) Dot product means multiplying the projection of ‘two vectors along x-axis (©) Dot product means multiplying a vector by the perpendicular projection of the other vector on the first vector. (@) Dot product makes the resulta scalar. BQN (2) anda) 3. Choose the comectstatementon te bass ofthe paragraph (a) Cross product is product of projection of one vector on the other and the other vector. (b) cross product (b) Cross product is the product of one vector with perpendicular projection of other on the first vector. (©) &= AxBmeans ¥, Gand Bore muwally perpendicular. Vectors 31 AxB means X 1 i and X 1 omy. @ (b) and (@) Read the follewing passage and answer the questions given at the end. twas thought until about 1956 that all laws of physics were invariant under another kind of transformation of coordinates, the substitution of righthanded coordinates system with a left handed system. In that year. however, ‘some experiments involving the decay of certain elementary particles were studied in which the result of the experiment did turn out to depend on the handedness of coordinates system used to express the results. C. N. Yang and T. D. Lee were awarded Nobel prize in 1957 for their theoretical prediction that it would be the case. In other words, the experiment and its image in a mirror would yield different results. This surprising result led to re-examination of the ‘whole question of the symmetry of physical laws. 1, Are the laws of physics invariant under another kind of transformation of coordinates? Experimentally, decay of elementary particles proved dependence on the coordinates system. ‘Theoretically Yang and Lee predicted the results should depend upon the coordinate system. 2. Do the vector laws vary in mirror image coordinate system? This is a matter of debate yet, It has not been confirmed as such. QUESTION FOR PRACTICE 1. A large number of particles are moving with same magnitude of velocity v but havingrandom directions. Theaverage relative velocity between any two pasticles averaged over all the pairs is x x oy wy 3 4 op @ 2, Aboy swims in astraight line to reach the other side of a river. His velocity is 5 ms“ and the angle of swim ‘with shore is 30°, Flow of river opposeshis movement at2ms" If widthofriver is 200m, where does hereach the other bank? Fig. 2.31 (a) 106m fromO™ downstream (b) 186m from 0" downstream (©) 186m iromO" upstream (@ 106m irom 0" upstream. 3. Two cars are moving with same velocity of 30 km maintaining a distance of $ km between them Speed 32 Pearson Gui of third car moving in the epposite direction and ‘meeting the two cars at an interval of 240 sis Fig. 2.32 (a) 45 km (b)30 kmh! {c) 55 kmh (d)35 kmh" 4. Given F = (41-10) and # = (51-37), compute torque. i (a) ~62 junit (b)62é unit (0) 487 unit (@)-48% unit, 3. Twoswimmers siart from point P on one bank of the riverto reach Q on the opposite bank, Velocity ofeach swwimmerin still watersis 2.5 kmir". One ofthe swimmers crosses the river along the straight route PQ and the other swims right angles to the stream and then walks, the distance which he has been carried away by the river to get to point Q. Stream velocity is 2 kmh'. If both the swimmers reach point Q simultaneously, the velocity of walking of second swimmeris 2 Fig. 2.33 (a) 3kmi! (b)4 km! (2kmn! (@)3.5 ko 6. A boat sails 2 kmeast, then 4 km northeast and then in an unknown direction, Final position of the boat is 5 km east from starting point. Unknown displacement is (@)-2.8km, 3°26" with north towards east (b) 3m, 2°26" with east (©) 3.5 km, 2°30" with south towards west (@)L8km, 2°36° with north towards east. 7. A parallelogram has diagonals expressed as A=5i-4j+3k and B parallelogram is 3429-2. Area of Fig. 2.34 (a) Ji17 units (b) Vi71 units: (©) J7it units © Vi07 units 8. Two particles are projected simultaneously in a vertical plane from the same point. These particles have different velocities at different angles with the horizontal, The path seen by each other is (a) parabola (b) hyperbola (c)elliptical (©) straight line. 9, Rain appears to fall vertically or a man walking at 3 kmh! but if e doubles his speed to the rain appears to fall at 45°. The real velocity of rain is (a) 33 kmh ', 45° (b) 23 kmh", 45° (©) ay kmir', 30° (6) 243 kmh", 60°, 10, A helicopter is to reach a point 200,000 m east of his existing ploce. Its velocity relative to wind blowing at 30kmb" from northwest taking scheduled arrival time duration as40 minute is Fig. 235 297+ 217 (21742795 (129) +127 (912547297 11, Two bodies move uniformly towards each other. They ‘become 4 m nearer in every 1 second, and get 4m closer every 10 second. If they move in the same direction with their previous speeds, the speeds of the bodies are (@) 1.8 ms, L8ms" (2.2 ms", 1.8ms" (b) 2.2 ms, 20 ms (©) 1.5 ms',25 mst a] [B [a] —, Lal Fig. 2.36 12, A man holdsan umbrella a 30° vertically to keep himself dry. He, then, uns at a speed of 10 ms and finds the raindrops to be hitting vertically. Speed of the raindrops w.rt. earthandw.rt. man are 30 Fig. 237 (@)20 ms",10ms'——_(b) 10ms'', 205 ms (@ 20s", 10/3 ms" 13. A nut is screwed onto a bolt with 12 turns per em and diameter 1.18 cm. The bolt is lying in horizontal direction. The nut spins at 216 r.p.m. Time taken by the nut to cover 1.5 em along the bolt is (©) 10 J ms", 20 ms! @2s (3s (as @5s Bolt fo Tar Fig. 238 14, A particle moves from one end ofa strip to the other end in time ¢ with uniform velocity. The particle then flies off vertically making the stripmove onthe smooth horizontal table. Further distance travelled by the strip in time T is equal to the length of the strip L. Then + J / Fig. 2.38 @T>r (oT rst (d) data is insufficient 15, An open umbrella is held upright and rotated about the handle ata uniform rate of 21 revolution in 44 s.1f the rim of the umbrella is a circle of 100 em diameter and theheight of therim above groundis 150m, then the drops of water spinning off the rim will hit the ‘ground at Vectors 33 150m , Fig. 2.40 (a) 200 em (b) Sem (o) 170m (4) 97 em. 16, ‘Two motorcycles M, and Mare heading towards each ther with a speed of 30 kmh each, A bird flies off M, at 60 kmh when distance between the motoreyeles is 60 km. It heads towards M, and then back to Mand 80 on, The total distance the bird moves till the motoreyeles meet is ea wii Fig.2.41 (a) 60 km (b) 40 km (@)50km (a) 30 km (e) none 17. Two towns T, and T, are connected by regular bus service. A scooterist moving from T, to T, with speed ‘of 20 kmh! notices that a bus goes past it every 21 minute in the direction of his motion and every 7 minute in the opposite direction. If a bus leaves in citherdirection every ¢ minute, the period tis Bs +2, me 1_ FH _ 20k" Fig. 2.42 (a) 1.5 minute (b) 9.5 minute, (c) 6 minute (d) 10.5 minute. 18. If B+G4R = 0 and out of these, two vectors are cequalin magnitude and the third veetor has magnitude 7 times that of any of these two vectors, then angles among the three vectors are (a) 45%, 75°, 75° (b) 45°, 90°, 135° (©) 90°, 135°, 180° (4) 90°, 135°, 135°, 19. A hunter is at (4, ~1, 5) units. He observes two preys a1, (1,2, 0) units and P. (I, 1, 4), respectively. At 34 Pearson Guide to Objective Physics zero instant he starts moving in the plane of their | (a)x,—x.x )-8 =x postions with uniform speed of S unis sina direction ox.-28 @se-x cept spear ote 2 a hesees Pand P, 59 Two particles are originally placed at P and Q distant! apart. At zero instant, they start moving such that velocity 7 of P is aimed towards Q and velocity 7 of iis perpendicular to 7 . The two projectiles meet at time T= Fig. 2.43, (0) 0.535 (6) 0.73 6 (035s (4) 092. ‘An aeroplane flies from P and O with speed v and then from Q and P with the same speed. If wind blows ‘normal to straight line PQ with the speed V, the totat time for to and fro motion is 23. A particle is moving in a circle of radius R in sucha ‘way that at any instant the a, and a, ate equal. If the speed at ¢~ 0 is vy the time taken to complete the first revolution is, (e)y,R R R oO; @ i 0-e"). 2L Z Oy 24, A boat which has a speed of 5 kml in sill waters crosses a river of width 1 km along the shortest possible 21, If @ and B are the intersecting face diagonals of a path in 15 minutes. The speed of the river in kmh is cube of side x in plane XOY and YOZ, respectively, @ 3 with respect to reference frame at the point of intersection of the vectors and sides of cute as the (4 Ova axes, the components of vector F = 4x6 are [Based on LT] 25. A particle is moving in a plane with velocity given by ¥ = iuy+ 7 aw cose ifthe particle is at origin at t= 0, Distance from origin attime 3 /2eis (a) for «Grn, 20) (&) for + Onn, oF y >, (2% ) os (ve @ +e) [Based on Roorkee} Fig, 2.45 26, Mass 4 is released from the top ofa frictionless inclined plane 18 m longand reaches the bottom 3 slater. At the instant when 4 is released, a second mass B is projected upwards along the plate from the bottom with a certain initial velocity. Fig. 2.46 Mass B travels a distance up the plane, stops and returns to the bottom so that it arives simultaneously with A. The two masses do net collide, Initial velocity (Sms (d) Tes [Based on LLT| 27. A particle is moving eastward with a velocity of 5 ms", If in 10s the velocity changes by 5 ms" northwards, what is the average acceleration in this, time? i () FymsNW (© jms EN (© Jame'NW (@ 243 me NW, [Based on LET.) 28. The driver of a truck travelling with a velocity v suddenly noticesa brick wall in front of him ata distance d. To avoid crashing into the wall (@) he should apply brakes (b)_ he should take a circular tum without applying brakes (©) both (a) and (b) alternately (@) data is insufficient. [Based on Roorkee] 29, To get a resultant displacement of 10 em, two displacement vectors, one of magnitude 6 em and another of § m, should be combined (a) at anangle 60° (&) perpendicular to each (©)parallel (anti-parallel 30. When mass is rotating ina plane about a fixed point, its angular momentum isdirected along (0) the axis of rotation (b) lino at an angle of 45° tothe axis of rotation Vectors 35 (0) the radius (d)_ the tangent to the orbit 31. Which of the following is a vector? (a) force (b) mass (energy (@) power 32. Whea two vectors 4 and B of magnitude a and bare added, the mognitude of the resultant vector is always (a) greater than (a+b) (b) not greater than (a+ 5) (© equalto (a+b) (2) less than (a +6) 33. Identity the vector quantity (a) heat (b) angular momentum (time (work 34, Which ofthe following quantities isa scalar? (@) magnetic moment (b) acceleration due to gravity (©) electric field (4) electrostatic potential 38, Which of the following quantities isa vector? (a) volume (b) temperature (© displacement (d) density 36. ‘The rectangular components of force 5 dyne are (a)3.and 4 dyne (b)2.5 and 25 dyne (©) 1 and 2 dyne (@) 2and3 dyne 37. Identify the scalar quantity. (a) work 4b) impulse (force (d)acceleration 38. Moment of inertia is ()sealar (8) tensor (@) vector (©) phasor 39. Ifthe magnitude of vectors A, B and @ are 12,5 ‘and 13 units, respectively, and 4 + B = C , the angle between vectors A and B is ns ox wo 40. Angular displacement is (b) 2 36 Pearson Guide to Objective Physi (a)ascalar (b)a veetor (©) neither (a)nor(b) (either (a)or (6) 41. A mosquito flies from the hole in a mosquito net top comer diametricelly opposite. Ifthe net is 3 < 2 2 m then the displacement of the mosquito is (a) iz m () Jim © Jim (d) none of these 42, Aman travels | mile due east, $ mile due south, 2 mile due cast and finally 9 miles due north, How far is the starting poin‘? (a) 3miles (b)S miles (©)4mites (d)between5 and 9 miles 43. Two forces of magnitude 7N and 5 N act on ap article at an angle @ to each other, @ can have any value. The minimum magnitude ofthe resultant force is @L2N (BN (@2N sn I started walking down a road to day-break facing the sun, After walking for some time, I tumed to my left then I tured to the right once again. In which direction was I going? (@) northeast (b) south (cheast (@) northwest 45. If 7 = B+ and the magnitudes of 7. B and & are 5,4 and 3 units, respectively then the angle between Gand @ is (a2 (b)sin-1 Gia (©) cos (3/5) (@cos'(4/5) 46. A boat which has a speed of 5 kmbr' in still waters crosses 2 river of width 1 km along the shortest possible path in 15 minutes, The velocity ofthe water in kmhr ‘is @4 ) Var ol @3 47, Iftwo waves of same frequency and same amplitude on superimposition produce @ resultant wave of the same amplitude, the wave differs in phase by (x5 203 © n4 (@z000 48, If jis a unit vestor in the direction ofthe vector 3, then @ nla Oarna n> ANal @n> AlAl 49, Two forces of 4 dyne and 3 dyne act upona body. The resultant force on the body can only be (a) between 3 and 4 dynes (b) between 1 and 7 dynes (©) more than 3 dynes (4) more than 4 dynes 50. Ariveris flowing from west to east at a speed of 3 m/ minute. A manon the south bank of the river, capable of swimming at 10 m instill waters wantsto swim the river in the shortest ime. He should swim in a direction (@) 30° west of north (b) 60° east of north (©) 30° east of north (@_dve north 51. The resultant of two equal forcesis double of either of the force. The angle between them. (aor () 60° (90° (a) 120° 52. Anacroplane ismoving on acircular path witha speed 250 kmbr!, What is the change in velocity in half revolution? (a)0 (©) 125 kmbr! (©) 250 kmhr* (4) 500 kmh 53. A body constrained to move in y direction is subject toforce given ty =(-2] +157 +6%)N. Whotis the work done by this force, in moving the body ‘through a distance of 10m along axis? (a)20) (0) 1505 (©1605 (a 1905 54, I walked 4 miles tumedto my left and walked 6 miles then tured to my right again and walked 4 mile, Which of the choice mentions the distance from the straight point to the place where / stopped? (2) 10 mile () 14 mile (1S mile (20mile SS. Aforce F = 6] -8] + 10j newton produces an acceleration of | ms” in a body, the body would be (a) 10 V2 kg (©) 6 V2 ke (0) 20 ke, (@) 200 kg ‘56. Maximum and minimum magnitudes of the resultant oftwo vectors of magnitudes P and Q are in the ratio 3: 1. Which of te following relations is rue? (a) PQ=1 b)P=20 ()P=0 (4) none of these 57, Whatis the projection of 7 on G2 @O.7 oOP.d 7-6 @PO 58, Rain is falling vertically 4 ms", Amanis moving due east with 3 ms“, The direction in which he shall hold ‘the umbrella with the vertical is (a) 53° east of vertical (b) 37° east of vertical (c) 53° west of vertical (d) 37° west of vertical 59, There are N co-planar vectors each of magnitude ¥ Each vector is inclined to the proceeding vector at angle 2 rN. What is the magnitude of their resultant? (a)22r0 () YN wor (@av 660, Which of the following operations between the two vectors can yield a vector perpendicular to either of them? (a) subtraction (b) division (©) addition (¢) multiplication 61. Three vectors 4, B and Csatisfy the relation and A. C= 0, The vector 4 is parallel to (@) BC ©) B @) Bx @é 62, Angle between the vectors (i+ j) and (/+#) is (b) 90° (d) 0° (a) 0° (c) 180° 63. Resultant oftwo vectors B and @ is inclined at 45° to either of them. What is the magnitude of the resultant? Pe © Fe ()P+Q )P-Q 64. A steamer is heading due North with 20 ms". The wind is blowing 10 ms”, The wind is blowing 10 ms’ due west. In which direction the flag on the mast flutiers? tant 4 west of non 2 (b) tan! 5 east of noth Vectors 37 (©) tar! north of east (@) tan 5 north of west 65, What is the angle between j + j + { and 7? (a) 3 (b) x4 (©) 76 (4) none of these 66. What is the maximum number of components into which a vector can be split? (2 3 4 (4) more than 4 67. What is the maximum number of a rectangular components into which a vector can be split in its own, plane? (aytwo () three (6) four (4) more than 4 68. A force of 6 kg and 8 kg can be applied together to produce the effect of a single force of (@)20kg (b) 1Ske iikg @ikg 69, To a person going cast in a car with a velocity of 25 kmhr', a train appears to move towards north with a velocity of 25 3 km/hr. The actual velocity (a)5 kmh! (b) 25 kmhr (©)50 kmhr? (@) 53 kmh 70, The area of a A formed with sides 5i + 3 - k and Bit 2)—his @ Ve (ON 3 5 of of ‘71, At what angle should be the two forces 29 and Jp ‘act so that the resultant force is p 10 (yas? (60 90" (@ 120° 72, Two cars are moving. A along east with 10 ms". At any instant it is 1500 m away from the crossing. B at the same instant is 1800 m away from the crossing and is moving towards the crossing with 15 ms, Whendo they come closest? (a) 1093s (11938 (b) 129.2 (9935 73. What is the angle between f and the resultant of F-B)2 38 Pearson Guide to Objective Physics ten (F-) . oa (b) tar" (/P) (o)tan-1 (PQ) (@) zer0 74, The length of seconds hand in a watch is 1 em. The change in velocity of its tip in 15 seconds is (a) zer0 © (Salon (Fens: © (G2) ems 75. Rain falling vertically downwards with a velocity of 3 kimi, A person moves on a straight road with a velocity of kmh". Then the apparent velocity of the rain with respect to the person is, (@) 1 ki? (b) 5 kmh" (4 kar! (3 kmh" 76, A large number of particles are moving towards eech, other with velocity v having directions of m randomly distributed. What is the average relative velocity between any two perticles averaged overall the pairs? (ayavin (b) 4mv rv wma 77. The magnitudes of the X and Y components B are 7 and 6. The magnitudes of the 4° and ¥ components of B + G are 11 and 9, respectively. What is she magnitude of Q? (a9 ms (©)6 @s 78, A swimmer can swim in still waters with speed v end the river flowing with velocity v2. To cross the river in shortest time, he should swim makingangle @ with the ‘upstream, What is the rato of the time taken to swim across in the shortest time to that in swimming across ‘over shortest distance? (@)sin 9 () cos 0 (o)tan 9 (coro 79. A vector of magnitude a is rotated through angle 8. ‘What is the magnitude of the change in the vector? (a) 2asin 6 (b) 2arcos (c)2asin (62) (@ 2acos(42) J Another vector that 80. Consider avector F = 4 isperpendicular to Fis (airy (b) 7k (03-4) (a 6 81. A helicopter is flying south with a speed of 5D km h-! A train is moving with the same spoed towards cast. ‘The relative velocity of the helicopter as seen by the passengers inthe train willbe S0./? km h- towards (2) northwest (b) southwest (©) northeast (4) southeast 82. A man is walking due east at the rate of 4 kimhr and the rain is falling at an angle of 30° east of vertical with a velocity of 6 kmbr'. The velocity of the rain relative to the man will be (@)Skmhr! (b)7.U18kmhr? ((8.718km he! (@ l0kmhr! 88. A truck travelling due north at 20 ms turns west and travels atthe same speed. Then the change in velocity is (2) 40 ms! northwest (8) 20 YB ms" northwest (©) 20 JB ms" southwest (@ 40 ms* southwest 84. Given that P isa point on a wheel rolling on a horizontal ground. The radius of the wheel is Initially i he point Pis in contact with the ground the heel rolls through half the revolution, What is the displacement of point P? @R GPa (yk fe ea (c) aR (a) 20k 85. A vector F, isvalong x axis. If F.F, is zero PF, could be @ygk &) GD 40+ @-4 86. A parallelogram is formed with q and j asthe sides. Let g, and d, be the diagonals of the parallelogram then a? + b= @(Greya (b) (4-3) @di+d (d) dj -d} 87. If|4|=|B |, then what is the angle between 4 + B and 4 -B (a) 90° (b) 60° () 30°, (ad) 0° Read the following passage and answer the questions given atthe end. A graphic artist is creating a new logo for her company's Website. In the graphics program she is using, each pixel in an image file has coordinates (x,y) where the origin (0, O)is atthe upper left comer of the image, the positive.xaxis points to the right and positive y axis points down. Distances are ‘measured in pixels. The atist draws a line from pixel location (10,20) to the location B (210, 200). She wishes to draw a socond line that starts at 4 (10. 20) and is 250 pixel long. The line is at 30° angle measured clockwise from the first line, Call this point C. The artist now joins BC. 1. Find the coordinates of C. (a) (97, 248) (7,258) 2. Find the length of BC (a) 137 pixel, 35°above rightlett (b) 137 pixel, 25°below stright leit (©) 157 pixel, 45° below straight left (@)__ 137 pixel 35° below skright left Lo T= {(x-10) 7 +(y-20) ~() 250 = x10 +20 (b) (87, 238) (007,268) SSI <3 _ (x=100)200+(y-20)180 aa 250%269 AD) Solving (1) and 2) we get C (87,258) 2.(b) BO = (210-87) + (200-258) = 137 _ (210-877-i) 13, 0s @ = OI A 010-25 Read the following passage and answer the questions given attheend, ALEnormous State University (ESU), the football wam records its plays using vector displacements with the origin taken to be the position of the ball before the play starts. Ina certain pass play, the receiver starts at (10, —5) where the unit are yards and 7, jare unit vectors along the right and the downfield, 6; +4 7 (zigs)and 127 +18 j (zags). Meanwhile quarterback has dropped straight back at 7 7 Vectors 39 41. How far must the quarter back throw the ball? (@) 35 yards (6) 33.9 yards (6) 37.2 yards (4) 31.1 yards 2. The direction in which he throws the ball is, (a) 31°36’ right ofdown field (b) 31°36 leftofdownfield (6) 28.3*rightof downfield (@ none 1@ Ro = xityj-=10i-5j+( 6449) + (125-189) 77 = 164-267 \el = Vig eae =2 6 2.(@) tan B= 56 or B= 31°36" right of down field, Read the following passage and answer the questions given. attheend. Allthe stars of the Big Dipper (part of he constellation Ursa Major) may appear to be the same distance from the Earth. But they are in fact very far away from each other. The Fig shows the distance fiom the earth to each of these stars. The distances are given by (light years), the distance that light travels in years. Mizar 1B Dubhe 105, Alhaid 18h Alioth aly nes Moral 01 Thy Fig. 2.47 (One light year = 9.451” 10m, Alkaid and Merak are 25.6" apart in the earth's sky. 1. Find the distance in light years from Alksid to merak. (a T6ly (b) 56 bv (€)663 hy (09h 2, Toaninhabitant ofa planet orbiting Merak, how many degrees apart in the sky would Alkaid and sun be? (129° (b) 108° (©) 100° @2r 1.(a) 77 1,38 =75.62 0 76 ly 2.(a) Angle between SA and SM=264, given angle between A, and 4, (with nearly same dimensions) will be around 26° \ ‘Thus, total 7 . total Fig. 2.56 44 Pearson Guide to Opjecuve rhysics 2U(oHere G =X7 +x j and =x j +x G Since R = axf weset R ax jw jaws Hom or (1-@,1)=e* orl Clearly, the components are R, =x?,R, =—¥7,R, - R “x or T=\) (he) 24(b) Speed of bout, v,= Skmh" = 25(a)Comparing the given equation with = Fig 287 (a)Comparing the given eq 22(d)Relative velocity of P and Q is (v —u cos 8). The we get v, =H, and 4 particles will meet when Bn = a $B < so cos imegratng x= Jud andy = Jaw [v-weos0) a= dan [veosddr= cosendtorx=u,i-+e,andy=asinat~c, At aT uf orvP -u = =dor(v?-ir) T= vd =a, Then distance from origin.d= Ja + 57 = Ja + Gu, 207 1 2G{o)Hlere for 4,18 =0%3+ 5 a%F ora=4 ms? for B, time taken to move up is given by, 1 ~w/a (-. the relation v = u + at here bezomes 0 = w —at,). Distance moved up is given by the relation 0 = «' — 2a Sie.,S Rie = wires Fae andy,= 72 ory= fe = ae Vg a2a But“ hen th + 3 4 Fig. 2.50 g anon ic, Ror= Rae, Ra=R 2 or 2 = 27(6)Changein velocity, CH )=(5 45) ONW=5 JF ms NW. 52 1 NW= 35 ms?NW 2, 28(2)For taking a circular tur to avoid. accident, the i-@, 0% acceleration acquired willbe “> Thusadditional ie, o> 1a, js dt a effort is required to meet this acceleration, Tab) == (1500-1047 + (1800~ 1517, for sto be a 1 mee pa PFOreTORS MIDI 44) aya 2vsin O2=2% x Fe de its frst derivative should be zero. Thus [> =0 4(oyDisplacement of = mR] +2Rj +=-20(1500 107) -30(1800 - 15) 29.23% Displacement of P= (aA) +(2R) = rYgioa Motion in One and Two Dimensions is said to be in motion ifit keeps changing its position with respect to its surroundings with the passage of time otherwise it is said to be at res. Frameof Reference A set of coordinates x,y,z and ris said to bea frame of reference. Its of two types, inertialand noninertial, Inertial frame of reference is one which is either fixed or moves with a uniform velocity in the same straight line, Non-inertial or accelerated frame of reference moves with an acceleration ‘a’, Newton's laws are valid only in inertial frame. Pseudo or inertial vectors are to be applied to make the frame of reference inertial from non-inertial so that Newton’s laws may be applied. One-Dimensional Motion _If the particle changes its position only in one of the x, , orz directions with respect to time, then the motion is said to be one-dimensional, Since the particle moves alonga straight line, the motion may also be termed as linear or rectilinear. Speed The time rate of change of distance is called de speed, that is, v= > unit ms! Velocity The time rate of change of displacement is, a& G-Unit ms"|.oms" and fis in SI,CGS and FPS system, respectively, v=LT". Displacement The shortest distance between initial and final position of the particle is called displacement. Acceleration The time rate of change of velocity is called velocity, that is, + a called acceleration, @ = 5, units is ms*, cm s* and fi sin SI, CGS and FPS system, respectively, a= LT, Speed, velocity or acceleration may be of four types. We define here velocity and others can be anticipated in similar terms. (@) Instantaneous Velocity The velocity, at a particular instant of time is called instantaneous velocity, for example, velocity, at 4.82 5 may be al expressed a5 7 = Gp. ds (b) UniformVelocity If = constant throughout the motion and direction of motion does not vary throughout then sucha velocity is called uniform velocity. Fig. 3.1 (a) shows displacement time graph and Fig. 3.1 (b) shows velocity time graph for a uniform velocity (mn) wa tims) Displecoment seo $-—____> se time time (9) @) (b) Uniform velocity de af Sis not constant but (© Variable Velocky 16 7 a& varies at different intervals of time or = is a constant but direction varies or both vary, then such a velocity is said to he variable velocity. Fig, 3.2 (a) illustrates x ~ ¢ graph for @ body moving with variable velocity and Fig. 3.2 (b) shows velocity Vs time variation fora body moving with variable velocity. (oy Displacement velocity Tne ee Time re) @ (b) MELE Veravie velocity (@ Average Velocity It is that uniform velocity ‘with which ifthe body would have moved it would have covered the same displacement as it does otherwise by moving with variable velocity. Thus total displacement covered total time taken Average Velocity in Different Cases Particles covering different displacement in different times: Assume a particle covers s, displacement in , ands, in time f, and so on then average velocity is Stutst rere (harmonic mean) (ii) Bodies moving with different velocity in different intervals of time. A body moves with velocity v, in time t, ¥, im time f, and so on then v, is given by Hy tat SS (Arithmatic mean) Then y,, = Equations of Motion fa)y=utat (smut 5 at ()¥ = =2as @s,.=04 5 Qn » ‘The conditions under which these equations can be applied 1. Motion should be L-dimensional 2. Acceleration should be uniform, 3. Frameof reference should be inertial. ‘While drawing graphs compare your equation with the following and then draw (matching the equation) graphs 1. y= ma +e straight line with positive intercept on svaxis, 1 = mx straight line passing through origin. ‘max e straight line with negative intercept (on =v axis). ‘Straight li 2. 8 +1'=aXcircle with centre at origin. (x AY + AP =P a circle with centre at (hk). represents ellipse. shows hyperbola, or.xy= krepresents a rectangular hyperbola, See Fig. 3.4 (a) 6 y=y,e"and y=y, (1 —e*) represents expotential ‘See Fig. 3.4 (b) and (ec) 43. Pearson Guide to Objective Physics 1 ve! \ \ “s, J... 4 T w sin 20 @) (by HorizontalRange R= - Note that the range will Hyperboia Exponential be same if projected at complement angles, i.e, @ and ames (90 ~ ) with same velocity. Maximum Range 2, when @= 45° © Exponent ge Trajectory vtan 6 el Bw cos 8 the problem into nso one-dimensional motions. Separate ¥ and y, similarly a, and a. Treat the motion in x- and y= Instantaneous velocity = || directions Projectile A freely falling body having constant horizontal = fea + ie ty, =a velocity may be termed as a projectile. In general, in one direction the motion be accelerated and in another direction the motion. uniforn, then such a motion isealted projectile motion, Fig. 3.5 shows acceleration in x-direction and Luniform velocity in x-direction. Such hodies follow parabolic tan B path. , Range and time of fight along an inclined plane Consider an inclined plane of inclination a. Let a projectile be fixed at an angle @ with the horizontal or at an ora angle (@ ~ @2) with respect to incline plane as shown in Fig. a7 2 The time of flight 7” LEEERE pete cin have Oblique Projectile Motion Assume aprojectile is fixed Eee atan angle @ with horizontal, with a velocity u from point 0 as shown in Fig. 3.6, Resolve velocity along x and y-axis Along)-axis g acts then maximum height attained, geosce 2u?sin(@ ~) cos {sin (28 a) ~ sin we z e goose oa ioe ac loi an talon © along the inclined is maximumif 20-0 24, or 0-7/4 Thats. ismasionum when the retin of projection bisects the a Je hat the inefined plane makes with Oy’ and R, [1 -sin a woos a Note: In projectile motion along the plane acceleration acts along x and axis both, “SHORT CUTS AND POINTSTONOTE _ | 1, Slope of'y 1 graph is velocity, slope of v is acceleration, t graph total displacement 2. Average velocity v= Sta ime taken e()maQ) tat ho nth Habody moveshalf distance with y, and other half 2uy. with», then y= Ifa body moves with different velocities in equal intervals of time then Wt ty tent TS (avithmatie mean) Area under vr graph is displacement, arca under a= 1geaph is velocity. 4, When a body leaves a moving body it acquires its velocity but not acecleration, a 5. Insamtancous velocity w(.)~ Fi» 6 Apply y=utat, s=urt Yar, wow =2a5,s,,=0t Y nV when (motion is one dimensional or made so if wo or three dimensional fby resolving]. i) acceleration is uniform. (Gil) frame of reference isinertial. Motion in One and Two Dimensions 49 Iaceslertion is variable then ston with , un and v= Freya 8 UF acveleration Ws variable and funetion ot displcoment or velocity, For example at sent ey then a gy oe JE faa 9. Note carefully the graphs for v= 1 ar as shown in Fig. 38 (ajand(b). Graphs forv=u+at in Fig, 3.9 (a), (b) and (©). FP nissre 'g Mais ve (a) ) stim: () ' LEER capticat ropresontation of s ut 4 at 11, Ifa particle starts fiom rest with an acceleration after acquiring a maximum velocity the particle decelerates with Band finally comes to rest in time t, then ‘50 Pearson Guide to Objective Physics ope’ (a+ B)” Bt Yaa” Gap Mdistence covered 12, Time cannot be negative in physics. 13, If in a projectile motion, direct formulae is/are inapplicable, convert the problem into two one dimensional motions. 14, Average acceleration v,, (vai tv 5/)—(rs here 49,3) and direction tan"! ( 15. As far as possible apply vector laws to solve two. dimensional problems if physical quantities involved are vectors. 16, Problems on relative velocity can even be solved using Vector laws, Use v, OF Vyp= (War Yes (Ma My )J 5 tan with respect tor direction o 3 Finaing direction in two dimension ‘motion: 17. Whenever solving problems for inclined plane, consider axis along the plane as x-axis and perpendicular to ivas axis. See Fig. 3.11 wcox(@-a1) Note: o' - _ osing | along the plane. usin(@—a) perpendicularto the plane ~gcoser i.e. use accelerated motion along both x and y axis, 2usin(O ~ cz) “goose is true everywhere, w° sin? (0—ar) Ls “gene Ze] is also truein all eases. 18, To find radius of curvature of a projectile at any point R= —. ‘The velocity v and radial or normal aeveleration at that point is used in the above relation. If-yand a, cannot be determined then use 19. Equation of trajectory means the relation between and y. Try to establish relation between x and y by eliminating « 20. To cross the riveralong the shortest path the swimmer shall strike at an obtuse angle to the flow ofriver so that resultant velocity v is along the normal as iMlustrated in Fig, 3.12. provided ¥o™ Vier From triangle law v= Jv? —y? where: of swimmer and v, = velocity or river. velocity ! Ifthe width ofthe river is then ¢= — = 21, To crossthe river in the shortest time (when v, Vg): Then the swimmer shall strike atright angle to the flow of theriver and 1. 22. A particle is projected from the top of an incline as shown in Fig. 3.13 ¢,=gsin @ and a,~~geosa. ~acoe 4 23. Net acceleration in cireularmotion Fig. 3.14) 4, celeration fe? +a} where a, is tangential and, is radial acceleration tan p= 24, If resultant of two motions is to be determined use (v,,+v.) +(y,+¥,) and p= 25. If. particle is projected from the top of a tower or from a height/ then consider the point of projection as origin. So that displacement is -h. Using — h= wsind ¢<— 2, Find rand range =x= (ucos@) 1, 26. If person can throw a ball to maximum height fr (vertically up) then the maximum horizontal distance up to which he ean throw the ball is 2. 27. Once aparticle is thrown in a gravitational field it will return only after time of flight. 28. If particle is thrown up, itwill have same speed at the same height during ascent and during descent. 29. 16 pons ~ Your then one has to reach the directly ‘opposite point on crossing the river. The drifted parton foot or by other means the minimise drift or ‘minimum toal time aspergiven problem. To minimize put first derivative zero, 30, Ifthe frame of reference istioninertial, make it inertial by applying pseudo vectors before applying Newton's laws or equation of motion. Projectile attains maximum rangewhen @ (angle of projection) is 45°, on the same level. If projected from a height and the projectile reaches ground then de angle is less than 45° and is determined using “7 31 =0. where x=u, (time of fight) or r=u, (time spent in gravitational field) 32, Range will be same ifa body is projected at 9or(90 — 6) (i.e. complimentary angle) with same velocity. 1. Inuniform motion ¥ = => Converse is, however, not true, Thatis if ¥,, = ‘motion may or may not be uniform. Tee, 2. Applying v=u+ar;s=ut +> af ete. even when acceleration Is not uniform, = When acceleration isnotuniform and motion isnot cireularotational, use a ory & =a Pl de dy og 7 52. Pearson Guide to Objective Physics 3. Applyingy=u-+ rete, withoutmodificationwhen 6, Sticking to the originat ground. frame of reference is non-incrtish ai > For example, ifthe [if is moving up with on vertical disp acceleration « then the effective qvecleration fora reaches the ground ish, ie we body falling from the ecliag is (¢ + a). ie. apply sector algebra or relative acceleration, . et oh = wsind 1-5 7. Notremembering common trignometic formal, => Remember tigonometie relations ke sin 20=2 sind cose, " sin (ISU ~ 6) = sin @ sin (4 + B) + sin (4 — B) = 2 sin l cos B and sin d sin B = cos (4 ~ B)~ cos 4 +B) cos. d con B= V5 feos (+ BY + eos (4 ~ Bh 4 Not differentiating berween average and 3. Considering vertical distance given in problems in instemaneous Yells projectile motion a “> lia particle travels: according to thevequation => Itis not necessary that vertical distance given be s+ 2¢+ 5 where xs in meues and in seconds, fig. Wits i, .then velocity at tis point is only is easantandous veWoaly: While horizontal velocity, i¢., venieal component of velocity is zero atthe highest point, Otherwise vse pop Where f, and f, ae final and intial 2 times. yea TE ose 5 Applying direct equations of projectile when sang or terminating pins afenot Me sane gy = stan o(1-=} vertieal height or vertical displacement between R i iv ore dioastet noice ao — Fven insome cases these equations arenot suitable, ply one-dimensional moti 7 ‘or make the problem lengthy then use along x- and the other along y-axis with time of ke the problem lengthy then use flight as combining factor. wekae andr somo 9 a 9. Considering x1, ratong on nied plane * aie => Along an inclined plane o, is slso present. Find out 1 4 and then apply x10 5 a, 4 10. Considering if the projectile strikes a wall or an ‘obstacle its time of flight will change. = Time of flight remains fixed unless it is trapped al somewhere SOLVED PROBLEMS 1, A ball is thrown up with a certain velocity so that it & touhesakeighth Hedbenaieatibesimen vin gdb! wh W341 g itis at : A 1 ° f-1 AB (oye =Igh: 5 = perorgr-2 OR O Be 2h fa 22h + eeh= Bel s or Vek 2. The displacement of a particle varies with time as x= ae" + be where a, ab, Bare positive constants. The velocity of the particle (a) willlbe independent of ax and (b) drop to zero when a= (©) goon decreasing with time (@) goon increasing with time, =a @ ¢ ~acte + bBel as increases decreases and of increases, 3. Convert given v—r shown in Fig 3.18 toa ~v graph. (IIT Screening 2005) a4 @ ) ©) @ Fig. 3.18 Motion in One and Two. {a) equation oF siven curve is ¥ & a0 4. Therelation between time randdistanvex ise = as be where a and fare constant. The acceleration is (a) 2a bie (b)2 be (e)-20°) (d)2a* (AIEEE 2005) b Nd Fath de stu ds Qav+by dt * Qar+b) 5. Acarstarting from rest accelenatesat the rate Fihrough a distance s. then continues at constant speed for time f F ‘rand then decelerates at rate > to come torest Ifthe total distance covered is 15 s, then @ss (bys (as {AIEEE 2005) (a) 1= 14 ond y,24=25 Disiance moved with uniform speed (15 ~ 3). = 12.8 ~ BF i sm ye Fig. 3.19 6. A projectile can have the same range R fortwo angles of projection. If, and ¢, are the times of flights in the i'son Guide to Objective Physics two cases, then product of the time of flights is proportional to 1 @R Oe J OR y= uw sin28 2K 4h aor 4 g g 7. Aparicle is moving eastwards with a velocity Sms" In 105, the velocity changes to 5 ms" northwards. The average acceleration in this time is ms?NE (b) 4 ms2N 1 @ Jy ms7NW (c) zero [AIEEE 2005} Si-Si wa, F Bape agg ime 8. Aparachuistaterailing ou falls 0. without rion ‘When parachute opens, it decelerates at 2 ms?, He reaches the ground with a speed 3 ms". At what height did he bait out? (91m (b) 182m (9293m Oy (= 2gh=2x 10% 50d 3 =2x10x50 = s0r| 2-20] aos 9, InFig 3.20 theposition time graph ofa particle of mass 011 kg is showa, Find the impulse at = 2 sec (2) 0.2kems" (b)-0.2 ke ms" ()0.tkgms* (@)-0.4kg ms" (@)dp= Fat=m(v,-0)=0.1 2)=0.2 kgms! 10, te 1 Fig. 320 When a ball is thrown up vertically with a velocity », it reaches a height h. Ifone wishes to triple the maximum height then the ball be thrown with a velocity (a) 3», sy, @O¥% 9», [AIMS 2005] (a) ¥ = 2gh or v= J2gh.ic., fe BY ‘A car starts from rest, moves with an acceleration a and then decelerates at b for sometime to come to res. If the total time taken is r, the maximum velocity is abt Om at oan (b) ath jt a © ab @ a+b {BHU 2005] (a) v= 0+ ay; 0= at, —b(r-1) abr ato See Fig. 3.21 From the top of a tower, two stones whose masses are in the ratio 1: 2 are thrown, one straight up with an initial speed w and the second straight down with same speed u. Neglecting air resistance, (@) the heavier stone hits the ground with a higher speed. (b) the lighter stone hits the ground with a higher speed. (©) both the stones will have same speed when they hit the ground. (@)_ the speed cannot be determined with the given data, © 13. Two runners start simultaneously from the same point on a circular 200 m track in the same direction. Their speeds are 62 ms and 5.5 ms"', How far from the starting point the faster will overcome the slower? (@) 150m away from the starting point () 170m away from the starting point (©) 120m away from the starting point (a) none (b) 200 = (6.2~ 5.5) ¢ or = 285.714 5 (6.2.x 285.714) ~ 1770 m (faster), 1770~8 « 200 70 ‘Thus 170 m away from the starting point along the track in the direction of run ° a Fig. 322 14, A particle moves according to the equation x= 2P—51+ 6, Find (i)average velocity in the first3 see and velocity at ¢=3 5, (a) ms", 7 ms! (©) 2 ms", 5 ms (b)4 ms ', 3 ms" (@)3 ms, 7 mst (a) x@)=2 GY -5 G)+6=93e 0) =6 x3)-x(0) 9-0 y= Sg sims 4 -5=4(3)-5= 7 ms* 15, Plotacceleration time graph of the figure shown, 0 10 2» 19) Motion in One and TwoDimensions 55 cms) ons) 40 rv sid 20, 0 wv 1) of gp Mt 2% 3, (@) () ue) o e Bp 14s) | + beri) t i ] 79 20 2 2 ©) (@) Fig. 2.23 @ 16, A girl after being angry throws her engagement ring from the top of a building 12 m high towards her boy friend with an initial horizontal speed of 5 ms", speed ‘with which the ring it touches the ground is (Sms 143 ms" (c) Sms! (d) 162ms! (v2 =2ay=2« 1012 v= 255240 = 16.2ms" 17. ‘The driver of a train A running at 25 ms" sights train Bon the same track with 15 ms". The driver of train applies brakes to produce a deceleration of 1.0 ms. If thetrains are 200 m apart, will the trains collide? (ayes (b) no (c) collision justavoided (@) none of these 18. Two cars 4 and B are Sm long each. Car 4 is at any instant just behind B, A and B are moving at $4 km/h and 36 knv/h, Find the road distance covered by the ear 4 to overtake B. (a)35m (b) 30m (c)32.Sm_ (275m ()y4g" 15 10= Sms" Fm, le 107 Fig. 3.24 56 Pearson Guide to Objective Physics Road distance covered =v, 1+ length of ear A ISx2+5) ‘Sm, 19. A flowerpot falls off a window sill and falls past the window below. It takes 0.5 s to pass through a 2.0m high window. Find how high is the window sill from the top of the window? Fig. 3.25 (@)10em ()12.5em ()7Sem @15em (hours a or u Using h 225 _orasm=125 2x10 20° metecen. 20, A particle moves according to the law a= ky. Find the velocity as a function of cistance y;v, is initial velocity. waagene (rang 2by (= 02-2? (@ none ow 4 wares or [lve = [)-tydy or 3 —v = hy ay 21, A particle moves according to the equation Bw where «and are consiants. Find velocity as a function of time. Assume body starts from rest. @: 2a oye Barer wow Ben (ov S en wo LA = Bla “ (@-By) _ a 6 Brory=B (1~e*) 22. A boat moves relative to water with a velocity vand river is flowing with 2». At what angle the boat shall move with the stream to have minimum drift? (@30° (b) 60° or (@) 120° (4) Let boat move at angle @ to the normal as Fig. 3.26 iftx= (Qv—vsin ) forx tobe minimum. i, 2 Jp =O! Gsee Otan O-sec*#) —_orsin 8 or 6=30° and 9=90+30= 120°, 23, Acarstarts moving rectilinearly from rest with 5 ms? for sometime, then uniformly and finally decelerates ‘a5 mis and comes to a stop. The total time of motion ‘equals 25 s, How long does the car move uniformly? Given V,, = 72 km‘h during motion. (5s (b) 10s 13s (208 (6) Total distance covered = area under v graph. From Fig 20x25 =5 12+ (25—24,) 54, or 512-1254, +500 or (i, ~5)(¢,-2)=0 = ,=5s discard t 10) Fig. 3.27 24. A ship moves along theequator tothe east with a speed 30 kiwh, Southeastern wind blows 60° to the east with 15 kmh-*, Find the wind velocity relative to the ship. (@) 39.7kmb,tan-! 6 Nor (b) 23.7kmh",tan' % Nor (©) 37.Skmb,tan! % Not E (@) none Y= %—M = (15005607 +15 sin60J)-307 WL = ¥G9.5) +(7.57 =39-Tkmb* tan B 28, 5 B ~ tax! Yf North of West. 25. A ball is thrown up with a velocity v, and it returns to the spot of throw. Plot y—r and v~ x graphs. veut at anda a straight line (Fig3) axis parabolic (Fig3) yeu +aranéa=—gisa tight Wine = Jax isparabolic (b) Fig, 3.28 26. From a tap 10 m high drops fall at regular intervals. ‘When the first drop reaches the ground, the Sth drop is, about to leave the tap. Find the separation between 2nd and 3rd drops. 38 ai @ym ym 27 opm (@) none of these 1 78h 10 or = V2s Motion in One and Two Dimensions 57 1 time interval r= “= 3s “Bs is v,, When itis im below O, its velocity is 2v. Find the ‘maximum height from O itwillacquire Fig. 330 2h » Oe o> hi ot (2h &Qvy-"=2¢2m) or sgh agg = He a SH eS 28. ‘The first stage of the rocket launches a satellite to a height of 50 km and velocity attained is 6000 kmh-' at ‘which point its fuel exhausted. How high the rocket, will reach? (2) 138.9 km (b) 188.9 km. (©)88.9km (@) 168.9 km (i= Fg *50km ‘58 Pearson Guide to Objective Physics _ (5000/3 = +50= 18809 2oxigg9 “= 188°km 29, A paniclemoves accordingto the equation = Jy +3, ‘when the particle comes to rest for the frst time @3s (b)2.5s (35s (d) none of these (x= (-3P =-2(-3)=0 orr=3s, 30. A particle of mass a is projected with a velocity 61 +8 4. Find the change in momentum when it ust touches ground. @o ) 12m © 16m (4) 20m ()Ap=m (u,v) =m [Gi ~8F)-Gi +8 Fyj=-t6m 7 [api= 16m 31. A particle is projected with a velocity 67 +8 3,3 m away from a vertical wall. Afier striking the wall it lands at... avay fom the wall (3m (b) 3.3 m (5.5m (d) 6.6m 2x8 @ 2S n16s 3 £ 6 rv =u (T-)=6(16—05)= 6.6m Zz [,_3m_,h —a . Fig. 3.1 32. The radius vector of a point A relative to the origin varies as r= ari + br F where a and b are positive constants, Find the equation of trajectory OF (A) none of these ; say or ocy=0(2) 7 + ox d where 33. A particle movesin theay plane as and Fare the unit vectors along x and y axis, The particle starts (fom origin at ¢= 0. Find the radius of curvature of the particle as a function ef x. @ tte &) efu(ey ba ae (b) Gy = 4 of x=ar ayy tb ay Yo 34. A man riding on @ flat car moving with 10 ms, He attempts to throw a ball through a stationary hoop 5 m above his hand such that the ball moves horizontally through the hoop. He throws the bell with 12 ms: with respect to himself. Find the horizontal distance from where he throws the ball (a) 15m (b) 142 m (©) 16.7m (@ 182m Solu A= 5" 35 4 = VEO = Vd = 167m 35. A body standing on a long railroad car throws a ball straight upwards, the car is moving on the horizontal road with an acceleration 1 ms. The vertical velocity give 9.8 ms“. How far behind the boy the ball will fall on the railroad car? @im 5m (c) i m (d2m wr= EF cael: a -tae-Loer-2m 36. Find the average velocity of a projectile between the instant it crosses one third the maximum height. Itis projected with w making an angle @ with the vertical (2) 1 cos 8 Ou (c) usin @ (d) wv tan @ (© Note carefully the vertical velocities atthe same height are in opposite directions and therefore their average sum =0. It is horizontal velocity which is uniform and hence v,, = 1 sin @(= Fig. 3.32 37. A person is standing on a truck moving with 14.7 ms" ona horizontal road. He throws a ball so that it returns to him when the truck has moved 58.8 m., Find. the speed of the ball and angle of projections seen by ‘@ man sianding on the road, (@)22.5ms',53° (©) 19.6 ms", vertical Sr (b)24.5 ms! 53° (d) none of these 38. Six persons are situated at the comers of a hexagon of side /. They move at a constant speed v. Each person maintains a direction towards the person at the next comer. When will the persons meet? i u @, 5, oat ox © e D Fig. 3.33 39. The compass needle of the aicplane showsit is heading due North and speedmeter indicates a velocity 240 km Jr', Wind is blowing 100 km h dve east. Find the velocity of airplane with respect to cart, (a) 260 ms, 25° EofN_ (b) 260 ms, 23° W of N (0) 260 ms', 32° BofN (d) none (a) y,.= 100 7 +2407 vy, = (240) +100? = 260 ms"; 100 4 = tan (Sip 228 BOE 40. Inan exhibition, you win a prize if you toss a coin into a small dish placed. The dish is on a sheep 2.1 m away ata height h from the hand. The coin is tossed into the dish ifits velocity is 6.4 ms" at an angle of 60°. Find h 60 Pearson Guide to Objective Physics (@)12m (135m () 1.5m (d)L.65m. (o)y=xtan 6- TYPICAL PROBLEMS 41. A projectile is launched from a height + making an angle 8 with the horizontal with speed v,. Find the horizontal distance covered by it before striking the ground. 1 “hay,sin Ord ge o 5 P-2y,sin 9¢-2h~0. = Resind + fant sin" +88h 2g x= 28 [), ors ora] 2 gin @0) % 2 er. cove Fig. 3.34 42. A baseball is projected with a velocity v making an angle @ with the incline of indication a as shown in Fig 3.35 (a). Find the condition that the ball hits the incline atright angle. ora Fig.9.35(a) (@) cot @= tan @ (b)sin @= cos a (©) tan @= sin (@cot = cos a 2vsind geosa Itwill hit vertically the incline if,=0. 0=veos @7-gsin aT? 2vsind geosa or veo 2v' sind goose [cos@ cosa —sinasin6] ~0 or cot@=tana A gcosar Fig. 3.35 (b) 43. An elevator is moving with 2.5 ms’. A bolt in the elevator ceiling 3 m above theelevator falls. How long does it take for the bolt to fall on the floor of elevator? @O.731 5 (b) 0.162 s (©) 0.7825 8315 ¥,,=O because bolt will 1 OZ 8PH3 thy also get a velocity 2.5 ms". VG = 07825 44, A point moves on the xy plane according to the law x=a sin ox and y =a (1 —cos ef) where a and «are positive constants. Find the distance covered in time t f@aat, (b) 2a? +2a° cosa, (20 (6) 2a S280 ¥, = aeacos oor and v, = asin @ Motion in One and Two Dimer or ¥ = aeeos cot 7 + aw sin ar} v-ve 4 ut = Jveosadr orut= or jy] = aa a 5 =vl4=a0r, or (=) =v 45. A particle moves with 2 deceleration «= Jj. Initial, velocity is v,, Find the time after which it will stop. x fe ve wre oF © whe (@ none of these Fig. 3.36 48, From poiui“d located on a highway, one has to get by a ‘ear as soon as possible to point B located in the field at av OG je Tk cere distance / from point D. Ifthe car moves ntimes slower oly [[-kdt orem inthe field, at whatdistancex from Done must turn off the highway 46. A particle moves according to the equation v= a Jy . Let v be the velocity in the field and mv in the Find the average velocity in the first s metres of the velocity on the highway. path. Then t, 4 and 1, vs ve i a oy oO 2 4 2a ou For tto be minimum < (1, +4) = 0 OF eft sess ce aHG or Ls Jade ort 25 7 47. Particle A moves uniformly with velocity v so that vector ¥ is continually aimed at point 8 which moves rectilincarly with a velocity uw = (ho) Gy) #2 (hm) y) 0 dy thy, Lan) +(e tes) (8) ig. 3.39 Pick the correct statements. ) 55. ‘Average speed of a particle can neverbe less than the magnitude of average velocity. dt ‘The average velocity of particle is zero in atime interval but instantanecus velocity is never zero in that interval (B) We may come across © Motion in One and Two Dimensions 63 (@) only A and Bare correct ()_ only Band Care correct (©) only A and Care correct (@) A, BandC are correct @ 56. ‘The velocity ofa particle i¢ zero at (= 0 then which of the statement is incorrect?” (a) The acceleration at ¢=0 must be zero may be zero (©) [fa=Ointhe interval O— 10s the speed isalso-zer0 inthe same interval. (a) lfthe speed is zero in the interval 0 — 10 s then a=Gind~ 10s, @ 57. An object may have ay (B)_ varying velocity without having varying speed. (© nonzero acceleration without having varying velocity varying speed without having varying velocity (©): nonzero scettention without bavine vanying speed (a) Aand Bare correct (b) Band C are correct (©) Band Dare correct (d)_ none of these © ‘58. A particlemoves accordingto the equation.x=a sin ax and y= a (1 —cosax). The path of the particle is (a) circle (b) parabota (©) hyperbola. (d) cycloid (©) ellipse @ 59. A girl throws a water filled balloon on the eve of Holi with a velocity 12 ms" at an angle of 60° with the horizontal. It hits a car at the sume height (of throw) approaching her at {5 ms. Find the distance of the car from her. @ 13m (b) 31m (44m (@ 18m (T= usin20 R Distance of cat from her= 13 +15 (2.07)=44m 64 Pearson Guide to Objective Physics 60. Amotorcyclistdoes a daredevil stunt. He jumps of 140 mwide river, The take ramp is inclined at $3° as shown in Fig.3.40, Motoreycle is 2m long. Find the minimum velocity for a successful attempt “I id 53 0=42 — 22 and 12083 1=42 oru= ae = 7 SP 15¢-56=0 or 31-112 34VOrTEES _ 10.33 er Silos Fig. 3.40 61. Aparticle starts from origin at ¢= 0 and movesin the x- xplane with a constant acceleration Grin the y-direction. ‘The equation of motion is y = Ax’, Its velocity component along x-direction is a (o)vaiable w fe F a a OR (d) Sk wee Boe coysntor © a2 F 62. A particle moves with an initial velocity v, and retardation kv, where vis the velocity at any time (2) The particle will cover a tata distance "/f, (0) The pani comes to estatr= ¥f (c) Particle continues to move for a long time (oy wis Y= te on [iden a fae Ba o “ih. fa pen # or Sp a font or v=vet Ay oOwhen (ve. 63. A particle moves slong x-axis. Iris at rest, Atr=0,x=0 and comes to rest at = 1 and x= 1. If @ denotes instantaneous acceleration, then (@) cccannot remain positive inthe interval 0 escape velocity which {bp towards Toft {d) in her hands trajectory itwill follow (elliptic {b) hyperbolic (0) vertical straight {d) parabolic o 5. Iwo particles are thrown with § ms! as shown in ‘one borizontally from s height of 40:m and te other froma height of 50 in making an angle 6U° with the vertical. They strike in mid ait. Find the ‘coordinates of strike point and distance between the buildings. i | t ‘s0mn, 7 | Fig. 3.42 1 yy +10=8 eos 601+ 5g +10=41.25 m = 82.5)-20m; = 8 sin 60% 2.5 = 19.32 m x = 20+ 1732=3732m PASSAGE 3 Read the following passage and answer the questions given attheend. We interpret average and instantaneous velocity interms of | the slope of @ graph of position versus time. In the same way, we can get additional insight into the concepts of ‘nd instantaneous acceleration from a graph with instantaneous velocity v on the vertical axis and time ¢ on the horizontal axis; that is, v,—1 graphas shown in Fig. 3.44, ‘The points on the graph latelled p, and p, correspond to points P, and P, as shown in Fig. 1. The average acceleration 4, ,~ Av At during this interval is the slope ofthe line pp AS point p, as shown in Fig 2 approaches point P., point P, in the v, ~F graph of as shown in Fig 2. approaches point p, and the slope of the line 2 approaches the slope of the line tangent to the curve at point p,.. Thus, on a graph of velocity asa function of time, the insianteneous acceleration av any point is equal wo the slope of the tangent 10 the curve a that point. As shown in Fig 3.44 the instantaneous avceleration varies with time, Note that by itself, thealgebraic sign of the acceleration doesn’t tell you whether a body is speeding up or slowing down, You must compare the signs of the velocity and the acceleration. When v, and a, have the same sign, the body is speeding up. If both are positive, the body 3s moving in the Positive direction wth inereasing speed. Iboth are negative, the body is moving in the negative direction witha velocity that is becoming more and more negative, and again the spocd is inereasing. When », anda, have opposite signs, the body is slowing down. If, 1S positive and a, is negative, the body is moving in the positive direetion with decreasing specs: ify is negative and «, is positive, he body is moving in the negative diteetion with a velocity that is becoming less negative, and again the body is slowing down, As Fig 3.45 illustratesall these possibilities. nd Prix carat two points on the siraightway. Fig. 3.43 Ay, -f graph of the motion in Fig. 1. The average acceleration beteen s, and ¢, equals the slope ef the line, p. ‘The instantaneous acceleration at P, equals the slope of the tangent at P, @ 1. Does the slope of v—findi Jerated motion? (a) Accute angle indicates aeceleraied motion and obtuse angle retarded motion (b) Acute angle represents retarded motion and obtuse angle represents accelerated motion. (c) Accelerated motion always gives curved path, (4) [acceleration is uniform path will be linea. 2. Motion along 4B is ™ 4 16) Fig. 346 (@)acceterated (b) retarded (©uniformvelocity ——_(@)eireular 3. Plot yr graph for given a ¢ graph. a | 118) [ PASSAGE 4 ] Read the following passage and answer the questions given atthe end. Average velocity ofa particle during an interval is defined AF _ displacement ime elapsed Suppose @ particle is moving in such a way that its average velocity measured fora number of different interval does not tum out to be constant. This particle issaidto move with variable velocity. Therefore, we define instantaneous velocity Velocity can vary by a change in magnitude, by a change in direction or path The limiting value ot Ly 4” is called instantaneous ee velocity. ic ar ad ‘The magnitude v of the instantaneous velocity is called speed and is the absolute value of # , That di a 0 ut 2(y, +15) (a) 2s (@) none is 2. The OA and AB part of the graph correspond to. 10 5 fo oo) ™ © «@ Fig. 3.47 /B n Gif Gis accute tan Gis +e: if Bis obtuse tan Bis ~ve. ‘a 2. (b}-z slope tan Bis negative. s $9109) 2 @) Fig. 3.48, 68 Pearson Guideto ObjectivePhysics (@) uniform retardation, variable acceleration (b) uniform acceleration, uniform velocity (©) constant velocity, uniform acceleration (@) uniform acceleration, varying velocity. 3. Find the average speed between 0 ~ 10s. ns") Fig.3.49 (@)8.25ms! (b)7.75 ms ' ()8.75ms" (@)792ms* 24 Dw, Bee ari aa 2 0) distance covered 3. @) Avg. speed= SE aa Read the following passage and answer the questions given at the end. If a particle is moving in such a way that its average acceleration turns out to be different fora number of different {ime intervals, the particles sid tohave variable acceleration. ‘The acceleration can vary in magnitude, or in direction or both. In such cases we find acceleration at any instant, called the instantaneous acceleration, Itis defined as ry Avice war a ‘That is the acceleration of a particle at time ¢ is the ay limiting value of [at time ¢ as Af approaches zero. The direction ofthe instantaneous aceeleration i the limiting direction of the vector change in velocity Av. 1. The limiting direction of vector change implies (@) the direction of the tangent atthe limiting point () the direction of normal atthe Limiting point. (©) direction of final velocity, (@)_ direction of initial velocity. A particle is travelling due north with S ms. tt tums east in 5 sec. and continues to move with S ms" Find the average acceleration during turning, (ayo (b) V2 ms* NE (6) V2 ms* SE (d)2 ms? SE 3. A particle is moving along a straight line with 10 ms". It takes a Ustur in 5s and continues to move along with same velocity 10 ms". Find the average acceleration during tuming (b)-2 ms? (d) none of these 4. A particle travels according to the equation such that its acceleration a e ~ v*, Find the distance covered when its velocity falls from v,to v. pratt r= 7 t % (x= klog, > (x= log, > @ 2 (a, 3 a, a 45 PASSAGE 6 Read the following passage and answer the questions given attheend. adr S 4, and a, are called the scalar components of the acceleration a. In one-dimensional motion. one, of the acceleration a, or a, will be 2ero. If motion is along vertical direction then a = gand we term it as motion under gravity, The sparkling example of two-dimensional motion under gravity is projectile motion. A projectile has uniform horizontal velocity and falls freely under gravity. Therefore, the path appears to be parabolic. Circular motion isanother example of two-dimensional motion. In uniform circular ‘motion, particle has time period fixed. If is the time period aking aac ie aii with time the particle is said to possess nonuniform angular velocity. 1. A particle moves accordingtto the equation x=asin ‘or and y= a cos ax. Then its resultant motion is, (@) uniform circular motion and wis clockwise. (b) uniform circular motion and wis anticlockwise. ©. romuniform circular motion with avclockwise (@ _romuniform circular motion with anticlockwise. 2. Inuniform circularmotion (@_scceleration is variable, (b) acceleration is uniform. (©. the direction and magnitude of acceleration both vary, (© _ifforce applied is doubled in circular motion, then angular velocity becomes double. 3. Which ofthe following is correct? (@ Radius ofcurvature of projectile motion isuniform ‘throughout. () Radius of curvature of projectile motion varies at ‘ach point. (Radius of curvature is maximum at the time of projection. (Radius of curvature is maximum at the highest point. 4. Aparticle moves under gravity. It implies (@ _itshorizontal velocity is zero. (b)_itshorizontal displacementis zero. (© _ithas only vertical displacement. (@ it may have horizontal displacement but ‘acceleration is along vertical direction. S.A particle moves according to the law x~ af andy = be then (®) its acceleration is under gravity (©) _itmove in a curved path in XY plane. (© _itmoves in a straight line path in X-V plane. (@ itdescribes ellipse. 6 change in position of the particle from A to B in time ‘Average velocity of the particle during this interval is 1 ‘A particle moves from A to B diametrically opposite in acircle of radius 5 m witha velocity 10 ms", Find the average acceleration. AO aay {a) zero oO) me a (@)none (a)at1=0,x=0,y=a atar=7/ s=ay=0 (a) (a) @ (b) av 2esing2 40 Oa a fe a Read the following passage and answer the questions given attheend. ‘The velocity of a particle is the rate at which its position varies with time. The position of a particle in a particular reference frame is given by a position vector drawn from the origin of that frame of reference. At‘~ ¢, assume the particle ‘sat andits position in xy plane being described by position vector Fj .At later time , the particle reaches at point B as shown in Fig 3.50 and its position vector is The displacement vector AF j describes the he displacement (a vestor) ‘» ~ “time elapsed (a scalar) Fig. 3.50 Theaverage velocity defined aboveis called average the average acceleration. 70 Pearson Guide to Physics for IT @ (by © the motion between 4 and Bis known, the motion between 4 and Bis erratic. the motion between 4 and B may havebeen steady orerratc co) A particle moves from 4 to Bsueh that x= Its average velocity from (ay6ms! the motion between 4 and Bis steady. (85 ms" particle moves according to the law = ax + hy +e (a) Theaverage velocity is same between x=3 mtox =9m. (b) The average velocity cannot be defined between x= 3mtox-9m © (a) The acceleration depends oncube of the velocity A body throws a ball to his friend 20m away, The ball reaches to the friend in 4 s. The friend then throws the ball back to boy and it reaches the boy in5s. ‘The average acceleration is uniform throughout. @) &) © The average velocity is 40/9 ms" ‘The average acceleration is zero. The average velocity is zero but average acceleration is nonzero. (a) Average acceleration of the motion cannot be defined, acceleration= 4 © x(5)—x2)_ (2545-3)-(442-3) = 3 i, ()t=axt+hyre dy =a ch ~ Qaxeby (as g aces. "PASSAGE 8 Read the flo atthe end. P. Kirkpatrick in February 1944 American Journal of Physics published @ paper titled “Bad Physics in Athletic Measurements’, He suggested in this paper how the choice of Olympic venue affects the records of athictes. In 1936 Jesse Owens (United States) established a world record in running long jump of 8.09 mat the Olympic games of Berlin (g= 98128 ms*), If the Olympies was held at Melbourne (= 9.7999 ms) his record would have been 8.1065 m. Similarly, be quoted many other examples. 1. _Istheclaim ofMr. Kirkpatrick right? (@ Yes (b) No © @ 2. Find the change in range of projectile ifacceleration due to gravity changes by dg, 1g passage and unswer the questions given cannot say may be correct or may be not de (dk=R dg =P (dk= 5, @ar-S8 2e f 4. Which ofthe spon is most affected by variation of ‘” (2) swimming (©) jump (horse riding (4) shooting (c)allare equally affect, bn RE BOPOSIE 6 gs ara) Sm Wsin20 2 R= 0) @ «differentiating (1) dk dg aR R 2) Motion in One and Two Dimensions 71 QUESTIONS FOR PRACTICE 1. * Acarmoves in a semicircular track of radius 700 m. boos) (oe) If it starts from one end of the track and stops at the ther end, the displacement of ear is f 1 (a)2200m (b)700m (6) 1400m (@) none of these ot Sr) 2. ispacementime graph ofa body i shown Below mol | Velocity-time graph of the motion ofthe body is é Fig. 3.52 4, A boy can throw 3 stone tomaximum height of 30m (c) (dd) Fig. 3.51 3. For Figure of Q.3, the acceleration-time graphof the erat . wo 3 4s) 3 i (a) (b) To what maximum range can he throw this stone and to what height so that the maximum range is maintained? (a) 100m, 100m (©)200m, 50m A player throws « ball upwards with an initial speed of 29.4 ms". The height to which the ball rises and the time taken to reach the player’s hands are (a) 22.05 m, 38 (b)44.1 m6 (0) 29.4 m, 6s (4)54.5 m9 Itwas known that a shell fired witha given velocity (b)100m,25m sx 3¢ radian fa target but a hill was found to obstruct n strike and at an angle of projection is path, The angle of projection to hit the target should be 13a (a) Data is insufficient (b) | radian 230 10x (ce) 3 radian For the velocity-time curve shown below, the distance covered by a body from Sth to 7th second of its motion is —-- fraction of the total distance covered by it Fig. 3.53 (b)92 23 (29 (t2 72. Pearson Guide to Objective Physics & A vehicle moves west witha speed of 50 ms" and then towards north with a speed of 50 ms" only. ‘Total time taken by the body is 5s. What is the average acceleration of the body? @0 (b)S0ms? (© l4ms? (€)204ms? 9. A body is projected at an angle 6 with the vertical with kinetic energy KE. What is the kinetic energy of the panicle at the highest point? () KE cos 0 (b) KE sin? oF @KEwn e 10. A ball is thrown fiom the ground to clear a wall 3 m high ata distance of 6 m and falls 18 m away from the wall, the angle of projection of ball is, (@ tant 2 want 2 sel. arattt (¢) tar 2 (d) ar a IL. Ifthe velocity of the motoreycle vis constant, then determine the velocity of the mass zs a function of 1x. Given that ends ? and & are coincident on Qwhen x=0. H+x Tee Ow HP +x? +e OV on 12, Three points are located at the vertices of an equilateral triangle having each side as at All the points move simultaneously with speed w such that first point continually heads for second, the second forthe third and the third forthe first. Time taken by the points to meet at the centre is 13. A wall clock has a $ cm long minute hand. The average velocity ofthe tip ofthe hand reaching 0600 brs. to 1830 hrs. is (a) 22x10" ems? (b) 1.2 10 ems" (6) 5.6 x 107 cms (@)3.2x 10° ems" 14, A particle leaves the origin at ¢ = 0 and moves in the positive x-axis direction. Velocity ofthe particle [sj ms" and r= 5s find the x coordinate of the particle at an instant of 10 s, at any instant is given by v 0 ces") Fig. 3.56 (a0 (b) 10 m (©) 20m (@)-10m 15. The acceleration of a particle is increasing lineerly with time ¢ as i. If the particle starts from origin velocity u, thedistance travelled by itin (ous pe (our + Be (oust Be (dart a 16, A drunkard takes a step of 1 m in 1 s. He takes 5 steps forward and 3 steps backward and soon. The time taken by him to fall ina pit 13 maway fromthe start is — — Fig. 3.57 (a)26s (31s (37s (als 17. A jeep moves at uniform speed of 60 kmh! on a straight road blocked by a wall. The jeep has to take a sharp perpendicular turn along the wall. A rocket flying at uniform speed of 100 kmh’ starts from the wall towards the jeep when the jeep is 30 km away. 100 ken ae @ kank" Fig. 3.58 The rocket reaches the windscreen and returns to wall, Total distance covered by the rocket is (2) 100 km (b) 50km_ (©)25km @75km, 18. A marble rolls down from top of a staircase with constant horizontal velocity w. by | L Fig. 359 If each step is y metre high and x metre wide, the marble just hits the edge of the nth step when = os Oe 2nd ant on os Motion in Oneand Two Dimensions 73, 19. A particle experiences a fined acceleration for 6s after starting ftom rest. It covers a distance of, in first two seconds, s, in the next 2 seconds and s, in the last 2 seconds then s, : 5, : 5, is, (a) 1:3:5 (b)5:3:1 (o:2:3 @3:2:1 20, A football dropped from a height onto an elastic ret, stretched horizontally much above the ground rebounds. The graph for the motion is f y © @ 21. A projectile is required to hit target whose coordinates relative to horizontal and vertical axes through the point of projection are (at ). If the ‘gun velocity is J2ger, it is impossible to hit the targetif (Bb p> te 3 3 (Bszo (p20 22. A stone isallowed to fall from the top of atower and. cover half the height ofthe tower in the last second of its journey. The time taken by the stone to reach the foot of the tower is @Q- V2) (4s ()(2+ J2)8 @ Q+V3) 5 st a velocity of 12 ms? packet is dropped from it when itis ata height of 65 m from the ground, it drops a packet. Time taken by the packet to reach the ground is, 74 Pearson Guide to Objective Physics (ass tb) Ss B (os zs 24. A particle is projected up an inclined plane of inclination ceto the horizontal. Ifthe particle strikes the plane horizontally then tan ce= ig. 3.61 (an B (by 21an p 1 (o) 3 tan B (a) 5 tan B 25, Three balls roll down three diferent frictionless pathsas shown below. If the masses of the balls are kg, 2 kgand 3 kg then their respective velocities 1,.u, and u, on reaching the ground are such that Fig. 3.62 (yn rue (b)n, sna, ©n iu , 26. A girl standing on a pedstal at rest throws a ball upwards with maximum possible speed of 50ms I? the platform starts moving up at § ms" and the girl again throws the similar ball in similar way. The time taken in the previous case and the time taken inthe second ense to retum to her hands are (a) 105,55 (b)5s, 10s (©) 105, 10 (a) 5s,5s 27. An acroplane drops a parachutist, After covering distance of 407m, he opens the parachute and retares a2 ms°, Ifhereaches the ground with a speed of 2 ims ‘he remains in the air for about 6s (o)3s (135 (a) 108 28. A huge rectangular compartment {alls vertically down with acceleration a. A gun fixed at comer P requites to hit Q. The bullet fired by it wil hit Q if e a Fig. 3.63 laya=g (bya>g (ace (d)any of (a), (6) orc) 29. A helicopters flying at 3564 m above ground. [fan angle of 30° is subtended at a ground point by the helicopter position 100 s apart, wht is the speed of the helicopter? Fig. 3.64 () 100ms* (b)150ms * (©)20ms" ()25 ms" 30. A particle is thrown vertically upwards fiom ground. It takes time ¢, to reach a height h, It continues to move and takes time /, (0 reach the ground. Its ‘maximum heights Fig. 3.65 goo ws ire OF Gray (ag (G+ 31. A tram moves with a velocity w. A particle moving. horizontally witha speed v enters through corner B perpendicular to u. This particle strikes the diagonally opposite comer 4. Ifthe dimensions of tram are 16 x 2.4x 3.2 mi, the value of wis Fig. 3.66 (a)3ms" (b)20ms (@)15ms? (4)30ms" 32. A driver driving a truck at a constant speed of 20 ms" suddenly saw a parked car ahead of him by 95 m, He could apply the brake after some time to produce retardation of 2.8 ms. An accident was (ayoss (008s Wis 33, Awater tap leaks such that water drops fall at regular intervals. Tap is fixed 5 m above the ground. First drop reaches the ground and at that very instant third drop leaves the tap. At this instant the second drop isat a height of sm (b) 0.758 iil Fig.3.68, ney @3m 4.5m (375m @25m 34. Aparicle moves alongastraight line as perequation P= og + 2Br+ ¥, where x is the distance travelled and a, 8, Yare constants. Its acceleration varies as @x (oy? (yx? @x 35, Atankmoves uniformly along s-anis.It fires ashot from origin at an angle of 30° with horizontal while ‘moving along positive.x-axisand the second shot is also fired similarly except thatthe tank moves along negative x-axis. If the respective range of the shot are 250 mend 210 malong x-axis, then Motion in One and Two Dimensions 75 ne Fig. 3.69 the initial velocity values are (2) 12nvsand21 mvs (b) 53 misand 49 avs, (c) 9Sm/sand S8m's (4) 79 msand 19 m/s 36, Engine of a vehicle can give it an acceleration of I ms? and its brakes can retaed it at 3 ms. The minimum time in which the vehicle can make a journey between slations 4 and B havinga distance of 1200 mis. (a) 55.68 (b) 65.68 (506s () 56.55 37. Onarainy day. a raindrop falls from very high clouds and faces retardation cue to air. This retardation is directly proportional to the instantaneous speed of the drop. An expression far the distanes travelled, by the drop in time ris, 38. A morble starts falling from rest on a smooth inclined plane of inclination a After covering distance h the ball rebounds off the plane. The distance from the impact peint where the ball rebounds forthe second. timeis Fig. 3.70 (a) 8h cos a (b) Sh sin (©) 2htan a (@shsin a 39. Whatis the maximum range thata ball thrown witha speed of 40 ms" can cover without hitting the 25 m high ceiling ofa ong hall? 76 Pearson Guideto Objective Physics (@)1505m (6) 100.25m. (©)1103m @)2005m 40. Two canons installed a the op ofa cliff 10 mhigh fire a shot each with speed 5/3 ms" at some interval. One canon fires at 60° with horizontal whereas the second fires horizontally. The coordinates of point of collision of shots are 14 (@)3.5 m3 m OoRmsm 14 Oymy_Em (d@) 53m, 5m Fig. 3.74 41, A lift moves upward with an acceleration of 1.2 ms", A nail falls from the coiling of the lift 3 m above the floor ofthe lft. Distance of its fall with reference to the shaft of the lift is (8) 0.75 m (0) 05m (im @1sm 42. A body travels 200 em in the first two seconds and 220 em in the next four seconds. What will be the velocity atthe end of 7th second from the start? (@)10ems* (b)20ems" (© 1Sems* (@)sems* 43. A caraccelerates from rest ata constant rate o for some time after which it decelerates at a constant rate to come to rest. Ifthe total time elapsed is 1, the maximum velocity acquired by the ear is given by o(F } o(F of} a(S} 44. A panticleis moving in a plane with velocity given by @=u,? +(awcos ax) J, where i and j are unit vectors along x and y axis, respectively. Ifthe particle isat originat ¢~ 0, the distance from origin atime 32/2wis (aso 20 7 (yet stm [Based on Roorkee 1985] 45. A train moves from one station to another in 2 brs time, its speed during the motion is shown in graph, Times in tes Fig. 3.72 The maximum acceleration during the joumey and distance covered during the time interval from 0.75 hour to 1.00 hour are (@) 60 km br®, 100 km (b) 160 km hr*, 10 km. (©)10kmhr?,10km (a) 260 km hr, 1 km 46. The displacement x of a particle moving in one dimension is related to time by the equation 1 = (Ve+3), where'x ia in m and ¢ ins, The dae placement, when velocity is zero, is f@)1m (b)2m (4m (zero 47. A soldier jumps out from an aeroplane with a parachute. After dropping through adistance of 19.6 'm, he opens the parachute and decelerates at the rate of I ms*. If e reaches the ground with a speed of 46 ms", how long was he in air (a) 1s (by 125 fe) 15s @i7s 48. A boat having a speed of 5 knv/hr in still waters, crosses a river of width | km along the shortest possible path in 15 minutes. The speed of the river water in knw is (1 6)3 (4 @ va (117.1988) 49, A particle moving in a straight line covers half the distance with speed 3 ms“, The other half is covered {in two equal time intervals with speeds 4.5 ms‘ and7.5 ms".respectively. The average speed ofthe particle during motion is, (a)4ms (o)5.5ms" (b) Sms" (@)48 ms 114.7.1992] ‘80. A paniclein uniformly accelerated motion travels a, band c distances in xth, yth, and zth second of its ‘motion, respectively. Then a (y=2)+ b(z—2) +c Gy . @t (wo 2 @3 SL. The driver ofa train moving with a speed v, sights another train ata distance s, ahead of him moving in the same direction with a slower speed v,. He applies the brakes and givesa constant develeration ato his train. For ro collision, s is _ avy we one gy. (Be (yaw 2d of eS @y [Based on LLT. 1984] 54. 4, B, Cand Dare points in a vertical line such that ‘AB = BC= CD. Ifa body falls from rest at 4, prove that the time of descent through 4B, BC and CD are in the ratio of 1: —:—. Motion in One and Two Dimensions 77 @ (V1): (4-8) oy (v3-1): (¥3-v2) ©) (E-8) (5) (-V3): (8-8) [Roorkee 1987] 55. A car is travelling at a velocity of 10 kmh" on a straight road. The driver of ear throws a parcel with a velocity of 10.J3_ kmh! when ear is passing by a man standing on the side of road. If parcel just reaches the man, the direction of throw makes following angle with the direction of car (a) 138° (b) 45° (c) tan (V2) (@)tan (1/¥2) [Roorkee 1992] 56. A ball is dropped vertically from a height d above the ground, It hits the ground and bounces up vertically to a height d/2. Neglecting subsequent motion and air resistance, its velocity varies with height A above the ground as = Be be © @ Fig. 374 ILL-T. Screening 2000] 57. An observer moves with a constant speed along. the line joining two stationary objects. He will ‘observe that the two objects (2) have the same speed (b) have the same velocity 78 Pearson Guide to Objective Physics © @) movein the same direction move in opposite directions Which of the following statements are true for a moving body? (a) (o) 59. Let v and a denote the velocity a fits speed changes, its velocity must change and itmust have some ac fits velocity changes, its speed must change and it must have some acceleration, Ifits velocity changes, its speed may or may not change, and it mus! have some acceleration, Ifits speed changes but direction of motion does not change, its velocity may remisin constant Ml acceleration respectively of a body @ ) ©) @ ‘can be nonzero when v= 0, amust be zero when v= 0. aamay be 2ero when v #0. ‘The direction of a must kavesome corelation with, the direction of y. 60, Let ¥ and @ denote the velocity and acesleration, @) () ©) @ spectively, ofa body in one-dimensional motion. | # | must decrease when d <0. Speed must inerease when g >0. Speed will increase when both # and a <0. Speed will decrease when # 0. 61. ‘The figureshows the velocity (») ofa particle plotted against time (7). @ co © @ Fig.3.75, “The particle changes its direction of motionat some point. The acceleration of the particle remains constant ‘The displacement of the particles zero. ‘The initia! and final speeds of the particle are the 62, A particle moves along the a-axisas follows: it starts fiom rest at = 0 froma point x and comes to restat/= | ata point x= 1. No other information is, available about its motion for the intermediate time {0 4 atsome point or points in its path @ must change sign during the motion, but no other assertion can be made with the information given 63. The displacement (x) of a particle depends on time lass ar pr. ) () © @ ‘The particle will return to its starting point alter time aif. ‘The particle will eometo rest alter time 20/38, Theinitial velocity of the particle was zero but its, initial acceleration was not Ze. No net furee will act on the particleat = a/32. 64. A particle moves with an initial velocity ¥, and retardation ox, where wis its velocity atany time 1 a o ©) «@ The particle will cover a total distance 1,/¢% ‘The parti! The particle will continue to move fora very long. time, ‘will come to rest afteratime Va. The velocity of the particle will become v, /2after atime 1/a 65. A particle starts from the origin of coordinates at time, ‘anid moves in the.xy plane with a constant acceleration 0: in the y-direction. Its equation of motion is y= Be. Its velocity component in the x- divestionin able b) pa yvariabe oye a z OR O35 66. In Fig. 3.76, pulley P moves to the right with a constant speed 1 The downward speed of 4 is vy, and the speed of 8 to the tight is ayy, 3 Fig. 376 ()y,auty, © @ 67. In vtuey, The two blocks have accelerations of the same ‘magnitude. Fig.3.77, the blocks are of equal mass. The pulley is fixed. In the positive shown, A moves down with a speed u, and v, = the speed of B. P. @ ) © @ AA Fig.3.77 Bwill never lose contact with the ground. The downward acceleration of 4 is equal in ‘magnitude to the horizontal acceleration of B. ucos 9 uicos 8 68, Two particles 4 and B start simultaneously from the same point and move in a horizontal plane. 4 has an initial velocity 1, due east and accelerstion a, due north. B has an initial velocity u, due north and acceleration a, due east fa) (b) © @ Their paths must intersect at some point They must collide at some point ‘They will collide only if a, = au, If, > w, and a, < ay, the particles will have the same speed at some point of time 69, Two particles are projected from the same point with the same speed, at different angles 8, and 8, to the horizontal, They have the same horizontal range. Their times of flights are ¢, and ¢, respectively. 70. A cart moves with a constant speed along a horizontal circular path. From the cart, a particle is, thrown up vertically with respect to the cart @ (b) © @ ‘The particle will land somewhere on the circular path. The particle will land outside the circalar path. ‘The particle will follow an elliptical path The particle will follow a parabolic path. Motion in One and Two Dimensions 79 71. A man on a moving cart, facing the direction of motion, throws a ball straight up with respect to imselt. @ O) © Cc) ‘The ball will always return to him. ‘The ball will nevereturn to him ‘The ball will return to him ifthe cart moves with a constant velocity. ‘The ball will fall behind him ifthe cart moves with, someacceleration, 72, A small ball isconnected toa block by a lightstring of length /. Both are jally on the ground. There is sufficient friction on the ground to prevent the block from slipping. The ball is projected vertically ‘up with a velocity x, where 2g! g. P will hitthe wall CD or the floor AD ifav. 71. Ariveris flowing from west to east at a speed of $ ‘metres per minute. A man onthe south bank of the 82. An object may have river, capable of swimming at LO metres per minute ' ‘ instill waters, wants to swim across the river in the (@) varying speed without having varying velocity (b) varying velocity without having varying speed (©) nonzero acceleration without having varying shortest time. He should swim ina direction (a) due north (b) 30° east of north velocity (c)30° north ofwest. (4) 60° east of north (0 senzeroraccilbislion ditsrhadina vecping 78. Inthe arrangement shown in figure, the ends P and speed of an inextersible string move downwards with uniform speed u, Pulleys and B are fixed. The mass M moves upwards witha speed. a straight ine 83, Mark thecorect statements fora particle going on @) _ Ifthe velocity and acceleration have opposite sign, the object is slowing down (b) If the pe ty have opposite sign, the particle is moving towards the origin. (©) Ifthe velocityis zero at an instant, the acceleration should also be zero at that instant, (@) If the velocity is zero for a time interval, the acceleration is zero at any instant within the time interval jon and vel 84. The velocity of a particle is zero at /= 0. (a) The acceleration at = 0 must be zero. (b) The acceleration at ¢= 0 may be zero, (©) If the acceleration is zero from #=0 tor = 10 s, the speed is also zero in this interval, (4) If the speed is zero from ¢ = 0 to r= 10 s the acceleration is also zero in this interval. 85, Mark the conect statements (®) The magnitude of the velocity ofa particle is equal to its speed. (b) The magnitude of average velocity in an interval is equal to its average speed in that interval. (©) tis possible to have a situation in which the speed ofa particle is always zero but the average speed is not zero. (@)_Itis possible to have a situation in which the speed of the particleis never zero but the average speed in an interval is zero. 86. The velocity-time plot for a particlz moving on a straight line is shown in the figure. vv) 10 o 169) 1X0 30 -20 20 Fig. 3.82 (@) The particle has a constant acceleration, (b) The particle bas never turned around. (©) The particle has zero displacement. (@ The average speed in the interval Oto 10 5 is the same as the average speed in the interval 10's to 20s, 87. Fig. 3.75 shows the position of aparticle moving on axis as a function of time, snd Two Dimensions 81 fm) 20; 10. + 10) d 4 Fig. 3.83 (a) The particle has come to rest 6 times. (b) The maximum speedis at=6s. (©) The velocity remains positive for 1=0 to 1=6 s. (d) The average velocity for the total period shown is negative. 88, Two extremely smnall blocks are lying on a smooth, uniform rod of mass M and length £. Initially the blocks are lying at the centre. The whole system is rotating with an angular velocity 0, about an axis passing through the centre and perpendicular to the rod. When the blocks reach the ends of the rod, then the angular velocity of the rod will be— we Fig. 3.84 Ma, Mos © am ©) Fram Ma. Mo, © Tom Ot 8m 89. A rocketis projected vertically upwards, whose time velocity graph is shown in, The maximum height reached by the rocket is - 2040 80 60 100 190 1s) —> Fig. 3.85 (@)1km ()10km (©)20km (@)60km 82. Pearson Guide to Objective Physics 90. Three blocks of mass m,,m, and mare lying in contact with each other on a horizontal frictionless plane as shown in the figure. Ifahorizontal force F is applied on m, then the foree at the contact plane cofrm, and m, will be Fig. 3.86 Fim tm) ae ©) (mi, +m,+m,) ©) (in, +m, +m) F(m+m,) mer © (nm +m,+m,) 91. Abulletis fred froma rifle. I'theriflereccils freely then the kinetic energy of therifle will be (2) equal to that ofthe bullet. (b)_ less than that of the bullet. (©) more than that of the bullet @ 200 92, A small dise is lying on the top of a hemispherical bowl of radius R. The minimum speed to be imparted to the dise so that it may leave the bow! ‘without slipping is Fig. 3.87 R (w= Vey (yv=2 fer (ov = Jerk v= Vigk 93, The quantity which remains constant for a body moving in a horizontal cirele, is (apkinetic energy (b) acveteration (@) velocity 94, Force F varies with time in accordance with the following figure. The mean force will be 7 F (©) force Fig. 3.88 A @F, oF (@2F, (d)Zeto 95. An object of mass 150 kg. is to be lowered with the help ofa string whose breaking strength is 100 Ke/ wt, What should be the minimum acceleration of the body so that the string may not break ? (93.33me (4)45m'e 96. Two blocks of mass 8 kg. and 5 kg are connected by aheavy rope of mass 3 kg. An upward force of 180 N is applied as shown in the figure. The tension in the string at point P will be tw Ke PsKe swe Fig. 3.90 (@)00N (b)90N (120N (@) 150 97. A body is released from the top ot a tower. The bodycovers adistance of 24.5 m inthe last second of its motion. The height of tower is (8) 598m (b) 44.1. m (392m (49m 98. A meter scale is suspended freely trom one of its ends. Its another end. is given a horizontal velocity ¥ such thet it completes one revolution inthe vertical circle, The value of vis () a3 ms, (0) V6 mis (© x mis. (4) mss 99. A block slips with constant velocity on a plane inclined at an angle 9. The same block is pushed up the plane with an initial velocity v,, The distance covered by the block before coming to rest is— oe 2 ony 100. A ballis dropped from a height of 19.6 m. The distance covered by it in the last second is (@19.6m (147m (4.8m (d)98m 101. A particle is projected upwards. The times corresponding to height h while ascending and while descending are t, and f, respectively. The velocity of projection will be @at, (selt,t) 102. A frictionless wire is fixed between 4 and B insidea sphere of radius R. A small ball slips along the wire. The time taken by the ball to slip from 4 to B will be al Fig. 3.91 2er 2yeR eos © geos6 Oe gk (© 2\R7e @ Jecoso 103. Starting from rest, a body takes 4seconds in slipping from top to bottom of an inclined plane. The time taken by the same body in covering one quarter distance on” the same plane from rest will be @ls (b)2s @4s (16s 104, A 150meter long tain is moving towardsnorth with a velocity of 10 mvs. A parrot is flying in the south direction parallel totram at Sm/s. The time taken by the parrot in crossing the train is (12s (8s (lss @ 10s 105. Auniform stationary sphere tarts rolling down from, the upper end of the surface as shown in the figure, 106. 107, 108. 109. Motion in One and Two Dimensions 83 and it reaches the lower right end 7=27 mand = 20m. The sphere will fallon the ground level atthe following distance from 4 nh Fig. 392 f@) 10m (b)20m (30m @40m A body of mass m starts moving with velocity F, at point 4 on a frictionless path as shown in the figure. The speed of the body at point B will be Fig. 3.93 (a) Zero oy, oy 2, oF 2, In the above problem the speed at point C will be (ay, wr, y, oF @ fiteh ‘A passenger train is moving with speed ,, on rails, ‘The driver of thistrain observes another goods train, moving in the same direction with speed v, (v,> v,). fon applying brakes, the retardation produced is a, then the minimum distance covered by the passenger train so that it may not collide with the goods train willbe ot) mee Za 2a ots (4) information isincomplete Figure 3.86 represents a painter in a swing by the side of a building. When the painter pulls the string thenthe force applied on thesurface is 450N, whereas the weight of painter is 1000 N. If the weight of the swing is 250 N then the acceleration produced in the swing will be 84 Pearson Guide to Objective Physics @ams (sms 110. The length of the arm of a nut-cracker is 15 cm. A force of 22.5 kg. weight is required to cut a mut ‘without cracker. Where should the nt be placed on the cracker in order to cut it by a force of 2.25 kg/wt (© Lemfrom fulcrum (©) 1.Sem trom fulerum (©) 05cm fiom folerum (@ 20cm fiom folerum 111. Two balls 4 and B are simultancously thrown. 4 is thrown from the ground level with a velocity of 20, rms“ in the upward direction and B isthrown froma height of 40 m in the downward direction with the same velocity. Where will the two bails meet? @ism (b)25m 35m asm 112, A body falls freely from the top of a tower. It covers 36% of the total height in the last second before striking the ground level. The height of the tower is (050m (b) 75m (100m (@) 125m 113, Two blocks are attached to the two ends of a string, passing overa smooth pulley as shown in the figure. ‘The acceleration of the block will be (in m/s*) Fig. 3.95 (2)0.33 (b) 1.32 ol © 0.66 114. In the above problem the tension in the string will be us. 16. a7. 9. 120. 121, (@)4s6n (b)850N (©) 1000 (@)2000N A ball is thrown from a height of 12.5 m from the ground level in the horizontal direction. It falls at ‘horizontal distance of 200 m, The initial velocity of the ball is (2) 40 mvs (©) 80 ms (6) 120m's (2)20 mis The distance traveled by a body in fourth second is, twice the distance traveled in seconds. If the acceleration of the body is 3 m/s', then its initial velocity is 3 5 (a) 5 ms (b) 5 mis 1 9 (5 ms @ 5 ms The diameter of the tep of a fire brigade pump is 5 cm Water is thrown by this pump at 2 horizontal speed of 18 mvs on awall. If water rebounds back from the wall, then the force exerted by water on. wall willbe (a)235 10° dyne (6)5.76 108 dyne (6) 636% 10°dyne (2) 10" dyne A.30kg sphere, moving with a velocity 48 mis, splits into two parts after explosion. The masses of these fragments are 18 kg and 12 kg. lithe heavier fragment comes to rest after explosion, the velocity of second fragmentwillbe (@soms (&) 100ms () lms @ 120ms A bullet of mass 20 gmandmoving witha velocity of 200 mvs strikes a sound and comes to rest after penetrating 3em inside it. The force exerted by the sand on the bullet will be (a) 11.2 10"dyne (b) 15.7% 10 dyne (6) 13.3 10%dyne () 18.6% 10" dyne A ballet, moving with a velocity of 200 m/s penetrates a wooden block and comes to rest after traversing 4 cm inside it. What velocity is needed for traversing a distance of 6 em in the same block (@) 104.3 ems (6) 1362 cm/s (©) 244.9emis (@) 272.7 em’s The diameter of a solid disc is 0.5 m and its mass is, 16kg. What torque is required to increase its angular velocity from zero to 120 rotations/minute in 8 seconds? (a "i Nim oF Nim (TINim (OnNm 122, In the above problem, at what rate is the work done by the toque at the end of eighth second? @nw (IFW rw @mw 123, Two projectiles each of mass m are projected with same velocity v making an angle cand fiom the same point in opposite directions. Find the change in their momentum at any instant. (2) 2m sin (e+ (b)2msin 2? (©) 2mv cos (a+ A) (@2mrcos 4B) 124, An ircraft is flying ata height of 2800 m above the ground. The angle subtended by it in 10 s is 30°. Find the speed of the aircraft G (@)150ms" “ (6) 100 ms* (© 140m" (125 ms* ° 125. A rifle witha muzzle velocity 1500 fis" shootsa bullet at a small target 150 ft away. How high above the target must the gun be aimed so that the bullet hits the target? (@)2.02 inch (b)1.72inch (©) 1.82 inch (@)1.92inch Electrons, nuclei, atomsand molecules like all forms of matter, will fall under the influence of gravity. Consider separately the beam of electrons, of nuclei, of atoms and of molecules travellinga horizontal distance of | m. Let the averagespeed of electrons be 3 x 10" ms", fora thermal neutron 2.2 x 10* ms, for aneon atom 5.8 x 10° ms and foran oxygen molecule 4.6% 10? ms". Thebeams move through vacuum horizontally with initial velocities mentioned above. A golf ball is also projected horizontally with 20 ms“ in vacuum, 1, Outof the given beams which deviates maximum in travelling 2m? (a) electron beam (b)neutron beam (©) neon atom (d) oxygen atom 2. Find the deviation of golf ball in travelling through 2m, (@)2cm (b)Scm (©8em ()3.6em Motion in One and TwoDimensions 85 3. Is there any effect of electron-electron repulsion? (a) Yes (b)No (c)insufficient datato reply (d) none 1 x 1 (xy 1. (Deviation y= 5 gf ands=— ory 42(2) % 1 (2y sive (2 sem 3. (b)Since the net velocity has already taken into account the repulsion, no effect of repulsion is to further added. Read the following passage and answer the questions given attheend. The instantaneous acceleration of a body, that is, its acceleration at one instant of time or at one point of its path, is defined in the same way as instantancous velocity. Let the second point Q in Fig. 3.96 be taken closer and closer to the first point P, and let the average acceleration be computed ‘over shorter and shorter intervals of time. The instantancous acceleration at the first point is definedas the limiting value of the average acceleration when the second point is taken closer and closer to the first. nia onan soe ganar sa tet 1.A particie travels according tothe equation A+ y. The acceleration at any instant is (a)2a (b)2on4 (©20 @207 2.A particle moves according to the equation x = ar’ +e. Find instantaneous acceleration at any time f, (a)6a+28 (b) 301+ 28 (@3a0+B ox2+ (6ar+26 Read the following passage and answer the questions given atthe end. We consider a particle moving along the x-axis as in Fig. 3.97. Its distance from the origin O is described by the coordinate x, which varies with time, Ata time tthe particle is at point P, where its coordinate is x,,and at time fit is at 86. Pearson Guide to Objective Physics point Q. where its coordinates x The displacement during the time interval from fof, is the vector from P to Q; the x- ‘compenentof this vectoris (x, ~»,) and all other components arezero Its convenient to represent the quantity x,—x,, the cchange in x, by means of a notation using the Greek letter ‘(capital delta) to designate a change in any quantity. Thus we write Arex, in which Ax isnota product butis to be interpreted as a single symbol representing the change in the quantity x. Similarly, we denote the time interval from, to , as a The average velocity of the particle is defined as the ratio of the displacement Ax to the time interval Af. We Fepresent average velocity by the letter v with a bar (+) to signify average value. Thus om Ae via a = Fig. 3.97 1. A particle moves half the time of ts joumey with v ‘The rest ofthe halftime it moves with two velocities ¥, and such that half the distance it covers with ¥, and the other half with ¥, Find the net average velocity. Assume straight line motion, uy #¥At2nVs 2u( +s) @ 3+) © ahh u(Ki+¥,) 2h oa Ouran 2. A particle moves according tothe equation x= # + 31+ 4, the average velocity in the first 5 is ()76ms" (@)38ms* (a) Sms" (64ms 3. A particle moves according to the law shown in Fig. 3.90. Find the average velocity in 6 seconds. a pions) 19 @20ms' (b)12.8ms! (0) 163 ms" (18.9 ms! 4. The resistive force suffered by a motorboat is = V? ‘when the engine was shutdown, Whenthe velocity is V, find the average velocity at any time & y = hot¥ Wh wry 70,47) h mrtg Bs VF, bog, 2Y, 109, o7st OS (%-¥) Wt¥) 5. A particle moves according to Fig. 3.91. Its KE Vs ‘time graph is anon) 1) 6, ao . 0 zim 3 2 NS 25m) ©) ® Fig, 399 a +h) +20, 2H 7) se 20 =30Q)-G =60- F = tata _ 20/3+606'2 ay q 7 360. i } 29 saome! = Noms on Ho) Gent Jer = Jit A/a © ERLE % av de tay a de site er wa Sy J tog, 110 tog, Yy=log, Y= tx MY (og, Ya/V xk or di i] vw VY, log, Y* oe “Gry Read the following passage and answer the questions given at theend. Radar is used for ranging ofthe projectiles. A radar observer on the ground is watching an approaching projectile. At a certain instant, he gathers the following information. The projectile has reached maximum altitude and is moving Motion in One and Two Dimensions 87 horizontally with a speed v, the straight line distance of the projectile is. The line of sight to the projectile is an angle @ above the horizontal. D isthe distance between the observer and the point of impact of the projectile. Assume observer lies in the plane of the trajectory and the Earth is flat in that par, 1. Find Din termsof ,vand 0. cor reaching him? (Yes (c) insufficient cata to reply (b) No u'sinacoser # g o =D = heot@ = £5 cot 2F 2(c) If @< a, the angle of projection of projectile, then the projectile will fll before reaching PASSAGES Read the following passage and answer the questions given at the end. ‘A determined student waited totest the law of gravity himself and jumps off the top of CN Tower in Toronto (553 m high) and falls freely. His initial velocity is zero. A rocketeer arrives at the scene 5.0 s later and dives off the top of the tower to save him. The rocketeer leaves the tower with an al speed v,. In order to catch the student and to prevent injury to him, the rocketeer should catch the student at a sufficiently great height above the ground so that rocketeer and student slow down and arrive at the ground with zero velocity. The upward acceleration that accomplishes this is provided by the rocketeer’s jet pack, which he tums on when he catches the student, before that rocketeer isin free fall. To prevent discomfort to the student, the magnitude of acceleration should not exceed 5 g 1, What is the minimum height above the ground at which the rocketeer should catch the student? 88 Pearson Guide to Objective Physics (a) 2.1m (0)4609m or gf=10(553—A) (788m (823m 2, What must be the rocketeer’s minimum downward | or gi?= 10(553- ~ gf) speed? a (a)90.2ms" (b)75.4ms" 6g" =5530 (082.3 ms" (€)65.5ms* or 1.@)2.0) or EXPLANATION ; : ee or 553—h= 553 (- hm geh-u-9)+ 5 ett) 1 1. 2g ce Fah = l-9h ght SE Ser u-5)=5e- BE 25g of STE - 50(9.6) 125 (3) 7 ~sye-0 or un US as ist 46 2% 5g (553-h) =F Answers to Questions for Practice & (0) © @ 2) 2 @ @ 1a (0) 2. (0) 0) a © a) @ 32 (c) 3 a) © a @) “a © @ 4.) 4) ® A ) 55. (@) @) 61 (bod) 82 (0) ence 68 (acd) 8% (abd) (od) 75. (ab) (aca) @) 82 (bd) (abe) 0.09) 86. (ad) () 88.) 8. (@) @) 91) 3.) © 95. (©) 96. (0) © 98.) 100. (b) @) 102) 108) (@) 105. (0) 107, (a) (@) 108.) 110.) @ 12 @ 14) ) 46 @ 0 (e) HE) HH.) wt. (0) () 123 @) tka) 1A) 4.(b) For maximum height, the boy has to throw the stone at Greatest height for maximum range is given by = 90°, then . He H = Fy ocu=2it o 2x9.8x50=980 (ms)? = se ax98 x _ 980 Maximum range R=“ = q = 100m 2 Motion in One and Two Dimensions 89 y Berge case a a % H = uty gP=DAi-5 08)E=0 nts bs sing Using g= + , where7is time iaken to reach the highest ¥ point, we get Fig. 3.102 et for? ~ 45°, 1 goon 3S - 24se= KE By symmetry, the time taken by the ball toreach from KE,, = KEsin’45°= >. the highest point to the hands of the player is 3 s. o.totaltime=3+3=6s, 10. (b) Using R= 6.0) The other engle of projection for the same range is ore _ sin26 w walt & Se git 236 16 mn ly=3m 8.(c) Change in velocity = Fig. 3.103 some Range, R = 6+ 18=24m € _ sin26 Sas w 24 Fig. 3101 : ° Asy =xtno-— 2 = 50°50" Rene 15000 forx = 6my=3m = {5000 =70.7ms" cs £ _ sin2@ acc Sleeatiln = nee Velo, and jr = ~3y weet . time sin2e 6 3 = 61am 0-95-34 cos Using sin 262 sin@ cos 6, we get 9. (c)Let ube the velocity of projection then KE = ; mv, tan (2/3) but velocity of body at highest point is ux v sin 6, +. kinetic energy of the body at the highest point is 41, (a) HereL= JH ay) or P= HP +x equal to Differontiating, 1 i 2 mncosingy= + musi: o= Ke sn? @ a i > ? 2 (G)-o-2 F 1B. vs 1s. 16. Pearson Guide to Objective Physics a . But “>> is velocity of the mass and = Gis velocity of motorey. old, a © OE” Tee 3 Su 3 aw Integrating a= J> ude or = au 1 (@) Displacement of minutes hand = 10 em ‘Time difference between 0600 hrs to 1830 hrs is 12 hr ‘and 30 minutes i.e, 45000 < 10 «+ Average velocity = F555 =2.2% 104 ems? , tes willl a ‘ 10 re roxte( (W) Using the relation, +t _. (oF Dr+2) Su for non-uniform accelerated motion, we get +5 _, D2) S sut (There a= Br so comparing it with a= kt" we get n= 1) (©) When he takes 8 steps, the displacement is (5 ~ 3) = 2m 11 and Or 20. Time taken for 8 steps = 8 8 “ Eek geen ++ Average velocity = 3 = 5 ms In the last Sstepsthe drunkard will not be able to come backward because he would fallin the pt. +. Total displacement required =13-5= 8m 4 (b) The ime taken by jeepto covera distance of 30 km. ‘Time required 328 _ Distance 30 Speed 2% ‘Total distance covered by rocket inthis duration = speed. x time, 1 = 100 x 5 = 50km (¢) Total horizontal distance 1 be covered Total vertical distance to be covered but mx = ut... (i) 1 n= ser oli Substituting the value of from (i) in (i), ny n ax?=2a 0 5,45, = bax(242)'=80 ti) iy l Fox +2429 18a ‘Total time = 32 + time requiredto cover last 5 steps = 32 #52378, (c) The football falls on the net with constant ‘acceleration. Firsily, the net makes the acceleration drop but due to elasticity it again acclerates the football which ultimately gains constant acceleration. h 1 22, (8) Using 5 = F Qn tyand i= 5 ent (where » = total time and last moment of motion is nth second). We getn? dn 42=0 saagrs bee 1 23. (a)Using = ur > ge we get SP 12-65 =0 Solving ¢= $s or-13/5s Time cannot be negative, thus 1=5 s. (@) As the particle strikes the plane horizontally, 24, velocity is parallel to horizontal axis so its vertical velocity is given by O=nsin B-ge(v=u+ at) -y ris time of flight) or sin B cos or= 2 fsin f cos a-—cos B sin a] or 20s Bsin a= sin Bos a sin B cos sina or 2—— cosa 1 7™p (4) Since the vertical and downward distances travelled by the balls are equal () therefore their respective or tan a= tan Bor tan 28. velocities are same i. each ball has velocity = 26. (@)Time taken to each he highest pont 30 2710 + ‘ Total time taken to reach back = 5+5=108 When platform starts moving with uniform speed, the acceleration is nil, Motion of the tall and the girl wr. each other is unaffected so total time taken in this case also is 10 5. i 2h 3 at. we et, 27. (a) Using h 28. 29. 30. or 32. Motion in One and Two Dimensions 91 Let 1, be the time taken from instants of jumping to the opening of parachute, then .-&- His velocity at this point is given by, 865 v2 = 2gh 29.840 FRdor v,=28ms! For the remaining journey, v= vtar, total time=1, +/,=286+ 13 = 15.86 = 16s (2) The huge rectangular box is supposed to have an upward acceleration « so that reaction depends upon (ga) ie. no net acvelerat ‘Thecompartment now behaves as if stationary and the bullet has no net, deflecting acceleration so it will hit the zim, (©) Here 30° = 1/3564 x=3564% 1000 m 2 onisies Too =20m8 (©) Leuirbe the velocity, then time taken by the particle to reach ground is. Speed of the helicopter: rs |. (b) Considering the vertical motion, 1 32 = 5 x10xiie.1=08s ‘The particle covers 2.4 m due to velocity component v and it covers 16 m due to velocity win 0.8 s 16 ~ 08 (b) Let the driver apply brakes at ¥, then S= 2x1 om =20 ms! and 38. Pearson Guide to Objective Physics where 1 is the time taken by the driver to react to the situation Using 2a5= uF — 1? we get, ~2x2.5%5,- 0-20 (cr of retardation) From (i= = 15 20.75, rom (i)!= 35 = 59 “0-755 (©) Distance covered by first drop is 5 m. If Tis the time interval between drops then time taken by first drop 10 cover this distance is 27. 5 oS 78 QTP=2gT* oa Distance covered by second drop in time 7, poke ey From (i) and (ii) D= foery=S=125m + Distance of second drop from ground = 5-125=3.75m (@) Givens? = (01? + 28+ 9) = (oP +2p+ ded va Ze (at? +2t+y) * (ort 2p) = ae 428147)? (ot+B) 1/2 (oot + 2Br+ 9" (2oa +28) (at+ B)+ (at +2pr+ y'2(a) (oP + 2ft+ >? (art P+ or(ar + 2Br4 yy? = (08 + 2Br+ Yy27(-0F? — oF — oft att? + 20Pi+ a -— me ar 8" * (at? +2Br+yy? ~~ Thus a « x? (b) Let ube the velocity of projectile w.r.t. tank’s velocity then for xaxis, U, = uc0s30°+0 U, = usin30° 2usin 30 and = (ueos30°+ v) cos 30° usin 30° 2usin30? « Tw, 2usin 30° 7 20830? — g (e0s30°— 0) a Then, B+ 8,= “7 (sin30€08 30) R-R 4u -R, = — vsin30 & Eliminating u, we get ge (R-Ry Y= Fan30 (+R) (250-200)! (250+200) Oo 36. Fig. 3.104 1! Usings= 5 v(i,+1) [Displacement = 1 er 1200 = 5 u(u+ 3) 2u? or 1200 = or = 3 gg0 2 or v = 424 Ath at 37. (a) Let the retardation produced by instantaneous opposition = au (where cris a constant) ‘Net instantaneous acceleration ~ ¢~ av dv ie. F = G@-av) In jev = F-e9 Se hey (i-ey . £ jeds = 2 (-eya a Integrating, [aS = =f faBfoma ofl or Ss £ @ Motion in One and Two Dimensions 93 38. (b) Velocity before strike w= JIgh ‘Component of acceleration along the inclined plane=g sin Gand the perpendicular component = g cos a 1 Saute det Using $= r+ Sar for vertical direction we get, 1 O= veos ar 5 gos at? and for horizontaldirection, 1 weusinat 5 gina? wt aa Py = 2 pein 24) (ore 2% sin 25 gsina(™) (0 - sing , 2using _ 4u°sing g g g 2ghxsino nay ee w sina 39. (@) Using H= 2 2u' sinwcosar g and oR wwe get? = sin? acost a E 8 Eliminating o, we get [toe-2er] g& d 8x25 a2 = [S250 -20.8025] 40. (d)Let1, and 4, be the times of shot from cannon at 60° and the shot from horizontal cannon to reach the point of collision 94 Pearson Guideto ObjectivePhysies and 41. and or or a2. Fig. 3.105 sx = (55 cos 60) Bt y = 10453 sin601,—> gn? 1 = 10-5882 (c) Loty, be the distance covered by the fan and y, be the disiane: covered by lift just before the fan reaches the floor of the lift, 3 (initial velocity of fan is upward and acceleration is downward) “rare gia = 241+ 068 = 1.68+2.68= Lm 1 (2) Using s= 11+ 5 ar we get 200 = uQ+ tae oud , distance covered is 220+ 200 = hms 2A =n O)+ 5 a(6F (i Solving (i) and (ii) and 43. and and 43. 46. a = 15cm u = 1Sems" + Velocity after 75= 1+ at = LI9-15x7=10ems* (Using, v=u ar, we get, For retarded motion, 0 = v-B2 ore wp ‘Total time “lt (& a+B ): 0s aot and velocity (©) Given, w=, F + (aeocos ox) I Thus velocity along y axis, along x axis, uv =n, Displacement at time #in horizontal direction, de x = fat =agt (2 v4] y = foweoneor dia sin at Eliminating 1, ya sin (ax/u,) Attime 32a, x~u, (3020) wees y = asin ‘Thus distance of particle from origin s = PS] oe (0 e-F) (b) Maximum acceleration during journey = slope of co DG DG 60-20 40° oG 55 7 pgs 7 160km hr? Distance covered in the asked interval = Area of rectangle KCGH + Area of A CDG, (025 x20) + + (40% 025) *5=10km (d)Heree~ Ye +3 0r Ve -1-3 x = (3P= 66149 or 41. or 49. or 50, a dt 1 =3s 3 = Ve +30rx=0 (@) Let 1, be the time before opening of parachute 1 Using h = ut> gf, we get 196 =0+4 98x12 Bs. = = yp nd orn.=28 Taking v, as velocity attained aftcr falling through 19.6 mand using v—12=2gh, we have v2-0=2x9.8x 19.6 Againtaking f,as time takenafter opening of parachute and using v= w+ af, we get 46 = 19.6-1x0, 1 7196-4, Ss s.total time =3, 4 ¢,=24 1 (b) Speed of boat v= 5 knvh 1 Tq 74 kmh Speed of boat in lowing water ¥,= 77 = Speedoof the river u, = (Fog =3kmb! (6) Using 5,=u+ 5 20 1 we get a sur Z el) “ [d=uniform acceleration) and and si. 32. From(iv)ay= day | Subtracting, o (y~2) =a Similarly, (39) = a (z—yx)+ ( ee y) = dtexnye Adding above 3 equations aQy-2)+b(@-x)t+e0—))= af (yar yey te ( a’ wee 2 ertroxtany)=0 (b) To avoid collision, the faster train should come to rest aflercoveringa distance d. Using v= 12 = 20S we get 0-F2 = 2tad (where V, vy) = wwe 2a a Collision can be avoided if {a) Using h =r + ar, we get WH = 330%) 5 x98X1 1 = (330-49)m. 96 53. For For 58. Pearson Guide to Objective Physics (4) Initial distance between sny two persons = x L Ny M Fig. 3.106 Considering KL, velocity component of Lalong LK, ‘vcos 9° =0 ‘Thus speed between K and Lis v a ctimer= = v |. (a) Lets, t,and , be the times to cover AB, BCand CD AB, h 1 AC IH = 5 tet? 1 2 AD.3h = 5 ott, +4, 41 tA tate) ea ERED) Thus 4, (8-1): 6-8 (a) When the car is exactly opposite, the man at least distance, the bag will reach him when thrown with velocity Y,. sin @ a = 45 Fig. 3.107 56. 51. or or or 65. 66. or or or 6. (8) Using v,-u, =: whenball is dropped, #* = 2gh (u=0) ‘The variation is parabolic. Ath=0, velocity ismaximum, Now direction of velocity is reversed and goes on decreasing such that it becomes zero at di2 Superimpose on the observer and the two objects & velocity equal and opposite to the velocity of the observer. The system now reduces to what is seen by the observer. ay oe v= O forse, [¥ =0 asit has acceteration only in the y-direction.) At any instantof time, let the length of the string BP = {and the length P=, Ina further time ¢, let move to theright by x and A move down by y, while P moves totheright by w. As the length of the string must remain constant, Ath =(j-xtuy+@ty) x =uty Bosuty. peed of B to the right = v,, > = downward speed Also, P= Y, Let the lengths of the sections of the string be BP =/, and PA ~I, inthe position shown, Let B move through a small horizontal distance x to BY and 4 move down through a distance y to 4’. Length of the string Fig. 3.108 or ¥, 69, Projectiles with the same initial speed have the same horizontal range for complimentary angles of projection, Here, 8, =90~ 8, = vy, 008 O= u. nD. As the block does not move, the ball moves along a circular path of radius J. The centre of mass of the system always lies somewhere on the string. RB. 4 5 Let v= speed of ball when the string makes an angle @ ‘with the horizontal. 1 ‘mut — mgl sin. 1 ‘The horizontal component of y= =v sin @= sin 6 2gisind av For F to be maximum, Ge 7 Or Which gives sin 3el Superimpase an upward acceleration «on the system, ‘The box becomes stationary. The particle has an upward acceleration a and a downward acceleration g Ifa=g, the particle has no acceleration and will hit C. Ifa> g, the particle has a net upward acceleration, and ifa=g, the particle has a net downward acceleration Consider the vertical motion of the particle after centering the compartment. Let itreach the floor intime (oF or 08S: Due to the velocity component », which remains constant, it covers a distance of 2.4 m in 0.8 s, For two shells to collide in ait, they must reaeh the same point in the same time. First shell is fired at ox angle and has a longer time of fight ¢. The second fixed a has less time of flight as OF ot oof aS tel Nero etc, cannot be used]. Alsonotey=ra, = a,=ra and eaten ce dt at 3. Since a, and a, are at right angles to one another. Therefore a, i$ given by 4 oq = Yap raj and tan B= 7 wnt [See Fig 43] 4. Ifa,=00re=0,nowork is donein circular motion ic., in uniform circular motion work done is zero. Note that centripetal force and displacement are perpendicular to one another. Therefore, centripetal farce doesno work. However, work will bedone if, #0 ora 0 since tangential force actsin the direction of displacement 5. Ifthe particle moves from A to Bin uniform eitcular motion with a velovity vthen change in velocity Av zy si S| =2vsin ($ ] and avenge necelerion 0) airsin( 2 = ee 100 Pearson Guide to Objective Physics lA NS Fig. 46 6 u 10. 1. 12. & my ; a =m oF Centripetal force Radius of curvature of a projectile atany instant R= Ia road is banked (the angle of bankin, ‘maximum speed a vehicle shall have while taking atumistan@ =" v A cyclist will bend at @= tant i ‘while taking tum, If friction is also present then on a banked rosd ‘maximum and minimum velocity is determined as yo a ee Pe 1 ptan® y= |zlene=w) | Te utand In vertical cireularmotion, minimum veleeity at any point P is Vax. = Sng 2rgcosO . Minimum ‘velocity at the highest point is v,,..., > 7g and. ‘minimum velocity at the Bottom Vag saa = VTE + ‘In vertical cireular motion T,,,..~T,, = 6 mg. Ifthe velocity at the top ofa vertical circle is known, then velocity at the bottom is given by Yawn = Yon +2 27) 8: Ifthe body comes down a distance h then %e Veg + 28h, AF the velocity at the bottom of a vertical circle is given, then velocity at any point P (say at a height ‘from the bottom) is given by YF = mee —2 gh 15. 16. 2. Inthis way velocity atthetop wll be vi, Agr. Tension at any point 7= as + mg cos 8. If the pariicleis not tied to the string, then tensionmay be Tead as action or reaction. The difference between rotational motion and circularmotion is that in rotational motion different particles of the body move in different radii about 4 fixed axis within or outside the body. In circular ‘motion all the perticles move in the same radius Ifthe distance between the wheels (side wheels) is 4, and centre of mass of the truckivehicle is at a height f then the maximum speed it can have without overtuming is v= eke Considering no acceleration acts in a uniform circular motion as v and w remain constant, Radial or centripetal acceleration is required for v changing direction continuously “= ra? is the radial acceleration acting in uniform circular motion. Applying equations like v = u + af, s= ut+ at ete. In horizontal circular motion v=u-+ af ete. cannot be epplied. Instead apply @,1+ Yat and @-@2+2 00. Assuming work mustbe done in circular motion as force isacting, In uniform circular motion, no work is doneas force ‘and displacement are mutually perpendicular, However, work will be done innon-uniform circular motion. Assuming minimum velocity to be given to the particle to complete vertical circleis rg . ‘Minimum velocity to be given to the particle to complete vertical circle is JSrg_at the bottom. Considering change in velocity in uniform circular ‘motion is zero, ‘Change in speeds zero. Ifthe particle moves by 8 8 then Av=2 sin 3) Assuming velocity at any point will be given by the equation v= /3rg+2recos0 = 0, at; = This isthe minimum velocity at any point. If velocity area is other than minimum, then use Vay pie = Y2q +2 gh See Fig. 4.7 (a). 70080 Weypun = Ney +2.gr(I 08 8) TE ggg 18 known the Vy ig From fig. 4.7 (b) Vie 2 Eh Circular Motion 101 Veyrint = Vem ~2.87 (1-008 6) If a car is accelerating on a circular road with acclerationa then assumingais the net acceleration. Since 2 also acts. Therefore, net acceleration at any instant will be a,, = rary Considering like horizontal circle tension in vertical circteis aso in a or r In vertical circle T= 2“ +¢mgeos@ where v is velocity at that point. % ‘A voluntecris rotated in a horizontal circle of radius 7 1m. Findthe period of rotation for which the acceleration is equal to 3g 2618 (©€)3.07s (2.875 (331s my an? oa-r(¥) =3¢ or7 | Fr |=3g or =3075. A ferris wheel with radius 14 m is turning about a horizontal axis through its centre. The linear speed of | the passenger onthe rim is 7 ms". Find the acceleration | of passenger at the highest point. (@)6.3 ms*downwards (6) 3.5 ms? upwards (©)13.3ms* upward (¢) none mw Fig. 48 | 4. Agiven shapedglas tube having uniform area of eross- section is filled with water and is mounted on a rotatable shaft as shown in fig. If the tube is rotated with a constant angular velocity @, then .S ms upward ‘An annular ring with inner and outer radii R, end R, is rolling without slipping with a uniform anguler speed. The ratio ofthe forces experienced by the two situated oon the inner and outer parts of the ring is ay x OR Oz & 1 OE @h- mek Fl mRo* RR, [AIMS 2005) 102 Pearson Guide to Objective Physics 21> Fig. 4.9 (@)_waterlevel in 4 will iseand fall in B (b) waterlevel in both 4 and B will rise (©) waterlevel in B will riseand will fallin A (qd) waterlevel remains same in both 4 and B (b) extra force F= "and extra pressure oi a mre A 24 Increases the level in the two. 5. A pendulum was kept horizontal and released. Find the acceleration of the pendulum when it makes an angle @ with the vertical. Fig. 4.10 (@)8 Vi+3c0s'@ (b)g Jis3sin7@ ()gsin @ (@) 2g c0s 6 ey =mg ios Bor “> = 2g cos 8 and = fata I(gsin@) +(2gcos0) 8 Vi+3cos°@ gsin® tang ng = = 8 * 2 7 6. A circular track of radius 100 m is designed for an average speed 54 km/h, Find the angie of banking. j(: fe (a) tar 0 (b) tan wo 3 (c) tant (3) (d) none of these 1Sx1S, 9 (8) a9 OF Faget aN" f® owe [3 7. A fighter plane is pulling out fora dive at 900kav> in a vertical citcle of radius? km. Its mass is $000 kg. Find the force exerted hy the airon it at the lowest point. (@) 2.0625 10'N upward (0) 2.0625 x10 N downward (©) 2.0625 x10°N upward (8) 2.0625 10'N downward 5x10? x(250)° © AS + mg= ig tS x 10h 2.0625 x 10* N upward. 8. A patticle of mass m rotates in a circle of radius a with ‘uniform angular speed o» It is viewed from a frame rotating about z-axis with @ The centrifugal force on the particles is @) mara (b) m aia om( 5) ()m ow,a o) 9. A motorcyclist is going on an overbridge of radius R. ‘The driver drives with a constant speed. As the motor cycleis ascending on the overbridge, the normal force on it (@) increases (b) decreases (©) remains unchanged (4) fluctuates @ 10, A particles going ina spiral path as shown in Fig. 4.11 with constant speed. Then \ Fig. 4.11 is constant (b)_ its acceleration is constant (©) magnitude of its acceleration is constant. (@) The magnitude of acceleration decreases continuously i) tc) A. A person applies a constant force # on a particle of mass m and finds that the particle moves ina circle of radius r with a uniform speed ¥ when seea from an inertial frame of reference. (a) This is not feasible (b) Some other forces whose resultant varies in ‘magnitude and direction also act on the particle (c) The resultant of other forces is me ‘towards the centre (qd) none of these OO) 12, Findthe tension inthe pendulum a the extreme position if amplitude is 8, oe (+ mg cos 6 {c) mg cos 4, (@)™ ~ng 00s 6, (o) At the extreme position v= 0; therefore ig COS 6, 13, Findthe radius of curvature of aprojectile at the highest point, fired with a velocity v, making an angle 6 with the horizontal. v' sin? ©, vy cos? © Oe Sm «x 14, A track consists of two circular points ABC and CDE of equal radius 100 m and joined smoothly as shown. Each pant subtends a right angle at its centre. A cyclist weighing 100 kg together with the cycle travels at a constant speed 1 km hon the track. Find the normal force between the road and the cycle just before and Just after the eycle crosses C. (cos)? g Circular Motion 103 B 7 x . a, 5 Fig. 4.12 (2) 682,682N (b)732, 682 N (€)732,732N (6682, 722N (@) just before crossing C N= mg.cos 4s — = 100x 10x .707 just after crossing CN =mg eos 45 + 707+25 = 132N 15. Aninclined plane ends into a vertical loop of radius R. A particle is released from a height 3R. Can it loop the loop? (ayes (by no (o)cannot say (@yesif friction is present (a) To complete the loop minimum height be 5 z R 16. A particle has velocity rg. atthe highest point in a vetical circle Find the ratio of tension atthe highest and lowest point. fa li6 (by 1:4 (e)1:3 (1:2 @)7,,= = —mg= 2 mg; ee ae) Img + 6mg 17. A car is moving on a circular track of radius R. The road is banked at 6. js the coefficient of friction. Find the maximum speed the ear can have. © oe stant cos +p1sin® ty | BelcosO+ using) ys © cos8 ~usind Rg(sin + 40058) )* © cose -psine (@)_ none 104 Pearson Guide to Objective Physics % ov, - | Rettane+m) ]* (2) 0.09 (b)0.1 (Max = [ T-atané (0.2 (@) none of these _ [ Re(sind + cos) J* (mr oF = w mg 030 = sin 19. A Lgcoinis placed on an L.P. record 10 cm from the axis of rotation. The coin is not displaced. The minimum m= value of coefficient of friction is___ifrotation speed 1 aie 8335 am. a TYPICAL PROBLEMS 20. A table smooth horizontal surface is rotating at a speed about its axis. A groove is made on the surface along a radius anda particle is gently placed inside the groove ata distance a from the centre. Find the speed of the particle with respect to the table as its distance from the centre becomes /. (a)v= ol (b)v= @(/-a) o(!+2) (ove 2 () OF dv dk apt vdv = wxde a. [os = feat Fig. 4.413 ‘A car moves on a horizontal track of radius , the speed wv increasing constantly at rate > = a. The coefficient of friction between road and tyre is 1. Find the speed st which the car will skid. (@) (Grg?+ ty)" ©) ner ()(Weg+ e771" @ Jar (€)oet sceleaton is E Se = Eilemcemeneen, 22. Two strings are tied 2m apart on arod and on the other end a mass 200 gis tied as shown in Fig. 4.14 (a). Each string is 1.25 m long, Find the tensions 7, and 7, if the rod is rotated with 60 rpm. 125% 2m i Fig. 4.14 (a) (@)6N,3N (6) 6.25N,3.75N (©)4.25N, 5.75N @S25N,4.75N (b) Resolve 7, and 7, Look into Fig. 4.14 (b) carefully T, cos 6+ T, cos @=mra2 T, sin 8 T, sin 8+ mg 3) or (7,+7,)(0.)=0.2 (3) (2a)? or 7,+T,=10 0.210 ap 725. J mre 7308 Fig. 4.14 (b) solving 7, = 6.25 Nand 7, =3.75.V 23. A car is moving oa a horizontal circular road of radius 100 m with a uniform speed 10 ms". It suddenly accelerates at 1 ms™, The acceleration is. (a) ims? (b) V2 ms* (@) JS ms* (c)2ms? b) 24, A chain of mass m, radius r (mass uniformly distributed) is kept on a cone, If the chain moves by an angular velocity @, then find the tension in the chain due to rotation. Fig. 4.18 (a) Consider a small element of the chain ‘Let 7'be the tension them Tsin d@ + T sin d0= Amro ue = md8, am= nr (rae = © do 27 sin d@ = = ra} ga ae Tr sin d0+ T'sin dO Fig. 15 (b) Circular Motion 105 2ra0 = ap 7 [sind = d@asd0is small] nro? on or T= If total tension including weight is taken into account suppose the chain slips with half angle a AN ssin a= gdm wD) 27 sin dO— dN cos c= dm or nd) on a6 solving (1) and @) m(o'r+ goot8) ln avin ac a cn Fig. 4.15 (c) 25, A particle isprojected with a velocity w making an angle 8 with the horizontal. What isthe radius of curvature of the parabola where the particle makes an angle 2 with the horizontal? v,=ucos 8; v,= usin @- gi usind or wsin @—g/ = ucos tan (82) = ecoso (I+ an? 2) - Se 2 * cos'(072) ; 4? cos" radius of curvature r= 7 = cos? (6/2)° 26. A massm isreleased from the top of a vertical circular track of radius r with « horizontal speed v,, Find angle @ where it leaves the contact with circular track. Vs yg + 2gh= vf +2gr(1—c0s 6) 2 gaa Te = Me + amg (1—cos ror or 2. os or or or uide to Objective Physics Condition of leaving mgcos 8 img cos 8 = “2 + Ame (1 ~c08 8) Fig. 4.16 Assmall object slides without friction from theheight H =50 em and then loops the vertical loop of radius r=20 ‘em from which a symmetrical section of angle 2 «has been removed. Find angle oF such that after losing contact at 4 and flying through air, the object witl reach point B. mg [2.5r—r(1 + cos a); h=25r—r(1 +605 a) Fig. 4.17 = gr(3—2eos @) vj sin 2a Range = AB=2rsina= or v3 cos = gr Seg 7 BF 2eos a] 2eos* a —3eos a +1=0 (cos 1) 2cos 1) = 0 oF cos cosa = 1/2 a = Oora=60° ast 0.0160, 28. A chain of length / is placed on « smooth spherical surface of radius r with one of its ends fixed atthe top of the surface. Length of chain is assumed to he /< © Acceleration of each element ofchain when upper endis released is fa r wif) kg ! Bly sind © ®( (b) Consider a small element al! making an angle dO dn = 7 t= force acting on the element AF ~ (dmg sin d= "sin 00 mre po p= E [sinoco 1 where total angle a= m 1 Pet re(l ~ cos) = rg [1~c084 «a= (trent - 29. A pipe of length { contains a liquid of density Area of cross-section of the pipe is A. It is rotated about one end with an angular velocity @ after sealing both the ends. Find the force acting on the liquid a Fig. 4.19 Consider a small element dk at a distance x 32, A tube AC forms a quarter circle in 2 vertical plane. from the axis of rotation. Mass of liquid in the element ‘The ball B has on area of cross-section slightly smaller d= p Ads :dF= dmx oo than that of the tube and can move without friction through it, Balls placed at 4 and is slightly displaced. The bal is 30, A spotlight is fixed 4 m fiom the vertical wall and is rotating at a rate | rads '. The spot moves horizontally ‘on the wall. Find the speed ofthe spot on the wall when the spotlight makes an angle of 43° with the wall i (a)4ms" (b)6 ms" i (co) Sms" (d) none of these Fig. 822 (©)sis the spotlight shown in Fig. 420.and.x ‘e is the distance moved by spot in time 1. (a) alwaysin contact with the inner wall (b)_ always in contact with the outer wall (€)_ initially in contact with the inner wall and at some point i comes in contact with the outer wall (4) initially in contact with the outer wall and finally comesin contact with the inner wall. 2 Solutio At the break-off = Fit (6) Atthe break-offpointcos 8= 5 , itcomes in contact with outer wall 33. A multrack is banked for speed v by making the height of the outer rail 4 higher than that of the inner rail. The distance between the rails is d. The radius of curvature Fig. 420 From Fig. 4.20 © =tang x = ytan@ of the tracks , then yp Bo cg % sng @ yey h ah v= Gi mysectg SP and TP =o 4 (by tan [sin v= 42) 10when 8-45", 9= 45% “Susi a = Sms (ota ws 3M. A coca-cola bottle is whisled in @ horizontal circle of radius ras shown in Fig. 4.21 Where will the gas @ stay. (34, When a ceiling fan is switched off, its velocity falls to half in 36 rotations. How many more rotations it will rake? (a)36 (b)24 NB (18 (12 raz (Ser 2-208 oF en; ( (a) at thonect (b)in the mide (c) at the bottom |d)anywhere ‘ Om or = 2a36r0). ene) Ge = c) F= mr oF the smaller the mass, the more is The radius to maintain the same force Note:This is the principle of cream separator from milk, = 2a(erot) 2) ‘centtifuge michince used in Jaborutoriec and even the 4 Earth is bull inthe same way. Heaviest masses closet ——_Solving(I)and Q).¢= 12 the axis and the lightest at the surface. 108 Pearson Guide to Objective Physics 38. A paricle P ofmass m attached toa vertical axis by two AY, ay; strings AP and BP of length / each. The separation AB (@) [AVael ~ V5 [Pa] 1. The point? rotates around the axis with an angular | SFele6 GE velocity ca The tensions in the two strings are T, and ©),(@) [AY | =2 Vsin 30 [AV 1, = 2¥'sin60 2 [Wal = MV 37._A particle moves along a vertical circle of radius rwith avelocity ffrg at YIT,, Ty Ty T, represent tension at A,B, Xand ¥, respectively, then ig. 4.23(a) @T=T, ()T,+T=marl ‘ (T7,-T,-2mg (a) BP will remain Fig. 425 (), (0, (@ (T,- 1) c0s 60= mg or T,-T, = 2mg (7, +7; sin 60~m o# sin 60 @7,=1, ()T,-T,-6me or T,+7, = mel (©T,-T,-6mg ()7,>T,26mg. 1 eo 1, = E mle! 2¢}for 7.20.02 fig] 11, 38. A riag moves in a horizontal circle of radius r with a velocity a in free space the tension is (2 mr oF (b) mer of ra? mo os Oa Fig. 4.23 (b) 36. Aparticle moves from 4 to B in uniform circular motion with a velocity @. Fig. 4.24 wo e0ne Em «7 ener: (a) The average acceleration during 4 to B = Avg lar acceleration during 4 10 X. Otobe small. ¢b) © mre Read the following passageand answer the questions given attheend, ‘Ifthe speed of the particle ina circle varies, it is a, example of non-uniform circular motion. A roller coaster car that slows. down and speeds up as it moves arcund vertical loop. The radial component of acceleration a, = fa which is perpendicular to the instantaneous velocity still acts and is directed towards the centre of the circle. Since speed has different values at different points in the motion, the values of a_,, is not constant. Radial acceleration is greatest at the point in the circle where the speed is greatest. In non-uniform circular motion there is also acomponent of the acceleration that is paraliel to the instantaneous velocity, This is the component a, or a, ie. tangent to the circle, 4... i8 equal to rate of change of speed, Thus a= ~ ab at 1. Find the angle that the resultant acceleration makes and with the tangential acceleration. Fig. 4.26 (on a= = A jeep is moving along a circular road of radius R with a speed v, A pendulum is hanging from the ceiling of the jeep. If the jeep accelerates, how does angle, the pendulum makes with the vertical vary? (a) Ifjeep accelerates angle decreases (b) If jeep accelerates angle decreases (©) angle remains constant whether or not jeep accelerates (@)_ none of these increases, tan (b) Angle will inerease. As v increases, Re Circular Motion 109 3. In vertical circle, can the motion be uniform circular motion? (Yes (®)No (©) Sometimes (None of these EINEM () No. as due to gravity the velocity will change. Read the following passage and answer the questions given at the end, ‘When vehicles go around turnings, they travel along anearly circular arc. There must be some force which will produce the required acceleration. Ifthe vehicle goes around a circular path, the resultant force is horizontal. Consider a vehicle of ‘mass m moving with velocity vmaking atur on cieular path of radiusr. If the road is horizontal, the normal force is vertically upward and the only horizontal force that can act towards the centre isthe fiction F This is static friction and is self adjustable. The tyres get a tendency to skid outward and the frictional force which opposes this skidding acts towards the centre, Friction is not always reliable at circular tums particularly if itis speed is high and sharp circular tums are involved. To avoid the dependence on friction, the roads are banked at the turns so that the outer part of the road is somewhat lifted up as compared to inner part. 1, You normally nete the speed limits mentioned on high ways (on straight roads without turns) why? (a) The road law forces to write the speed (b)_Itis just a fashion (c) On horizontal road speed limit has no meaning (qd) Whill overtaking one encounters sharp turns (d) When a vehicle overtakes the other it has to ake a turn which is quite sharp. Hence speed limits arementioned. 2. If the road is banked, then there should be a range of velocities which is permissible. Write the permissible velocities. Fein «) Fig. 4.27 Assume vehicle has V sin @ + F, cos 8 as tendency to move up. 110 Pearson Guide to Objective Physics ot |. Find the averageacceleration between points 4 and Nain 0+ Freoa8 “(ly Baan ala epee o mg = Neos @+F.sin@ (2) (a) nearly 240 ms*radially inward F= uN -Q) (b) nearly 240 ms*radially outward From(1),(2)and (3) (c) nearly 6.28 ms? radially inward oe - (@e4) (@)_ nearly 240 ms along AB, mn & ” 2. Will the average acceleration be same if the bikes _ tan +n \} moves in a vertical circle with uniform speed? Thus vat | e (a) Yes, because speed is constant i” . and - [=(24] ; (b) No.as g affects fey ne (©) No, as biker cannot keep speed constant Whatis the order ofcoetfcienofstatc ston betweer (4) None ofthese tyre and road? ‘ @>1 <1 3. Find the change in velocity from xo yin vertical circle assuming biker completes the circle in 3.14 @<.5 (a) 25.11 ms" (b) 32.5 ms" (©) 21.75 ms" (@) 27.61 ms* Read the following passage and answer the questions given attheend. Ee Ina circus show a Joker is riding on a motorbikeiin aspherical shell at constant speed v in a horizontal circle along the longest diameter. Your physics teacher is sitting along with you. You notice that the motorcyclist completes one rotation in6.28 s. The radius of the sphere is 6.28 m. You suddenly tum to your teacher and ask him how could the bikes move with a constant speed? The teacher tells you the biker is continually accelerating. 12x3.14x6.28 2 3. ave 2vsin = 180°) QUESTION FOR PRACTICE 1. A body ismoving with uniform speed von a horizontal — circle from 4 as shown in the Fig, 4.29. Change in N velocity in the first quarter revolution is aw e uv (b) V2 v southwest c (©) V2 northwest (@)2vwest, Is Fig. 4.29 2. Two particles A and B are moving on two different concentric circles with different velocities v, and v, then angular velocity of B relative to Aas observed by Aisgiven by: v,-v, », @ 7 (b) r, sv +0, © on Fig. 430 3. Asscooteristis approaching a circular tun of radius 80 m, He reduced his speed from 27 kmb'at constant rate of 0.5 ms*. His vectoraccleleration on the circular tum is (a) 086 ms*, 54° (c) L0ms?, 45° (b)0.68ms* 45” (d)0.5 ms? 45° Fig. 4.34 4. A particle movesina circle of radius 4 em clockwise at are unit constant speed of 2ems!.If ¢ and § acceleration vectors along x and y axes, respectively. the acceleration of the particle atthe instant half way between PQ is given by (b) 4+ 8) Fig. 432 5. Circular Motion 111 circular track of radius 300 m is banked at an angle $ radian. ifthe coefficient of friction between wheel ofa vehicle and road is 0.2, the maximum safe speed of vehicle is (a)28ms! (b)38ms! ()18ms! (@)48ms" ‘A small block slides with velocity 0.5 gr on the horizontal frictionless surface as shown in Fig, 4.33. The block leaves the surface at L. The angle cis Fig. 4.33 3 jer: (ayeos! 3 (b) cos ¢ 13 (sin Z @ ‘A mass attached to a string of length L is revolved in a vertical circle with least velocity. The siring breaks when the mass ceaches at the highest point. The mass describes a parabolic path, then range of body ws.t. the lowest point is, (al? (ye (c)2L @ iL Apparticle slides from rest from the highest point of a vertical circle of radius r, along asmooth chord. Time of descent of the particle along the chord is Par fir Oye @ fa ‘A pendulum is vibrating with an amplitude of 5 radian. ‘Value of ecfor which resultant accleration of the bob directs along ths horizontal is 412 Pearson Gulde to Objective Physics () sin! 1/ ) cos" If 13, A jeep rans around the curve of radius 0.3 km at a ee ©) 3 constant speed of 60 ms". The resultant change in ig B velocity, instantaneous acceleration and average (©) cos! @ sin acceleration over 60° arc are 10, A small block is placed at the top of a sphere. It slides, on the smooth surface of the sphere. The angle made by the radius vector of the block with the horizontal ‘when the block leaves the sphere is, < Fig. 4.38 (@)30ms*, 11.5ms?, 12ms? (©) 60ms", 12 ms?,11.5ms* (©) 60ms*, 11.5 ms? 12ms* (@) 40ms", 10ms*,8 ms? Fig. 4.38 14.4 Gall 'ni6ve¥ along a wast Laxton road with (ap24? (b) 36° fixed speed at all points on the road. The normal. jar @is reaction of the road on the body at two different points Aad Bis 11, Asmall ball descrites a horizontal circle on thesmooth j inner surface ofa cone. Speed of the ball at a height of gz e 0.1 mabore the vertex is [ (a)4ms" (b) 3s" ()2.ms (@) ms" 7 @ Fig. 439 (a) maximum at A (b)maximum at B (c) minimum at B (@) same at 4 and B 15, A hollow vertical drum of radius rand height Hfhas a small particle in contact with smooth inner surface of the upper rim at point P. The particle is given a horizontal speed u tangential to the rim. It leaves the Fig. 4.36 lower rim at Q vertically below P. Taking n as an 12, Two balls P and Qare at opposite ends ofthe diameter ofa frictiontesshorizontal circular groove. ?is projected along the groove and at the end of T second, it strikes, Sin Der ball Q. Let difference in their final velocities be (n= oP ©) Baie proportional to the initial velocity of ball P and ‘coefficient of proportionally is e then second strike lm integer for number of revolutions, we get 2nr Z ‘occurs at n= pale (@)n= 55, ATs 2) 2} Fig. 437 (a)2Te (year (e2er (@T26. 16, A particle moves down the inclined surface and ‘completes a vertical circleat the foot of the plane. Ratio of heights H and h is Fig. 444 @s (4 2 @3. 17. The radius of front wheel of arrangement shown in Fig. 4.42 is aandthat of rear whee! is b. Ifa dust particle driven from the highest point of rear wheel alights on the highest point of front wheel, the velocity of arrangement is (c+a~bye-a+5)]" H-a) © ls lng — ’ ; Fig. 442 18, There are n small elastic balls placed atrest on a smooth, horizontal surface which is circular of radius & at other } t end. The masses of the bells are M, 5°, 53 ze M rt: espestivey. The least velocity which should be provided to the first ball of mass Mf such that nth ball completes vertical circle is mh, © 9000 Fig. 4.43, Circular Motion 113 19, Amotorcaris travellingat 30 msona circular road of radius 500m. ILis increasing speed atthe rate of 2 ms 2, The acceleration of car is (@)2ms* (b)2.7 ms? (3 ms? (3.7ms? 20. A string of length Im is fixed at one end and carries a mass of 100 g at the other end. The string makes 2/t revolutions per second around the verical axis through the fixed end. Ifangle of inclination of the string with the vertical is cos"'5/8, the linear velocity ofthe mass isnearly (a) mst (0) 2ms! (@)3 ms" (@4ms* 21, Acar'is moving on a circularhorizontal track of radius 10m with a constant speed of 10ms '. A plumb bob is, suspended from the roof of the car by a light rigid rod of length 10 m. The angle made by the rod with the track is (a) zero (0) 30° (ase aor 22, Aparticle Ps sliding down africtionless hemispherical bowl, It passes the point 4 and ¢=0. At this instant of time, the horizontal component of its velocity is v. A bead 0 of the same mass as P is ejected from A at along the horizontal string 4B with speed v. Friction between the bead and the string may be neglected. Let t,and f, be the respective times taken by P and @ to reach point B, then 2 4 2 . Fig. 4.44 (4) ag (@)_ The foree of friction makes an angle tar"'(v'/ay) with the direction of motion at the point of slipping. 42. The density ofa rod AB increases linearly from 4 to B. lts midpoint is O and its centre of mass is at C. Four axes pass through 4, B, O and C, all perpendicular to the length of the rod. The moments of inertia of the rod about these axes are J, Jy, J, and J, respectively. > Bh< Olle @Mlo7, 727, (co) T=T, (4) The relation between 7, and 7, depends on whether the rod rotates clockwise or anticlockwise. $1. A simple pendulum having a bob of mass m is suspended fiom the ceiling off car used ina stunt film shooting. The car moves up along an inclined cliffata speed vand makes a jump to leave the cliff and lands some distance. Let Rbe the maximum height of the car from the top of the cliff. The tension inthe string when thecaris inairis (ng ()me~ (omg (ze 52, Let 6 denote the angular displacement of a simple pendulum oscillating in a vertical plane. If the mass of the bob is m, then tension in the string is mg cos® (a) always (b) never (©) atthe extreme position (@)__at the mean position. 53, An object follows a curved path. The following. quantities may remain constant during the motion. (a) speed (b) velocity (©) acceleration (d) magnitude of acceleration, 4, Assume that the Earth goes round the tun in a circular orbit with a constant speed of 30 kms (a) The average velocity of the earth from Ist Jan, 90, to 30th June, 90 is zero. (b) The average acceleration during the above period is 60 kmvs (c) The average speed from Ist Jan, 90 to 31st Dee, 90 is zero. ()_ The instantaneous acceleration of the earth points towards the sun. ‘55. The position vector of a particle in a circular motion about the origin sweeps out equal area in equal time. is (a) velocity remains constant (b) speed remains constant (©) acceleration remains constant (@) tangential acceleration remains constant. 56. A particle P is going in a sprial path as shown in Fig. 4.50 with constant speed. — iA ig. 4.50 (a) The velocity of the particle is constant. (b)_ The acceleration of the particle is constant (©) The magnitude of acceleration is constant. (@) The magnitude of acceleration is decreasing continuously, Circular Motion 117 A.carofmass Mis moving on a horizontal circular path of radius r. Atan instant, its speed is vand is increasing ata rate a. (a) The acceleration of the car is towards the centre of the path, (b) ‘The magnitude of the frictional force on the car is greaterthan (©). The friction coefficient between the ground and the car is not less than a/g. (@) The fiction coefficient between the ground and the car ise tan" ‘58, A circularroad of radius r is banked for aspeed v= 40 km/hr. A car of mass m attempts to go on the circular road. The friction coefficient between the tyre and the road is negligible. (a) ‘The car cannot make a turn without skidding (b) iP the car tums at a speed less than 40 krnv/r, it will slip down (©) if the car turns at the correct speed of 40 knv/hr, th fre bythe codon he carin equal to (@)_ Ifthe car turns at the correct speed of 40 knvhr, the force by the road on the car is greaterthan mg ax walla grater han 2 59. A person applies a constant force F ona particle of ‘mass m and finds that the particle moves in a circle of | radius * with a uniform speed vas seen from an inertial frame of reference. (a) This is not possible, (b) There are no other forces on the panicle. (©) ‘Theresa of te other frenia wards a sai (@) The resultant of the other forces varies in magnitude as well as in direction, 60. A panicle is moving along a circular path with uni- form speed. Through what angle docs its angular ve- locity change when it completes half of the circular path? (@)36" (0) 180° (45° wo 61, A fly wheel rotates at « constant speed of 3000 rpm. ‘The angle described by the shaft in radian in one second is 418 Pearson Guide to Objective Physics (30002 (b) 00" 50m @2n 62. A scooter is going round a cireular track with a speed of 30 ms", The radius of the circular track is $0 m. ‘The angular velocity of the scooter is (8) 1000 rad s* (0) 0.6 rad s* (6) 2.5 mad 5" (a) 04 rad 63. A particle is moving along a circuler path of radius 5 1m and with uniform speed 5 ms"!. What will be the average acceleration when the particle completes half revolution? (@) 10m" (b) 10/ms? © 10ms? (A) zero 64. What determines the nature of the path followed by the particle? (a)acceteration (b) speed (o)velocity (4) none of these 65. A particle moves alonga eiccular path of radius r with uniform speed v. The angle described by the particle in one second is given by (yr? () (vr (@) 66. To enable a particle desenbe a circular path, what should be the angle between its velocity and accelera- 'r tion? (180° (090° as wr 67. A panicleis describingthe circular path of radius 10m every 2 s. The average angular speed of the particle, during 4 sis (@)20nrad s" (b)4 ands (02 mrads" (d) none of these 68. The angular speed of the minute hand of the clock in degrees per second is (10 (b)0.1 (©) 0.001 (4) none of these €. The angular speed of a motor increases from 600 to 1200 rpm in 10 s. What isthe angular acceleration of the motor? (2) 60 rads? (b)2 mrad s* (©) 60 mrad s* (d) 600 rad s* 70. A wheel is subjected to uniform angular acceleration about its axis. Initially its angular velocity is zero, In the first two seconds it rotates through angle 8. In the next two seconds it rostets through angle @,. What is the ratio 0/6)? @4 2 (b)3 @ 71, ‘Two bullets are fired at angle 6 and (90 ~ @) to the horizontal with the same speed v. The ratio of their radii at the highest points is (a) c0s°@ (b)tan’@: (o)Izh (d) core: 72, Match the following (@) Acharged particle of charge q mess m ismovingwitha velocity v (b) A paniclereleased from moving train (© Apanticlereteased with a velocity > v,., (P) describes circle if placed in magnetic field (Q) generates magnetic field (R) generates electric field (S) describes parabolic path (U) describes elliptical path (W) describes hyperbolic path 73, A metal ring of mass m and radius r is placed on a smooth horizontal table and is set rotating about its axis So that each part of the ring moves with a speed v. ‘The tension in the ring is om mi? oF 74, A particle moves in a circular path of radius 7. In halt the period of revolution its displacement and distance covered are (a2, Ora (2,20 Orie 75. The angular acceleration of a particle moving along a ‘circular path with uniform speed (a) zer0 (b) variable (©) uniform but non-zero (@)incomplete information 76. A bucket tied at the end ofa 1.6m long string is whirled. in avertical circle with a constant speed. What should be the minimum speed so that the water fromthe bucket does not spill out during rotation (g = 10 ms)? (b)6.25 ms" (@) none of these 77. A sioneof mass m tied toa string of length! is rotated in acircle with the other end ofthe string as the centre. ‘The speed of the stone is v. If the string breaks, the stone will move (a) along a tangent (©)away from thecentre (b) towards the centre (@will stop. 78. Anautomobile is tuming around a circular road of ra- divs r. The coefficieent of friction between the tyres and the road is #. The velocity of the vehicle should not be more than @ y= (©) Jing (ws. (dugr 79. The roadway bridge over a canal is in the form of a circular are of radius 18 m, What isthe greatest speed with which a motoreycle can cross the bridge without leaving the ground? (a) 18.98 ms? (b) 1819.8 ms" (©) Jog ms" (a) Ji8x98 ms 80. Anelectric fan has blades of length 30cm as measured, from the axis of rotation. Ifthe fan is rotating at 1200 pm, the acceleration of a point on the tip of the blade is about (a)2370ms? (6) 3085 ms* (e) 1000ms* (0/4740 ms? 81. A bottle of soda water is held by the neck and swing, briskly in a horizontal circle, Near which position of the bottle do the bubbles collect? (a) inthe middle ofthe bottle (b) near the bottom (©) bubbles uniformly distributed (4) near the neck 82. Two cars c, andc, are going round concentric circles, of radiir, and r,, They complete the circular paths in thesame time then 20S. ‘seed of ¢, arin, (by rr, ol (@) none of these 83. A fan has 3 blades. The edges of two blades are | m ‘apart, A coin of mass 2g is placed on one of the blade edge. If the fan rotates with 600 rpm, the force ‘experienced by the coin is necrly @3.1N (b)46N ()52N (@)28N 84. A stone of mass | ky is tied to the end of a string I m long. Itis whirled ina vertical circle. IFthe velocityof stone at the top be 4 ms", What is the tension in the string at the lowest point? Take g = 10ms? (ON (b)66N ()52N (76N 85. A body of mass 1 kg is rotating in a vertical circle of radius Im, What wll be the difference in its kinetic energy at the top and bottom of the circle? Take =10ms? Circular Motion 119 (@)30) (b) 30) (©)20) (ay10s 86. A thin uniform rod of length / is hinged at the lower end to the level floor and stands vertically. If the rod is allowed to fall, its upper end will strike the floor with a velocity (a) Beh tb) Sah ®) Pel © Ja 87, Acaris takinga tum ona level road. t may be thrown ‘outwards because of the (a) reaction of the ground (b) fictional force (c} weight (a) lack of centripetal force 88. The velocity ofa particle at highest point ofthe verti- cal ciscle is JSrg . The tension at the lowest point if sass of the particle is m is (a)2mg (ong (oy 4mg (ad) 8ng 89. A second is pendulum is suspended in a ear that is travelling with a constant speed of 10 ms" round a circle of radius 10 m. f'the pendulumundergoes small ‘oscillations, the time period will be (@)2s (4s (c} less then 2s (d) greater than 2 s but less than 4 s 90 A hemispherical bowl of radius ris rotated about its axis of symmetry whichis kept vertical. A small block iskep' at a position where the radius makes an angie @ the vertical. The block rotates with the bow! with- ‘out any slipping. The friction coefficient between the bblock and the bowl is. The maximum speed for which the block will not sip és [ (sind — ucos6) J} rsin@ (c0s0 + sin g{sind-+ycos8) (b) | Fein (000 + psin®) © g(sind + p1c0s0) rsin@ (cos0— usin0) (4) none of these 91. Acaris travelling with linear velocity von a circular road of radius r. If it is increasing its speed at the rate ofa’ ms*, then the resultant acceleration will be 1420 Pearson Guide to Objective Physics @ {> (©) YF @ fa-# ? 92. A toy car istied to the end of an unstretched string of Tength ¢. When revolved, the toy car moves in a hori- zontal circle of radius 2a with time period 7. If itis now revolved ina horizontal circle of radius 3awith a period 1” with the same force, then © = @r (T=P @ 93. A proton goes round in a circular orbit of radius 0.01 m under a centripetal force of 4 < 10° N. What is the frequency of revolution of the proton’ (a)2.5 10" Hz (b)4* 10” Hz (sr 10Ps! (@) 167 108s 94, A block of mass m at the end of a string is whirled round in a vertical circle of radius r. The critical speed of the block at the top of its swing below which the string would slacken before block reaches the top is () re (b) arg © fre (a) fire 95. A ube of length is filled completely with an incom- pressible liquid of mass m and closed at both the ends. ‘The tute is then rotated in a horizontal plane about ‘one of its ends with a uniform angular velocity «. The force exerted by the liquid at the other end is. (b) mo 96. What isthe work done in seconds by a body of mass ‘m moving in a circular path of radius r witha constant angular acceleration. Initially the body was at est. Oa (b) 22m (V2 mee RE (ayzer0 97. A string can withstand a tension of 25N. What is the greatest speed at which a body of mass 1 kg can be Whirled in a horizontal circle using Im length of the string? (a) 10 ms" ()7.5ms" (c)Smst (@)25ms" 98. A car travels north with a uniform velocity. It goes over a piece of mud which sticks to the tyre. The par~ ticles of the mud are thrown (a) vertically inward (©) vertically upward @ horizontally to north 99. Acaris moving with a speed of 30 ms"! on acircular path of radius $00 m. Its speed isincreasing atthe rate of2 ms, The acceleration of the car is (a) 2.7 ms* (c) horizomally to south (}2 ms? 100, Ifthe overbidge is concave instead of being convex, ‘the thrust on the road at the lowest position will be (b) mg+ a @ 7s 101. A body crosses the topmost point of a vertical circle with a critical speed, What will be the acceleration when the string is horizontal? @)3g (0) 6g sg (2 102, Ata curved path of the road, the road bed is raised a litle on the side away from the centre of the curved path. The slope of the road bed is given by (a)tan = rghv? (0) tan B= rigy? (c)tan 6= ving (tan = gir 103. A truck hasa width of 2.8 m, Itis moving on a circular road of radius 48.6 m. The height of centre of mass is 1.2 m. The maximum speed so that it does not over- tum will be nearly (a) 243 ms-1 (0) 27.2 ms-1 (©) 134 mst (4) 15.7ms1 Read the following passage and answer the questions given atthe end. ‘The cosmoclock 21 Ferris wheel in Yokohama city, Japan, has a diameter of 100 m. Its name comes irom its 60 arms each of which can actas aseconds hand, that i, it completes cone revolution in every 60 seconds. A passenger who was shown his weight 660 N by a machine fixed on the ground sits on one of the arms. The ferris sets into rotation faster. The passenger is frightened when ferries arm carrying him reaches the top as she finds herself weightless. 1, What is the speed in rpm if the apparent weight of the person at the highest point is zero? (2) 1.87 rpm (b)2.87 pm (€)3.32 rpm (€)2.08 pm 2. What will be the minimum weight of the passenger if the Ferris revolves at its normal speed? (a) 600N (b)827N (€)586N (@)396N 3. What will be the maximum weight of the passenger if the Ferris revolves at its normal speed? (0) 693.3N (b) 7033 N (c) 813.2N (@)none of these Lob) 044260 on = 2873p 2n 2.0)9F = name =teo-662 5022) = 62567N 3a) F.,.= mg+mrv?= 6933 Read the following passage and answer the questions given attheend. ‘A naughty boy always disturbs his mother when she is in the kitchen. His mother reads a book and designs a machine for her son. It is a 11 type tube as shown in Fig. 4.51. It is filled with water upto a fixed level asshown. Before going to the kitchen she sets the Lu type tube spinning about 4X and asks her son to see, as soon as this sets rotating the level of water in each limb as it changes. She asks him to bring his notebook, stopwaich and a measuring scale and find out theoretically what the level will bein each armand check this result with the experimental observation, till she finishes her Job in the kitchen. The student measures r = 10 cm, time of revolution (25) Fig. 451 1. Findthe levelin(0). Circular Motion 124 @2mm (by2em (400m (@80em ‘The liquid level in limbs ABC forms — shape. (a) straight line (b) parabolic (e) hyperbolic elliptic ‘What will be the shape if rotated about limb B ? @) Oy © @ Fig. 4.52 10) 2 par (2r)’ 2 arse. POE P= Pat ple fxd =p, + POL Ap = par ar} » (by) fa) Read the following passage and answer the questions given attheend. ‘One problem in humans living in outer space is that they are ‘apparently weightless. One way around this problemis todesigin ‘a spece station that spins about its centre t a constant rate. This ‘creates “artificial gravity" at the outside rim ofthe sution. The physics teacher asks you to think of a space station of diameter 800 m. This space station isa waiting area for travelers going to the Mars. It might be desirable to simulate the acceleration due to gravity on the Martian surface (3,7 ms?) 1. How many rpm are needed to simulate g=3.7. (2) 0.62 rpm. (60.821 pm (60.783 rpm. (60.18 rpm. 2. Ifyou have to create artificial gravity equal to that on the Earth, how many rotations per minute will be required? 122 Pearson Guide to Objective Physics (149 pm (©)1.6:pm (©)L78xpm (@) none ex(rom)= 355 x $E =.918rm Ld@ror ao= 28 m 2ado= fae 49 0p RON cke Mae aac Tae 1 © 2 @ 2 @ a) 5 . @ 7 © a % © 0. ©) 1% 2 @ 3 @) 4 @) 6 6. @) 7 © 18 a) 1% @) a © a © 22. (a) 23. 2. @ 2 a) a © 7 @ 2. ©) 2. 30. bed) 31. Cd) =) 3B. (cd) 35.) 36. (@) 7 © 38. (abcd) 38) 4 co A) 43. (0) “4 © 45.) 6 fa) a. © a @ 0 @ 0. (0) 1. @ 52) so) 55. 0 5) sr. (be) 58.) A). ) 61.) 62. (b) 8.) es. (a) 6. 0) 6.) o. ) 6.) 8 ©) 7.) in. @) @FP.ARO+S,C3W) 7 @) ar) 75. (@) 76. (a) mm @) 7%. ©) mad) 8. @) at.) 82. tb) 33.) 4. 0) 8. ©) 86 (a) 7. @) 8.) a © 90. (o) a. @) 2 0) 9 a) %. (©) 8.) © ee) 3 @) 9 00) tO.) 102.) ® EXPLANATION 1(b) For quarter revolution é vav,(-f)-n7 Bs (a) Centripetal acceleration vi _ (271000) Ay Av- Jean? - V2" Tr [ 3600 Alsoor=tan > =45° =0.703ms* soul ste Netacceleration, a= 2a) Assuming the particles to be the closest, relative sic gel wale ae a = Joos s05 =0.86ms? and and relative position vector, 1 0.703 _ s40 6 tar! S70 = 54 Using o= ©, we get relative angular velocity. 4(¢) Mid point acceleration, v = ms" at 45° withx Sib) Maximum velocity, - (ease) 0.2+ tan15® = -[l (es PES poms] =38ms" rh (a) Here cos a= or -h=reos ar or h=r(1—cos.a) AtL, mg cos. r 0) + 2eh)] ot mgcosa= (qert2en ers 2ett-n) 9 or 3c0s. a= 7 orcos a= 3 or @=cos'3i4 Circular Motion 123 Te) Velocity at the highest point, va Height of the mass = 21, Tine taken to fa'= f?%24 [AE & g Horizontal distance covered Val x itig v, =a 8(d) Note HH 0s oF Using s= 1/2 ar, we get 2rcos a= 1/2 (g cosa) or Mn | 8608 a4 H Fig. 4.54 9%(b) Tangential acceleration, a, g sin o and radial acceleration, _v _ 2gteosar Fig. 4.55 124 Pearson Guldeto Objective Physics wherev= 2gh Rah and sin @= “Z4 we get mx2gh po me Rah a mg sin 8= mg A= or h=R/3 and sin @= 43 or Ox 47° 1(d)Using Re cos @andmg = R sin 6 Fig. 4.58 or v= J10xG.1 = ms" 12(a)Let ube the initial velocity of ball P, then w= 32 and difference in final veloc 2ar__ 2ar Time for second strike = 32" = 2 2er__ ar elt e Ir 13(b)4 p =2vsin Instantaneous acceleration 60" ry 03x1000 Time taken to cover the are 2ms* Av average acceleration a= 9% mm Arp respectively. Clearly r,>r, therefore centripetal acceleration at 4 is more, Thus, reaction at B is less than 4 because here the normal reaction depends on upwards force or acceleration. 15(d)For vertical motion Loe H~ 5 short \Bilig For horizontal motion, distance covered is given by, 2am= ut or tmn= Pig o n= Hig lar 16(a)Velocity a the bottom of the circle Ser Velocity atthe top of the circle Jer Using PE.=K.E., we get 1 L Z mg i= 3 m (far) = 3 m (Ser) @ alsomgh= 4m (fgr) = 4 mgr @ Dividing) and Gi, we get n Bes 11(c)Let be time of flight of the particle from P to 9. Since e is the distance between the centres of the two wheels, the horizontal distance between the wheels, ye? -(6-ay If vis the velocity of the carriage, ¢,= 0 (horizontal range) FT @aaye or = t In this time. the particlehas covered the vertical distance =2(b=a) then Motion 125 20(¢) Using u=ra= rx 2af =I sin 6x 27, we get _f-teta-bMe-a+6)]? or v= |B aay 18(a)Let x be the speed of frst ball, then fora sitike between first and second ball Fig. 487 bag aay and 4 are} aoe solving (i)and(ii) ps 2 1 2 22(a) Horizontal displacement of both particles is equal. For particle 0, the horizontal velocity is always equal to v. For particle P, tthe start the motion will be accelerated ‘one, causing increase in velocity, Thust, <1, 23(d) Here va wee 2g! 2g! Ai) in this way v,= (J u Fig. 458 Since the velocities are mutually perpendicular, change in velocity Av= ti) (substituting the value of v? from (i) aa = 1.8ms* 24(b) Here average velocity _ diplacement 48 a= Jalsa? = Sige time i 2 = 2.7 ms? -2-2ms 2, $n2 126 Pearson Guide to Objective Physics 25(a) Here, speed of the block at the highest points in all the four cases will be same. The normal reactions at these points will depend upon the centripetal acceleration experienced by the block. Centripetal acceleration 4 Radius of curvaturein the first case will be minimum, so centripetal acceleration and hence reaction will be maximum in this case, 26(c) Motion of a simple pendulum is an example of non uniform circular motion. The bob has some tangential ‘as well as radial acceleration, Fig. 4.59 Thus, ota! acceleration (4) will be vectorial sum of radial acceleration (@,) and tangential acceleration (a). 27(a) The force my*/r directed outwards, called centrifugal foree, is nota real force ALA, od 7, tng. Fig. 4.60 28(b) For v, to be minimum, 7,=0. ered 29(4) Applying the principle of conservation of energy between A and B. Also, T~ mgcos 0= "= 2mg (cos @~c0s 6) 7 or = mg (3c0s - 2005 6). Fig. 461 31(c,d) As no forces act on the system, @ remains constant. Hence, 7 = mia? remains constant, However, T's the sum of electrostatic force of atiraction on the bob and the force exerted by the string due to elastic strain init. |As the positive charge on the bob increases due t0 photoemission, the electrostatic force on it increases, and the elastic strain on the string must decrease, 32(b.0) Li Fig. 462 me Tsin 9= moy*sin 9 or T= marl. = 008 = oes Teos = mg or maviecos 6=mg or w= 7-5 =F uP =2gt —ui magnitude of Ay = Ju —2gl)+u? = 200 — gh - 34(b, c,d) A ABP is equilateral, Let 7, and 7, be thetensions in PA and PB, respectively. T, cos60°= T,cos60°* mg or T,-T,=2mg 7, c0s30°+ 7,cos 30° = mai/cos30° or 7,+ 7, change in velocity 45 ¥, ~ 2 T= 5 mlail-2g) For 7,20, 02 J2g71 . 35.4) D rar de Fig. 4.64 ‘The mass of the element = dn = (m/l) di. ‘The force on the element towards the axis= 7 (T+ dT) Circular Motion 127 Fig. 465 ‘Themass of the element = din = (mL) de. The force on the element towards the axis = (F +a) — FedF. d= (dn) oP eee Fax. ME + constant the constant = 0, Fig. 466 Let = tension in the ring. Forthe section 4B, the net forceiis 27 sin 8. As @is small, therefore 27 sin 6 = 270, and the net force is towards C. Mass of section A2 ~ (2r 6)m,For circular motion, 278=(2r Om) root or HEN = the normal reaction of the track, =2mg or F=N+2mg. As N20, F 22mg. ‘Taking the speed at B and D to be the same as at C, at B,F—N=2mg or N= F~2meg (centre of curvature is O)at.D,N—-F=2mg or N=F+2mg (centre of curvature is’) For ZAOM=6, mg (r= ros 6) = 3 mv? or 2mg (1 = cos Njor2mg (I ~cos 8) ~mg cos 0+ FN. For F=N, cos @= 2/3 128 Pearson Guide to Objective Physics 39(b) Dueto the rotation of the earth about its axis, different points onits suface have the same angular velocity but different linear velocities, which is maximum at the equator and decreases at higher latitudes. ‘The earth rotating from west to east imparts an eastward velocity to the wind mass. The wind mass moving northward fiom the equator retains its eastward velocity, which is greater than the eastward velocity of the surface at higher latitudes. Hence, relative to the carth’s surface the wind mass shifts to the east. 41(6,4) e a, d Fig. 4.87 The tangential acceleration a,and theradial accelerat 4,— Vir being mutually perpendicular, have a resultant acceleration a = [a2 + (v%r)1!. The only horizontal force on the body is the force of friction, F= ping = ‘ma, ecting in the direction of a. 67 w TD mrads*. vile _ veosta n vile” sin? 76 9 92 95 vo Sng - J5x1-6K10 9ms"' apy maa{ ] mia fr din C_ > Fig. 4.68 where 4 is area of cross-section pak im? 103 mgi2-"™ eR ley’ (overtuming torque = restoring torque) [ert _ floxsoxia 2h 12 4.3m", {es pAl=m} Newton’s Law BRIEF REVIEW Force A pull orpush which generates or tends o generate motion in a body at rest, stops or tends to stop a body motion, increases or decreases the magnitude of velocity of the moving body, changes or tends to change the shape of the body. Newton's First Law of Motion A body at rest will remain at rest and a body in uniform motion will remain in the state of uniform motion unless it is compelted by some external foree to change its state Inertia The inherent property of the body with which it cannot change its state of rest or of uniform motion unless acted upon by an external force, is called inertia, Hence, Newton’s first law of motion may also be called law of inertia. Note that the term external force was used. It means there would be internal force also. InternalForce Ifthe force applying agent is inside the system, force is internal. Internal force cannot provide motion. For example, if you are sitting ina car and you push the car, ear does not move. If you come out of the ear and apply the same force, car moves. When you were inside the car, the force applying agent was inside the car, hence, the force was intemal and car did not move. When the force applying agent (you) had moved outside, the car moved. The straight line along which force act of action of the force. Ifabody is to accelerate or decelerate then unbalanced force is needed. is called Line of Motion A system of bodies on which no external force acs is called a closed system. For example, two bodies moving towards each other due to their mutual electrostatic or gravitational force. ‘When many forces act on « body at the same point, they are called concurrent forces. The system of concurrent forces may be: (@)_Collinear, thats acting along the same straightine. (6) Coplanar, thats in the same plane. (©) Generally directed, butnot in the same plane. Mass _Innewtonian mechanics mass is considered to be a measure of inertia of a body and is considered independent ofits velocity Iisa sealar quantity. Unit kg (SI system). Momentum The total quantity of motion contained in a body iscalled momenturn Itisa vectorquantty. Unitkg ms" (S) B= Iftwo differentmasses have same momentum, then the lighter one has more kinetic energy (also more velocity). Newton's Second Law of Motion The time rate of change of momentum is directly proportional to force (external) applied on it and the change in momentum occurs in the direction of force . dp . dp_mdv Fear k Newton considered mass to be constant. Unit of Force is Netwon (N) or kg Wt (kilogram weight) or kg f (kilogram. 130 Pearson Guide to Objective Physics ‘If mass is varying and velocity constant F dd <7 ifboth mass and velocity vary. Impulse Product of force and time for which it acts is called impulse. Fe £ or F. dt = dp ie, impulse = change in ‘momentum F, .1= Ap is called impulse momentum theorem Newton'sThirdLawofMotion To every action there is anequal and opposite reaction, ie., F, = —F,, . Moreover, action and reaction act on different bodies. According to third law fercesin nature oceur inpairs, Single isolated force is not possible. Note: In certain cases of electrostatics and in springs Newton's 3rd law fais. Law of Conservation of Momentum If no external force acts then the total momentum of the system is conserved ap Pd Equilibrium Translatory Equilibrium — When several forces acton a body such that resultant force is zero, i.c., LF = 0, the body translatory equilibrium. ¥=0 implies SF, 0, It means the body is in the state of rest (static equilibrium) or in uniform motion (dynamic equilibrium), a If the force is conservative then F 0 means a potential energy w= maximum, minimum or constant. Stable Equilibrium If on slight displacement from equilibrium position, body has the tendency to regain its original position. In such cases centre of Mass (COM) rises on slight displacement. Note that PE is minimum du ca Unstable Equilibrium {fon slight displacement from equilibrium position the body moves in the direction of displacement, the equitibrium is known to be unstable. The COM goes down on slight displacement, PE is maximum du # Neutral Equilibrium Ifthe body remains atthe displaced position after a slight displacement then such an equilibrium and ve for unstable equilibrium. is neutral. The COM does not change and PE is constant but not zero. Fig. 5.1 illustrates all the types of equilibrium: stable, unstable and neutral PE lunsable equilsium PE coast neural eauiibeigm PE mi sable equilibrium GEIEREEE Types of equilibrium explanations Strings String is considered to be massless unless stated and hence, tension remains constant throughout the string. String is assumed to be inextensible unless stated and hence, acceleration of any number of masses connected toit isalways equal or same. Ifthe pulley is massless and smooth, ‘and string is also massless then tension at each point (or two sides of string) is constant as shown in Fig. 5.2 Tension in string fora light and smocth pulley If the string changes tension changes as illustrated in Fig. 5.3. Tension in affront strings 7,, 7, and T, in Fig 5.3 are different as string changes. InFigs3. 7,~27, 2-2) Pa—_______+F If forces are equal and opposite ona massless string as shown in Fig 5.4 then tension 7 is equal to either of the two ‘Themaximum tension which astring can bear is called its breaking strength. Ifthe string has mass tension is different at each point a illustrated in Fig. 5.5. a —SSSS Sse rere ERE tiustration of tensionin astringrod having mass ‘Mass per unit length .= =~ . We have to find tension 7 M(=x) pe at P. Mass of (1-2) part is M( Tension at P= “= Springs Springs are assumed massless unless stated. Restoring force is same every where, ie. F =k Springs can be stretched or compressed. Stretch or ‘compression is taken positive. Restoring force is linear as is clear from F = —kx. k is called spring constant or foree cons!ant. kee + (kalso depends upon radius length and material In parallel kageuye= Ay ++ If masses m, and m, connected to aspring as shown in Fig. 5.6 are oscillating or both masses move then find reduced toa 1 seaside em If the spring has mass m, then “is used to produce extension. Psuedo Forces The hypothetical forces added while dealing with problems associated with non-inertial or accelerated frame of reference, so that Newton's laws may bbe applied are called psuedo forces or inertial forces. If a frame of reference is moving with an acceleration a, then force on a particle of mass m is ma, In the force equation a force ~ma, will be added to make the frame of reference Newton's Lawoof Motion 131 Friction _ Ifwe try to slide a body overa surface the motion is resisted by the bonding between the body and the surface. This resistance is represented by a single force called friction. The friction is parallel to the surface and opposite to the direction of intended motion. Remember static friction is a selfadjusting force. Ifa body is at rest and rot being pulled, force of friction is zero. Ifa pulling force is applied and the body does not move, friction still acts and is called static friction. The maximum value ofstatic frctionis called limiting friction. See Fig. 5.7. If we apply the force beyond limiting friction, the body begins to move and frietion slightly decreases called kinetic friction. Fiction Fores applied Friction illustration Limiting Friction F,, = 1, N where N’is normal reaction. H,= tan 6 where 8 is the angle of limiting friction. Note: 1, > H,> fly Where 1, stands for coefficient of static friction, 1, stands for coefficient of kinetic friction and jt, stands for rolling friction, Friction is independent of surface area of contact. However, it depends upon the nature of material of the surfacesin contact, their roughness, smoothness, inclination, Nomnally friction between too smooth bodies is more. If the bodies are made extra smooth by polishing the bonding force of cohesion or adhesion inereases resulting in cold welding, Inpractice 0<1<1 but > 1 is observed. For example; H,= 1 for glass/glass, and, 4, = 1.6 for Cu ~ Cu. Friction is a non-conservative force. If force is applied and still the body is at rest then the force of the friction is equal to force applied. Equation of motion for centre of mass (COM) ay, m Seo 5 “cou = SHORT CUTS AND POINTS TO NOTE Tension isa reaction force produced in a string or rod 2. Ina massless string (if not passing aver a pulley) tension is equal at each point. 3. If pulley is massless and smooth and, string is massless and passing over a pulley as shown in Fig. 5.8. then 432 Pearson Guid Objective Physics (m-m)g ,_ 2mmg m+ My 4. Ifthestring changes tension will change. Assume, in Fig. 5.9 Pulley is smooth and massless. String 's also massless. Then, Is Lem +m)— m +m, #7 2m, +m) g mga), 17=2T 5. If the pulley system of fig. 5.8 moves up with an_ acceleration a! then, and 2nmlg+a') im, +m, 6. If the pulley system shown in Fig. 5.9 moves up with an acceleration a’. Then, = Le tm= meta im, tn, + my . = 2m, +m, )m,(g +a’) m+n +m, T = m,(g+d-a) 7. IfF>2T in Fig 5.10 is applied on the pulley to move the system upwards. F-2T (n,=mye re) Then, d= mm, _ 2mms(g +a’) a t IF <27, thena’=0 anda = imam Me) nme T= +m F a ir 4 fb 8. Ifthe springs arein parallel then their displacements are equal. For example, in Fig. 5.11 (a)and(b),the springs are in parallel, Le. kip + hy a OOD (b) 9, Ifthe springs are in series, 2s shown in Fig. 5.12 streches in spring are un-equal and.x = x, +x, 1 2 l % 1 10. Ifthe springs cutk== 7 . Forexample, ifaspring of spring constant iscut in the ratio 2 :3 then shorter 3 spring has k’ = = and bigger one has spring Sk constant R= >. 1, InFig.5.13, ifthe block or pulley moves down by x, spring moves down by 2r, Thus 7= ae InFig. 5.14, ifthe block movesdown by x then spring x or pulley moves down by 5. F = T, F” T= k Fr x Can a Fig. 5.14 12 As shown in Fig. 5.15, ifthe pulley moves forward by x then block moves forward by 2x. Fae = 2 Spay? Goes 13, Since force is @ vector, apply vector algebra Whenever there are two or more forces. 14, Draw free body diagram before you solve the problems. They make the problem very simple. Newton's Law of Motion 133 18. If force is applied on the body and body does not move, then, friction = force applied and not pV wthere N is normal reaction, 16. 4,> 1,> H,. Barring few exception 1,< land hence WAI. 17. In conservative forces work done depends upon initial and final position. It is independent of the path followed. Net work done in a closed loop equals zero. Gravitational, electrostatic, magnetic forcesare conservative. Friction isnot conservative. 18. If there is no friction then acceleration down an incline is a= g sin @ as shown in Fig. 5.16 (a) N mind income me 0 19. Ifthere is friction and coefficient of friction between OF Fong mg (sin 8-108 6) - 5.16 (b) 20. Ifthe biockis tomove up the incline with constant velocity then F., = mg (sin 6 + cos 6) (See Fig. 5.17) Ifitis to move up with an acceleration ‘a” also then F.,= mg (sin 8+ 11 c0s 6) + ma, 134 Pearson Gulde to Objective Physics 21, On a horizontal plane deceleration due to friction is bg. 22, Ifalif moves up with an acceleration a then effective or apparent weight is m (g + a) as ma acting downward is pseudo force to be added to make frame of reference inertial Similarly if the pulley is moving down with an acceleration a then apparent weight of the body is m(g-a). 2B. If the force is a function of distance or velocity then use: bar? ae aes 24, It is always helpful to choose axis along the incline as x-axis and axis perpendicular to the incline as y- 25, Remember frictional force and normal force are always perpendicular and F, 26, Pulling atan angle decreases the kinetic friction as Normal reaction decreases as illustrated in Fig. 5.18. ahong, y iF A > Feo wwe— N= Mg—F sin 0. or, F,= 4, N= u,(Mg—Fsin 8). 27, IEF, ‘velocity asin case of viscosity. F= 6 xn rv (Stoke's Jaw) vis terminal velocity. 28. If a bodyiparticle of mass m moves with a lineer velocity valong the diameter ofa tum table then an extra force is experienced by the body called coriolis force ena) =2 VO. Where @is angular velocity of the turn table. 1. Applying Newton’s law without caring about inertial/non- inertial frames. = Innon-inertal frames of reference, first apply pseudo ‘vectors to make the frame of reference inertial, only afier that apply Newton’s laws. 2, Considering action and reaction always act on different bodi > In case of elastic bodies and springs, action and reaction act on same body. That is, in case of restoring force in a spring or deforming force in elastic bodies, action and reaction act on same body. ‘These forces are therefore called internal forces. 3. Considering Newton’s third law is always valid. => Incertain cases of electrostatics Newton’s third law. fails. 4, Assuming friction always acts in 2 direction ‘opposite to the motion. = If the friction causes motion then the friction acts in the direction of motion. 5S. Considering force constant of aspring does not vary when spring is cut. 1 = Spring constant k= 5. 6. Assuming friction is always equal to s2V. = Ifthe body is moving. friction = 1, N. Ifthe body is stationary then friction is equal to force applied. 7. Assuming if pulley is massless then tension in the string on twosides of the pulley is unequal asshown in Fig. 5.19, => Ifpulley is massless and smooth 7, = 7, Ifpulley has mass then only 7, and 7, are unequal. 8. Notunderstanding constraints, => In problems like shown in Fig. 5.20, ifthe pulley ‘moves forward by x. then thread 2xis used r below and.x above which will be supplied by the block sideas other is fixed. Therefore, block will move 2r. Hence, tse, =2 tey 9. Considering in equilibrium body must be at rest. = In static equilibrium body is at rest. In dynamic equilibrium, it moves with uniform velocity. 410. Assuming there is no tension if the rope is pulled by equal and opposite forces on two ends. = Tension is equal to either of the force applied. 11. Considering impulse always provides acceleration, => Sharp impulse only provides velocity. 12, Considering rough surfaces have more friction. => Ingeneral it may be true. Butpolished surfaces may offer more friction. For example, coefficient of friction between glass/wood is 0.23 and glass and glass is 1.0 and between Cu ~Cuis 16. 13, Considering horizontal planeasxaxisand therefore normal force perpendicular to x-axis as shown in Fig. 5.21 (a). Newton's Law of Motion 135 Wsing ¥ e030) ean) = Considering axis along the incline plane ay x-axis ‘and perpendicular to itas y-axisis more convenient way of solving problems as shown in Fig. 5.21 (b). SOLVED PROBLEMS 1, A smooth block is released at rest on 245° incline and then slides a distance d. The time taken to slide is times as much to slide ona rough incline plane than on a smooth incline. The coefficient of friction is cease (1) Without friction d= case (ii) With fiction d= & [sin45—p1, cos45] # r? ... @ From (1) and (2) 4,= 2. The upper half of an inclined plane with inclination is periectly smooth. While the lower half is rough. A body starting from rest at the top will again come to rest at the bottom if the coefficient of friction for the lower halfis given by @2sing ()2 cos (©2mn¢ (@ung IAIEEE 2005} (mgs sin 9= mg cos 9 5 oF u=2 tang. 3. A block is kept ona frictionless inclined surface with angle of inclination a. The incline is given an acceleration ato keep the block stationary. Then a is ‘equal to og (b) gcosee a Og (@)gtana [AIEEE 2005) (@) macos a=mgsina or a= gtana 136 Poareon Guide to Objective Physice (a) b) Fig. 5.22 4, Aparticle of mass 0.3 kg is subjectedto aforce F = kx with k= 15'N mv, What will be its initial acceleration if it is released from a point 20 cm away from the (b) 15ms? (@) 10ms? IAIEEE 2005} 5. A. Frictional forces are conservative forces R. Potential energy can be associated with fictional forces (@) Aang R both are true and Ris correct explanation of 4 (b) A.and & are true but R is not correct explanation ofA (© Ais correct but R is wrong (© Both A and R are wrong [AIMS 2005] EE @ 6. Which is true for rolling friction (1), static friction (1) and kinetic friction (4,); Our m>H, OH o> (2k @ak (Ok (a) k= 2+ Dh ak het 24, Momentumis closely related to (2) force (b) impulse (c)velocity (a) kinetic energy [cE 1997) © 28. A force F =k 1(t~ 1) acts on a particle of mass m, which is at rest at r= 0 where k isa constant. Find the momentum of the force when theaction ofthe force is discontinued. ws wh ok, ory kel ket (odp= [Fat = fia(e—nae =~ a 6 140 Pearson Guide to Objective Physics TYPICAL PROBLEMS 26. The velocity of a ballet changes from v, to v after the bullet has passed through a distance fin the plank. Assuming resistance offered is proportional to v, find the time of motion in the plank. (b) (= v, vlog, (a) ()F=-k*, o fe g & fi si> (I) ‘ae i = fta@ot-4-tiw fiom (I) and 2)¢= 020A v,vlog, “= 27. A body of mass m rests on a horizontal plane with a friction coefficient 4. At f= 0, a horizontal force is applied (F= a 1) where a is a constant vector. Find the distance traversed in first ¢ see. oer Oe (a) After the application of force body begins to move after a time /, such that 28. A horizontal disc rotates with a constant angular velocity @ about a vertical axis passing through its centre, A small body m movesalong a diameter with 2 velocity v. Find the force the disc exerts on the body when it is at a distance r located from the rotation axis. (a) mra?+2mvo &) mg+ mr? o* +(Qnnvo) © wig rOmaF tre fear Gear @) me? +r°a* + Qve) (a) The force mg is vertical, 2mvo perpendicularto vertical plane and mr oF outward along the diameter. The resultant force is, Fem (esrar+Qvoy - 29. A bead A can slide freely along a smocth rod bent in the form ofa half circle of Radius R. The systemis set in rotation with a constant angular velocity @ about a vertical axis OO’, Find the angle @ corresponding to steady position of the bead. (b) forces acting along the tangent tothe radius sin 6. ing sin 8 mr aF cos 0= 0 2x8] -0 o0- or (38 30. A block of mass m is placed on a wedge of Mass M. coefficient of friction between them is 4 > cot @. The o nesnals wedge is given an acceleration to its left. Find the ‘maximum acceleration at which block appears stationary relative to wedge. Fig. 5.34 (sind ~ 1.6058) g(sin + pcos) cos +usind 030 — sind (e050 + sind) © "Find—peoso —*(@) None ® 3. N = m(g0cos 0+ asin 6) ma cos 0= mg sin 6+ Nv ma cos ‘ma (cos 9 1 sin 6) = mg sin @+mg cos 0) img sin 8+ mg cos 0+ pi ma sin @ __ glsind + 40088) Fig. 535 In the arrangement shown in Fig, pulleys are smooth and massless. Thieads are massless and inextensible. Find acceleration of mass m,. [2rmm, + mC, — mi Ne © aman mton oy [ema + mtn =m, jg in an, + m, (em, +m) fo Bamcmim me ‘amin, +m +) (@) none ywton’s Lawof Motion 141 mf oT me Gi Fig. 8.36 (b)m,g-T=m,a, {fromFigQ31(ii)] =m,a, —_ [fromFigQ31(i)] ma=2T — {fromFigQ31()] a,+a,=2a [Armyn, +g Ig Solving we get o, = Amite tm(es “Ng meee es Ging, Fmglam +) A train of 2000 tonne moves in the lattitude 60° North, Find the magnitude ofthe lateral force that the train exerts on the rails if it moves with a velocity S4kmh". 2m wsn 60 32. 2nw @ unter Fig. 8.37 (@)24x10'N ()3.6x10°N (0)36x10'N (@)24x10'N (OF=2mveosin60 ; isxae = 2x2 10% FA 60x60 % 9” = 36x10°N 33. The arrangement shown in Fig, mass of the rod is M_ ‘and mass of bead is me and M> m. The bead slides with some friction, The mass of the pulley and friction in its axle are negligible. At the initial moment the bead was located opposite the lower end. If set free both the bodies move with constant accelerations. Find the frictional force between the bead and the threadso that {second after the release ball reaches the upper endof the rod. Length of the rod is 2 142 Pearson Guide to Objective Physics (a) (b) Fig, 5.38 Both rod and bead move down with acceleration a, anda, such that a, > a, Since both are downwards, relative acceleration fs 4, ~ a, forrod. 1 a P= Zl@-a)F 0) Ma, (2) mg-F,= ma, .G) multiply (2) by mand (3) by Mand subtract (4—m F, =mM(a,~a) 2mMt OE Ome 34. A particle of mass m moves along the internal smooth surface of vertical eylinder of radius r. Find the force with which the panicle acts on the cylinder wall if at 6, its velocity isv, and it makes an angle ewith the horizontal. | | Fig. 5.29 mw mi @ RR (b) R cos a 38. Find the magnitude and direction of force acting on a particle during its motion in a plane xy according to {he law x= asin et and y~b eos apt where. band are constants Foxityj-acoswri +bsinarj ola, a : : vy = GF =-aasinor i +6acos of j ér i ¢ a = Fr e-aat eos ati ~b 0? snot j =o F Fo=ma--me i ‘Twoblocks one of mass 4= | kgand B=2kg. A force of 5 Nis applied on a {sec Fig]. Coefficient of friction between 4 and & is 0.2 and that of between B and horizontal surface is zero, Find (a} acceleration of 4 and B (b) The time taken for the front surface of A 10 coincide with that of B. ICBSE PMT Mains 2005] x an t Mola eS ta rae 37. As shown in fig. mass of bodies is equal to m each. If coefficient of friction between horizontal surface and ‘mass is 0.2. Find the acceleration ofthe system [CBSEPMI Mains 2005} Lee ee woo T me zs me Fig. 5.41 mg-T=ma ..{I) T—pmg=ma ° Adding (1) and (2) and solving za 7 or a = 0ag=4ms*, observed from the frames 5, and S.. The frame S, moves with respect to S, with an acceleration a. Let F, and F, be wo pseudo forces acting on the particle when seen from S, and S. respectively. Which of the following are not possible. (FL A0F #0 (by F,#0,F, #0 WF, 40,7, -0 () F,-0,F,-0 @ 39. Fig, shows displacement of a particle going along the x-axis as a function of time, The force acting on the particle is zero in the region om Fig, 6.42 (a) 4B (ac cD (DE (a) and (c). In regions AB and CD 1s v = J = constant ndv Ce 40. A person says that he mesured acceleration ofa particle to be non-zero while no force is acting on the particle. Then, (@) heisaliar (b)_hisclock might have run slow (©) his meter scale might have been longer than the standard (@) he might have used non-inertial frame of reference (€) Pseudo force will act in noninertial frame. 41. In the fig. m, > m,. Pulley and string are massless and Pulley issmcoth. The system isreteased from rest. After 2 seconds of motion m, was held with the and. Newton's Law of Motion 143 Fig. 5.43 (a) thread on the side of m, remains light (b) thread on the m, becomes slack and it will remain slack until m, is released. (©) thread on m, becomes slack and after some time itwill again be tight. (A) justafter m, isheld the tension in the thread is 0. (end) 42. Inthe Fig table is smooth. Then an a Fig. 5.44 g (a) acceleration of the 3m block ~ (b) the force on the clamp cis 2 mg. 3 (6) the foree on the clamp is 2 mg (4) the fore onthe lamp is 3/3 me. and (a) and (c).ma=mg~T £ T = 3 masolvinga= £ 3 3 T = FeandF,..= 2 78 43, Inthe fig shown 44. Consider a vehicle going on «horizon oad towards ‘east. Neglect any force by the air. The frictional force con the vehicle by the road (a) istowards east when vehicle is accelerating, (b) is zero when vehicle is moving with uniform velocity. (©) must be towards west, (@)_ must be towards east. — Fig. 6.46 (@). (0), To oppose the rotational torque force should act along east. 45. A manholds a thin stick at its two ends and bend it in an arc like a bow without a string. Which of the following figures correctly show the directions the force exerted by him on the stick. Neglect gravity. m™~OD (b) Resultant Si forces be zero 46. Twobodies faces mand M(.M>> m)are attached to the two ends of a light string passing over a pulley ight and smooth). What is the tension in the string when the masses are in motion? (2) (M-m)g (b)2mg (Me @im+ Mg Mme. OT Free ASME mT = Img 47. Aman pulls a block heavier than himself with a light rope. The coefficient of friction isthe same between the ‘man and the ground and between the block and the ground, Then (@) block willnever move (©) the man can move even when the block is stationary (0) if both move a4,> dea, (4) none of these (b) and (©) 48, A long block 4 is atreston a smooth horizontal surface. Asmall block B (whose mass is half of 4) is placed on A at one end and projected along A with a velocity v. “The coefficient of friction isp. 2) theblockwilleach a final common velocity 4 (b) the workddone against friction is 34 of initial KE of B. (©) before the blocks reach a common velocity, the acceleration of 4 relative to Bis 34 ug. (@)_before the blocks reach a common velocity the acceleration ofA relive to Bie 34 1 (a), (b), (d) since F,,, of A + B system is zero. “Therefore momentum of 4+ B system is conserved. mie (conver i 5 s os Eee apy 2 (Va om (4) = 8 = 3 (pm Force of friction between the blocks = jung. Acceleration of 4 (towards right) = SE = BE Acceleration of B (towards left) He. Read the following passageand answer the questions given attheend. ‘The instrument most commonly employed to measure forces in static procedure is the spring balance. The third law is tacitly used in static procedure because we assume that the force exerted by the spring balance on the body is same in magnitude as the force exerted by the body on the spring. ‘This latter force is the force we wish to measure, First law is also used, because as we assume F = 0 when a= 0. Note that if a # 0, the body of weight H will not siretch the spring to the same length as it did when a = 0. 1. What will be the force measured by the spring balance ifthe block (a) is moving horizontally with acceleration a (b) is kept in a lift moving up with 5 ms"! (2) ma (b) mg. 2, Whats the meaning of static procedure of measuring force, making the EF — 0 by applying an equal and ‘opposite force. Read the following passage and answer the questions ‘given at the end. Before Galileo’s time most of the philosphers thought that some influence or force was needed to keep a body moving. ‘They thought body was in its natural state when the body ‘as at rest. They assumed that some external agent had to continually propel it otherwise it will stop moving. Newton ‘carried to full fruitation the ideas of Galileo and others who preceded him. In 1686 he presented the three laws of motion in his work Principia Mathematica Philosphiae Naturalis, Galileo esserted that some force was needed to change the velocity of body but no force was necessary to maintain the velocity of the body. 1, The Galileo principle adopted by Newton is known as Law of inertia, 2. Inwhich ypes of frames Newtons laws can be applied Solution: Inertial frames 3. Earth is considered to be an inertial frame while it is actually pseudo frame. Why’ REISE Since the lab in which experiment is performed also situated on earth, Therefore its relative impact ‘on the body is negligible. ‘Why pseudo forces tobe added in non- inertial frames? ‘Norvinertial frame of reference moves with an acceleration. By applying « pseudo force on the body the effect of this acceleration is made zero. Newton's Law of Motion 145 Read the following passage and answer the questions given atthe end. You are riding your motor cycle one day down a wet street which slopes downward at an angle of 20° to the horizontal. {As you start down the hill, you notice a construction crew has dug a deep hole in the street at the bottom of the hill. A tiger escaped from the city zoo has taken up sheltor in the hole. To save yourself to become his iunch you aplly the brakes and lock your wheels at the top of the hill where your speed is 72 km’ h. The inclined street in front of you is 50 m long. The coefficient of frinction is 11, 0.9 and .7 between the tyres and the wet road. = 10 ms? (sin 20 = 0.3420, cos 20 = 0.9507) 1, Will you fall into the hole and become tiger’s lunch ? (a) yes (b)no (©) as the fate will be (@) insuffiuent data w calculate 2. What must your speed be to stop just before the hole. (2) 18 ms" (176 ms" (©173ms" (0) 169 ms" BRIM 1.) = 2.5 a= pg c0s20°—g sin 20° a = (0.9307*7-03420)g=0.31 g=3. . 20° ~ 2x31 2.(b) v=2%3,1*50=310 or v <176ms" Read the following passage and answer the questions given atthe end, You are part of a design team for future exploration of the planet Mars, where g= 3.7 ms?. An explorer is to step out of asurvey vehicle travelling horizontally at 30 ms-!. When it is 1200 m sbove the surface and then fall freely for 20s. ‘At that time, a portable advanced propulsionsystem (PAPS) will become ON. It will exert a constant force that will decrease the explorer’s speed to zero at the instant. She touches the surface the total mass (explorer + suit + PAPS) is 150 kg. Assume change in mass of PAPS negligible and no air resistance, Find the force in vector form. (242 )1425 (@)85_—_(d)None How much time explorer takes to reach the surface of mass? (@ILIs (324s (©4538 (85.65 3. The maximum force exerted on a passengerby the floor of the elevator isnotio exceed 1.6 times the weightof the passenger. The elevator accelerates upward with 146 Pearson Guide to Objective Physics ‘constant acceleration for 3m. Find the maximum. velocity ofthe elevator (a)4ms" (b)Sms* (@)oms* (7 ms" 1 RIMM |. (2) v, = 27 =3.7x20= 74 ms 'h= > gh = 5 * 3.7 Q07=740m. TAx74 74x37 _ 1369 2x(1200-740) ~~ aed ~ 230 91 ms? Yee. ar F =m(a,j +4,j) 1502.42 7 +5917) 4 (363 7 +8865 7)N 2 (b) t= 20+124=3245 3. (C)1.6m=m(g+a) @ = 6.2ms* Vas v 6ms' VOD Read the following passage and answer the questions given, atthe end. ‘A man is driving a classic 1954 Nash Ambassador with a girlfriend sitting to his left on the passenger side of the front ‘eat. The ambassador hes flat bench seats, The man wishes tobe closer to his girlfriend and decides to apply physics to achieve his goal for the romance by making a quick turn. The coefficient of static friction between the girlfriend and seat is 0.35 and the man is driving at 20 ms ! 1. What is the maximum radius of the turn so that she slides the man’s way? (@) 120m (b) 40m (©) 180m (46m Be ‘way the man should turn to achieve his romantic goal (@)left (b) right (©) anyway left orright (4) can not say 20x20 SM! 2 () QUESTION FOR PRACTICE A toy train consists of three identical compartment 4, B is being pulled by aconstant force F along C. of the tension in the string connecting AB (1:3 @r2 2. A block of mass M is pulled slong a smooth horizontal surface with a rope of mass m. The acceleration of the (ut block will be (@) FAM +m) () FM—m) (OFM @Fim 3. Abody of weight 50 Nis dragged on a horizontal surface with a force of 28.2.N. The frictional force acting on the body and the normal reactional force will be zt ¥50N Fig. 5.50 @2N3N (6)5N,7N, (©) 10N,1SN (d)20N,30N 4. Two blocks of mass 4 kg and 2 kg are placed in contact with each other on a frictionless horizontal surface. If ‘we apply a push of SN on the heavier mass, the force ‘on the lighter mass will be (a)2N (b) 4N (3N (d) none of these 5. A jar coniaining water is placed in a train, The train accelerates from left to right. Which of the following shows the wate level in ajar comectly? Ta 14 (a) (b) ) Fig. 5.51 A block of massm isplaced ona smooth inclined plane of inclination @with the horizontal, The force exerted by the plane on the block has magnitude (a) mg tan @ (b) mg cos @ (e) mgicos @ (amg ‘The work done in dragging a block ot'mass 5 h inclined plane of height 2 m ix 15 Joule. The work «lone against the frictional force will be (2)200 Joule (by 150 Joule (©) 100 Joule (4) 50 Joule ‘A boy of mass m with his mass centred at he standing in a train moving with constant acceleration a. Ifhis legs are wide spread witha distance 2dand he is not taking the help of any support then normal reactions at his feet are given by © 37 }s(-2) (b) m(a+g),mg Hina 2 -) @ Fg |e (@)_mg,mg 1 2 Fig. 5.62 Two masses m and M are lying on a surface moving with acceleration @, Only the given supporting and moving surface has coefficient of friction as. The frictional forces for y > alg and f= aig are 10, ML. Newton's Law of Motion 147 (2) ma, ma (b) ma, pmg (©) mg, pong (d) pmg,ma A small sphere of mass m is attached to a spring of spring factor kand normal length /. Ifthe sphere rosates with radius rat frequency v then tension in the spring is Fig, 5.54 (ayer (RUD fo) mr 2m (dy ‘Two masses each equal to mare lying on x-axis at (a, 0) and (+ «, 0) respectively. They are connected by a light string. A force F is applied at the origin along axis resulting into motion of masses towards each other. “The acceleration ofeach mass when position of masses at any instant hecomes (~ x, 0) and (+x, 0) is given by ™ (30) ao iF Fig, 5.54 Aweightless string passes through a lit over a pulley. ‘The slit offers frictional force /to the string. The string carries two weights having masses m, and m, where 1m, > m,, then acceleration of the weights is 0 Fig. 5.55 13, “we 15, (my =m g-f Lm, = mg Om Ons (my +m)e—f mg~f © Ga=my (Dn, +m.) ‘A particle of mass m is suspended from a fixed point O by a string of length /. At = 0, itis displaced from equilibrium position and released. The graph which shows the variation of tension Tin string with time ris — @) —s © @ Fig. 5.56 i ‘Two blocks of masses m, and m, areconnected to each other with the help of aspring. Ifpushing force is given to mass m, providing acceleration a to it, then acceleration of mis ma-F ja A stone weighing 1/2 kg is tied to a string 1/2 m long having withstand capacity of 20 kg. The stone is in horizontal circular motion over a frictionless table with a speed of 1.5 ms", Iftension in the string is equal 10 the breaking force of the spring, the speed attained is 16. 17, 18, Fig. 5.58 (@i4ms* (b) 11 ms" (o)24 ms" (17 ms" A body takes 1 times, the time to slide down a rough inclined plane ast takes to slide down the same inctined plane when it is perfectly frictionless, The coefficient of kinetic friction between the body and the plane for an angle of inclination of 45° is given by 12 ‘Two blocks of masses? kgand 5 kg are a reston ground. ‘The masses are connected by a sting passing over a frictionless pulley which is under the influence of a constant upward force F = 50 N. The accelerations of 5 kg and 2 kg masses are SON ore Lo Fig. 5.59 00,0 (@ 1 my, 2.5 ms* (@)0,2.5ms (6)2.5 ms, 2.5 ms A body starts t slide from P, down an inclined frictionless plane PQ having inclination @ with horizontal and then ascends another smooth inclined plane QR with angle of inclination 2a. Neglecting impact at 0 typ fon () fog foe (i =h e 19. A rod of length Z is rotated in horizontal plane with ‘constant angular velocity «A mass issuspended by a light string of length L. from the other end of the rod, Ifthe angle made by vertical with the string is @then angular speed, «o= Fig. 5.61 | ‘6 [ee] ® [es ‘giand gine | gin} olfeel © 20. A stone of mass 1000 g tied toa light string of length 10/3 m is whirling in a vertical circle. If the ratio of the 4 and g~ 10 ‘ms®, then speed of stone at the highest point of circle is ‘maximum tension te minimum tensior (a)20ms" (b) 10/3 ms ©)5V3 ms" 21, A man tries to remain in steady state by pushing his feet and hands against two parallel walls. Then for (@10ms* Fig. 5.62 (8) force of fiction should be equal on the two walls. (b) force exerted by him oa both walls should be equal and the walls should not be frictionless, (©) he should press his fest with greater force. (@) coefficient of friction should be equal for both walls. Newton's Law of Motion 149 22, A machine gun is mounted on a flat railroad car. The ‘gunis firing bullets at the rate of 10 bullets per second each of mass 10 g. The bullets are fired at velocity $00 ms" relative to the car. Calculate the acceleration and force of car atthe instant when its mass is 200 kg. - Fig. 8.63 (@) 0.25 ms®, SON (b) 2.5 ms*, 500 N (©) 25 ms*, SON (6) 8 ms?, 1600. N 23, A block of mass 1 kg is connected by a light string passing over two smooth pulleys placed on a smooth horizontal surface as shown. Another block of | kg is connected tothe otherend of the string then acceleration of the system and tension in the string are rig Ig Fig. 5.64 (a) Sms*,5N (by ims, tN (©) ms*,5N (@)5ms*,10N 24, Armas of2 kg isplaced on a wolley of 20 kg sliding on a smooth surface. The coefficient of friction between the mass and surface of trolley is 0.25. A horizontal force of 2N is applied to the mass. The acceleration of the system and the frictional force between the mass and surface of trolley are 3g |-——» 2 Fig. 5.65 (@) L8 ms?, 0.09 N (b) 0.9 ms? 18 N (©) 0.00 ms, 1.8N, @1ms2,2N 2S. In the shown system m, > m,. Thread OR is holding the system. If this thread is eat, then just after cutting. 180 Pearson Guide to Objective Physics 26, 27. 28. Se ore Fig. 5.56 (a) acceleration of mass m, is zero and that of m, is directed upward ) i 9, directed downward, acceleration of bth the blocks will be sume. m—m,) ©) (€) acceleration of system is given by where isa spring factor. ‘Three blocks of masses3 kg, 6 ky and | keare connected by a string passing over two sinooth pulleys attached atthe twoends of a frictionless horizontal surface, The acceleration of 3 ky mass is ke, (ims? (b)2ms ()3ms? (@4ms? ‘A peat] of mass mis in position to slide overa smooth wire. At the initia instant the pear is in the middle of the wire. The wire moves linearly in horizontal plane with an acceleration a in a direction having angle @ with the wire. The acceleration ofthe pearl withreference towireis (@) gsin 8~acos@ (c)gsin 0+acos 0 (b)g sin 8~g 00s @ (d)gcos 0+ asin @ Fig. 5.88 ‘A mass is resting on a rough plank. At initial instant a horizontal impulse is applied to the mass. Ifthe velocity of mass at instant f is v and displacement upto this instant is S then correct graph is 30. a. y s ff \ fa) x af . © A straight tube of length L. consis incompressible liquid of mass and the closed tube is whirled in horizontal plane about one of the ends. If «is the tuniformangular velocity, he force exerted by the liquid on the otherend is, Fig. 5.70 @ (o2ML a 4 © OMe 4 A light rope passes over a pulley. One section of the rope is held by achild and the other section by a men, then 674 the man and the child have same vector acceleration. the man and the child have same magnitude of acceleration but in opposite direction, @ (by the man and the child have: acceleration, ifferent magnitude of (@) the man and the child have accelerations which, keep on interchanging with each other. A.uolley is under the action of a constant force F. The sand contained by itis poured out through @ hole in the floor atthe rate of m per second. [finitial mass of sand 2. and trolley was M and initial speed wes u, then acecleration of trolley is given by Fig. 8.72 Oem Oem ‘A smooth track of incline of length fis joined smoothly with circular track of radius R. A mass of m kg is projected up from the bottom of the inclined plane, ‘The minimum speed of the mass toreach the top of the track is given by. v= (a) (2g(/cos 0+ R)C1 + cos 8]! (b) (2g/sin @+ Ry (©) [2g {I sin 8+ R (1 —cos 6)})!"? (d) (2g! cos 6+ Ry? Fig. 873 ‘A massless string of length / passes over a frictionless pulley with horizontal axis, Two monkeys hang from the ends ofthe string at the same distance /2 from the pulley, the monkeys start climbing upwards simultaneously. First monkey climbs with a speed 0 relative to the string and the second with speed of 20. Both monkeys have got same masses. The time taken by the first and second monkeys in reaching the pulleys are respectively. Fig. 5.74 (b) ez fi Newton's Law of Motion 151 34, Neglecting the masses of the string and pulley and ignoring the friction in the system, we find thet 4 Fig. 8.75 (0) weights fall freely. Pulley B rotates clockwise and pulley 4, C rotate anticlockwise. (b) the two weights have different accelerations. Pulley C rotates clockwise and 8, C rotate anticlockwise. (©) acceleration of masses will be zero and the system will be at rest. (@)_ acceleration of masses is equal to g. Pulley 4 and C rotate clockwise whereas B rotates anticlockwise. 35. A cage revolves around a vertical circle of radius R with constant linear speed «/gR . The cage is connected to the revolving arm in such a manner that a boy of ‘mass m remains always vertical while standing on a weighing machine kept inside the cage. It is found that Fig. 5.76 (@) the reading at lowermost point L is greater than the reading at highest point i7 by two times. the readings are same at all the points on the vertical circle. () (©) the reading at lowermost point Z is less than the reading at highest point 4 by two third. the reading at lowermost point L is five times the reading at the highest point #7. ‘A simple pendulum is vibrating with an angular @ amplitude of + ‘The value of a for which the resultant acceleration has a direction along the horizontal is. §a62-Rearson Guide Physics seg " oil | wlan? (@zer0 © Tomar ‘osu; lluey beg” Pye call moot k—maP (mia @ 39. A ropeis stetched"between tv boas at rest. A salor in the first boat pullsthe rope with a constant force of Fig. 877, 100 N. First boat with the sailor has a mass of 250 kg @= aca pig W)180® ‘whereas the mass of second boat is double of that mass. atelee If te initial distance between the boats was 100m, the rah +f loi cd “(4 time taken for two boats to meet each other is oe fio 08 98701 A) body of hrass-yn Starting’ from rest slides down a frictionless inclined surface of gradieit fixed on the l2vefldobof ait abdelerating upward with acceleration a. Taking width of inclined plane’as'¥?;'the time taken by borat. bddyto:shide from top to bottom of the planes Fig. 5.80 soini ativata Sy hh wi mek 9 vibe Wo obaiy Hpbi 6 bruens sda 9: . (0)3.188 x (318s, 40. A block of mass M is situated on a smooth horizontal 1a hay en table, A thregd ted tothe block passes through a hole aff Have at al inthe table and carries a mass ma its other end. Ifthe nue alae isnyy Zee org length of thread above the table is/ and Mis revolving eit safe td gga in horizontal circle with angular speed‘a on the table, —I——— then value of m so that it remains suspended at a Fig. 878 constant height his a aw ¥ ols a] ® “ aw weaqu 4 o(gtex) cr vane eta ec i i Fig's.01 38. A very small massiin3s fied to one end of a massless" ' spring of constant & and normal length /. The spring (a) Maio ey NGlak aa “ati the mass ate rolated about the otfier end of the la a ee “Gpllag with angular speed co Nepicel the eect of Mio? Foner 21 Oe, Ektension in'the spring is” ” o @ Map 41. A parachute of mass.m starts coming down with a constant acceleration. Defermine the ballast mass to be released for the'parachute to have an upward nest co #1 \ Meng 4 poate acceleration of me magnitude Neglet i drag aulugae ne hat oat r 8) o ay snl ata id 8 8 * ma 2m Oa, og Fig: 582 42. Block 4's keen ‘ee HloplaB (mass of B > mass at ee Bc ie Acta SY oie eae Ae aac eawealy Tap nea ole noieralobon ag nlite @ ig. 6.83 43. A circulac table has #adius of Imand mass 20 kg. It has 4 legs of 1 mceachifixed symmetrically on its circumference. The maximum weight which can be placed anywhere on ghis table Without toppling itis (@)83ke (34. ske, ©483ig (438g 4091 HH ie) eu ovtalA BR AMPRIND SrARaRiSh Hibs 44, A board is balanced on afough horizontal semicircular Jog. Equilibrium is obtined with the help of addition of a weight to one of the-ends of the board when the boerd makes an angle with the horizontal. Coefficient of friction between the Jog and the board is (tan 8 (b) cos @ (c)cot 7 Nosine ved wa ta to be Fgh 888 1" y are travelling at uniform speed producing rope A. The stronger planethen accelerates tion a. Tension in rope B just after the ‘acceleration is 88.2 09 pr =, “Fig. 5.26 MT sew siwarniase Ann OTETPM jovi & 8 bel A ces nase ph iso SOM MAL VN ash ei ia Niloety's a iba lo’ alte passing, through Ia boul Gt altedl df Pak of halsance ie directly proportional to the ae Time of motion of bullet in-the board is given by w= Rens @t0-8) a wheg,*i vlog i © @ 9 tlog, : woog, 47. A rocket of mass mis fired vertically upward and after the fuel burning it weighs’. Ejection of fuel gas is at ‘constant rate of m, per second witha constant velocity of v, relative to the rocket. Final speed of rocket after the complete bureaus of fuel is given by U= sad SPs mia cg 4 vic (8) 4g logy “orl i 4 8p ica lial me dm (014g tog, (the 154 Pearson Guide to Objective Physics Fig. 7 48. A chain of length is lying in a smooth horizontal tube such that a fraction of its length A hangs freely and the end touches the ground, Ata certain moment the other end of chain is set free. The speed of this end of chain ‘when it slips out of the tube is es a (a) [eens ) fh at @ (2st wong 4. A block of mass A with semicircular track of radius R rests on a horizontal smooth surface. A cylinder of radius r slips on the track. If the cylinder is released from rest from top, the distance moved by block when cylinder reaches the bottom of the track is M(R~r) (a)R-r (b) Mom © Hay AP Oman Fig. 5.89 40. Let pt be the coefficient of friction between blocks of mass m and M. The pulleys are frictionless and strings are massless, Accleration of mass mis 2ng (0) 5M +2um ( —Yne Sim 2am 25 SY21 © ane @ te +5m+2pm m+VSm+J2pm w ut Fig. 5.90 SL. A chain of length / is placed on a smooth spherical surface of radius,r with one ofits ends fixed at the top of the surface. Length of chain is assumed to be / < 7 acceleration of each element of chain when upper nd is released is Fig. 591 fs r ws {1-55} a ran?) oF ( sine 52. A board of mass Misplaced on a rough inclined plane ‘and a man of mass m walks down the board, If the coefficient of friction between the board and inclined plane is 1, the acceleration of the man, such that plank does not stip, is given by Fig. 6.92 @) as (cos @+ ysin |g (sin 6+ pcos 8) ¢ @ (sin 6+ eos 8) g Cr) () a2 (“22 eine noose (*") (rr) Newton's Law of Motion 455 $3. large free mass Mand a small mass mare connected Gi Bing toa string such that m moves in horizontal circle. Length Opie: ae of string is /and is the angle this length makes with Set Mee) vertical. The frequency. ‘of rotation of mass m so that M56, A block of mass m slides down an inclined tightangled remain at rest is trough. If the coeffisient of kinetic friction between the block and the trough is 4,, acceleration of the block down the plane is Fig, 5.93 © x Ra Fig. 8.96 (a)g (sin @- 21,008 6) (b) g (sin 8+ 2, cos 0) ot (©) g (sin 8+ JZ, 005 8) (A) g (sin O- 14,cos 0) 2e Vind 57. Acylinder of radius r= 1 mand mass m= 5% 10° kgis 44, Two blocks connected by a massless string slide down at rest on the edges ofa structure as shown. Distance a 4n inclined plane having angle of inclination as 37°. ‘The masses of two blocks are 4 kg and 2 kg with jt as 0.75 and 0.25 respectively. Bare (@ The common acceleration of two masses is 1.3, ms? and tension in string is 5.3. (©) Tension in the string is 14.9 N, (©) Acceleration of the mass is 3 N. (@) The acceleration of mssesis 5.3 ms and tension inthe string is 1.3N. i0Smandbe 2m Raton ton eget Fig. 6.07 he (8) 24.6KN,24.6KN (6) 42.6 KN, 42.6 KN (©)42.6KN,24.6KN (4) 52,6 KN, S.OKN $8. The force acting after time s on the bottom of a beaker of area A when water of density p falls from a tep at a height hat uniform rate vm Fig, 58, Accelerations ofthe vehicle and mass m,, when pulleys ‘are light and all surfaces are frictionless, are : ame ” (oop [st+24( 4 } (b) up [ar+ 28h] m, mg ty (e) each rp +m {®) Sim, +m,* dm, +m, (wgh Z u cath Bie wow [nee] 156 Pearson Guide te Objective Physics 59. ‘Two blocks B, and B, of masses m, and m, respectively are comnected with the help fa pulley’ string as shown. Upper surface of vehicle it smooth but vertical PR IRR ds cil ‘re tito pra pg lh a] id 02) TBs elt aves Fig, 5.98 Given a = g/7 and m, =7.5 m,. Coefficient of friction between block B, and side of vehicle is (@o4 (0S (2) 0.6 882 gd 03 (60, Sixteen bealsin-a string aesplaced of a smooth inelined Jane of inclintion sin! (1/3) such that some of them ie aldng the incline whereas the rest hang OVer the top lof the plane. If acceleration at first beads: g/2, the? perabgprecainChends isthat sul na 10 Lat eyby negated few a batt 20 4 Fig. §:100 @ 12hang vertically. (©) 10tieatong incline plane. 4 een ia bE AL AEE Um OPE ret PADAE AAD SL (a) 61. A mass M is hung with a light inextensible st "tease in hofionil pal oF site 07" (a) J3 Mg Me () Vi (044190) 62. Aship of mass 310" kg inal a est npaled By force of 5 x 10'N through a distance of 3 m, Assuming that resistance due to wetas is eae id speed of ship is ; iP esther PAPLOLMT sit yank A 2 ssf) + CELT. 1990) 63. "A built of hiais Mig fied ith bel of 0 mis! at an angle @ with the horizontal. Art the highes! point of trajectory it collides wit a bob of mass 3M suspended A 10 vertically by amasslessstting oflength =" mandgets, ‘embedded into it, After the collision the string moves through aa angle 120°, what is the angle of throw 8. {a) cos {c) cos" 64. Acaris. ving Jnaggircular norizon icklofzadius 10 mwith’a condtant speed of 10 ms /A'phittb bob is, nu sspRna from. Abm aot af saraxalightsiaid sad of; “length mit Fhe'angle madelby tod witithe traéleis ‘Wi AS ton yA be cake sada 26.4 bra 25.0 a OE can on te aneralySD Gaus aut rs woianon nus eT. 1992) peat ot ns apreboung nih qo pecs The o's apebouny wt ese sped, The tall remains: wordt Witt ne. e afice for 1.01 S'The \irawetage fored exerted by the aurfacd om tall is. (>) (10 “eytontn ot (IN OS @OIN . (Roorkee 1993) 66. Tension in rod of length Zand mass M at a distance y from F, whei the red isacted oa by two unequal forces F, and F, where (F, < F,) at its ends! @)F(l-yiL)+F,QIL) (by EAy/L) + F, OIL) (FC. +y/L)+ FGM)? WB F, (1+ y/L) +P, GL) coli da eee Dia al te TOOT 67. The ili UEOkcd Gin ALE Ba aly Mies with dime (in )a8 shownyA@, Cand ED ate straight line seements, The magnitue of total impulse of force cn the body from 1= 4 jg to 12916 05 Vlas exzznin fT woo I 0. 00 a a tous te or . TH is ‘Fime (us) > ay to We ne, Fig. 5.102 WA) BSOHOANS sb AY ey Ssevonge (tet eve! (6x 109NE 68. A smooth semicircular wire Gack of radiis Rs fixed in Nertical plane! One ed ofa massless spring of nitural vo ength3R/4 is attached so the lowest.paint Qof the wire track. A small sing of mass m which can slide on the tothe other end of the spring, The ring Gs he seit ‘at pdint P such tha the le Ai anangle 0" withthe verical Sprig nist = mg (0 RUT spring force is“ nats een ngutn2) > 4a noitsiit annie ¢2) anitoiit faito) sin auc ero lRgeBADBOiraiit quintet chy 2s! moa bors wall dguen x no bovalg zi Auold A on QB exit 0 aer0t oA Gprigitgys 2° ant Vo ould inssttib 101 baw ei Aoold oft ne Wolk uno wy atad bottolg 2 tq © Fe vroleto vel wai Ag 04 oY cn “CLL.T!1996) i re ora pct on oa Fixed! trial pil! Weage' bY nieitis OF inssiess, inextensible string and a frictionless pulley'as shown Tre costtcentet even beer ost dane, tion berwe LoVe si lait are 1 tw joni Myler wa dqe a rnalinw @ hia yavzoy ey vit i ows A g:ilizulay vd aun od teat capsdinsatstt (a)zerojoyontott ily) Dima > (©)2ms? ow ay @)3 ms ys lo esbusingem ot insavino A haw TO: 1A! large Neavy box is wiiding without friction:dbwn a ‘snidotk phate of énclination-@)Frota a/poiab Pioa the botiom pf the bony particle is projgcted-ipsidg the box. Thg inital sped of particle wih rspet tothe box is wan ion of projection makes fi tingle ie dt tcc noob ‘distancoidlongt the botpmof hex. between. the, point, of projection P vBY3 AON) XE ccieyts Newbettactalriof Meter 52 and point © where the: particle lands. (Assurne that the particle doesnot hitony other surface of the box, Neglect on ESRI 1 rier is eis Fig. 6.406. uw’ sin2a, = w sin? a ot geod | w'sinze it wsing © "Fe0s0 @ sore of (LLT. 1998) T1. A spring of force Cdhstant K is cut iiild tivo fitces such that one piece is double the length of the other. Rae wea i ave a forre omeRL Ty @snk’ antoanenet TA) (6K ssnyit sie rasianibog oft tial a ie fat i oh rire surface with coef ie teak fozoaldPiorce FFis applied ontheblock as shown. 1f the coefficient of ufejently high so that lock does not slide before tapping. the minimum foie required to [wr cor. pit Ota) 0 Fig. 5-406 ao) wae Payal a i” Oye gid i i i Hie) hastenoo yninyita goin teobl 0A, SDB a cue PBB tin Aion uitige oh dive boeuat Ald Ee Sremmting, 200) ‘Tah ao daweet erate A NeiEbMeH EdD SPRL Very slowly. The coefficient of friction between the sutface and the insegt.pv1 df the line joining thp gentee pf the hemispherical supfqes to the insect makes an,apigle with the Tertiat the maximum possible RNs oPeris colar nA ar at Ave = Aor 4 (@)cota=3 (e)sec a3 (bytan a=3 (Acosec a= 3 (LLT. Screening 2001) ‘A string of negligible mass going over a clamped pulley cof mass m suppor a block of mass M as shown in the figure. The force on the pulley by the clamp is given by Fig. 8.108 () 2 Mg () V2 mg © giMrmrm = Ag (Mim + 80. A bloc! (LET, Screening 2001) The pulleys and string shown in figure are smooth and of negligible mass, For the system to remain in equilibrium, the angle @ should be © © ez ca] (vam [a] a. Fig. 6.109 wor (030° (ease (60° (LIT. Sereening 2001) ‘An ideal spring with spring constant é is hung from the ceiling and a block of mass Mis attached to its lower end, The mass is released with the spring initially unstretched. Thenthe maximum extension inthe pring ®2- is (a) 4 Magik (b)2 Mg/k (c) Mgik (¢) Mgizk (LIT. Screening 2002) 83. Let F, F, and f denote the magnitudes of the contact force, normal force and the friction exerted by one surface on the other kept in contact. If none of these is 2210, (F>F, FS wroe OF -SeFekytf 78. 79. ‘The contact force exerted by # body A on another body B is equal to the normal force between the bodies. We conclude that (@)_ the surfaces must be frictionless. (b) the force of friction between the bodies is zero. (©) the magnitude of normal force equals that of fiction, (@) the bodies may be rough but they don’t slip on each other. Mark the correct statements about the friction between two bodies. (@) Static friction friction. Coefficient of static friction is always greater than the coefficient of kinetic friction, Limiting friction is always greater than thekinetic, always greater than the kinetic (b) © (d) Limiting friction is never less than static friction. placed on a rough floor and 2 horizontal force F is applied on it, The force of friction /by the floor on the block is measured for different vaiues of F and a graph is plotted between them. (@) The graph is a straight line of slope 45° (b) The graph isa straight line parallel to the F-axis. (©) The graph isa straight line of slope 45° for small rand strsightline parallel tothe F-axis for large rn (@)_ There is a small kink on the graph, ‘Consider a vehicle going.on a horizontal road towards ‘east. Neglect any force by the air. The frictional forces ‘on the vehicle by the road (0) istowards east if the vehicle is accelerating, (b)_ ts zero if the vehicle is moving with # uniform velocity. (©) must be towards east, (@)_ must be towards west. ‘When Neils Bohr shook hand with Wemer Heisenberg, ‘what kind of force they exerted? (@) Gravitational (b) Electromagnetic (Nuclear (Weak Let E, G and N represent the magnitudes of electromagnetic, gravitational and nuclear forces between two electrons at a given separation. Then @N>E>6 (E>N>G ()G>N>E (@E>G>N ‘The sum of ll electromagnetic forces between diferent particles of a system of charged particles is zero 86. 87. 89, 90. on @ (b) © only ifall the particles are positively charged. only ifall the particles are negatively charged. only if half the particles are positively charged ‘and half are negatively charged. (@) irrespective of the signs of the charges. A.60 kg man pushes 240 kg man by farce of 60. The 40 kg man has pushed the other man with a force of (@)40N wo (60 (20 ‘A neutron exerts a force on a proton which is (a) gravitational (byelectromagnetic, (©) nuclear (weak [A proton exerts a force on a proton which is (b) electromagnetic (@) weak (@) gravitational (c)nuclear ‘Mark the correct statements: (a) The nuclear force between two protons is always ‘greater than the electromagnetic force between them, The electromagnetic force between two protons, is always greater than the gravitational force between them, ‘The gravitationel force between two protons may be greater than the nuclear force between them. Electromagnetic force between two protors may be greater than the nuclear acting between them. [fall matter were made of electrically neutral particles such as neutrons, (@) there would be ro force of friction. (b) there would be no tension inthe string. (©) it would not be possible to sit on a chair. (@)_ the earth could not move around the sun. Which of the following systems may be adequately described by classical physics? (@ motionofacricket bal. (®) motion of adust particle. © (d)_ a neutron changing to a proton. The two ends of a spring are displaced along the length of the spring. All displacements have equal ‘magnitudes. In which case or cases the tension or compression in the spring will have a maximum ‘magnitude? (a) the right end is displaced towards right and the leflendtowards left. (b) both ends are displaced towards right. (b) © @ @ hydrogen atom. 22, 93. 94, 96. 97. both ends are displaced towards left. the right end is displaced towards left and the left end towards right. Action and reaction (a) act on two different objects. (b) have equal magnitude. (© have opposite directions. (@) have resutantzero. ‘A force F, acts on a particle so as to accelerate it from rest toa velocity v. The force F, is then replaced by F, which decelerates it to rst. (a) F, mustbe equal to F, _(b) F, may be equal to F, (€) F, must be unequal to F, (d) none of these ‘Two objects A and Bare thrown upward'simulteneously with the same speed. The mass of dis greater than the ‘mass of B. Suppose the air exerts a constant and equal force of resistance on the two bodies © @ (a) the two bodies will reach the same height (b) Awill go higher than B. (©) Bwillgo higher than 4. (@) any of the above three may happen depending on the speed with which the objects are thrown. Assmooth wedge 4 is fited in a chamber hanging from a fixedceiling near the eerth’s surface. A block 8 placed atthe top of the wedge takesa time Tio slide down the length of the wedge. Ifthe block is placed at the top of the wedge and the cable supporting the chamber is, broken at the same instant, the block will (@) take atime longer than Tto slide dowa the wedge. (b)_ take atime shorter than T to slide down the wedge. (©) remain at the top of the wedge. (@)_jump off the wedge. Inan imaginary atmosphere, the air exerts a small force F onany particle in the direction ofthe partile’s motion. A particle of mass mprojected upward takes atime, in reaching the maximum heightand in he return joumey to the original point. Then @) <4 ny © 4m (4) the relation between , and t, depends on the mass of the particle. A person standing on the floor of an elevator drops @ coin. The coin reaches the floor ofthe elevator in atime 1, iftheelevatoris stationary and in time f, if itis moving uniformly. Then @inny Ost, 460 Pearson Guide to Objective Physics © (@ 4, <4, ort, >4, depending on whether the lift is going up or down, 4>h 98, A free 7" nucleus kept in a train emits an alpha particle. When the trains stationary, a nucleus decays and a passenger measures that the separation between the alpha particle and the recoiling nucteas becomes x at time ¢ after the decay. Ifthe decay takes place while the train ismovingat a uniform velocity v, thedistance between the alpha particle and the recoiling nucleus at atime t after the decay as measured by the passenger is (@ xt ()x-w @x (@) depends on the direction of the train, A reference frame attached to the earth @ o is aninertial frameby definition, ‘cannot be an inertial frame because the earth is revolving around the sun. is an inertial frame because Newton’s laws are applicable in this frame. cannot be an inertial frame because the earth is rotating about its exis, 100. A particle stays at rest as seen in a frame. We can conclude that © o © © @ the frame is inertial, resultant force on the particle is zero. the frame may be inertial but the resultant force on the particle iszero. the frame may be noninertial butthereis anonzero resultant force, 101. A particles found to be at rest when seen from a frame S$, and moving witha constant velocity when seen from another frame S,, Mark out the possible options. (@)Boththe frames are inertial, (b) Both the frames are noninertial. @ (©) 5, is inertial and 5, is noninertial. @ 102. Figure shows the displacement ofa particle going along the X-axis as a function of time. The force ecting on the particle is zero in the region () 48 (b) 8c (co (@) DE S, is noninertial and S, is inertial Fig. 5.110 103, 104, 108, 106. 107, 108. 109, Figure shows a heavy block kept on a frictionless surface and being pulled by two ropes of equal mass m. ‘Ats=0, the force on the left rope is withdrawn but the force on the right end continues to act. Let F, and, be the magnitudes of the forces by the right rope and the left rope on the block respectively. II 4 Fig. $.111 @ F,=F,=Fforrs0 ) F=Fi-F+mgforr<0 (©) F\-F,F,=Ffort>0 @ F0. ‘A monkey of mass 20 kg isholding a vertical rope. The rope can break when a mass of 25 kg is suspended from it. Whatis the maximum acceleration with which the monkey can climb up slong the rope? ()7ms* ()10ms? ()Sms* (2.5 ms A force of 5 Newten acts ona body of weight 98 Newton What is the acceleration produced in ms*? @osi (1.46 (049.00 (@)5.00 ‘A body of mass m is released from the top ofa rough inclined plane of length 1. If the frictional force is f then the velocity of the body of the bottom in ms" will be @) [2erat—p fen oe A block of mass 2kg is lying on a floor.The coefficient of static friction is 0.54. What will be the value of frictional force ifthe force is 2.8N and g=10ms” (&)2gh-fl (@)zero (a)z2r0 (e)2N (928N @8n A cube weighing 10 Nis lying on a rough inclined plane of slope 3 in 5. The coefficient of friction between the plane and the cube is 0.6. The force necessary to move the cube up the plane will be (a) 64N (b)108N (©)21.6N (6) 108N A block of metal is lying on the floor of a bus. The ‘maximum acceleratin which can be given tothe bus $0 that the block may remain at rest, wll be @ue (He ug {d) ig 110. Abody of weight wis lying atrest on a rough horizontal surface. Ifthe angle of fiction is 8, then the minimam force required 10 move the body along the surface will be (@)w cose (b) w tané (Owsin 6 (4) weote LIL. A block of mass 0.5 kg. rests againsta wall exerting a horizontal force of 10 N on the wall. Ifthe coefficient of friedon between the wall and the block is 0.5 then the fictional force acting on the block will be (2) 499N (0) 98N (490N (4) 049 N 112, A rope of length /is pulled with a constant force Tis the tension in the rope ata point distant x from the end ‘where the force is applied. Then Tis @fU-ail (b) fl =x) Y-x fl or > 113, Two masses m, asnd m, are attached to a string which pass over frictionless fixed pully. Given that m, = 10 kg asnd m, = 6 kg and g = 10 ms, What is the acceleration of the masses? (@)2.5 ms? (b) Sms? (© 20ms* (a) 40 ms* 114, A block isying on the ble. What isthe angle between the action of the block on the table and the reaction of the table on the block? (@ 180° (e)90° ase @r 11S. A parachutist of weight w strikes the ground with his legs fixed and comes to rest with an upward acceleration of magnitude 3 kg. Force exerted on him by ground during landing is @4w (b)3w 2 (dw 116. The force that prevents the relative motion between the layers of a liquid is called (@) stati friction (©) sliding frietion (© contact friction (d) none of these 117. Gravels are dropped on « conveyer belt a the rate of 0.5 kgs". The extra force required in newtons to keep the belt moving at 2 ms*is jos wy @2 a4 IB. Starting from rest, a body slides down a 45° inclined plane in twice the time it takes to slide down the same Newton's Law of Motion 161 distance in the absence of friction. The coefficient of friction between the body and the inclined plan« @)025 (60.33 (€)0.75 (0.80 119. When we walk once, we should take small steps to avoid slipping, This is because smaller steps ensure (@) larger friction (b) smaller friction (©)largernormal force (4) smaller normal force 120. A chain of length Z and mass m is allowed to fall ona table such that the part falling on the table comes to rest instantaneously. The force acting on the table when ‘part of it has lied on the table is Smal ml — oO nile 3mig oF Om 121, Two balls of mass tkg and 2 kg respectively are connected to the two ends of the spring. The two balls are pressed together and placed on a smooth table. When released, the lighter ball moves with an acceleration of 2 ms. The acceleration of the heavier ball will be (a) 0.2 ms? (b) 1 ms? (©)2ms? ()4ms? 122, A fireman wants to slide down a rope. The breaking load for the rope is 3/4° ofthe weight ofthe man, With what mizimum acceleration should the fireman slide down? Acceleration due to gravity is g. (a) zero ) = o* oF 123. A rain drop of mass 0.1g is falling with uniform speed of 10 cnn’. What is the net weight of the drop? (a) 10° N (B)109'N ()2*10°N (d) zero 124, A heavy unifrom bar is being carried by two men on theieshoulders. The weight ofthe bars. [Fone man lets fall fom the end care ty him, what wll be the weight experienced by the other? (a) none of these (b) wid te) wi2 (dw 125. The coefficient of friction of an inclined plane is 1/ 3. If it is inclined at angle 30° with the horizontal. What will be the downward acceleration of the block placed ‘on the inclined plane? 162 Pearson Guide to Objective Physics @o (©) 3 ms? 126. A body is projected upwards with 2 kinetic energy of 100 J. Taking the friction of air into account, when it returns on earth, its kinetic energy will be (a) more than 100 (b)less than 1003 (©1005 (4) none of these 127, Which ofthe following is a self adjusted force? (@)sliding friction (b) static friction (©) limiting friction (@ dynamic: 128. A body isplaced over an inclined plane of angle x ~ 6. ‘The angle between normal reaction and the weight of the body is (@) equal tothe angle of friction (b) more than @ (6) less than @ we 129, The frictional force dueto air ona body of mass 0.25 kg falling with anaccelerationof 9.2 ms: will be (@)0.15N ()LSN (SN (@)zer0 130, Ifa rough surface is polished beyond & certain limit than the magnitude of frictional foree will (@ nothing can be said (b) some time increases and some time decreases ©) increase (@ decrease 131. A car is moving on a straight horizontal road with @ speed of 72 kmh1. Ifthe coefficient of static friction between the tyre of the car and the road is 0.5, then the rninimum distance, within which theearcan be stopped willbe @72m (40m ©30m @20m 132. When wekicka stone, we get hurt. Due to which one of the following properties does it happens? (a) velocity (b) momentum (©)inertia (@) reaction 133. A cricket player catchesa ball of mass 100g and moving witha velocity of 25 ms". Ifthe ballis caught 0.1s, the force of the blow exerted on the hand of the player is @4N (e)40N (©25N (@)250N (b) J2 ms* (a)3ms* ton Read the followingpassage and answer the questions given attheend. Suppose that you were called upon to give some advice toa lawyer concerning the physics involved in one of his cases. ‘The policeman has charged a driver for breaking speed limit of 60 kni/h and had arrested him. The length of the skid marks are 15 m when he made an emergency stop with brakes locked. The policeman had made a reasonable assumption that the maximum: deceleration of the car could not exceed & 1, Is the policeman right in arresting the driver to break speed limit? (a) Yes (b) No (©) insufficient data to reply (d) the policeman demanded bribe 2. Candeceleration of car exceed ‘g"? (a) Yes ()No (©) depends upon friction coefficient (d) none 1. (a) Since the deceleration will be due to friction. Frictional force F,= HN. Assuminglevel road F,= Mg or a= pg. Since <1 +. deceleration normally cannot exceed g. Distance covered by speeding car due to skidding s 2X10 2. (b)a> gif x> 1 which occurs only in extreme cases. Blexzon lens i Lett pore Y eflection system Fig. 5.112 Read the following passage and answer the questions given attheend. In a cathode ray tube as shown in fig. 5.136 electrons are emitted by heating the cathode and focussed using focussing system on the flourescent screen. B is the magnetic field applied. £ is the electric field acting. The electrons go undeviated when both electric and magnetic fields are applied. Under this condition a voltage is applied on yplates, Flourescent ‘and the spot shifts down by 2 cm. The separation between deflection plates and screen is 20 cm and separation between. }plates is 2 cm. B= 2 mT and F = 2.0 kV ar! 1, What voltage on y-plate will give such a deflection. @ory iy uv (@nore 2. If cathode has potential -200 V. what should be the potential on 4,, (a) +300V (b) +500 V (©)-100V (@)-20V 3. The path with which electron travels from y plate 10 sereen is (@) parabolic (b) hypetbotic (©) circular (@) straightline (©) helix a 1p = Lg IOV AIO 4 * 29X10 x2KIO™ 10s OV eg < VeayoS0 that electrons are replaced. 3. (a) as horizontal velocity is constant and vertically electric force acts, Read the following passage and answer the questions given attheend. ‘An adventurous archaeologist crosses betwen two rock clffsby slowly going tand-over-hand along @ rope stretched between the cliffs. He stops to rest at the middle of the rope as shown in Fig. The rope avill break if the tension in it exceeds 2.5 * 10°N. The mass of the archeaologist is 90 kg. ‘The rope makes an angle ¢= 10° [sin 10 =.1736: cos 10 = 9848] 4 B Fig. 5.113 1. Find the tension in the rope. (@3210N ()2970N (©)2867N (@2580N Newton's Lawof Motion 163 2. What should be minimum value of @ so that rope does not break? (aI? (2° (co) LS° (d)2.2° 1d) 2 Tsin 9=Mg Tene 6+ Tsin ‘Read the following passage and answer the questions given atthe end. ‘A man is working fora shipping company. His job is stand at the bottom of a8 mlong ramp that is inclined at 37? above the horizontal. He grabs packages off conveyer belt and propel them up the ramp. The coefficient of static friction is 0.5 arc. coefficien of kinetic friction is 0.3 between the packages and the ramp. His coworker is a lady whose job is to grat the packages as they arive the top of the ramp with 2e19 velocity. She misses one and it slides down back and reaches back to the man, 1. Find the speed with which man should push the package? (2) 154 ms? (b) 143 ms (©) 12.8 ms! @ 116 mst 2. With what speed the package returns back to the man? (25.5 ms" ()7.3 ms" ()11.6ms" (@84ms! we ot got wt Fig. 5.114 4184 Pearson Guite to GojectvePrysics o 1(d) deceleration a= (g sin8 + ug cos 8) go tl ott v = Jas =/2(gsind +g c0s6)8 ° = -[2x10(6+3x8)8 ee Fig. 5.115 v = Jed =4(2.9)=11.6ms! foxes = JPe10(6=3%8)8 = Jiexaa 20) v= J2(gsind—pgeos8)5 = 73ms! tua ae} @ 2 2 4 @ 5 . 7 © @ 2 0) v ©) 1 ©) 12 (a) a) 4 ©) ® 1% ©) 7 (3) 19. (a) 2. 2. (©) @) 2. (3) 2. (6) 2. (@) 28.) 2. 2. fe) 30.) a) 2 © 3. (a) 4 @) 35. (@) ©) a) Bd) » 0. ©) “a @) a2 (c) 44. (a) 4%) 4%. (2) 7. @) a) 49. () () 51. (>) 2. (0) 53. (@) (a) 55. (0) 56. (c) (e) 58. (a) 59. (b) 60. (4) (a) 62. (b) 63. (c) (ce) 65. (b) 6. (a) 7. (c) (d) 69. (a) 70. (c) (b) T (e) Ta) 4 (a) (c) 76. (b) 7. (aba)| (b.d) 78. (b,c,d) B. (c.0) ‘81. (a,b) 82. (b) 83. (d) 84, (C) (s) 86. (a,0] ‘87. (a,b,c) $8. (b,0,d). 89. (abc) 90. (a, b) 94. (ad) (b) 94 (b) 95. (c) 96. (b) 97. (a) 98. (c) (c,d) 101. (a,b) 102. (a,c) 103. (a) 104. (d) 108. (¢) (©) 0.) 0.) HO) © 1% @ He @ 4 He. ) 1). (d) Aa}, ®) 128, (Ce EXPLANATION 8. (a) If, and N, are the normal reactions at one foot and ~ the other foot, then or N+N=me o For equilibrium, (ma) H= aN, -dN,=d(N,-N,) (taking moments about centre of mass) ad N= = («-#) 9. (6) The maximum force of friction, Fig. 5.116 Som = HIM Taking a less than a,... friction fis also less than /... Here, ifthe value of t> alg then f¢ pmg rather f= ma. (c) Tension in the spring is actually the centripetal force given by mroy’ie., mr (2xvy. 10. AL, (c) Let be applied at origin from figure F =2Tcos F 2eosd Then force causing motion is given by T Tsin6 = (a3) sin @ 26088 F jr \ Boexo 00.0) a Fig. 8.117 <-aceelerati 12, (a)Heremg-T,=m, a, and T,-mg= ma —e2 Also(T, +) —@ (cs tension below the slitis 7, and above the slits 7). —) Solving the above equations, 1B. 14, and or or Newton's Law of Motion 165 (mm, =m a~f (m +m) (b) The particle is displaced from mean position and then released, i.c., at = 0, the tension is minimum because the particle is at the extreme position where the tension has to balance only the radial component of the weight of the particle. Tension is maximum at ‘mean position because it has to provide the weight as well as centripetal force also. (b) The force on mass m, is F, = ma and force onm, is, F,= ma but = F,+F=matmd F-ma . (8) Here, 20x9.8= 7“ = 20x98 v = Ji0x08 = 14ms" (©) Letate the acceleration down the rough plane and a be the acceleration down the frictionless plane. ‘Taking £ as the length of the inclined plane, we get. a = g (sin 0- p036) a = gsinO=g os ged oe Then = 3 att= 5 at, L 1 = (1-pr orp= (5) (a) The masses will be lifted if the tension of the string. is greater than the gravitational pull on masses. SON no hr 20N_ SON Fig. 5.119 166 Pearson Guide to Objective Physics ‘Weight of S kg mass = 5 x 10 = 50 N and 2 kg mass Tug Ut gh+ oh =2x10=20N Then from (jy = = SATE EE From free body diagram: - 50-270 or 7=25N . 10 So5 kgweight can not belifted(-; acceleration = 0) but maaabuairy 2 kg weight will be lifted. oe = 3 2.28-20=2a0r1 or 30} =300=0r y= 10m! 21. (b) For equilibrium, the forees exerted by both walls on the man should be equal so as the horizontal forces 18, (d) Planes PQ and QR are frictionless and impact is neglected, so mechanical energy will conserve, nd may balance but the vertical forces can be balanced 19. (d) Radius of horizontal circle of ball even if the forces of friction on the two walls are =(L+Lsind) unequal. CP. Acceleration=(L+Lsin a (1. a=ra%) Here me= Tos 6 22, (a), Here, reacting force = relative velocity x Fe and mar (L+L ‘ixil’ Dividing Gi) by i) =x 000 ) sow w(L+Lsind) (taking time = 1's) & Using F= ma, we get F_ 50 oo @ Fn 39 725s? : 23. (a) Considering free body diagram, 20. (4) Ty = “EE +g Gor hanging mass) and T= ma (for mass lying on surface) ‘Adding mg=(m+m)a or a= 3 2m Fig. 5.120 mp and Tyg = eB and T=1x5=5N 24, (©) Limiting frictional force = mg my =0.25x2x10=5N Te pe: Then + So the block and trolley will not have relative motion Ton M8 ng for a foros of 2N. & Here,2=(20+ 2a _ Utgl . or “ o Using ¥ 1? =2 a5, weget 2g(2L)=—4gh 28, (a) On cutting of string QR, the resultant force of m, 2 +4gh remains zero because its weight m gis balanced by the ‘ the spring but on block m, a resultant upward or 26. or 2. 28. 29. force (m,—m.)g is developed. Thisblock m, will have no resultant acceleration whereas m, does have an upward acceleration given by “1B _ (b) Here 7,-7,=6a, T,~Ig=laand 3g- 1, r, is Fig 5.121 Addition of the above three equations give 10a=3g-1g=2g 2 2 7g *10=2ms (@) Let @, a, and a, be the net leftward horizontal acceleration of bead, net downward vertical acceleration of bead and relative acceleration of bead with reference to rod respectively. Then = 0,005 8+ a 1, =a, sin 8 Projecting forces vertically and horizontally mg —N cos 0= ma, sin @ Ci) =m (a, cos 8a) (i) From (i)and (i) and sin friction. Thus, v~ ¢ graph is a straight line having negative slope and a positive intercept on vaxis, s~f curve will be having a decreasing slope till it reduces to zero because velocity of block decreases continuously and remains positive til the block comes to rest position. (c) Let there bea small element of length dl ata distance 4 from the end of rotational axis. Mass ofthe element = “a ‘Small radial force on this element M “ (£4) te? iy. «. Total force: \(e« er Newton's Law of Motion 167 30. (b)Considerthe manand the childto be simple masses hhung from two ends of 2 string passing over pulley. Being e system, the man and child both have same ‘magnitude of acceleration but opposite directions. 31. (@) Instantaneous acceleration, dy ‘di ~ Tnstantaneous mass Constant force a F or a= Mami Ge rate of fall of sand per second ism) (©) Using ui = 2aS we get w-r=2C)H 32. Fig. 5122 je, -W=2C-9)H,+h) but h, =/sin @ h,=R(1—cos 6) 2g (Isin 9+ R (1 —cos ) or u=[2g {I sin 8+ R (1 ~c0s }!2 (@) Relative velocity of monkeys =v+2v=30 Tos distance covered= 4 + 34. (@)Herem,g—T=m,, (tating a, as downward acceleration of m,) 27-mg- moe, (taking a, as upward acceleration of m,) 2r-T=0 Thus, T=0 - ‘Thus, the masses will have free fall. Clearly pulley B rotates clockwise and the other pulleys, in anticlockwise direction. and Gz mass of pulley is zero) 168 Pearson Guide to Objective Physics 35. (a) Athighest point v= J/Rg and radial acceleration, a, (tress ta } Thus, the weighing machine will not record the weight * Gtasiner of the man. At lowest point, the boy has a resultant force equal to w 4 _mg acting upward so the normal reaction has to balance - [eaten the weight ofthe boy mg and als to provide net upward force also. Hence, the machine will record double the va weight ofthe boy. _ [ aw : . Geosarsinaye ta) 36 (Ota, = 2 = Catena * _ aloes - lates 7 = 38 (4 Here spring fre = centripetal fre =m (+ x) @ (where = is the extension in the a, sin 90° 2gcosa = 28090, Begcone length of th mad tan = a Taine lengih ofthe spring) (2a, =gsino) ie 4 ae ie 39 (b) The force of 100N acts on both the boats ; fence : 1.2504, ie. tan? a= 2 or sec? a= 3 * w anda ~ 0.2 ms? or cosa - . ‘Then relative acceleration = a, +0,=0.6 ms* Using $= ur + 1/2 at?, we get 100 = (1/2)x 0.6 x# or f= 1835. 40. (c) The centripetal force is provided by the hanging weight ie, Mia? = mg 1 or a = cor (3) ew & 2. esa Zeon 41, (@)1FTis the upward dst on pach, hen oot ma = mg—T on Let’ be the mass to be released, then T(m—mi)g=(— ma Gi) ‘Adding (i) and (i) mm’ g=2ma-m'a t- oy 2ma atg or 42. (a) The two blocks move with same acceleration til force of friction between them is not reaching the limiting, £23 45, value. After reaching this value, acceleration of B becomes constant but acceleration of A continues to increase at faster rate. . (c) Let the weight W be placed on the nearer edge. Distance between two adjacent legs, d= xin RxI= am Then mg>1 = 20 +m) 52 20g - = 20(J2 +De mg = 483gkg m = 48.3kg (a) Let m, be the mass of board and m be the mass placed at one end of the board. sat Fig. 5.126 Then for equilibrium HNcos O=Nsin @ he, #=tan@ (@ Before accelerating, tension at rope Bis T= + x ty ae Just after acceleration, total tension in rope B is given tension at rope A qT, ttma 2 (@) foree of resistance, bY Tras wo or dv oi, mde ie,m = =—evie, dt 41. 48, Newton's Law of Motion 169, YL ed ig og Mat ics log, 7 =~ ie.tog, 5 = ied Big 00 Au —v) From (i) and (i) vac, +) =u,,l0g, 7 me (d) For hanging part, mgh-T=mha and for part in tube, T= ma wii) ‘Adding the above equations, we get mgh~m (tae -@ ett Oy et One dt” ax dv dk gh dv gh ded hee de hie 470 Pearson Guide to O! Physics or dun 2 ate ae ie, v-[2ettos,4] 49, (@) Let be the distance moved bythe block when cylinder moves from top to the bottom. Here (M+ m)x=m(R-r) mie) M+m or x= 50. (c) acceleration of mass m is Various force equations are 2T-N= Ma, N= ma, and mg—wN-T— ma, 2mg. Seline = F755 Bion da ame and 4,” 755+ 2m __wWme “ M+Sm+2um T Consider an element of chain of length i subtending, an angle of d@ at the centre of spherical surface. S41, (b) Letmass per unit length of chain = m ‘Mass of element dim = m at~ F rao 7 Force acting on element, dF = dm g sin 8 = 7 resin 649 a 71s wf rg(1-c05 a) (0) Let F, be the force between the man and the board and F, be the force of friction between the inclined plane and the board. Here F, can have a value between Mg sin @~ Ht (M+ m)g.cos Band Mgsin 8+ — (M+m)gcos Ai) Limiting value of F, = pN, = M(U+ m) gc05 8 the force equations are F,+mgsin @= ma ie, F,=ma—mg sin 8 From Mg sin 0- (+m) geos OS F,< Mg sin 8+ 1 (M+m) cos ile. Me sin 8- (M+ m) gc0s 05 ma—mg sin 8S mg sin 8+ (M+ m) cos @ -_ (s \omornosas (@) Here T= Mg and Tcos = Mg Also T'sin @= mail sin ie, T= ma?! radius = |sin = Be Me ie, dave yoy eves yor 7+ Mg sin 37 — 1, Mg cos 37- Ma (i) ‘and for mass M, Mg sin 37 —T—u, Mg cos 37= Ma a) For mass M, Newton's Law of Motion 174 1 81. (sin 5 or a=30 and sin B= ao Fi rig. 5.128 Adding the substituting the given values, we get 2g 0.6-+4z x 0.6-0.25x2gx08-0.75x4gx0.8 =(442)a g(1242.4-0.4-2.4) 6a or a=13ms? Using (iy Fig. 5.190 2X9,8 K0.6-T-0.25X2X98X0.8=2%13 or por or TH11.76-3.98-26 Then F, =mg cos @ =520N =5% 109.8% 00830 55. (b) If mass m,travels, m, travels by s/2 =42.6kN -: if acceleration of m, isa, then acceleration of massm, and. F,= mg cos B / 5x 10°x9.8 cos 60 =245IN. Here 7= Oi 58, (a) Height through which water falls in time dt after ¢ Sovinga = seconds /i~N’), where'isthe height of water already 56. (c)Note ™ allected 6] REKERe Mass of water collected = vp dt (@) itional forces up the plane and votocty of wares = [IRIE @@) resctens N authown: Total force= weight of wateralready collected + impact Then mg sin @-2,4,.V= ma @ ne x ¥ (ss there are 2 surfaces) = ups + op 2g (hh) also mg cos 0~ J \ we =p [s2e(#—] x 59, (b) For equilibrium Tema A and r= mgruma (i) From (iy and Gi) Q\\/ 9 3 HS ai manmg+ uma =m From (i) and(ii) mgsin 8-2u, a =ma or a=g(sin@~ V2 p1,cos 172 Pearson Guide to Objective Physics 15m —1 _ e080 - me 05 or ve 030 Y 60, (4) If mballs each of mass m are hanging vertically L au = ) shavnng fi=nme Using the given values we get 16 or cast = 55 orcos O=4/5 25 or @ or 600s 4/5 Seracesanete 64. (Tin b= me =isnmea and Tos = mg on ME (16m mg 3 =(16-m) (using (i) or nga 3° 6 | 803 or n= Gx 7710 61, (A)Here 7,=7,c0830= 7, > rol” & . i wr tan O= Fe = Tonto ik or wne=tore=45" Also Mg =T, sin 30° = 3 65, (6) Impulse ~Fr~ change in momentura =mv-(-m0) =2mv=2x0.01 x5 on Fa apy TION 66. (a) Acceleration along F,, RoR M sas efegty ome My 62, (ty Using F = ma, we get ; F sx 5 go 3 P 5-2 pa THT gain using 2 os = v8, we get al 5 | e WwFKIOKI=F 63, (€) Velocity at highest point woe 8 : Then Mu cos 6=(M+3M) 0 Fig, 5.133 If Tis tension in this length, then De) p-tema=(Hfy L -F)yik 2 T=F-(F-F)yilk or T=F,(1-y/L)+F,Q/L) 67. (c) Impulse = f Fat = area under greph +. Total impulse from 4yisto 161° = Area EBCD = 1/2 200+ 800) 10+ 2 x80010%10¢ =5x10°Ns 68, (4) CP: ZCPO= LPOC=60° Thus AOCP isan equilateral A OP=R «= Extension ~ R ~natural length of spring BR a) ms 4 foce=me=("©)(2 Thus spring force = kr = (= } (4 oak a4 Fig. 5.194 69. (2) fa for A= 1, (mg.c08 45°) Fig. 5.135 Also f,., for B= pt, 2mg cos45) Newton's Law of Motion 173 ~ 13 mg v2): 3 78 ‘Total fitional force =F mgt Butpulling force rn ah v2 2 + system can not accelerate. 70, (c) Acceleration of particle w.r block = Acceleration of particle acceleration of block =(gsin9 7 +g0os6j)-gsinoi =gcos 8 j Motion of particle with reference to block is parabolic w? sin 2a 2. PQ= range = 2 Te (b)h=2,= 51 Force constant Ks iength of spring 3 SKK 72. (c) Block willtopple if oe a L “4 He fe I Ip me Fig. 8.137 eee) 174 eon Guide to Objective Physics 73. (@)Forequilibriu Lhorizontal forces = 0 ie, Nsin a= Neos iescota= Vt or cota= 74. (a) Force applied by clamp = resultant of Zand (mg +7) rc = (ey Home Mey? = ala (mM +my 75. ()HereT=mg (i) and2Tcos0=(J2m)g (i) Dividing (ii) by (i) ’ Fig, 5.139 Work, Power and Energy BRIEF REVIEW Work In physics, the term work is used when a particle is displaced by the action of a force. Work isa scalar quantity. Unit is Joule ($1) and CGS unit is erg, Practical unit of work (particularly in electric consumption) is kWh. 1 kWh ~3.6x 10* Jand 1J= 10" ergs. Sometime eV is also used. | eV= 1.5, X10" J, In problems of heat | calorie= 4,186 WF ids W = F . 3 = Fs cos 8 if force is constant throughout. W = JF. dsif force is variable Work can be positive or negetive depending on the value of 6, for acute angles cos @is positive and hence, work is positive, For obtuse angle cos @ may be negative making ‘work negative, Positive work is parallel to displacement and negative work is opposite to displacement Work done in lifting a body up (against gravity) is positive and work done by the force of gravity (vertically downward motion)is negative. No work will be done if the body is in static or dynamic ‘equilibrium, ic, W=0ifEF=0, No work is done if displacement is zero or force is perpendicular to the displacement, Thus, work done by ‘centripetal force and work done by moving charged particle in a magnetic field is zero i.c,, F= q (3B) will dono work. Workdone depends upon the frame ofreference. With change in frame of reference displacement may vary and hence work done could be different in different frame of references. Ina conservative field work done is path independent W = aPE= [Fads In a force versus displacement curve, work done is area under the graph. The algebraic sum of the area is to be fourd out as illustrated in fig. 6.1 w= Jj, PaV Area underPressure(P)and Volume (P) curve is work done, W = AKE, i.e, work = change in KE, This is also called work energy theorem, For positive work KEy.,, > KE,,,.. Work energy theorem is velid forall types of forces (intemal or external; conservative or non conservative). 1 In case of a spring W’= ~ kei where xis extension or compression in the spring ‘Powers a scalar quantity, Its SL units Watt (W) oc J/ _$ Practical unit of powers HP or ip (britsh horse power) __Lbip = 746W = 550 Ibi = [P.. di or area under P —t graph. n never be negative while PE can be both . Potentil energy is defined only for Ke Joc and is taken positive in all cases 44: Electric PE= = may be negative or positive, GMM, Grayitational PE = may, be negative or positive “Mechanical éneray~ KE + PE is conserved if intemal forces are conservative and no work is done by nonconservative forces. if some of the iniernal forces are nonconservative mechanical energy of the system is not conserved, Total energy -E + PE internal energy Internal energy is directly related to temperature. Largerthe internal energy, higher is the temperature of the body. Thermal energy is related to random motion of molecules while intemal energy isrelated to motion as well as their configuration or arrangement. = me" is mass energy relationship. Quantization of energy radiations emitted by a black body are quantized. Quantum. nature of energy is confirmed in atomic and subatomie World. Even light energy is quantized. Planck has shown that the Ae Workidone WF : constant isc aia Cal, dh : if force is vatiable ae a eg” W = APE ieee forces) aes mea tt Wa 5k F = Pe'eos 8 if foree F is ‘ina spring’ Nonconscrvative forces do not sow PE. Ifa particle ‘moves ina circle then binding energy =KE > PE= ; PE =—KE, Ina bound system like this PE is negative. Se 4. In conservative forces work done isindepentgent of path followed. It depends only on the initial and Spal ponin, Total werk esi a FP He Joni ae itt bel a Penk £5 Gina spring and is onky positive). tderiegal 4 0 ea ayil 20 ASAE (in gravitational fields). It r ayy may be positive or negative PE = migh | i ‘ith iskmall Sst (ol ei td It may be sho! Siu fh pena 104,20) positive or negative, 6. Ifabody snr egte ie we =0. | (workidone by this 1 ni 8. Mechanical enerey = internal forces are conservative and. 9. KE PEis not conserved ifnoiico rées Beat ee et, sso 30. gros 2 TaaenBalqaibs Ie snob ai AoW OA , 1 oraoaalaal alt of agusibn 10. RES Font i onein tthe bs 1A, Iai dd a hd ae sla ee 12. Ifa lighter and heavier body have equal momentum then lighter body has more KE. Oe 13. Area under Power time graph gives work. 14, AU=change in PE=[F dr forconservative forces. WAP Oniesiny wu. 15, f < = 0, body is said to be invequilibrium. _— Equilibrium is stable if Wis minimum, unstable if | Uismaximum and neutral if — eng 16, If th part ofthe chain hangs then the work done oo oe eb eens be i abv leg uroytob xs10n auoarieannta 1 jg wit bon {o pull up the hangingichaimis, 5. [See Fig 02] us Gee PPM Fhitlbtbhim displacement in asprin is to be found i use 1 = + i steady state displaceme 1a ee as hee spring is tobe found use F = kx. OU bees w dion buen ail yore ek ws A r 18, Fosaelinehos KEmig mei Agha 19. Equation of dynamics of a body with variable mis i (8) i Tae * Gpbtiif Teference frame ig aheRst oe ‘eh! ‘wht ud = oe ti ushers on 5 Oi) (RING 20. Change i@ SUN Encra3RE = W, + Wy elo(ooneonsetyativedd zi u 01 zou to ctuinag 0 Tei tai SERED ston now ot] OR Bing OB TNA Pes 8 abel”? => When'for is Varidfle use W’= JF. ds or “ARE or WB bs Reith) UT orctaac #00€SIMMl work done even when the force is perpendicular to velocity. = No work will be doné'tidn forge is pagpendi to velocity. For example, no work is qe ‘centripetal force in a circular motion. No work is doneby the rpaenetic force | ¢y xB) a8 and ¥ are perpendicular. “ol 30-1 aietinlgi wininen held aumeasitABiherbdy snivogmmeoreditcoa batt nsds Sow id Pig? 43, it tinpaits: its complete KBtotie sprit) ~ 1. tion! ovido 2 coieyl AUST ROMA AID EDEN 177 + => Ifa body, would have ot been connected. to the spring only then complete KE of the body would have been converted to PE stored in the spring. But _ sng gall gpbody Is connected then firs conserve jum. The KE of system after collision will be equal to PE stored in the spring 4.) Tofindmaximum displacement ina spring applying the force equation die Rr mAR| pinto 00 SbF = Lee will provide steady state displacerient, "- Makiinum displacement soblained when we equate bonieHengyoiseev Thuan ult svete sto bail iMod at rd protige nh es Considering ov doe ina sringisalways. 3 ae. => If the force moves the block slowly and ae ORXOT KEY ~ us|, Ib Mth > aha (a) then IV ~ 5 kx but ithe movement of the black is zm Ob (0) serv fast(ora foree applied fora very shortinterval called sudden, force/impulse) then, work, done,by crust ROFABLEE cA asieanqeross bre A iutenw: 6. Conserve enersyleven Whos noncohservutve forte! 8 are present. > Ifnon conservatiy. is not conserved. forces perform work then energy agi ‘ 7 Assuming even ring body has KE = 0 ve = Rolling body possesses both linear KE and rotational KE; (foal KE is sum ofthe wo ib) “eno¥ x 1 1 rook Byaitig t+ 3 for 8 Assuming gravitational PE is only mgh, = The grabitdtlokal Bis\gh whdncalithnedt MONO aoe ‘i GMa sai al vi byw ysrane atte Reh 1, Ifthe attraction between two bodiesis invélvéd then ppm a2 (0) ANE) jeoos tatty 9, In a system of mutual forces considering only KE ‘2dlucto one particle is equal to APE. MS Penida te nS () iConlder Kefdueto bothihe piles” | SOUNEA 178 Pearson Guideto Objective Physics 10, When a vehicle is moving up an incline and the efficiency of an engine is given then applying efficiency ina wrong manner. Forexample.a truck of mass 20 ton is moving up an incline of 1 : 10 with velocity 10 ms". The friction is 500 N per ton. Efficiency is 80%. Find power. Thea using P__,= 0.8 (mg sin @+F) v. = Apply P., = (mg sin 0+ F)v ANd Prague (0-8)= Pe oe Pi he = Cemeetes oo 08 08 SOLVED PROBLEMS 1. A spherical ball of mass 20 kg is stationary at the top ofa hill of height 100 m. Itrolls downa smooth surface tothe ground and then climbs up another hill of height 30 m and finally rolls down to a horizontal base at a heightof20 m above the ground. The velocity attained by the ball [AIEEE 2005} (@)40ms" (&)20ms" (@)10ms" (4) 10.35 ms" 1 BRIM ©) mich = > moor v= J2gh = JIx10%80 =40 ms 2. The block of mass M moving on the frictionless horizontal surface collides with a spring of spring ‘constant K and compresses it by L. The maximum ‘momentum of the block after collision is i xe | Fig 6.4 KP (@) VMK L (b) IM a (Ze or {AIEEE 2005] GRIMM 9) 5 0-2 orp= VK 3. If'Sis stress and Yis Young's modulus of material of a wire, the energy stored in the wire per unit volume is Oy (say @2sy 2s? IAIEEE 2008] sls 1 sensed EMM) U- } sresxsmin= fsx $F. 4, Abody of mass mis accelerated uniformly from rest to speed vin a time 7. The instantaneous power delivered to the body as a function of time is ote mo? m' or OFF : BOM «P= maven'r=n (7) « ‘A car is moving on a straight road with a speed 100 ms", The distance at which car can be stopped is ..... (u,=0.5) (a) 800m (b) 1000 m (€) 100 m (4400 IATEEE 2005] b) D= = = EMM %) o-2 6 A particle of mass 10 g is kept on the surface of a uniform sphere of mass 100 kgand radius 10 em. Find the work to be done against the gravitational force between them to take the particle faraway from thern (0) 1334x105 (0)3.33« 10°F (6)667x10"5 (6)6.67x 10-5 [AIEEE 2005) (@yw=au= St _ 6.872107" 1005107 Ta =6.67x 10"), 2 shown infig 6.5. The work done by the forcein moving the object from x= Otox=6 mis Fig 6.5 (a) 185 (b) 13.55 (e)9d (a)4.s [CBSE PMT 2005] (b) W= Areaunder Fx graph, 8 A bomb of mass 30 kg at rest explades into two pieces of masses 18 kg and 12 kg. The velocity of 18 kg mass is 6 ms". The KE of other mass is (a) 324) (b) 4865 (c) 256) (a) 5245 ICBSE PMT 2005} 18x6 ' (b)m, v= m, v “w= >> =I ms KE = 3x12 P86 9 A block of mass 10 kg is moving in x-direction witha constant speed 10 ms", It is subjected to a retarding force F=-0.1 x Jim during its travel from x= 20m to x= 30m, Find the final KE. (a) 475 4 (b) 4503 (2755 (6) 2505 IAUMS 2005), (@kE,= [PF de +KE.., = [Pocnnis =F 10x10 10, Energy required to break a bond of DNA is approximately (@~lev (b)~0.1 eV ()~0.01 ev (@)~21 eV IAIMS 2005) 3} II. A particle is moving with centripetal force —>. Find the total energy associated. [CBSE PMT Mains 2005} Work, Power and Energy 179 k Total energy = KE +PE=—KE =~ 12. A spring does not obey Hooke’s law. Rather it follows, F= kx be~ cP Ifthe spring has natural length /and ‘compressed length / then find the work done. is kom 2 ye We [Pede=FU-n 3-4 e ary 13. A space shuttle of mass 86400 kg is revolving in a circular orbit of radius 6.66 x 10* m around the earth. It takes 90.1 minutes for the shuttle 1o complete one revolution. On a repair mission it moves 1 m closer to a disabled satellite every 3.0 s. Find the KE of shuttle relative to the satellite, (@)48001 (€)2.59x 10"J (04805 (269x105 1 1 1 @ym Eresaton (2) ss 14. A particle of mass 6 kg moves according to the law x= 0.22 +-0.02 7, Find the work done by the force in first 4s (IIB (b) 2.6428) (©)2.1324) (d) 1.6428) 1 ‘1 (We 5m (7-07) 2 = S041 060 v= Fp s041~.06 = $ x6 [(74)*-0] = 16428) 15. A moving electron has KE *K’. When a certain amount of workis done, it moves withone quarer ofits velocity ‘in opposite direction. Find the work interms of K. “17 xis ag K () 5K @yZK a () Gy wt or Fe 16. A brick of mass 1.8 kg is kept on a spring of spring constant K = 490 N a. The spring is compressed so 480 \Paatsbh Guide td ObPEtive Physi that after the release brick rises-tp 3.6 m. Find the compression in the spine. nme (a)021m (oaian ()0322m @osi4m ‘ OER mE or; Bx18x10x3.6 3.6 490) 17, (75 KW. engine is generating full power. provide a 700 kg airplane a speed 2.5 ms Find the fraction of engine power used, (ay 71 305 18! Tn an ice rink’ skator is ‘moving’ at SYinegAd encounters a rough patch that reduces her speed by 45% ch 6 ation force that T2506 of hed Weight. Piha the Iengtr ofthe rough patch’ @ (a) 156m (b) 146m (136% 126m = FA-(357)' te ag ns = 18 (7) = 126m, fant 19, A pump having efficiency 75% lifls 800 kg water per minute from 14m deep well and throws ata speed of 18 ms". Find the power of thé pump. (@2060W @2490W (c)3218W (d) 1400 W 800 254 2 BOM) Pe Ge gh Gy Ox Ld _ 5800 7m f Fyn § 22800 _aps.88 ‘xg 88 20. The heart takes and discharges 7500 / of blood in a day. Density efblood ~ 1.05 10? kgm”. [fon anaverage it takes a Regt of 116m’ Find the bi weld pump. : (a) 163'W ()1L36w a (e196 (ay2.40W wv dm ' (p= Ge ehap a ah | 1,05%10"x7500%10 2axooxgo — *10%x16 s= 1638 21, In the system shownafind the speed with which 12 ke block weight hitthe ground this @2 Jig. mst!) (6) 4, (3 mst“ ous Te sea (2 ¥5\ms" (8395), pees janes Ved Aaa a Fig 6.6 1 (©)m, gh—m, gh= or 8x10x2= 2x low Eee gy mn dN en 'A skier starsat the top of a stowball with negligible speed and skis straight down the side. At what point does he lose contact with the shotvball,’ | ) O= 0081.5 (a) 0=sin) > Aone oe ©. @ none Fg67 y, 8) Be meh = me GOS 6 [rons etait | ea | Condition of losing omicen "to = micas 11 mv’ or “= 2:mg (1 —ces @) E cay of, 2 mg (1.x £08 8) = mg cos Bor 8= cos! = She ne TAT 23. ‘Theforce P= C3? j with Casnegative constant is_ (@) conservative (b) restoring (©) nonconservative (@) none 0) Wag= Wyg* Wye * Wey 0% [)-C [-ora=o wo,c ‘oo eo Figgas ‘Since the work done in’a round trip is zero. ‘Work; Power and‘Energy ° 181 24, A particle has PE vs x curve as shown in Fig. 6.9. The Unstable equilibrium occurs at @a B (oc @D Fige.9 (©) E F= Oand PE maximum for unstable ‘equilibrium, 25. A particle is released from the top ofa quarter circle of radius 1.6 m, It stops at C, 3 m away from B. Find coefficient of friction which is present only on the horizontal surface. (@)0533 (©0333 (0438 (d) none AR 24 3m ——_» Fig 6.10 3gh 16 n= gf ~~ 0533 26, A 500 g ball is released from a height of4m. Each time it makes contact with the ground it loses 25% of its energy. Find the KE it posses after 3rd hit. (@iss (by11.253 (844s (A) none 3 conxe=nen( 5] /. The following data is obtained from a computer simulation for a patted baseball with mass 0.145 kg | Including air resistance. Find the work done by the air con the base ball as it moved from manimum height to back to its position 16) =e) 8 0} aut 0 30 0 3.05 702 53.6 18.6 0 659 1244 0 120 30 (@0 (0) 105.56) (0)763 (2135 1 pig 2 w= > m (vi +H; UE, - UF) 1 easgoar io soil 28. A locomotive of mass m starts witha velocity v=a-v¥x « Find the work done by all the forces acting on locomotive in first rsec. ma't” mat Oe oa ma'‘t ma‘ (c) @ o> dv dv da 2 OG Re a7 Me OBIS 29, The KE of a particle moving along a circle of radius R depends upon the distance covered x as T= ax*, Find the force on the particle. ax” Solut er) dv _ 4axv differentiating (i) 20 SP = 2ax o acceleration o, = = Hence net force = m Ja? +a? = mf 228.) (248) aN aR m -2= fA) 30, Twoblocks of mass m, and m, are connected by anon ‘deformed light spring resting on a horizontal table. The soeticient of friction between the blocks and table is ut. Find the minimum force applied on block 1 which will move the block 2 also, See Fig 6.11. aay) Fig 6.14 If xis the compression in the spring when block my, is just to move then Kx =m, g (1) formin, force; Fx Repu, gx=Oor =F-pm,g Q) From (1)and(2) wet) -ne [m+ 31. A chain of mass m and length / rests on a rough table with part overhanging, The chain starts sliding down L by ivelF if overhanging par is 4. What ill be the work performed by the friction forces acting on the chain by the moment it slides completely off the table. Mg 2, oe o= 2Mgl zt o> o# Fig 6.12 » 1 ‘pees SOM yu 5 mi ‘Assume at any instant the length of the chain on the 1M ule iain et icin == Hg sg Werk done against tition fy? H=T-98 de 2 (42) Mat 249 9 32. A horizontal plane supports a stationary vertical cylinder of radius R and a disc attached to the cylinder by a horizontal thread 48 of length, 3 showin ig. 6.13. Disc isgiven a velocity u,, How long will ittaketo strike the cylinder. Assume no friction. Me 7 = 2 . 1 mi jo 2 L @y, O Ry, & & (©) ZRe, © 3Ra, Fig 6.13 (©) Lets be the wot distance covered by disc. Then t= = Atany instant ds = (1, R®) 40 t ~ 2Ro, 33._ In the system of two masses m, and m, tied through a light string passing over a smooth light palley. Find the acceleration of COM. (Centre of mass). (m=m;) ‘Ona = Ware @ m, mam, © (2 ie (a) (2 aie ) oy = mda ‘com ty _ omg mem. 34. Two blocks of masses m, and m, joined to @ non deformed spring of length /, and stiffness K as shown in fig. 6.15. Ifa force Fis applied on block of mass m., Find the maximum separation between the blocks, jo jee ht Taye) ()he+ Emm) pe nae ay, + 2m O46 am, @l, m+n, ‘ 7, eon,» F Fig 6.15 (@)x, andx, be the maximum displacement in 'm, ard m, respectively. mE m, +m, 1 Ray tag mF ~ em, HD 2m ws Em +m.) Thus maximum separatior xtay a Km, +m.) 35. A particle of mass m is moving in a circular path of constant radius r such that radial acceleration a, Fr, Find the power delivered to the particle by the forces acting on it (a)2am Prt (bymE rr (© ymk re (ao @)*-e Fr or = mde v= ker Pa me me and Power P= F .# P= mkr(tk=mRPt 36. You lifta suit case from the floor and keep iton a table, ‘The work done by you onthe suitcase does not depend on (@) the path taken by the suitcase. (b) the time taken by you in doing work, (©) weight of the suitcase. (@)_ initial and final position. (a) and (b) gravitational force is conservative; ‘work done is independent of path and time. 37. A particle of mass m is attached to a light string of length J. The other end of which is fixed. Initially the string is kept horizontal and then particle is given an upward velocity v so thatit is just able vo complete the Je. Then (a) the string becomes slack when the particle reaches the highest point. (b)_ the velocity of the particle becomes zero at the highest point. 184 Pearson Guide to Objective Physics ~ mgt. (a) the particle again passes through initial position O.@ 38. In the Fig. 6.16 pulley is light and smooth. Thread is massless. On applying force F, KE increases by 20 in Is (2) tension in the string is Mg. (b) the tension in the string is F (©) work done by the tension in I sis 20 J. @ the work done by the force of gravity is 20, in 1, +e Fig 6.16 ® 39. Ina factory, 2000 kg metal is lifted by 12 min 1 minute by a crane. The minimum horse power ofthe engine is (2) 320 bhp (b) 32 bhp (©) 8.3 bhp (4) 6.4 bhp wie (ope = 2000 60 «98% 12 P(bhp) 3 bhp ‘One end ofa light spring of stiffness tis fixed to awall and the other end is tied to @ block placed on a smooth horizontal surface, The work done by the spring on lL displacement xis + kx’. The possible cases may be (@) _thespring was initally compressed by x and finally ithas acquired natural state. the spring was initially stretched by x and finally in natural state. ) (6) the spring was initally in natural state and finally compressed by x. @ the spring was stretched by x. (a), (b). In (c) and (d) we do the work on spring ily in natural state and finally a] Read the following passage and answer the questions given at the end. Consider a man standing on frictionless roller skates on a level surface facing a rigid wall, He pushes against the wall, setting himself in motion to the right. The forces acting on him are his weight ”, upward normal forces n, and, exerted by the ground on his skates, and the horizontal force F exerted by him on the wall, There is no vertical displacement. Hence W,n, and n, do no work. F is horizontal force that accelerates him right but parts of the body where that force is applied (the hands of the man) do not move. Thus, force F does no work, 1. Where does the man’s kinetic energy come from? @ (b) © (@)__from muscular motion (a) We can not represent man as a point mass. Different pars ofthe body must have different mations. Though his hands are stationary but the forces may appear at other body parts and do work on another par. This happens in this case. from his muscle power from the force he applied on the wall whatever he ate. 2. Can we consider work energy theorem as a generalization in Physics for any system? Yes No, in some cases it fails particularly in thermodynamics {c) can not say, (b) In many cases we can apply. But in thermodynamics dQ=di+dw @ (b) Read the following passage and answer the questions given at the end, Ata point where w(x) has a minimum value such as x=r,, the =a a |, ~O-A article atrestat this point will remain atrest, Ifthe particle is slightly HH ends to bingit back and Fe tendsto bringit back ar slopeofthe curveis zero, ic, Fy, displaced from, then F(x) particle will oscillate about the equilibrium point. Such an equilibrium iscalled stable equilibrium. Ata point where u (x) has a maximum value say ate =x, the slopeof the curve is 2010, i.e, P(x,) = A particleat rest at this poin will remain at rest. However, if the particle is displaced even slightly from this point, the force 4 |... 7 F Gy) will push it further apart from equilibrium point. Such an equilibrium is termed as unstable equilibrium, 1. Assumeat.x=x, 1 (x)is constant. Slope particle is displaced slightly from x=x, Then (@)_particle will retum back tox, after oscillating. () particle will move further way from. (©) particle wil stay atx, atu ()_ particle will come to a point where >= is ‘maximum or minimum, {b)Since there is no repelling or restoring fore at this point, particle stays there at the displaced position. 2, Assume at any point u (x) ~ 0. Which kind of equilibrium exists at this point? (a) stable (b) unstable (c) dynamic (d) static (e) none (€) Since the particle does not possess any energy initially. Iti very difficult to displace it. 3. A particle isat D andis slightly displaced (@)_itwill execute SHM and returnto D finally. (b) it will oscillatate (but not SHM) and return to D finally. (©) itwill crossover £ and stay at F (@)_ iewillstay atz. e) Fig 6.17 (Ona winter day in Srinagar (Jammu & Kashmir), a warehouse worker is shoving boxes up a rough plank inclined at an Work, Power and Energy 185 ‘angle ccabove the horizontal. The plank is partially covered withice, with more ice near the bottom of the plank than near the top, so that the coefficient of friction increases with the distence x along the plank y= Ax, where 4 is positive constant. Bottom of the plank is considered at x = 0. Coefficient of kinetic and static friction for this plank are equal and equal to 1, The worker shoves the box up the plank so that it leaves the bottom of the plank with velocity Ve. 1. Find the velocity ¥, that the box reaches the height, T= hsin a @ © rer AE (none 2. Find the condition that if box once comes to rest, it remains at rest (somewhere in between the top & boitom. 2 _ 2esin?ar agsin“@ OM hose OO feos 200% 3gcos’ (O jsine (0 sing , sing: 1c) my = mgh + f."" umg cosads ,1 “= ght eA cosa |) xdr 2(b) The box will come to rest if final velocity becomes zero before reaching highest point, Fig 6.19 186 Pearson Guide toObjective Physics 2 mg sinax+ [° Amgcosadx or = 2gsinax + Agcosax* it will remain at rest if the net force at that place is zero ie, mg cosa= mg sinot Read the following passage and answer the questions given attheend, ‘A diver jumps offa high board into a swimming pool. Before he jumps the diver bounces on end of the board he hits the ‘water moving preity fast. As the diver plummets towards the ‘water, he loses speed. The gravitational potential energy and elastic potential energy is converted to his kinetic energy. This energy is lost in doing work against the water resistance. Energy transformation occurs at every step but energy is not lost. We say energy is conserved fiom one form to the other. 1. Asthe diver enters the water the work done by water ‘on the diver is --—~ and work done by gravity on him is - (a) negative, negative _(b) negative, positive (c) positive, positive (@ positive, negative 2. Inwhich form the diver’s KE is transformed in water. (a) electrical (b) heat (c) viscous (@) surface tension 1 O2 & 1, A motorcycle of mass m resting oma frictionless road moves under the influence of a constant force F. The ‘work done by this force in moving the motorcycle is given by F°¢/2m, where tis the time in seconds, Fig 6.20 Ratio of instantaneous power to average power of the T second is 2:1 (@t:2 2. Two ends ofa uniform garland of mass m and length Lare hanging vertically as shown. At instant rthe end is released. Ifyis the distance moved by this end in time dt, change in momentum of an element of length dy is given as x © F 4 gy)? Gownward) m 1 ©) F 4 (gy)? (upward) © FP aray ay (upward) @ 4y (gy)? (upward) 3. A compressed spring of spring constant k releases a ball of mass m. If the height of spring is / and the spring is compressed though a distance x, the horizontal distance covered by bell to reach ground is, Fig 6.22 th ahh @)x ca OT [2k me. Ox ving Om 4. A mass m starting from 4 reaches B of a frictionless ‘rack. On reaching 4 it pushes the track with a force equal tox times its weight then applicable relation is 5. Coefficient of friction between a tol and grinding whee! isp. Power developed in watt by the wheel of radius r running at 7 revolutions per second when tool is pressed to the wheel with F’kgfis. Fig 6.24 (@uF rom) O)HF gm) (ourr @urg 6. A minute particle resting at frictionless surfaceis acted | upon by @ constant horizontal force. Neglecting frictional force the graph between work done on the particle w and speed of particle w is represented by |) | — ] fo apy ° © > @) — Fig 6.25 Work, PowerandEnergy 187 7. A particle of mass m slides along curved — flat —curved track. The curved portions of the track are smooth. 1f the particle is released at the top of one of the curved portions the particle comes to rest at flat portion of length 1 end of coefficient of kinetic friction = f,.,. after covering length of Perens! Y\ Fig 6.26 (2) () HH oe @ Hasse 8. A light rigid rod of length L has a bob of mass M attached to one of ts end just like a simple pendulum Speed at the lowest point when it i released is (ute werted and Figo? @ Jee () J2gt (©) 2g @ 5 9. Astronauts Mr. ¥and Mr. Y float in gravity zer0 space ‘with no relative velocity to one another. Mr. ¥ throws 44 mass of 5 kg towards X'with speed 2 ms". If Mr. X catches it the changes in velocity of ¥ and Y are ke ‘ 120 ke. g 20k y Fig 6.28 0.21 ms", 0.80 ms" 9.80 ms", 0.21 ms! (©) 0.42ms", 0.08 ms" (@) 0.08 ms", 0.12 ms" 10. A particle is dropped from a height 4. A constant horizontal velocity is given to the particle. Taking g @) (b) to be constant everywhere, kinetic energy E of the particle with reference to time is correctly shown in — — @) io) | 1) | z = — © @ Fig 629 1, An aeroplane is flying with a velocity U,, The plane takes in volume V of air of mass srper second to bum mass m of fuel every second. The exhaust relative to plane has a speed of U,. bh.p. of engine of the aeroplane is y, Fu, Fig 630 Ui m) UU =m), @ Te Ulu +m)—-U oy ® 746, UL itm) —Upet a 12. A vehicle of mass M is accelerated on a horizontal frictionless road under a force changing its velocity from 9 vin distance S. A constant power P is given by the engine of the vehicle, then v= we BPS" by ( S+ toe +57 (57 x) 3PS os Y(t 13, Two frames, one stationary and the other moving. are initially coincident. Two observers in the wo frames ‘observe a body initially at rest in the coincident frame. A constant force F starts acting on the body along horizontal axis when moving frame starts separation from fixed frame. Work done “W" ac chserved by ‘stationary ftame and W” es observed from moving frame are compared to each other as (aw=w" (yew 1 , owns (Ww 20 14, A proton (mass 1.67 10” kg) is accelerated along a straight line by3.6 x 10" ms*. Ifthe length covered is, 3.Semand initial speed of proton was2.4x10'ms" the goin in kinetic energy is (a)1.32MeV (b) 1.23 Mev (©)2.13MeV (@)3.12Mev 15. A mass m is thrown vertically upward into air with initial speed w. A constant force F duet air resistance acts on the mass during its travel. Taking into account the work done egainst air drag the maximam distance covered by the mass to reach the top is © se Fim ® gt Fim 16. A 20 g bullet passes through a plate of mass 1 kg and finally comes to rst inside another plate of mass 2980 g.It makes he plates move from rest to same velocity. The percentage loss in velocity of bullet between the plate is — 20g - ig 298g Fig 6.31 (a0 (b) 50% (0) 75% (625% . Aparticle of mass 1 kg has potential energy, PE. =3x + 4y. ALT= 00, the particle is at rest at (6, 4). Work done by external force todisplace the particle from rest to the point of crossing the x axis is (@)251 (201 (15d (@s2s 18, Abody is lifted over route I and then routelI such that force is always tangent to the path. Coefficient of friction is same for both the paths. Work done L = Fig 6.32 (@)_on both routes is same. (b) on route Lis more. (© onroute His more. (@)_on both routes is zero. ). Interaction between atoms of a diatomic molecule is described by the relation PLE. (x)= © - 8, P-E. ispotential energy. I the two atoms enjoy stable equilibrium, the distance between them would be “Gl GT off of 20. A hammer of mass falls from a height h repeatedly todrive a pile of mass mintothe ground. The hammer ‘makes the pile penetrate in the ground to a distance d in single blow. Opposition to penetration is given by — where Fig 6.33 mi nigh (@) gS; © Gtema ttme M'gh Oya (@) Garang Meme 21. The height from which a car of mass m has to fall to gain the kinetic energy equivalent to what it would have gained when moving with ¢ horizontal velocity of (u + v) is given by Work, PowerandEnergy 189 Fig 6.34 22. A gun requiring the support of shoulder shows intensive recoil when fired with Fig 6.35 (@) the butt held tightly with the shoulder. (b) the butt held loosely against the shoulder. (©) butt held loosely or tightly against the shoulder, (@)_abullet of greater mass. 23, A mass mis allowed to fall on a pedestal fixed on the top of a vertical spring, Ifthe height of the mass was H from the pedestal and the compression of the spring, isd then the spring’s force factor is given by ve T Fig 6.36 H+d) (H-d) (@) Mg a , (b)2Mg Mg (H+d) oF Fe (2g Fs How high can a man weighing m kg climb using the ‘energy from a choclate producing 100 calories in him? Let his efficiency be 7 190 Pearson Guide to Objective Physics (0) 2 metre (2% mnete mig me (© h mete (@#R2nmemere mg 25, Atoy car moves up 2 ramp under the influence of force F plotted against displacement. The maximum height attained is given by PES Ebb —- Fig 6.37 Ying = 20M Yn = 15 C= LOM (Yq 7 5 26. A block is placed on a plank which is placed on a horizontal plane. A massless butelastc string deviates by an angle @ with vertical when a force F is applied to the plank to shift it to the right making the block slide over it. fF, and F ,are frictional forces between plank and plane and between block and plane respectively then work done by applied force is given by Fig 6.38 (a) work done against friction acting on plank + ‘energy in the elastic string - work done by friction acting on block, (b)_ work done against frictions acting on plank and block + elastic energy. (©) elastic energy of string. (@ difference of work done against friction acting ‘on plank and block. 27. A truck tows a jeep of mass 1200 kg at a constant speed of 10 ms! on a level road. The tension in the coupling is 1000N. If they ascendan inclined plane of 1 in 6 with same velocity, the tension in coupling is Fig 6.39 ()2000N ()2960N (©) 1680N (@ 1000N 28. A screw jack of pitch § mm is used to lift the tyre of a vehicle of load 200 kg with the help of a handle of length 0.5 m. Neglecting the friction force between serew and nut ofthe jack, the least force required to raise the load is 0 [ Fig 6.40 @12N (b)2.2N (32N (@42N 29. A light rod of length Z can revolve in a vertical circle around point O. The rod carries two equal masses of mass m each such that one mass is connected at the ‘end of the rod and the second mass is fixed at the middie ofthe rod. wis the velocity imparted to theend P to deflect the rod to the horizontal position. Again ‘mass min the middle of the rod is removed and mass atend P is doubled. Now vis the velocity imparted to end P to deflect itt the horizontal position. Then vi o 2am Fig 6.41 @ (6/5) 1 (0) 2.5 (d) 6/6) 30. A coconut of mass m falls from the tree through a vertical distance of s and could reach ground with a velocity of vms" duc to air resistance. Work done by air resistance is 1 @)- 3 Qgs-v) (5 mv Comes (m2 2mgs e , Fig 642 31. A body of mass m is in ve influence of gravity then || motion under the (a) _ work will benegativeand kinetic energy increases during fall. (b) kinetic energy decreases when body is projected up and work will be negative. (©) kinetic energy increases when body is projected up and work will be positive, (4) work will bepositive and kinetic energy decreases during fall. 1 32. A smooth chain PQ of mass M rests against ¢ — circularand smooth surface of radius r. If released, its velocity to come over the horizontal part of the surface Fig 643, 33. A block of mass m has initial velocity w having direction +x axis. The block stops after covering distance S causing similar extension in the spring of spring constant X holdingit, fis the kinetic friction between the block and the surface on which it was moving, the distances S is given by @ fig xt @ > rk, Power and Energy 191 (@) (b) (mKu' = ?m?g?)? 1 2 © & Wimig? amKi2 +s)? © & Uimig Kid sume)? Fig 6.44 34, A rigid cord and an elastic cord support two small spheres of same mass M. The are deflected from the mean position through an angle of 90°. When the spheres pass through the mean position the lengths of the two cords become same. If v, isthe velocity of the sphere attached to rigid cord and v, is the velocity of the other sphere, then 1 Rigid cond Elastic cord Fig 6.48 (@)v,=0, (b) v, is more than v, (©)v,2m @y=Sm Inthe figure6 61. the block B of mass m starts from rest at the top of a wedge IV of mass M. All surfaces are smooth. HV can slide on the ground. B slides down onto theground, moves along it with a speed v, has an elastic collision with the wall, and climbsback onto IV, 196 Pearson Guide to Objective Physics (@) Bwill reachthe top of W agai (b) From the beginning, till the collision the centre of mass of 'B plus #” does not move horizontally. (© After the collision, the centre of mass of 2mv m+M” W’ moves with the velocity (@)_ When B reaches its highest position on W, the speed of Wis yz 2 yf a6 x Fig 6.62 Thering R inthe arrangement shown can slide alonga smooth, fixed, herizontal rod XY. Iti attached to the Block B by a light string. The block is released from rest, with the string horizontal. (@) One point in the string will have only mation. @ the wall, eo © FB plus @ pand E are conserved if both X and Fare free to move. (A) istruc only if Nand ¥ have similar charges. If Yis fixed, £ is conserved but not p. If is fixed, neither E nor p is conserved. 66. Ina one-dimensional collision between two particles, their relative velocity is 7; before the collisionand 7: (a) (b) © @ vertical @ (©) Rand B will always have momenta of the same magnitude. (© When the string becomes vertical, thes R and B will be inversely proportional masses. (@ Rill lose contact with the red at some 64. A strip of wood of mass M and length Iis placed ona smooth horizontal surface. An insect of mass m starts at one end of the strip and walks to the other end in time ¢, moving with a constant speed. (a) The speed of the insect as seen from the ground wt ext. (b) The speed of the strip as seen from the ground is M 1 ata (©The speed of the strip as seen from the ground is L(_m 1 Mem } (A) The total kinetic energy of the system is oo(f). &) speeds of () to their @ point. (a) ® © @ and the collision. if the collision is elastic. ifthe collisionis elastic. [5 =I imal cass, ~k ¥, in all cases, where k 2 1. 67. Asphere A moving with a speed u and rotating with angular velocity ©, makes a head-on elastic collision with an identical stationary sphere B. There is no fri Distegard gravity. mn between the surfaces of 4 and B. A-willstop moving but continue to rotate with an angular velocity 0. A will come to rest and stop rotating. Bwill move with a speed u without rotating, B will move with a speed u and rotate with an angular velocity 0 68. Inanclastc collision between spheres d and B of equal ‘mass but unequal radii, 4 moves along the x-axis and Bis stationary before impact. Which of the following is possible after impact? A comes to rest. ‘The velocity of B relative to 4 remai in magnitude but reverses in direction. Aand B move with equal speeds, making en angle 0f 45° each with the x-axis, Aand B move with unequal speeds. msking angles (of 30° and 60° with the x-axis respectively. the same 69. Ina one-dimensional collision between two identical perticles A and 8, Bis stetionary end 4 hasmomentum. A @ b) © 65. A charged particle X moves directly towards another charged particle ¥. For the ‘X plus Y" system, ‘momentum is p end the total energy is E- ‘the total @ _p before impact. During impact, B gives impulse to ‘The total momentum of the ‘A plus B” system is p before andafter the impact, end (pW) during the impact. During the impact, 4 gives impulse J to B. The coefficient of restitution is 22 — 1, P The coefficientof restitution is z +1 70. When acannon shell explodes in mid-air, (a) the momentum of the system is conserved in alt cases (b) the momentum of the system is conserved only if the shell was moving horizontally (©). the kinetic energy of the system either remains ‘constant or decreases (@) theki ‘71. A cannon, shell is fired to hita target at a horizontal distance R. However, it breaks into two equal parts at its highest point. One part (4) returns to the cannon, ‘The other part (0) will fallata distance of R beyond the terget. (0) will fal ata distance of 38 beyond the target. (c)_willhitthe target (d)_ have nine times the kinetic energy of 4 72. A particle moving with a speed v changes direction by an angle @, without change in speed. (@) The change in the magnitude of its velocity is {&) The change in the magnitude of its velocity is 2vsin (02). (©) The magnitude of the change in its velocity is 2vsin (02). (4) ‘Themagniude of the change in its velocity is v(1 = eos 6). 73. A block of weight 9.8N isplaced on a table. The table surface exerts an upward force of 10 N on the block. Assume g = 9.8 mist, (@) The block exertsa force of 10N on the table. (b) The block exertsa force of 19.8 N on the table. (©) Theblock exerts force of 9.8N on the table. (4) The block hes an upward acceleration. 74, The blocks B and C in the figure have mass m each. ‘The strings 4B and BC are light, having tensions 7, and T, respectively. The systems in equilibrium witha constant horizontal force mg acting on C. ic energy of the system always increases (a) ten 6,= 12 (T= V5 me ‘Work, Power and Energy 197 75. A particle of mass 70 g, moving at $0 emis, is acted upon by a variable foree opposite to its direction of motion. The force Fis shown as a function of timer. (a) Its speed will be 50 em/s after the force stops acting. (b)_ Its direction of motion will reverse (©) Its average acceleration will be | m/s! during the interval in whieh the force acts. (2) Its average acceleration will be 10 m/s? during the interval in which the force acts. 76. A monkey of mass m kg slides down a light rope attached toa fixed spring balence, with an acceleration, «a. The reading of the spring balance is W kg. [g = acceleration due to gravity]. (a) The force of friction exerted by the rope on the monkey is m (g—a).N. We (©) m=W ( + 2) g (d)_ The tension in the rope is Wg N. 77. Ablock of weigth 1 is suspended from a s balance. The lower surface of the block rests on a ‘weighing machine. The spring balance reads WW, and the weighing machine reads #7, (WF, W,, W, are in the same unit). (b) m (@) WW, + W, ifthe system is at rest. (b) W> 1, +1, ifthe system moves down with some acceleration. (©) W, > W, if the system moves up with some acceleration. (d)_No relation between #¥, and W, can be obtained with the given description of the system. 78, A simple pendulum witha bob of mass m is suspended from the roof of a car moving with a horizontal acceleration a. (@) The sting makes an angle of tan" (aig) with the vertical. (0) The string makes an angle of tarr' ( | with the vertical Pearson Guide to Objective Physics (©) The tension in the string is m¥a"+@° . (@) The tension in the stingis mV&"— 2°, 79. Two masses M and m (M> m) are joined by a light string passing over a smooth light pulley. Fig 6.65 ayia (a) The acceleration of each block is ( Ere Je (b) The tension in the string is 2M. Mem (©) The centre of mass of the ‘Mf plus m* system M-my ‘moves down with an acceleration of g (2) (@)_ The tension in the string by which the pulley is attached to the roof is (M+ m) g. 80. In the previous question, the blocks are allowed to move for some time, after which M is stopped ‘momentarily (brought to rest and released at once). Afier th (@) both blocks will move withthe same acceleration. (b) the strings will become taut (under tension) again ‘when the blocks acquire the same speed. (©) thestring will become taut again when the blocks cover equal distances. (@)_at the instant when the string becomes taut again, there may be some exchange of impulse between the string and blocks. 81. A block of mass 1 kg moves under the influence of external forces on a rough horizontal surface. At some instant, it has a speed of | mis due east and an ion of | m/s? duc north. The force of friction (b) F acts due south, (©) F acts in the south-west direction. (@) The magnitude of F cannot be found from the given data, 82. A long block 4 is at rest ona smooth horizontal surface. A small block B, whose massis half of 4, is placed on A atone end and projected along 4 with some velocity 1. The coefficient of friction between the blocks is Fig6.66 (@) The blocks will reach a final common velocity (b) The work done against friction is two-thirds of the initial kinetic energy of B. (©) Before the blocks reach a common velocity, the acceleration of A relative to B is 3 Hg. (@) Before the blocks reach a common velocity the rt east 83. A 10-kg block is placed ona horizontal surface. The coefficient of friction between them is 0.2. A horizontal force P = 15 N first ects on it in the eastward direction. Later, in addition to P a second horizontal forse Q= 20 ets on it in the northward direction. (@ The blockwill not move when only Pacts, bt will ‘move when both Pand Q act. (©) Ifthe block moves, its acceleration will te 0.5 m/ s (©) When the block moves, its direetion of motion 4 ssa roca (d) When both P and @Q act, the direction of the force 3 ites dacwel estisow(2) west of south. 4. A block of mass m is placed on a rough horizontal surface. The coefficient of friction between them is ‘H. Anextemnal horizontal force is applied to the block and its magnitude is gradually increased. The force exerted by the block on the surface is R. (@) The magnitude of & will gradually increase. RemgVi +1, (© The angle made by R with the vertical will ‘gradually increase, (@ The angle made by R with the vertical < tan" ys. ‘A man pulls a block heavier than himself with a light rope. The coefficient of friction is the same between the man and the ground, and between the block and the ground, Fig 6.67 (@) The block will not move unless the man also moves. (b) The man can move even when the block is stationary. (©) _lfboth move, the acceleration of the manis greater than the acceleration of the block. (@)_ None of the above assertions is comect. 86, Acar Cof mass m, rests on a plank P of mass m,. The plank rests on a smooth floor. The string and pulley are ideal. The car starts and moves towards the pulley with acceleration. im) Pe) (2) If m,> my the string will remain under tension. (b) fm, < my the string will become slack. (©) Ifm,=m, the string will haveno tension, and C and P will have accelerations of equal magnitude. @ C and P will have accelerations of equal magnitude if m, m,. 87. A man tries to remain in equilibrium, Fig 6.69 (©) he must exert equal forcesion the two walls. (©) the forces of friction atthe two walls must be equal. © friction must be present on both walls. (@ the coefficients of friction must be the same between both walls and the man. 88, Twomenof unequal masses hold onto the two sections of a light rope passing over » smooth light pulley. Which of the following are possible? Work, Power and Energy 199 Fig 6.70 (a) The lighter man is stationary while the heavier man slides with some acceleration. (b)_ The heavier man is stationary while the lighter ‘man climbs with some acceleration, (©) The two men slide with the same acceleration in. the same direction. (@) The two men slide with accelerations of the same ‘magnitude in opposite directions. 89. Two blocks 4 and Bof the same mass are joined by a light string and placed on a horizontal surface. An extemal horizontal force P acts on 4. The tension in the string is 7. The forces of friction actingon A and B are F, and F, respectively. The limiting value of F, and Fis F,, As P is gradually increased, a aa +a Fig 6.71 (a) forP2F, T= > (d)_none of the above 90. A block is placed at the bottom of an inclined plane and projected upwands with some initial speed. Itslides up the incline, stops after time #,, and slides back in a further time f,. The angle of inclination of the plane with the horizontel is 0 and the coefficient of frietionis p. Fig 6.72 @1>4 OA<4 200 Pearson Guide to Objective Physics (©) The retardation of the block while movingup is g (6) its kinetic energy is constant. (in 6+ 1 cos 6). (4) itmoves in a circular path. (a) The acceleration ofthe lock white moving down 95. Consider two observers moving with respect to each is ¢ (sin 8 10s 6). other et aapced v log a straight line: They observea 91. The two blocks 4 and B of equal mass are initially in block of mass m moving a distance /on a rough surface. ‘contact when released from rest on the inclined plane. The following quantities will be same as observed by The coefficients of friction between the inclined plone the two observers. and A and B are 1, and y, respectively. (©) kinetic energy of the block at ime (8) workdone by friction, B {c) total work done on the block. é (@) acceleration of the block. 96. You lift asuitcase from the loor and keep it on a table. The work done by you on the suitcase does net depend @ on Fig 6.73 (@) the path taken by the suitease. (©) fy, >t the blocks will always remain in contact. (b) the time taken by you in deing so. (b) If, wy the blocks will have a common 97, No workis done by a force on an object if 1 (0) the forceis always perpendicular to its velocity. acceleration = (jt, +H) sin 8. 2 HFA (b) the force is always perpendicular to its (d) If #, < Hy the blocks will have a common aoceleration, 2 (©) the objectis stationary but the point af applicstion Bes og of the force moves on the object. tla (@) the object moves in such a way thatthe point of 92, A ball of mass m is attached tothe lower end of light application of the force remains fixed. vertical spring of force constant k. The upperend of 98, A particle of mass m is attached to a light string of acceleration = 94, A pas the spring is fixed, The ball is released from rest with the spring at its normal (unstretched) length, and comes to rest again afler descending through a distance x. (a) x= male (b) x= 2mgik (©) The ball will have no acceleration atthe position where it hes descended through x/2, (4) The ball will have an upward acceleration equal togatits lowermost position. 93. The total work doneon aparticle is equal to thechange i kinetic energy. (a) always. (b)_ only ifthe forces acting on it are conservative. (©) only if gravitational force alone acts on it. (only ifelastic force alone acts om it, je is acted upon by’ force of constant magnitude which is always perpendicular to the velocity of the particle. The motion of the particle takes place in a plane, If follows that (6) its velocity is constant. (b) its acceleration is constant. length /, the other end of which is fixed. Initially the string is kept horizontal and the particle is given an upward velocity v. The particle is just able to complete acircle. (2) The string becomes slack when the particle reaches its highest point, (b) The velocity of the particle becomes zero at the highest point. (©) The kinetic energy of the ballin i al position was 4m? =mgl as m= me (d)_ The particle again passes through the initial position. 99, ‘The kinetic energy of aparticle continuously increases with time (a) The resultant force on the particle must be parallel to the velocity at all instants. (b) The resultant force on the particle must be at an angle less than 90° all the time. (©) Its height above the ground level must continuously decrease, (® The magnitude of its linear momentum is increasing continuously. 100. One end of a light spring constant kis fixed to a wall and the other end is tied to a block placed on a smooth horizontal surface. In a displacement, the work done by the spring is $ Jo2. The possible cases are (@) the spring was initially compressed by adistance x and was finally in its natural length (b)_ it was initially stretched by a distance x and finally ‘was in its natural length. (©) _itwas intially in its natural length and finally in a ‘compressed posi (O _itvwas initially in its natural length and finally in a stretched position, 101. A block of mass Mis hanging over a smooth and light pulley througha light sting, The other end of the sting, is pulled by a constant force F. The kinetic energy of the block increases by 20 Jin 1 s. (@) The tension in the string is Mg. (b)_ The tension in the string is F. (©The work done by the tension on the block is 20 in the raid 1's. (@)_ The work done by the force of gravity is 20J in the raid 1. 102. Aman and a child are holding a uniform rod of length L in the horizontal direction in such a way that one fourth weight is supported by the child. Ifthe child is at one end of the tod then the distance of man from another end will be- 3L L oF ws L uw OF @F 103. A perfectly hard billiard ball of kinetic energy Ek collides with another similar ball at rest. After the collision the kinetic energy of the ball becomes E"k Then-— (0) Bk Ek (b) E'k> Ek (EES eK 104. The law ofconservation of momentum can be perfectly applied tothe collision between two particles ifthe time of collision is— (@) very large. less. (©) will depend upon the special circumstances. 105. A body covers a distance of 10 meter under the influence of $N force. Ifthe work done is 25 Joule then (b) very shor, the angle between the force and the displacement will be— or (30° (60° (ao 106, A bullet of mass P is fired with velocity Q in a large body of mass R. The final velocity of the system will tee R Pt © bag © eR fe) (ex0) {d) Peto 107. A sphere of mass m moving with a constant velocit collides with another stationary sphere of same mass. The ratio of velocities of two spheres after collision willbe, ifthe cotficient of restitution is e— de eect ® Ite o) e+] lee att Orn Oo 108, An electric motor produces a tension of 4500 W in a load lifting cable and rolls it at the rate of 2m/s. The power of the motor is— (9KW (bs KW () 25 KW (6)9x 10° HP 109. A body of mass m is accelerated to velocity v in time 1°, The work done by the force as a function of time ¢ will be— ifm ofa} am (a) oF 110. A ball falls from a height of Sm and strikes the roof of a lif, [fat the time of collision, lif is moving in the upward direction with a velocity of Inm/s, then the velocity with which the tall rebounds after collision be— (2) 11 m/s downwards (6) 12 m/s upwards (c) 13 m’s upwards (@) 12 m/s downwards 111. Ifthe force acting on a particle is zero then the quantities which are conserved are— (@ momemtumand angular momentum, (b) momentum and mechanical energy. (©) momentum and charg (4) angular momemtum and mechanical energy 112, The co-efficient of restitution depends upon- 202 Pearson Guideto Objective Physics (a) the masses of the colliding bodies. (b) the direction of motion of the coll (c) the inclination between the colliding bodies (@) the materials of the colliding bodies. 113. The value of e for plastic bodies is— @t 200 os (2) arbitrary 114. A motor of 100 H.P.is moving with a constant velocity of72 km/hour. The forward foreeexerted by the engine on the car is— (@)3.73 x 10'N (6)3.73 x1 N (©)3.73x10!N (@) None of the above 115. Uniform constant retarding force is applied in order to stop a truck. If its speed is doubled then the distance travelled by it will be— (2) four times (©) double (o) halt @) same 116, The kinetic energy of a man is half the kinetic energy of a boy of half of his mass. If the man increases his speed by 1 m/s hen his kinetic energy becomes equal to that of the boy. The ratio of the velocity of the boy and that of the man is— 2 1 @st o> 3 4 OG OF 117. Inthe above question the initial velocity of the man will be— ()3.S71m5 (©)2.415ms ()5.718mis (4127s 118, Two elastic bodies P and Q having equal masses are moving along the same line with velocities of 16 m/s and 10 m/s respectively. Their velocities aftr the elastic collision will be in mv’s— (a)Oand25 (b) Sand 20 (©) 10nd 16 (4) 20and5 119. A squirrel of mass m is moving on a disc of mass M and radius R, in a circle of radius 2/2 with angular velocity @ (Fig, 439). The angular frequency with which the disc will rotate in the opposite direction will be— oe) Fig 6.74 om or om oe 120. A ship of mass 3 > 10" Karis initially at rest, Itis being pulled bya force of S x 10'N through a distance 3m. If the air resistance is negligible, then the speed of the ship will be— @) Sms 2) 15 ms (©) 60 mis (6) 0.1 mis 121, A metal ball does not rebound when struck on a wall, whereas a rubber ball of same mass when thrown with the same velocity on the wall rebounds. From this it is inferred that— (@) change in momentum is same in both, (6) change in momentum in rubberballis more. (©) change in momemtumin metal ball is more. (®) initial momemtum of meta bat is more than that of rubber ball. 122. The unit of the co-efficient of restitution is— (a) mis (b)sim (xs (4) none of the above, 123. A particle moves in a potential region given by U= 8x! — 4x + 400 Joule. Its state of equilibrium will (b)x=025m (d)x=25m 124, Keeping in view the law of conservation of momentum, which of the following figures is incorrect? 3 i, ™ @ Fig 6.75 Work, Power and Energy 203 (a) 051m (b) 0.062 m (025m (@)0.72m 128. A ball with velocity 9mvs collides with another similar stationary ball. After the collision the two balls move in directions making an angle of 30° with the initial direction. The ratio of the speeds of balls after the collision will be— 128. The graphbetween JE, and I/pis (Z,=kinetic energy and p= momentum) | p—> “p—> 1 (@) & VE vo —> ine—s © @ Fig 6.76 126. The graph between U and X in the state of stable equilibrium will be— uy u Read the following passage and answer the questions given attheend, ‘Your physics teacher who gives you tutions asks you to do experiments on the springs which do not follow Hooke's law faithfully. He gives you a spring and asks to fix one end, Connect a mass Mon the other end (The system is ona smooth table). The spring is stretched by /and released, Then he asks you w bring another identical block and provide @) * ©) ¥ velocity vo that it pushes the spring by x. Thespring follows the law F'= kx — r?. The teacher asks you to find u uv CHO Fig 680 © x @ * 1, The velocity of the block fin case (i] when the block Fig 6.77 has reached mid point after the release 127. Two masses m, = 2kg end m, = Skg are moving on a F/T Ein ae directionless surface with velocities !0m/s and 3 m/s (a) F(Z) 3) (3 8) respectively m,is ahead of m,. An ideal spring of spring constant k= 1120 N/m is attached on the back side of, ‘my, The maximum compression of the spring will be, 3a TeP V1 ifn collision the two bodies stick together — io) sa fig none 2. The maximum compression in the spring incase (ii) x is the compression so that it satisfies 204 Pearson Guide to Objective Physics kere’ (6) satisfies > - > Me 3 ker? Me OFF none 3. The work done by the spring in ease (i) ae wee O>-G Oo wo woe os @ 2-2 OF Oy-s 2 1 [Pade + > Mi (conserve energy) or 4 beawory at 20) 2.) = Meo y= 5 li, mol vy 2 k- = 7 OM) (3] z ‘The compression produced is x such that BB ae 3° 2° 4 ke? 300) W= > y Read the following passage and answer the questions given atthe end, ‘The brothers of fota, Eta, Pi fraternity build a platform, supported at all four comers by vertical springs in the basement of their frat house. A brave fraternity brother ‘wearing a football helmet stands in the middle of the plat forrn, his weight compresses the springs by 0.18m, Then four of his fraternity brothers pushing down at the comers of the platform, compress the springs by another 0.53 m until the ‘op of the brave brother helmet is 0.9 m below the basement ceiling. They then simultaneously release the platform. Ignore the masses of spring and platform. The dean of students suggests them to perform the stunt at other planet. L. Find the velocity with which fratemity borther’shelmet hits the ceiling. (@)3.14me" (6) 2.80 ms" (©)2.93ms" (04.13 ms! 2. Ifthe ceiling would not have been their, how high he would have gone? (24m (lIm @2.1m @l4m 3. How high will he reach if he performs the stunt at a planet which has g equal to half the acceleration due to gravity on the earth? (@)L4m (b)2m (Jim (d) none of these a) ee 22h or v = ye(I 14 ms" HAC) Me Se Ee ke mi (8) =sen 220 ae onhe 3 (a) The height reached will not be attected by g if compression of the string remains same. ‘Read the following passage and answer the questions given attheend, Inan experiment a proton of mass 1.67 10” kgis propelled tan intial velocity 3 « 108 ms directly towards a uranium nucleus $m away, The proton is repelled by a force po 21210 x objects. Assume that uranium is at rest. As the proton approaches the uranium atom, repulsive force slows down the proton until it comes momentarily to rest, after which proton moves away from the nucleus. 1, What is the speed of the proton when itis 8 A” way from the nucleus ? (a) 1.85 * 10° ms (b) 1.85 10! ms (©) 1.85 * 10% ms! (@) 1.85 «10 ms 2. Find the closest distance of approach, Nin where x is separation between two (a) 14 A" (b) 2.8 A" (c) 2.8 pm (d) 2.8 fn 3. What isthe velocity of proton when it reaches back $ maway? (a) 1.8 * 10° mst (b) 2.3 «10° ms" (c)3 «10° ms? (d) 2.7 « 10*ms* 1 1 4 pedo!” F de L@ my Le 2g B2x10 oy orf = Ge toy 2A2IO 9 «108 67x10" x9x10" aa You have either reached a page that is unavailable for viewing or reached your viewing limit for this book. 206 rearson Lulde to Objective Physics 1 Eel A(b) For instantaneous power aw de FP ap = — Se ai am” Om P@-0 FT But average power= > = 5 Instantaneous power : average power = 2: 1. (a) Let XO= OZ = >. When end Z goes down by y then 2 ‘velocity of bend v = Pe 2 because bend falls through y/2. = = dy (gy)? (downward) = a wa (upward) x z 3 Fig 6.81 '3(c) Spring energy = kinetic energy 1 L he 5 hats = mv 2" 2 Time taken by ball to reach ground, 1 t fiom $= ut+ — at’ (fiom 7a) Horizontal distance covered = v1 a) K.E.ofblockat B= PE. atd—PE. at B 1 5 mv =mgh—mgr=mg (h—2r) ie P=2g(h-2n) =) Also —mg=xmg r or Wale Ire Equating (i) and (ii) 2g(h—-2)= (+ Der oe 2gh = (er gr rage 5(b) Using P~ Fu, we get P =F + v=r@) HereF = uN (where Mis normal reaction) are (:s Lkgf= g newton) P = pF gro=2nynF gr (-s @= 2m) 1 6 (a) Workenergy relationship ie. = > mi indicates that the graph between w and u should be a parabola towards w axis. 74) use PE. = Work done mH Heine BS or 8 z Hence The particle covers the length / or not, or covers it repeatedly is determined by the above relation. ‘c) PE. with reference to lowest point = Mg(2L) From conservation of energy, ie. v=2 Jel. Xd) Using conservation of momentum, (12045) »,=5%2 1 y MP2 MeL aa You have either reached a page that is unavailable for viewing or reached your viewing limit for this book. aa You have either reached a page that is unavailable for viewing or reached your viewing limit for this book. 26(a)When plank is moved, the string is under tension. Work done by force Fries to neutralise the frictional force of plank and elongation of the siring due to non sliding of the block due to its friction. Thus, work done by F is equivalent to option (a). 27(b)On ascending the plane, component of weight of tractor along the inclined plane will add to the given tension when the tractor and truck were moving ona level road ce. new tension, T=T+mgsind 1 = 1000+ 1200 9.8x = 1000 + 1960 = 2960 N 28(c) Work done in one rotation ~ work done against gravity 4 5 he FH 2mH 7 = 200 x10%5 x10 (Fx 2x1 = (mg) x pitch) 10 sya 73 8N 31a 29. Using conservation of energy 1 Lo fuy mere Lats el] cmmteme E gh + me > 2 = (ge ou (Fe) Insecond case FO) v=2mel v= Gaby or v- (SF 5. 30(a)Work done = Change in K.E. i die vette bt where W,=mgsand I, isthe workdone by airresistance manners Lm? Work, PowerandEnergy 209 Cv +2¢8) Og) ws ls 31(b)In case of vertical motion under gravity, when the body falls, force of gravity (a conservative force) is in the é ‘of motion of so K.E. increases but when the body is projected upward, force of gravity is opposite to motionso K.F. of body decreases. If kinetic energy of a body increases the work is taken as positive and vice versa 32(c)Work done = AKE, of c.g. of the chain) Solving for , 1a cs Se 5 (ut nt gmk? — yng) 34. (b) Forrigid cord, mgl= 5 my Equating i) and (ii) Do 4 mg = 4 mot + Lacay 2 2 Clearly v,> v, 35(a)For equilibrium Ma,,. x h= Mg Xp (q will not matter as per statement of the question because when the car is about to topple, the whole reaction is on the rear wheel as front wheel isno more in contact with the ground and here & is the reaction of the rear wheel) Fig 6.82 210 Pearson Guide to Objective Physics 36(4)Force x Time = Change in momentum Fig 6.83 Am = or FxAr=Amv V3 oF =F v v2 Al Vix pa, (3 Am= pad or F=v J? pav=¥ V2 pa (5) 37(a)Work done by man to stretch the spring I “ (54) (+2 Work done by man on the floor 1 - (3 «) © + Total work done= kee 2 38(b)Let a small displacement be given to the system in vertical plane in frame such that ST remains horizontal thea let vertical displacement of centres of rods UP and QR be y then vertical displacement of centres of UTand RS will be 3y and that of TS willby 4y. Equating the total virtual work to zero we get P 2 w a y Wy yee y 4 Y uF 3H + » Fig 6.84 (W+ W) y+ (H+ W) 369+ W (48) - T (465) =0 (where Tis the tension in thread) or 2W+6W+4W=4T or T=3W ‘39(C)CG. of first slab= : ‘Weight of each stab = volume x density xg = pg CG. of columnof slabs Total height of N slats ~ foalheshaiNss Nb “2 Height of displacement of force Nb_b 4 “272A g rt ne =ses'poxer-y 3 Vaasa, = 7 O-N)B pg 40(b)me+ Fos O=R Where m is mass of wiper, F is applied force towards centre and R is normal reaction of ground upward. If motion is allowed on the floor, Fsin 0 yN20 ie F cin Ou (mg+ Feos 8) 20 bi ie sin@-u cos > “TE or tan Op and in limiting case tan 6,1, or 6,=tan" p, 41(d)Let AB = BC= CD = I unit and end A moves slightly towards right such that /is increased to (1+ &). Let the total work be zero, we get F,Q.H-F,(&)=0 (-e displacement of A =3 6! and that of or F\= 42(d)Let the extensi enemy, we get 4 Mg (LBL) = 5 (BLY K (8) +2-2Mgl -2Mg5L=0 or K(&L)-2 Mg6l-2MgL Ma + fa 2K (By solving the quadratic equation) 26) (2 ve sign is meaning less) 43(a)Let the particle be at adistancex at any instant and Tbe the tension in the string then T=K(1? ~P)!?=Kx +4Kx2MeL, or ie. bL Net force tending to make the particle move further through dx, v= |S (2S-3pl) « +-[Bas-a] 44(a)Let the pulley be given a small rotation @ in anticlockwise direction, the pulley and load will te lifted by r,@ and P moves down by (r,6~ 7,8). quating vital work done to 210 = 18) We Poin (T~SRRIN 60,000 ay 45(a)Here Brake power = (6000 Brake power er (T-S) ary _ 8000%5.25 © 2ex05%220 =455.8N = 456N an Distance travelled Ss=Sie.2x5=10m Work done = Fx $=3.92 x10 -30.2) 1 41(o)Here may mY fs, ths, 4 jes may — Hg cos Bs, ~ jumg cos ys HV gaee| pH opus or y= e019) 48(0)P = Fu=e ( or sept 49(a)Here momentum of third fragments Po ptr fimo Final K.E. of the sysiem aa You have either reached a page that is unavailable for viewing or reached your viewing limit for this book. {hthighes point venta component (Le. eonponen) is zero] or speed of stone at highest point Using conservation of momentum y mV =2m¥, 3 5225 mst '86(a)At horizontal position, tension is zero i.e, velocity of combined mass is zero. Vennins ~21= 0 32m and W,= [-Kvitaj).dyj Ka =-Ka [ay WWW Kat '59(d) The graph of L(x) with x is as shown in figure Potential energy is zero at x= O and maximum at ‘As mechanical energy has fixed value i.e. k/2, the Kinetic energy has to be maximum at x= 0 and maximum at x= 40. Tow Fig 6.88 60(4)Using F=— a we get which is satisfied by graph (4) 661(a,c)When the string breaks, the particle starts from the point (2, 0, 5) with speed u, = 3 mis, and moves as a projectile. It will reach the xy plane in | second. z rs Fig 689 62(bed)The horizontal momentum of the system is conserved (= 0) till the collision, as there are no horizontal forces acting on the system. At the collision, ‘an additional impulse 2my is given by the wall to the system, G(ac)There are no horizontal forces acting on the ‘R plus B’ system. Hence, its centre of mass will move down vertically, nd horizontal momentum will be conserved. (4(a.0) nm, = Mv,, where v, and, are speeds of m and M, as seen from ground, y, MULL .—— Fig 6.90 ‘The velocity of m relative to Mis oy tot Hence, 1= <= = > ory tyait ‘65(a,c)if the system is isolated (no extemal forces), pand E are conserved. Electrostatic forces are intemal forces. To fix ¥, extcmal forces must act on the system, viz., on ¥. In that case, p is not conserved. However, these extemal forces do no work, as there is no displacement of YE is conserved. {67(b.,4)There is an exchange of linear velocities. However, the two spheres cannot exert torques on each other, a5 their surfaces are frictionless, and the angular velocities of the spheres do not change. (69(be)Let v= speed of 4 before impact. Thus, p = mu. Let, v, = speeds of é and B after impact, 214 Pearson Guide to Objective Physics u=v,+ andy, 1 1 yu(l-eandyy= 5 u(I +e) Jemy m[ utes) Spite) 74(a,b,¢,d)Draw the free-body diagrams for B and C. Balance horizontal and vertical forces separately for both. 75(a,b/The impulse given to the particle is equal to the area underthe F- graph = 0.07 kg m/s =2 xinitial momentum of the particle. Hence, the particle will reverse in direction and move with its initial speed. 76(a,b.d) The torce exerted by the rope and the monkey on ‘each other= the force of friction between the rope and the monkey = tension inthe rope = reading of the spring balance. 77(a,b.d) The readings of the spring balance andthe weighing ‘machine are equal o the forces exerted by them on the body. ™ Aw, w Fig 6.91 ‘79ac\Mg-T=Ma T-mg=ma 2Mmg Mam Ma-ma _{M. My. ao Mem (Mm) atm) 82(a,d)Forve of friction acts opposite to the direction of ‘motion or tendeney of motion. (a Solving, a tan ean ‘83(a,b.A)As there are no external forces acting on the ‘A + B system, its total momentum is conserved. If the masses of A and B are 2m and 2 respectively, and v is the final common velocity, mu = (m+ 2m)v or v= 1% Work done against friction = loss in KE = Gm)" $4(a,5.d)R is the resultant of the normal reaction, N= mg, and the force of friction, F'< amg. As Pis increased, F (EP) increases, while N is constant Pp af Fe Fig 693 85(ab,c,¢)Let T=tension in the rope. Force T acts on both The limiting force of friction is larger for the block than for the man, Each body will move when T exceeds the force of friction £86(a,b,c)Let T= tension in the string, F-= force of friction between Cand P. If the string is under tension, the acceleration of C 10 the right = acceleration of P to the left= a. 5 m Plan + o er ® Fig 694 Ti Pomak-T- ma mg (d) none of these aa You have either reached a page that is unavailable for viewing or reached your viewing limit for this book. aa You have either reached a page that is unavailable for viewing or reached your viewing limit for this book. aa You have either reached a page that is unavailable for viewing or reached your viewing limit for this book. 230 Pearson Guide to Objective Physics Bins! Fig. 7.26 3. A rocket having fuel as its bulk is initially at rest. Neglecting the effect of gravity, when fuel is burning ata constant rate, acceleration a of the rocket with respectto time tis best represented by one of the graphs given below. ss |” — — @ o) (©) @) Fig. 7.27 4, Two girls of equal mass m jump off from the border line of a stationery carriage of mass M with same horizontal velocity u relative to the carriage. ‘Nelgecting the effect of friction Ll O-O-O Fig. 7.28 (@ they will impart grester velocity to the carriage by jumping of simultancousty. (©) they will impart grester velocity to thecarriage by jumping one after the other. (C) they will impart greater velocity to the carriage in ‘whatever manner they jump off. (@ insufficient data to reply. In the shown system friction is meaningless. The car- riage of mass has constant initial velocity w along a straight horizontal track when at = 0, it startsraining. The rain drops have a vertical velocity u/ and result into addition of mass m per second to the carriage. The velocity of carraige after T second of start of rain, is Mu Mus mui © Seem ©) lem (utu'\(M +m) Mutu’) oy OS Fig. 7.29 A boy of mass m kg boards @ trolley of mass 2m moving with constant speed along a horizontal track. Neglecting friction, ifthe boy jumps vertically up with reference to the trolley to catch hold of a branch of a tree, the speed of trolley after the boy has jumped off O (au ©F @ ‘Two similar bogies A and B of same mass M (empty bogie) move with constant velocities v, and v, towards each other on smooth parallel tracks. At an instant a boy of mass m from bogie 4 and a boy of same mass from bogie B exchange their positicin by jumping in direction normal tothe track, then bogie 4 stops while B keeps moving in the same direction with new velocity v,. The initial velocities of bogie A and B are given by aa You have either reached a page that is unavailable for viewing or reached your viewing limit for this book. aa You have either reached a page that is unavailable for viewing or reached your viewing limit for this book. aa You have either reached a page that is unavailable for viewing or reached your viewing limit for this book. 234 Pearson Guide to Objective Physics (@) 12mst (b)1 ms ()2ms" (@)4ms* 48. A bullet weighing 10 g and moving at 300 ms" strikes a 5S kg block of ice and drops dead. The ice block is sitting on frictionless level surface. The speed of the block, after the collision is (0) 60 me" (6) 60 ms" (©) 6ms' (4)6 ms? 49. A body of mass m moving with a constant velocity hits another body of the same mass moving with same velocity v but in the opposite direction, and sticks to it, The velocity of the compound body after collision is (a) zero Og (©)2» (ave 50. A bullet of mass “a” and velocity “6” is fired into a large block of wood of mass ‘e’. The final velocity of the system is ab b oe & Zero ob b © Grp @za+h 51. A mass m, moves with a great velocity. If it strikes another mass m, at rest in a head on collision it comes back along its path with a low speed efter collision. Then (a) m,>m, (c)m Ce Two) TEE 1(d) Using the conservation of momentum, velocity of manis given by MV =mvie. ¥= 5% upward. Time taken by ball to reach ground is 3 mo) h change in the distance of man = ( ral : Total distance of man from ground = h+ ah “8) 9 (4% Resultant momentum of two pieces of equal masses is p = ont)sery = 212 kgms? From conservation of momentum 3¥=21 yj xo) (where is velocity of 3kg piece) or V = 7J2=9.87kgms! ‘3(@) Accleration ofa rockets given by a P_, (My mt) With the burning of fuel, mass of rocket decreases but engine keeps the force constant so accleration is picked up in the manner shown in (A) 4(0) When boys jump one by one total velocity of the sys- tem will be more. (because from conservation of mo- ‘mentum, velocity of car in this case will be 1 1 «.7"(qim i) ‘ 1 1 teat ote (37355 *a7sm ‘when they jump simultaneously. 5(2) From conservation of momentum Mu = (M+) V Mu * "Mem vertical down pour of rain does not effect the aa You have either reached a page that is unavailable for viewing or reached your viewing limit for this book. aa You have either reached a page that is unavailable for viewing or reached your viewing limit for this book. aa You have either reached a page that is unavailable for viewing or reached your viewing limit for this book. 242 Pearson Guide to Objective Physics about an axis parallel to the axis passing through COM and at adistance x from itas illustrated in fig. 8.9is — rE leat Mx? where Jpoy ib the MOI about an axis passing through their COM. Perpendicular Axis Theorem It can be applied only to plane lamina bodies. Ifx- and y~ axes chosen inthe plane of ‘the body and z-axis be perpendicularto this plane, three being ‘mutually perpendicular, then +1, where I, and /, are MO! about x-axis and y-axis respectively. Perpendicular ail theorem Hustraton do Angular velocity (instantaneous) © = deo Angular acceleration or= 7 df Linear velocity v= rw; tangential acceleration a, = rar i O=O- 01 ;0=af+ > ar; @= 024208 Torque (1) t= 7 x F o | # |= Force x perpendicular from the axis of rotation. a # = Gj where Lis angular momentum, Note: Torque is moment of a force about a point. Though dimensions of torque are same as that of energy but itis not energy. Its unit is N-m. Dimensional formula is Wwe ry If line of action ofa force passes through its COM thea such 2 force will not form torque. Angular Momentum is moment of momentum (linear) Fxp about a point, ie. £ E=10:\E|=Z.p. (perpendicular distance from axis of rotation). Note: If external torque is zero then angular moment conserved. Dimensional formula of = [MZ? T-!,ltsunitis kem? s! and is same as that of Planck’s constant h. Angularimpulse J. 1 Rotational kinetic energy = 5 1 Note if a body only rotates about a fixed axis then it posseses only rotational KE. If, however, a body rolls then it possesses both rotational KE and linear KE, ie. Total KE, = Rotation: LEE 1 Tce KE + Linear KE= 5/7 + > my Work done W'= [# dé; Rotational Power P..= # Acceleration of body rolling down an incline plane: In Fig. 8.11 a h | EXEIE Acceleration of body rolling down an incline gsind 1a ‘Time taken to reach the ground 1 ~ For a system to be in rotational equilibrium 3 € For a system to be in linear equilibrium YF =0 For total equilibrium (Rotational + Linear) Euless. aa You have either reached a page that is unavailable for viewing or reached your viewing limit for this book. aa You have either reached a page that is unavailable for viewing or reached your viewing limit for this book. aa You have either reached a page that is unavailable for viewing or reached your viewing limit for this book. 246 Pearson Guide to Objective Physics => Perpendicular axis theorem can be applied only to plane lamina, Parallel axis theorem is valid for all, types of bodies. 10. Confused, what to do in rotational collision?” = Conserve angular momentum. SOLVED PROBLEMS 1. The moment of inertia of a uniform semicircular disc of ‘mass M and radius R about. line perpendicular to the plane of disc and passing through the centre is MR 2. OF (0) 5 MR (MR oe IATEEE 2005] eR warea( ort ME 2. AT shaped object with dimensions shown in fig, is Iying on a smooth floor. A force F is applied at point P parallel to 4B such that the object has only translational motion without rotation. Find location of P from C At——— | ——_+8 | r—— u c Fig. 8.19 2) 37 3 3 A oF! @! IATEEE 2008) (©) P should be COM. Take Cas origin 3. Acireulardise of radius 84 is cut from a circulardise ‘of radius R and mass 9 Mas shown. Then MOI of the remaining dise about O perpendicular to the dise is ; Fig. 8.20 (a)4.MR* (b)9 MR* 3 ge 2 sae? © 9 MR (@ 9 {IIT Screening 2005] 9M R® M(RY 2R or [BST om FF] =4MR carmel [El RR’ 3 4. A sphere is rolling ona frictionlesssurface as shown in Fig 8.21 with a translational velocity ums", Ifit isto climb the inclined surface then v should be art t Fig. 8.21 fo (az ra (b> gh 10 (©)2gh (> gh IATEFE 2005] toa (@) 5 10+ 5 mv? > mgt (2mm? gest more or | 57" | ot 5 mez meh aa You have either reached a page that is unavailable for viewing or reached your viewing limit for this book. aa You have either reached a page that is unavailable for viewing or reached your viewing limit for this book. aa You have either reached a page that is unavailable for viewing or reached your viewing limit for this book. 250 Pearson Guide to Objective Physics i“ laa Mw an EY wr (jue we) vee 23. A boy of mass M stands on a platform of radius R capable to rotate freely about its axis. The moment of inertia of the platform is J The system is at rest. The friend ofthe boy throws. tall of mass m with a velocity Mf. 2cta’ vhorizontally. The boy onthe platform catches it. Find law ee ican lot eth sem ne pce 24. Find the MOI of a uniform square plate of mass m and oe, je ‘edge & about one of its diagonals. remy T+MR p mk mR o, Deed / Teme? Tarek (ymvR=[F+QremR'] mR al 08 Tere 24, A ball of steel rolls down an incline of inclination 8. Find the ratio of rotational KE to linear KE. 2 2 oF ©) 5 Ma Me OF oO On Tan h 2, jie © sig Ta or dere 7” 25. The pulley shown in fig hes MOI 0.5 kg m?and radius 10 em. Assuming no friction any where, find the acceleration of 4 kg block 22, Two spheres each of mass Mand radius R are in contact, 1a shown. Find the MOI if they are rotated about the common tangent, Fig. 8.32 Fig. 8.31 (a) LU ms? (b)0.75 ms? > > (©)05 ms? (4)0.25 ms? (@) gue ) 5 MR (d) 4g cos 45~T,=4a wel) (.-T)R=1a () SMR ( tMR n-ne ot 7 5 3 a 7,-T= Zr ~@ aa You have either reached a page that is unavailable for viewing or reached your viewing limit for this book. aa You have either reached a page that is unavailable for viewing or reached your viewing limit for this book. aa You have either reached a page that is unavailable for viewing or reached your viewing limit for this book. 254 son Guide to Objective Physics O Wem a] Fig. 842 (a) 089 ms (c) 0.69 ms? (a) 2a=2g-T Tr =1a (b) 1.12 ms? (@) none ®) 02a 1 oe Pe on or T a @ From equations (1) and (2) 28 (+20) 189 ms? A paper roll of Mkg and radius R rests against the wall and is held in plave with brackets as shown, Assume ro friction. MOI of the paper and the rod is /about the axis. The other end of the bracket is connected to the wall by @ smooth hinge making an angle 6 as shown. Neglect weight of the bracket. The kinetic friction coefficient is 14. A Force F is applied on the paper to unroll it. Find the angular acceleration Fig. 8.43 and M g pass through COM and hence do Tot contribute to torque. PLR+UNR=10 Ms Fig. 8.44 (usinga= ra E ges i tou ee 37. The door of an almirah is 6 ft high, 1.5 ft wide and weights 8 kg. The door is supported by two hinges situated at a distance of 1 ft from the ends. Assuming forces exerted on the hinges are equal, the magnitude of the force is @ISN (ION (@28N (A3N -——— ke F cos Fig. 8.48 Taking torque about O 3) Bel 5 |=4F cos 6 3 ense( Fy) nr s¥73 =43N 38. A uniform rod of length 6a and mass 8 m lies on a smooth horizontal table. Two point masses m and 2m ‘moving in the same horizontal plane with speeds 2 v ‘and v strike the rod as shown in the Fig 8.46. Find the velocity of centre of mass and angular velocity about COM. Also find KE just after collision. Conserve momentum as external force is zero, 2m aa You have either reached a page that is unavailable for viewing or reached your viewing limit for this book. aa You have either reached a page that is unavailable for viewing or reached your viewing limit for this book. aa You have either reached a page that is unavailable for viewing or reached your viewing limit for this book. 258 Pearson Guide to Objective Physics rotates once every 0.0331 » and its period is increasing by 4.22 » 10's for each second of time that elapses. Theories ‘of supernova predict thal theneutron star in the Crab Nebuls hhas a mass about 1.4 times that of the sun. Assume neutron star asa uniform solid sphere |. Assuming energy lost by neutron star is equal to the rate at which energy is released by Nebula, Find the ‘moment of inertia, (a) 8.5 * 10 kam? (b)2-* 10 kam® (62 10" kem> (a)2* 10" kam? Find the radius of neutron star. (@) 183km (b) 13.64m, (3 *107km (6) 13.6 « 10'km, What iy the linear speed at the equator of a neutron star? (a) 2.6 « 10% ms! (b)2.6 » 10° ms" (©)2.6 * 10" ms! (4) none of these 4. What is the density of neutron star? (2) 2.4 10%kgm (b)24 «10% kgm? ()24* 10" kgm * 1 LW 5 ha (d) none 5x10" som] EE) or 2 0) R? 2(b) 5 (L423 10%R = 1.810" or R= 136 * 10 oF 13.6 km 34ayr= kor = 1.36% 10! = 2.6 10° ms! M. MEE eeagi 3 3x1LAx2x10" ~ 4x3.14x(130x10') = (136) = 247 10" kgm? PASSAGE 4 Read the following passage and answer the questions given. attheend, Kitehen food processor has become a necessity now-a-days in every kitchen. An old man has a food processor which works ona de motor. At f=0 the direction of current in the moter of the processor is reversed. It results intoan angular displacement 0(1)=250 1-202 1501 When does the velocity become zero!) (07s (b)06s (03s (035 2, Find the angularaccelaration when the angular velocity (a)-630 rads * (b) 40 rads? (©)-670rads* (a) none 3. Find average velocity in the time interval Oto the time calculated in question 55 ? (a) 113.75 rads! (b) 143.25 ads! (c) 102.5 rads" (d) none 4. How fast was the motor shaftat r= 0? (@) 125 nds! (b)250rads-* (375d wo do 1.2) = 250-401-4507" = 0 0 or 45/441 Vi6+4500 90 do 2.0) Se 40 - 9001-40 9004.7) --670 rads 3.(6) (0) =0,0(.7) = 250(1)=20¢77 1507) = 175-98- 51.45 = 113.75 (113.750) = SP oa.srads aa) A hes 4.(b) = 250 rad s~ aa You have either reached a page that is unavailable for viewing or reached your viewing limit for this book. aa You have either reached a page that is unavailable for viewing or reached your viewing limit for this book. aa You have either reached a page that is unavailable for viewing or reached your viewing limit for this book. aa You have either reached a page that is unavailable for viewing or reached your viewing limit for this book. aa You have either reached a page that is unavailable for viewing or reached your viewing limit for this book. aa You have either reached a page that is unavailable for viewing or reached your viewing limit for this book. aa You have either reached a page that is unavailable for viewing or reached your viewing limit for this book. 266 Pearson Guide to Objective Physics fe Fig. 8.65 @mor (b)mg os (@ma*r+ me 76. A Yo-Yois.atoy inthe formofadise with a concentric shaft. A stringis wound on the shaft. [itis suspended from the free end, then the string unwinds and winds so that the Yo-Yo falls down and rises up again and again. The ratio of the tension in the string while descending and ascending is Fig. 8.66 (liz (yrs (ORR, (RR 77. Two masses of 200 gm and 300 gm ere tached to the 20cmand70 em marks ofa light meterscale respectively. The moment of inertia of this system 2bout an axis passing through 50.em mark will be 240 om 300 om 20 50 70 100 Fig. 8.67 ()0.3Ke/mr (b)003 Keim? (0.15 Kyim* (a)2er0 78. The ratio of angular frequency and linear frequency is (2m on 1 £ OF @z 79. A mass Mis moving with constant velocity parallel to sraxis, Isangular momentum about the origin {a)is zero. (c) decreases. 80. Which of the following bodies of same mass has maximum moment of inertia about its geometric axis? (b) increases, (d) remains constant (a) abarpendulum. (bja solid sphere, (dja circular dise, 81. Which of the following relations is wrong ? (c)acireular ing (a) JarxP (b) (©) §=@xF @ 82. If the position vector of a particle is #=(3/=4)) mere adit ngularveloiyie @=(}=24) nde then its linear velocity is (in m/s) (a) -(87-6) +38) (c) (37-6) +88) i —6]+6E) a (67 -8F+38) 83, Moon is revolving round the earth us well a itis rotating about its own axis. The ratio of its angular momenta in two cases will be—(orbital radius of moon = 3.82 10° mand radius of moon = 1.74 x 10" m) (a) 1.22% 10 (by 22x10" (c) 122x108 (122x109 84, The moment of inertia of a solid cylinder of mass M, Jength L and radius R about the diameter of one of its faces will be (@ uf ()ze10 85. Equal torques are applied about a central axis on two rings of same mass and same thickness but made up of different materials. Ifratio oftheir densities is 4:1 then theratio of their angular acceleration will be (a)16:1 (b)1:16 (o)8:1 (di:2 86. A circular hoop of mass M and radius R is suspended from a nail in the wall. Its moment of inertia about an axicalong the nail will be (be (a) 2080. ()2MR aa You have either reached a page that is unavailable for viewing or reached your viewing limit for this book. aa You have either reached a page that is unavailable for viewing or reached your viewing limit for this book. aa You have either reached a page that is unavailable for viewing or reached your viewing limit for this book. 270 Pearson Guide to Objective Physics 1 2 % 1 . (a) 2 (b) 5 (a) 1 Kem? (b) 2 Kem £ L fT vag Pega Oy Ws (©) 75 Kem’ OF gm’ 125. A solid cube of side 18 made wo oscillate about 8 128:The diameter ofa solid disc is 0.5m and is mass is horizontal axis passing through one of its edges. Its 16Kg. What torque will increase its angular velocity mopar atllbe for covet 20 tans inane in econ? ® Bas ot Nim 2 Nm Sp N (d) 2 Nit () 2m id) im 129. In the above problem at what rate is work done by the torque atthe end of 8th second ? 126. The radius of gyration of 2 plane lamina of mass M. ewe owe length Zand breadth B about an axis passing through ee . its center of gravity and perpendicular to its plane will ey? Watt (aay Wat te 130, Two point masses are lying ona smooth uniform mass -M and length. Initially the massesare in the middle of the rod, The system is rotating about an axis passing ‘through the center and perpendicular to the rod with angular velocities @. No external force acts on the Perl aR system, When the masses reach the ends of the rod, © {e+e 2 (d@) (2 +82 then the angular velocity of the system will be 127, Two spheres each of mass IKg are attached to the two (a) (+B 12 (by (+8 ys Me Me ends of a rod of mass IKg and length Im. The moment oy a wz me of inertia of the system about an axis passing through a ae ws contre of gravity and perpendicular to the rod will Meo, Ma ts ()ie6m (8 Trem POEs Lo Tt sae ao 1 (o) 3. @ bcd 6. a) © 8 10. faba) (od) 13. (abe) (eda) 6 (ea) 47. (s) fed) 20. (a) : 22 2. (¢) 2a. (abcd) (bed) 7. (abc) (ab.c.a) a ». @) 3 © wb) (oc) 3%. a7. (ay) 38. © @) 4. (©) @ 4, 4 a) s © ©) 4& (a) () 50, s. @) 52. © @ © 7. 58. (b) 50. © @ @ (e) @) 64, 0) 86. @ en) @ 1, m (0) BB. @ @ % i) ©) 8. m @) 80. © ® 8 @ ©) 25, 6. (e) a7. © ©) 9. ie) ® 92, 93. (a) 94. @ @ 7 a) .(b) 9, wo. (0) wt. © (@) 106. fa) 108. (e) 108. 107. (@) 108. 109. (c) @ tH © 12) 13, 4d) 15, 16.) ©) 18 te) 6) 20, wm. me 123. (0) ©) 25.) ©) 21, 128 (a) 18. 130. (0) aa You have either reached a page that is unavailable for viewing or reached your viewing limit for this book. aa You have either reached a page that is unavailable for viewing or reached your viewing limit for this book. aa You have either reached a page that is unavailable for viewing or reached your viewing limit for this book. BRIEF REVIEW Newton's Law of Gravitation Newton in 1665 formulated Feemm. Where G= 6.67 x10"!'N im? kg? and is called universal gravitational constant. The value of G was first measured bby Cavendish in 1736, The value of G measured for small distances (r <200 m) is less by about 19% and pethaps gives ‘an indication of a fifth natural force. Note, gravitational field is independent of the nature of medium between the masses. e._# GEER sravitational force between two masses Zam Gravitational Field Intensity Gravitational force per ‘unit mass s called gravitational field intensity. Gravitational field intensity of earth is ‘g’. F ™ om 5 and g E, Gravitation the axial ineas illustrated in Fig. 92 is Gravitational field due to a dise at any point on the axial te 26M J, 2GM — |1-ee 1 —cos 9] in terms of [yee] oa angle Gravitational field intensity due to a shell See Fig. 9.3 aa You have either reached a page that is unavailable for viewing or reached your viewing limit for this book. aa You have either reached a page that is unavailable for viewing or reached your viewing limit for this book. aa You have either reached a page that is unavailable for viewing or reached your viewing limit for this book. aa You have either reached a page that is unavailable for viewing or reached your viewing limit for this book. aa You have either reached a page that is unavailable for viewing or reached your viewing limit for this book. aa You have either reached a page that is unavailable for viewing or reached your viewing limit for this book. aa You have either reached a page that is unavailable for viewing or reached your viewing limit for this book. 282 Pearson Guide to Objective Physics Grain, OF= Gaye gravitational field inside the shell is zero. Oil does not play any role. 13, A 75 kg astronaut is repairing Hubble telescope at a height of 600 km above the surface of the earth, Find shell exerts no force as its his weight there. () 740.N (b) 700 (©)650N (a) 610N SIE 4) mg’ = mg ( 3) = 75 x 10(1 ~-185)=610N 14, 5 kg and 10 kg spheres are 1 m apart. Where the ‘gravitation field intensity be zero fiom 5 kg block. @oam (0.3 m (©)025 m (@)0.35 m v2 T ® 1 e+ 15. A Ring hes mass M, radius R. A point mass mis placed ata distance xon the axial line as shown. Find.x so that force experienced is maximum, or a Fig, 9.16 w Me wh (© RV? (a) R/VB moor A 16. A saiteliteis in sufficiently low orbit so thas tencounter air drag and if orbit changes from r to r— Ar. Find the change in orbital velocity and change in PE. [GM Gémar 2 Yt a? (A) none GM. OM eae Av = 17. When an object isin a circular orbit of radius r, its time period of revolution about the earth is T and orbital velocity is vwhen its orbit is r+ Ar. Find the change in time period AT and orbital velocity Av. aes 2dr ir Oar On ae ante dae Ea 5 OF (qd) none r aa You have either reached a page that is unavailable for viewing or reached your viewing limit for this book. aa You have either reached a page that is unavailable for viewing or reached your viewing limit for this book. aa You have either reached a page that is unavailable for viewing or reached your viewing limit for this book. 288. Pearson Guide to Objective Physics and 33. At what height over the earth's pole the freefall acceleration decreases by 1% (@ 64am ()16km (o8km (@)32km owe Bal apes s'=8 R199 "92K 34, A satellite of moon revolves around it in a radius n times the radius of moon (R). Due to cosmic dust it experiences a resistance F = crv, Find how long itwill stay in the orbit. vn -1) m (da-l) Og» 35, Aparticleis projected with a velocity 15 kms". Find its velocity in the space far off from the earth, (@38kms* ()7.6kms* (©) 10kms" (N2kms* (6) bf vf— 02 = 15°=(11.29 of v= 10k” 36. Find the minimum velocity to be imparted to e body so that it escapes the solar system @ ) (@)none © joa | Me +O] 1 1 SEMEN) 5 mut= 5m u2+ 5 many = yur +(2-1 0? 2G, _ GoM, where v= JE andu,= JE Rg the orbital distance of earth from the sun. AR, is the radius of the earth. A tunnel is dug along a chord of the earth at a a R Perpendicular distance 3 from the earth's centre, ‘Assume wall ofthe tunnelis frictionless, Find the force exerted by the wall on mass mat adistance.x from the centre of the tunnel. Fig. 9.24 (o) 28 oF @ 7 eet 9 9 (©) F=ng’=mg) I~ neg. % 38, A body weighs 1 kg by a spring balance at the north pole. What wil ‘at the equator? (2)0977kg, (60.967 kg, ()0987kg, ()0.997 ke. aa You have either reached a page that is unavailable for viewing or reached your viewing limit for this book. aa You have either reached a page that is unavailable for viewing or reached your viewing limit for this book. aa You have either reached a page that is unavailable for viewing or reached your viewing limit for this book. 290 Pearson Guide to Objective Physics @», (b)15y, VTS» @2¥, 10, The escape velocity for a planet is v,. A particle is projected from its surface with a speed v. For this, particle to move as a satelite around the planet, @ F 2 J/R/g - 13, A satellite close tothe earth s in orbitabove the equator With a period of rotation of 1.5 hours. Iit is above a point P on the equator at some time, it wll be above P again after time @ 1.5hours (b) 1.6 hoursif itis rotating from west to east. (©) 24/17 hours ifit is rotating from west to east. (4) 24/17 hours ifit is rotating from east to west. 14, For a satellite to be geostationary, which of the following are essential conditions? (2) Semust always be stationed above the equator. (b) Itmust rotate from west to east, (©) Iemust be about 36,000 km above the earth, (4) Its onbit mustbe circular, and not elliptical. 15, Two small satellites move in circular orbits around the carth, at distances r and r ~ Ar from the centre of the earth. Their time period of rotation are Tand 7+ AT. (Ar < Rand r,> R Mr ifr, > Rand r,> R itr, (e) none of these 28. A springhas mass M and spring constant k. A mass 's attached to it as shown in Fig. 6. Find the extension produced in steady state. aa You have either reached a page that is unavailable for viewing or reached your viewing limit for this book. aa You have either reached a page that is unavailable for viewing or reached your viewing limit for this book. aa You have either reached a page that is unavailable for viewing or reached your viewing limit for this book. 366 Pearson Guide to Objective Physics 2. The spring in Fig. 2 is compressed, on releasing And Bmore with velocity V and F, The ratio of the KE-A to Bis: ‘ mk Ty 4 as Fig. 2 my ne nyt, my — my O-3,—, Onam (©) none of these rocket at the end of 60s is, (@)2740ms* (€)3200ms* (b)3328 ms" (@) none of these 4 Co emits rray of wavelength 0.932 < 10-? m. Assuming the cobalt nucleus was at rest before emitting rray then recoil velocity of Co is ‘mass of neutron or proton is 1.66 = 10°" kg. (@) 931 ms! (b) 821 mst (07.13 ms! (d) none of these 5. Aparticleof mass m collides elasti initially atrest. Ifthe KE of massm isK, The maximum KE itean lose during the collision is 4m (9) (=m) 2mMk © (tem & A particle of mass 2kz is moving along a line Y= x + 4 with a velocity 6/2 ms“, The angular momentum of the particle is (2) 48 units (b) 24 units (6) 48.2 unit {d)24 3 units 7. A baal of mass m and radius is spinaing whill falling vertically. ls acceleration is 3 @e bse 5s 2 Os? Assume the ratio of initial to finel ma@of the rocket is 4. The fuel is consumed in 60s, The relative velocity of sas emerging out is 2400 ms". Then the velocity of the A bicycle wheel has a rim of radius 0.3 m mass Ikg and 50 spokes. Find its moment of inertia assuming each space has mass 1.01 kg (a)0.105 kgm? (b)6.09kgm? (60.1125 kgm (60.9075 kgm? (e) none of these A cylinder of mass m and radius r rolling ona smooth horizontal plane with » rpm. If a wall appears in its path, how high on the wall the cylinder will rise? lenin’ os ©) 360 (©) none of these 10. A cylinder of radius 2 is spinned about its axis to the angular velocity @, and then placed into a comer as shown in Fig. 3. The coeff. of friction between the wall’s and eylinder is yz. How many turns will the cylinder take before it steps. Fig. 3 wo 2k ait) ug anu slug ogra oga(ue +1) eu Dug (© elus LL. A uniform cylinder of mass m and radius R starts descending at a moment 1 = 0 due to gravity. ‘Neglecting mass of thread, find the time dependence of power gained by gravitational force (Fig. 4) ria a mgt tonne eet 2 3 od mer 3 met aa You have either reached a page that is unavailable for viewing or reached your viewing limit for this book. aa You have either reached a page that is unavailable for viewing or reached your viewing limit for this book. aa You have either reached a page that is unavailable for viewing or reached your viewing limit for this book. 370 Pearson Guide to Objective Physics 9) @ © a) 6 @) te) & ©) 2d) i © 2 @ 2 14 (0) 1% (a) 16. (b) () © 19. (0) 20. (@) 2. (a) 2. (c) Be @) @ @ 26. (a) 2. () 2 (©) 2.) 2. (2) @ oy 2 © Ad) 98 (0) 38.) m @) © @ 40.) a. © a2 () Read the following passage and answer the questions given 2.__ Find the time period of oscillation of the system shown attheend. in Fig. ‘The simpleharmonic oscillator of mass m coupled by a spring of force constant ktoa solid wall. The wall i rigidly connected tothe earth, so that this system isreally a two-body system, connected by a spring, one of the bodies being effectively of infinite mass. Tais solid support remains at rest in an inertial reference frame so that the change in length of spring is ‘equal to the displacement of the mass m, the other end of spring does not move. Inthis case define the potential energy L(x) of the escillating system to be a function of the [ern ye]a displacement x ofthe mass m alone. This again is equivalent @ T= 2n Lathe le ‘to assuming that one end of the spring is connecied to an Kish infinite mass sothatthe extension of the springs determined bby the motion of mass m alone. iM Often in nature we find two-body oscillating systems ©) TH 2 Ee in which we can’t take the mass of one of the bodies to be 7 infinite and we must consider the motions of toth bodies in anappropriate inertial reference frame, Examples are diatomic (© T=2". (kh +h +h mn molecales such as #,, Co, Hel ete which can oscillate along Tih thik + hk their axis of symmetry. The coupling between the atoms that make up these molecules is electromagnetic but we may (@) none of these imagine them, for our purpose, to be connected by atiny, 3, A hydrogen atom is attached to chlorine molecule and massless spring. ‘oscillates with 10" Hz with an amplitude 0.1 A°. Find 1. A spring of force constant kis connectedto two blocks the force aciing on the hydrogen atom. of mass m, and m, as shown in Fig. Ifthe spring is ioe igw mopeclty vedic fndtietinejenatat | ()S6%10°N @ asm eeN cscillation. (6) 6.6 10°N (€)6.6* 107N 4. A spring has spring constant k, mass I. Its loaded | COOH] Fig.1 (T= 2m, { R = a, [me (T= 2H ©) T= 2855, +m) - mm ree with a mass M. Find the time period of oscillation. Fig.3 aa You have either reached a page that is unavailable for viewing or reached your viewing limit for this book. aa You have either reached a page that is unavailable for viewing or reached your viewing limit for this book. aa You have either reached a page that is unavailable for viewing or reached your viewing limit for this book. 374 Pearson Guide toObjective Physics 30A For one dimensional motion the angle between acceleration and velocity should be zero. 31A A simple pendulum hanging from the roof of moving train may have the string inclined towards the rear side ofthe train, 32A Range of projectile is maximum when the angle of 4 33A A body with constant acceleration always moves along straight line 344 The velocity ofa particle may vary even when its speed is constant. [35A The time rate of change of speed in one dimensional motion depends on the insianeous velocity. 36A Distance covered = Magnitude of displacement projection is 37A Relative velocity of A with reference to B is greater than the velocity of either, when they are moving in opposite directions, 38A A body moving uniformly may be accelerated. 39A Trajectory of projectile is aparabola. 4A Heavier bodies fall with greateracceleration ALA The acceleration ofa particle in uniform circular motion is constant both in magnitude and direction. 42A IT the string of an oscillating simple pendulum is cut, when the bob is at the mean position, the bob falls along a parabolic path, 3A The displacement magnitude of a particle is equal to the algebraic sum of the areas between the v~ graph and the time axis, 44A A sail boat can not be propelled by air blown at the rail from a fan attached to the boat. SA A parallelopiped with sides.a, 2s and 3a is in the most stable equilibrium, when the length 3a is vertical. 46A The centre of mass and centre of gravity always coincide. 4TA If the atmosphere suddenly condenses into a solid mass and forms a thin layer around the surface of the ‘earth, then the duration of the say will increase. 48A Angular momentum ofa system of particle isalways ‘conserved, 49A Langer the size of the body, smaller i of inertia, ‘50A Moment of inertia depends on the axis of rotaion and itsmoment the nature of distribution of mass of the body. SIA Internal forces can change the momentum of the nw R R R ‘One dimensional motion is on a straight line. The train is retarding, Range of projectile depends on the angle of projection, A body with constant acceleration may not speed up. ‘Such a body may move along a circular path, Speed = Magnitude of velocity. Particle moves with acceleration always directed along the direction of instantaneous velocity Relative velocity of A with reference to B= ¥,—V, Acceleration does not imply speed up or speed down along. Projectile possesses two dimensional motion. Gravitational force is more on heavier bodies, Inuniform circular motion, the particle experiences centripetal acceleration alone. ‘The bob possesses horizontal velocity, at the mean position Area between the v— t graph and the time axis is equal to the distance covered Action of the air on the sail and reaction of the sail, both act on the boat and their resultant is zero. ‘The paralleopiped is in the most stable eq centre of gravity is at the least height. Centre of mass depends on the distribution of mass but the centre of gravity depends on the distribution of mass as well as the variation of acceleration due to gravity. ‘The moment of inertia of the earth about its own axis will increase, so the angular speed of rotation of the earth will decrease. ‘Torque = time rate of change of angular momentum. librium when Moment of inertia of a body depends on the distribution of the mass of the body. ‘Moment of inertia isthe rotational inertia of the body. Internal forces are mutual forces. aa You have either reached a page that is unavailable for viewing or reached your viewing limit for this book. aa You have either reached a page that is unavailable for viewing or reached your viewing limit for this book. aa You have either reached a page that is unavailable for viewing or reached your viewing limit for this book. 378 Pearson Guide to Objective 16. 1». 20. 2. 2. 2. 24, 25, 26. 27. 28. wD. ai. 3. AxB ABsindA ABcosé ‘where j is unit vector perpendicularto both 3 and 3. However tan@ aE \a+8l-14_B! £+B+24B e050 = A+ BE2AB cos @ Hence, cos 80 which gives 8= 90° Also, vector addition is commutative. Hence 45° Bea Net displacement Time elapsed Average velocity = For 6= 180° which is greater than ¥, oF vy [Displacement < Distance covered. 3 = axi, The expression G = Fx7 iswrong. Reason iscortect explanation, IFangle between j and +7 is 6, then Ma) cos = Fis] ~ inva VE [AxB] aBsine TB” ABcose For giving a zero resultant, it should be possible to represent the given vectors along the sides ofa closed polygon and minimum numberof sides ofa polygon is three. In fact FycB = Bc The two cross products are directed opposite to each other [A+B] = 42 + B+ 248 cos @and [4-3] 2AB cos 6. Ifwe equate the two then we find cos 9= 0, which gives @= 96° Magnitude of insiantancous velocity is always equal to the instantaneous speed, irrespective of the nature of motion, Displacement 7-7, Forone dimensional motion angle between velocity and acceleration may be zero or 180°. For the rearding tain, the ring is incfined towards the Frontside, Range of projectile is proportional to sin 28, where Bis angle of projection, And sin 20 isis maximum when o= x8, e+ BE 33. Projectile is an example of motion with constant acceleration, it moves along a parabola, Also, it is 4 case of body whose speed first goes down and then Boes up. Particle can move along any curve with constant speed. In such a case direction and henee, velocity changes. The time rate of change of speed in one dimensional motion is magnitude of acceleration and itis independent of instantaneous velocity The particle moves along a straight line path with no change in the direction of motion and hence, distance covered = magnitude of displacement. 37. Since velocities are in opposite direction, therefore Vig 34 35. ell apr agwnienassemee tin 8, 38. A body moving uniformly on a circular path possesses centripetal acceleration, which is due to the change in direction of velocity Assertion as well as reason are statements of facts but reason is not the explanation of assertion. 40, Bodies fll wit the same acceleration uncer the influence of gravity. 41. The direction of ceatripetal acceleration changes continuously. 42. Because of the horizontal velocity, the bob behaves as if projected horizontally from a cersain height and soit follows a parabolic path. Both statements are true but reason fails to explain the assertion 44, Both assertion and reason are true and reason statement is the comect explanation of the assertion. 39. 45,_Inthis case the height ofthe centre of gravity willbe the largest. 46, ‘Thecentre of mass and centre of gravity coincide only when the value of the acceleration due to gravity is same at all locations where the particles of the system are located, 47. Both the mass as well as radius ofthe earth will increase 48, It isa law of nature and no reason can be given for the conservation of angular momentum. 49. Larger the size of the body, greater is its moment of inert Momentof inertia /= Em r* . Herer, isthe distance of the particle from the axis of rotation and m, is mass of the particle. So m,y;) depends both on the axis of 50, rotation and the distribution of mass. 51. ‘The momentum of the system changes only under the influence of external forces. The velocity of centre of mass remains constant when the extemal foree on the body is zero. Pure rotation can occur about any axis, when each particle of the body describes concentric circular paths, 54, For describing the motion of large sized bodies whole 52, 53, aa You have either reached a page that is unavailable for viewing or reached your viewing limit for this book. aa You have either reached a page that is unavailable for viewing or reached your viewing limit for this book. aa You have either reached a page that is unavailable for viewing or reached your viewing limit for this book. Copyrighted material aa You have either reached a page that is unavailable for viewing or reached your viewing limit for this book. aa You have either reached a page that is unavailable for viewing or reached your viewing limit for this book. aa You have either reached a page that is unavailable for viewing or reached your viewing limit for this book. 4386 Pearson Gulde to Objective Physics a Me T= 2% files \ \ » i Conical Pendulum QE Physical Pendulum 7 T =2n |, mel EAE _, [asia tT -2n* +0 ik oe FY or Lille Torsional ‘righ Torsional Pandulum [Note in physical pendulums is maximum if! Tis minimum ifk=/ Second’s pendulum: Ifthe time period of asimple pendulum is 2s. itis called seconds. Rk & = 84 min, 36 s. for an infinitely long simple pendulum) Oor!=cand where & is radius of the earth IFL=R, the radius of the earth then T= 27 60 [R 2g minorth, SHM under gravity _If'SITM occurs due to restoring force provided by weight or acceleration due to gravity then T= af Fg liquid in a U-tubevertical cylinder/piston. Motion of a ball in concave mirror/ bow! and a floating cylinder as illustrated inFig. 12.9. ‘Some of the examples of this type are motion of @ — a fe ra an (E irpat dees me O TOF VE roll butslips. P= 20 5 sf the bal rol. aa You have either reached a page that is unavailable for viewing or reached your viewing limit for this book. aa You have either reached a page that is unavailable for viewing or reached your viewing limit for this book. aa You have either reached a page that is unavailable for viewing or reached your viewing limit for this book. aa You have either reached a page that is unavailable for viewing or reached your viewing limit for this book. aa You have either reached a page that is unavailable for viewing or reached your viewing limit for this book. 392 Pearson Guideto ive 18, Ina spring pulley system: Fig. 12.25 and Fig. 12.26 (a) (©) I pulley has mass m and MOI then ‘sing conservaticn of ccergy and differentiating ‘ergy to find force and relating F'=— kx we find or 19, SHM ofliquid in U-tube: InFig. 1227 (@) ier] Ifone side of liquid has length as shown in Fig 12.27 (>) then f ron fe 20. SHMin eylinder— piston Ina vertical cylinder It \ T sey Fig. 12.28 (a) Ina horizontal cylinder/piston system having a gas of bulk modulus B or pressure P. Volume V, ome In (isothermal conditions) 21. If pendulum is in a carraige moving vertically up 7 * Veta use + ve sign for upward motion and use — ve sign for dowaword motion, ‘or down with an acceleration a, then T= aa You have either reached a page that is unavailable for viewing or reached your viewing limit for this book. aa You have either reached a page that is unavailable for viewing or reached your viewing limit for this book. aa You have either reached a page that is unavailable for viewing or reached your viewing limit for this book. aa You have either reached a page that is unavailable for viewing or reached your viewing limit for this book. aa You have either reached a page that is unavailable for viewing or reached your viewing limit for this book. aa You have either reached a page that is unavailable for viewing or reached your viewing limit for this book. aa You have either reached a page that is unavailable for viewing or reached your viewing limit for this book. aa You have either reached a page that is unavailable for viewing or reached your viewing limit for this book. aa You have either reached a page that is unavailable for viewing or reached your viewing limit for this book. aa You have either reached a page that is unavailable for viewing or reached your viewing limit for this book. aa You have either reached a page that is unavailable for viewing or reached your viewing limit for this book. aa You have either reached a page that is unavailable for viewing or reached your viewing limit for this book. aa You have either reached a page that is unavailable for viewing or reached your viewing limit for this book. aa You have either reached a page that is unavailable for viewing or reached your viewing limit for this book. aa You have either reached a page that is unavailable for viewing or reached your viewing limit for this book. aa You have either reached a page that is unavailable for viewing or reached your viewing limit for this book. aa You have either reached a page that is unavailable for viewing or reached your viewing limit for this book. aa You have either reached a page that is unavailable for viewing or reached your viewing limit for this book. aa You have either reached a page that is unavailable for viewing or reached your viewing limit for this book. aa You have either reached a page that is unavailable for viewing or reached your viewing limit for this book. aa You have either reached a page that is unavailable for viewing or reached your viewing limit for this book. aa You have either reached a page that is unavailable for viewing or reached your viewing limit for this book. aa You have either reached a page that is unavailable for viewing or reached your viewing limit for this book. aa You have either reached a page that is unavailable for viewing or reached your viewing limit for this book. aa You have either reached a page that is unavailable for viewing or reached your viewing limit for this book. aa You have either reached a page that is unavailable for viewing or reached your viewing limit for this book. aa You have either reached a page that is unavailable for viewing or reached your viewing limit for this book. aa You have either reached a page that is unavailable for viewing or reached your viewing limit for this book. Melde’s Experiment aft ¥ 2 FE sp mabe atop Longitadinal mode Bir £ af where P= number of loops de ‘Transverse mode [EEREEAIDY) t's exporimont = Peoene Pragmins = OY ‘Velocity of a wave on the surface of aliquid is where T is surface tension and p is density of the liquid. at 2 3. Velocity of Torsional waves in a rod is To produce longitudinal waves the medium should possess bulk modulus of elasticity. ‘To produce transverse wave the medium must possess shear modulus of elasticity. ‘y=, sin( ox — kx + @) is the equation of a plane progressive wave in positive direction. dis initial phaseangle orepoch. Normally, we writea simplified equation, y= y,sin(ax ~ x). A more general equation of wave is y= y,e%"-™) i propagation vector or wave number. de Wave velocity is eee and Waves in String 421 called phase velocity: Ita dispersive medium ‘waves travel with group velocity v, given by jd v, <1 tncispesive mediums waves of different wavelengths travel with different velocity. Bets Se at a (slope) Maximum particle velocty = y,00 7. Frequency ofthe wavedoes not vary when a wave passes from one medium to the other. 8. Power (average) transmitted along the string is. 6. Particle velocity = Intensity = 2° P% f2v where 1 = mass/length and p= density of the medium. 9. Interference of waves travelling in the same direction is obtained using vector laws Yo Ya + Voi din CSD and sing Vo) + Nap C080 if Y8in(ex — oo) and Ys =YSin(ex— for 9) interfere, im _ Out Yn) Ie ee ae ‘nin (Ye) ‘Maximum intensity is obtained when phase shift is zero ot path difference is an integral multiple of wavelength. Minimum intensity or destructive interference occurs when phase shift is an odd iogral multiple of wradian or odd integral mutiple of half the wavelength. 11, Reflection from a denser medium causes a phase shift of 180° and reflection trom rarer or lighter ‘medium occurs without change of phase in the string. 12, Standing waves result when two waves having same ampliude and sume frequency travelling in opposite directions superpose. 13, y=2y,sin kv cos ox represents a stationary wave in strings. Note that (2,$in kx) shows amplitude and is a funetion of distance. At certain places. amplitude is maximum antinodes) and at other places amplitude is zero (nodes), Separation between consecutive nodes or antinodes is 2/2. Distance between a node and consecutive antinode is 4/4. aa You have either reached a page that is unavailable for viewing or reached your viewing limit for this book. aa You have either reached a page that is unavailable for viewing or reached your viewing limit for this book. aa You have either reached a page that is unavailable for viewing or reached your viewing limit for this book. aa You have either reached a page that is unavailable for viewing or reached your viewing limit for this book. aa You have either reached a page that is unavailable for viewing or reached your viewing limit for this book. aa You have either reached a page that is unavailable for viewing or reached your viewing limit for this book. aa You have either reached a page that is unavailable for viewing or reached your viewing limit for this book. aa You have either reached a page that is unavailable for viewing or reached your viewing limit for this book. aa You have either reached a page that is unavailable for viewing or reached your viewing limit for this book. aa You have either reached a page that is unavailable for viewing or reached your viewing limit for this book. aa You have either reached a page that is unavailable for viewing or reached your viewing limit for this book. aa You have either reached a page that is unavailable for viewing or reached your viewing limit for this book. 434 Pearson Guide to Objective Physics @ VE, () JE, & @ & E, 80, Fora pulse moving in a heavy striug the junctions of the string behaves as a (a)perfectly rigidend (6) free end (c)pantally igidend ——_() rigid end 81. The intensity of a soundwave in an elastic medium falls by 10% on travelling 2 distance of 1 m. If the initial intensity of the wave is 10(% then on travelling. a distance of 3 m in that medium the intensity will become (ay81% (b) 70% (72.9% Wom 82. A75 em long wire isreduced by 0.5 emthen it produces three beats per second with the fork. The frequency of the fork is (aoa7Hz (4.07 Hz (047.7 (@HT0He 83. When two identical wireson asonometre are kept under samme tension their fundamental frequency is 500 Hz. In order to produce five beats per second the percentage change in the tension of one of the wires will be (0)2% (4% (06% (aR% 84, The minimum distance between the sound and the reflecting surface, in order to hear an echo, must be (a) 0.65 m (b) 1.65 m (©)16.5 m (a) 165 m 85. The total mass of a sonometer wire remains constant, On ineceasing the distance between two bridges to four times, its frequency will become (a)0.25 times (b) 0.5 times (c) 4 times (4) 2 times 86, ‘The lowest pitch out of the following sources is that ofa (a)man. (b) boy, (blion. (@) mosquito. 87, Ifthe frequency of a sound wave is doubled then the velocity of sound will be (ayzero. (by halt, (e)double, (@) unchanged. 88, Ifthe wavelength of'a wave is decreased by 20% then its frequency will become {a)20% less (c)20% more, (b)25% more, (a) 25% less. 89, Ifthe energy density and velocity ofa wave are wand respectively then the energy propagating per second per unit area will be (a)ule (bye (che. (doin 90. In which ofthe Following isthe energy loss maximum? (@) sonometer (b) tuning fork (©) thin tude (d)broader pipe 91. What is produced at a rigis displacement wave? (a) beats. (b) node and antinode. (©) amtinode, (d) node. 92. If the equations of wo sound waves are y, reflecting plane for a 5 sin252 sand y, = 5 sin 28077 respectively. Then the number of beats heard per second will be (2) beats will not be heared, (6 (12 34, 93. Ifthedensity of materials oftwostrings of same length, tension and area of cross-section are 2 kgm”? and 4 kgm" respectively then the ratio of their frequencies will be fal: V2 (b)221 (2 @) V2 31 94, The time taken by a particle to travel between a trough and a crest in a transverse wave is (@)T (b) 3714 (om wna 95, The second overtone of a closed organ pipe P, and the third overtone of an open pipe P, are in unison ‘witha tuning fork. The ratio of the lenath of P, and P, willbe (3:8 (by 1:3 (98:3 (3st 96. Two open pipes, whose lengths are 50 em and 51 em respectively. produce five beat per second when sounded together. The fundamental fiequencies in Hertz will be (@) 65 ané 60 (b) 90 and 95 (©) 95 ané 90 (d) 255 and 250 97. The mass of a4m long string is0.01 kg. It isstretched by a force of 400 N. The velocity of the transverse wave propagating in the string is (a) 100 ms* (b) 200 ms" (©) 300 ms" (€) 400 ms" 98. The length, mass and tension of a string are 1000 cm, 0.01 kdg and 10 Nrespectively, the speed of transverse waves in the string will be aa You have either reached a page that is unavailable for viewing or reached your viewing limit for this book. aa You have either reached a page that is unavailable for viewing or reached your viewing limit for this book. aa You have either reached a page that is unavailable for viewing or reached your viewing limit for this book. 438 Pearson Guide to Objective Physics 150. In strings, the position of antinodes are obtained at (@)A 2034 (b)0,42,A (€)22,4,64 (@)44,34, 504 aS __PASSAGE1 ] Read the following passageand answer the questions given at theend, ‘Your roommates have lost the TV remote control and no amount of searching can find it, Rather than buy @ new one, ‘you build a low cost replacement. You attach one end of a small liver mechanism to the TV channel changing button, ‘You plan to attach the other end of the liver to a 3 m long siring that will run from TV to the couch. When you pul the string tight and pluck your end of the string a wave will travel down the string and trigger the fever, changing the channel. Your design assumes you will disturb the string vertically by 5 mm when you pluck it and that your wave will take only 0.2 s to travel horizontally along the string from your endto te lever. Unfortunately you could not find single string 3 mlong. You could only find two |.5 m long sirings one weighing 90 g and the other weighing 10 g You tie the two pieces to make a 3 m long string and attach one end of the combined string to lever mechanism. You then take the other end in hand and head for the couch. 1. How hard do you have to pull to make it stretch taut? (@I5N (b)75.N (ISN (@)138N 2. How many loops will be seen in each string” (a) thinner wire and three in thicker wire, (b). 3inthinner wire and | in thicker wire, (©) Leach (@)_ 3.each. 3. What is the frequency of the wave? (a) 21.5 Hz (b)20H2 (©) 143 Hz (@) 1 Hz, Lay or 37=225N or T=75N 2.(a), Read the following passage and answer the questions given at theend, A boy of Sth standard is playing with the cloths line. He tunties one end, holds it taut and wiggles the end up end ‘down sinusoidily with frequency 2 Hz and amplitude 0.075 im. The wave speed is 12 ms"!. Atr=0 the end has maximam displacement andis instantaneously a rest. Assume no wave bounces back from the far end to muddle up the pattern, 1. What is wave number? (a) 1.05 me! (b) 132m (©)0.78m* (@)234m* 2. Write a wave function describing the wave. (a) y= 0.075 cos (1.05¢ — 4m) (b)_ = 0.075 cos (1.05 ~ 20) (©) y= 0.075 sin (1.05: — 4) (A) y=0.075 sin (1.05x-21) 3+ Write equations for the displacement as a function of time 3 m of the boy’s end of the clothesline =0.075 cos4z ——_(b) y= (- 0.075 cos 4) (4) none on ke = 5 105m! ESM 2.) 0075 cos 2 3. O y=, cos >| 7-2) 6 v1 a= FF 6m 075 cos (x- 4m) 0.075 cos 4a Read the following passage and answer the questions given attheend. One of the strings of @ Guitar lies along the x-axis when in equilibrium. The end of the string atx = 0 (the bridge of the guitar) is tied down. An incident sinusoidal wave travels the string in the -x direction at 143 ms" with an amplitude 0.75 mm anda frequency of 440 Hz. This wave is reflected from the x = 0 end (fixed end) and the super position of incident and reflected travelling waves forms a standing wave. 1. The equation of the wave representing stationary wave is (a) 0.75 sin 19.3x cos 880m. (b) (0.75 10°) sin 19.3x.c0s 8807 (6) 15% 10? sin 19.3x cos 880m. (d) 150 10°sin 19.3x cos 402. 2. The separation between the two nearest points on the string that donot move at all is, (2) 0.163 m (60325 m (6)0.202 m ()0244m (c)none aa You have either reached a page that is unavailable for viewing or reached your viewing limit for this book. aa You have either reached a page that is unavailable for viewing or reached your viewing limit for this book. aa You have either reached a page that is unavailable for viewing or reached your viewing limit for this book. 1442 Pearson Gulde to Objective Physics when temperature rises by 1°C velocity of sound | increases by 0.61 ms“', | Intensity 1 =27°P¥ fy Rv Fe PERS Ry v 2B” 2pv | | 1 Antensity fe 4 (for an isotropic source) | Jee + (for cylindrical source) where r is the distance between the source and observer. Effectof Pressure Velocity of sound is not affected by pressure. 1 Effect of density vo 7 where pis density. With increase in humidity the density of air decreases and, hence, speed of sound increases. Appearance of sound in human air is characterised by three parameters — pitch, loudness and quality. Pitch is related to frequency. Higher the pitch sweeter is the sound. Children and Ladies speak at higher pitch as compared to men, therfore, their sound appears sweeter. Higher the frequency higher is the pitch Loudness is correlated with sound level. Human ear can hear a minimum intensity 10? Wim? whispering 10 dB normal talk > 6043 L sound level in dB SL = 10 log,, (4) Even at 80 dB (heard continuously for sometime) headache begins, A(130 (2% ron sete Ifwind of velocity v, blows in the direction of sound then change v to v + v, or vv, depending upon ‘wind is blowing in same or opposite direction, If ‘wind isin the direction of sound then. eS ny 24, Ifthe source or listener move with a speed greater than the speed of sound then Doppler effect cannot be applied. tos = (vay, aa You have either reached a page that is unavailable for viewing or reached your viewing limit for this book. aa You have either reached a page that is unavailable for viewing or reached your viewing limit for this book. aa You have either reached a page that is unavailable for viewing or reached your viewing limit for this book. aa You have either reached a page that is unavailable for viewing or reached your viewing limit for this book. aa You have either reached a page that is unavailable for viewing or reached your viewing limit for this book. aa You have either reached a page that is unavailable for viewing or reached your viewing limit for this book. aa You have either reached a page that is unavailable for viewing or reached your viewing limit for this book. 454 800+2400 2 A driver of a car approaching a vertical wall notices that the frequency of his car horn has changed from 440 to 480 Hz when it gets reflected from the wall. Find the speed of the car if the speed of the sound is 330ms" ith fea = 1600 He, 35. (@) 16.3 ms! (©) 143 ms (b)153ms* (@) none of these (©) Let the velocity of car be 36. Two sources of sound 5, and S, vibrate at the same frequency and are in phase. The intensity of sound detected at a point Pis J, if @= 45°, what will be the intensity of sound reaching P if one of the sources is switched off. What will be the intensity if @= 60° Fig. 14.19 fe te ty 2, be @OTG OF ar be he oe O34 Owns (©) Since the waves reach in phase J, = 47 Irae & 0, * Yea) * 2g) = 4 Yin = AE 1, ~ 1 tris independent of 37. An electric train in Japan runs with a speed 1.3 Mach. I is approaching a station and blows a whistle of frequency 800 Hz. The frequency of the whistle heard by a stationary observer on the platform is (2) 800Hz (b) 1600 Hz. (100 Hz (@ insufficient data. (@) Since the velocity of source > veloc ‘sound, Doppler effect is inapplicable. A person Pis 600 m away from the station when train is approaching station with 72 kawh, it blows a whistle of frequency 800 Hz when 800 m away from the station. Find the frequency heard by the person. Speed of sound =340 ms? 38. 600m Fig. 14.20 (a) (a) 800 Hz (c) 829.5 Hz, (b) 839.5 Hz (@) 843.5 Hz he —v,6080 0 = Sgq cig * 800 = 839.5 Hz Fig. 14.20 (b) | 49. Phenomenon of heats can take place @ for longitudinal waves only. for transverse waves only. for both longitudinal and transverse. o © | @)_ for sound waves only. © 40, Inthe absence of teecher a class of 50 students make a noise level of 50 dB. $0 more students enter the class. ‘Assuming each student on an average produces same intensity of sound then the noise level increases by (a)504dB (6) 25dB (sssa3 (348 aa You have either reached a page that is unavailable for viewing or reached your viewing limit for this book. aa You have either reached a page that is unavailable for viewing or reached your viewing limit for this book. aa You have either reached a page that is unavailable for viewing or reached your viewing limit for this book. aa You have either reached a page that is unavailable for viewing or reached your viewing limit for this book. aa You have either reached a page that is unavailable for viewing or reached your viewing limit for this book. aa You have either reached a page that is unavailable for viewing or reached your viewing limit for this book. aa You have either reached a page that is unavailable for viewing or reached your viewing limit for this book. aa You have either reached a page that is unavailable for viewing or reached your viewing limit for this book. aa You have either reached a page that is unavailable for viewing or reached your viewing limit for this book. aa You have either reached a page that is unavailable for viewing or reached your viewing limit for this book. aa You have either reached a page that is unavailable for viewing or reached your viewing limit for this book. aa You have either reached a page that is unavailable for viewing or reached your viewing limit for this book. aa You have either reached a page that is unavailable for viewing or reached your viewing limit for this book. aa You have either reached a page that is unavailable for viewing or reached your viewing limit for this book. aa You have either reached a page that is unavailable for viewing or reached your viewing limit for this book. 3 4. 3 Three sources of sound of equal intensity having frequencies 500, 501 and 502 Hz are sounded simultaneously. The number of beats produced is (ao 1 2 @3 cylindrical tube open at both ends has a fundamental frequency 210 Hz in ait. It is vertically half dipped in water. The new fundamental frequency is (@ 10sHz (0)210Hz (9315Hz (0)420Hz ‘An empty vessel is filled with water, its frequency (a) increases. (b) decreases. (c) remains unchanged. (4) insufficient datato reply. ‘Two sound waves of intensity /, and4 , interfere, The 3a intensity at a point where @= — is 2 @B1, (b) V5 J, @v7h @ 3%, The velocity of sound in Hydrogen : Velocity of sound. in Nitrogen is @l 7 ) 7:1 (©) via #1 @) 28:1 ‘Two tuning forks of nearly same frequency are producing 3 beats. The velocity of these beats will be. a Naswarssasr (b) equal to sound waves. (©) 3 times the sound waves. (@) none of these, ‘Two sound waves y,=4sin 400 mand y,=2 sin 404 mt when superpose produce Fas 9 (b)A beats with 7 Fos (@) beats with I, fx (©)2beats with 7 =9 (d)2 beats with 7 =3 8. Two organ pipes have frequencies (fundamental) 300 Hz and 305 Hz, their lengths are in the ratio (@)60:59 (©)60:61 9. A small spread Ak in wave number corresponds to a spread in wavelength AA given by @ zat (dy 20k 2k 2m ak OD oF 10. Fathometer is used to measure (@) height of an aeroplane (b) depth of ocean. (©) frequency of ultrasonics. (@)_ velocity of supersonics. 11. The velocity of sound in hydrogen is 1400 m/s at 0° C. Find the velocity of sound in a gasseous mixture containing 2 parts hydrogen and 1 part Nitrogen by volume at 10°C (617.3 m/s (b)5173 mis (0)9173 mis (@)8173 ms 12, Find the coeff, of refraction of a sound wave at an interface of air and glass. Young’s modulus of glass is, 6.9 « 10 Nm®, the density of glass is 2.6 « 10" kg/m’ and temp. of surroundings is 20°C. Also find the minimum angle at which total intemal reflection will (@) 14.96,3° si’ (€)0.1496, 1° 51" (b)1.496,3° si” (4)0.067, 3° si’ aa You have either reached a page that is unavailable for viewing or reached your viewing limit for this book. aa You have either reached a page that is unavailable for viewing or reached your viewing limit for this book. aa You have either reached a page that is unavailable for viewing or reached your viewing limit for this book. 474 Pearson Guide to Objective Physics (0345 (b)035 (02s (0.19 (©) none of these 58. A particle is subjected to two SHMS one along x-axis and other making an angle of 60° with x-axis. The two motions are given by x =x, sin ao, s =5, sin ax. The r r OF OF E r OF Os (e) none of these amplitude ofthe resultant motion i 61. x=, sin of and y=, sin 2ov act ow a body along:x () (+d + Jing) and y axis. The resultant fisur (b) xy +5; +455, © aw C.D Yah +55 + VB x55 @ &) '59. A particle is subjected to 1Wo SHMS.x,~ 3 sin 314 xt and x, = 4 cos 314 mi. Find the amplitude and phase shift ofthe resultant @5,37 (b)5, 53° ©5,90° (5, 60° ()5,30° 60. Find the time when the displacement of a particle © @® exerting SHM is half the amplitude _ 1 2 © @ 4 © 5 ( & 0) 2 © 8 9 © 11.) 2 @ B © * © 5 ©) a) @) 8 6) a a (6) a ©, 2 © 2a © 23 2 (a) 2 © ma ©) 23. (a) 3. @ 2 ©) a @) uO) 5 © 36) a7. (a) @ 32 ©) a) a ©) 2 © a © “a @ 46.) 47. (a) 4 a. 50. () 3. @ 3. ©) a O) 3.) %. @) 57.) 8.) 6.) a © Read the following passage and answer the questionsgiven at theend. ‘Your friend in the hostel room sounds a stopped organ pipe ‘ear a guiter.Itis found that one ofthe strings starts vibrating With long amplitude. The length of the string is 80% of the length of the stopped pipe. The string and stopped pipe both vibrate attheir fundamental frequency. The temperature of the day is 35° C, 5 m of the string weighs 10 a. 1. The tension in the string is nearly (a)250N (b)350N ()160N (d) none of these 2. The velocity of sound in organ pipe (2332s! (6)353 ms? (€)367ms" (@) none of these 3. The velocity of wave in string is nearly (a) 141 ms" ()332ms" (6) 163 ms? (4) none of these Read the following passage and answer the questions given at the end. In the physics laboratory a student was performing an ‘experiment on sonometer io find the frequency of AC mai He kept the tuning fork as shown in Fig. The electromagnet aa You have either reached a page that is unavailable for viewing or reached your viewing limit for this book. aa You have either reached a page that is unavailable for viewing or reached your viewing limit for this book. aa You have either reached a page that is unavailable for viewing or reached your viewing limit for this book. 478 Pearson Guide to Objective Physics {@) conic section, (b) spherical. (cylindrical (4) straight line. 24, In the equation of plane progressive wave y= y, sin (cor + kx), ts wrong statement is @ ©) © @ 25, An equation of plane progressive wave ¥ 2x =P is its wave length. T_ ib velocity of wave, Fp is fequency of wave. k — is maximum velocity of particle. Pr ty of part (t= 3x). Its direction and phase angle are x (a)+X—axisand x (b)-Xaxis, > x ()—Xaxis and (d)+ Xaxis, > 26. For cylindrical progressive weve amplitude depends upon position of particle as @x (x? @x? xe? a 27. In the equation of plane progressive wave y~ sin 2n 3 (-¥]}. If maximum velocity of particle and velocity of wave are equal, Now = willbe 6 @2 ()32 jin (d)none of these 28. Considk of ey: Elk He . Consider equation of wavey=Ssin 5 [1-5 | Here displacements inem and timein see. At4.em position of the perticle the isplacement is 2.5 cm, the time interval is (@)05s (iss (25s @4s Now “A™ represents @ ©) © @ velocity of wave. inverse of velocity of wave. wave length. inverse of wavelength. 30. During the wave motion, the pressure of wave is, (a) maximum at rarefaction, (b) minimum atcompression (©) constant throughou, (4) none of these. 31. A wave is incident on fixed target as y ~Ssin 2a (4 }. the equation of reflected wave , ) y-+sinzn (+3) © y-+ssn2e("-4) © y-rsemzx (+3) \ 32. The equation of wave y = > sin (-3) where displacements isin om and time insec. The phase angle between two particles at displacementas 12.cm is wer () 6 radian os (3 radian 33. After superposition a wave equation is y 4 sin? or, ‘The amplitude and frequency of periodic wave before superposition is @ 4 @ = OF a ote OG 2m 34. An equation of complex wave after superposition is a . =a sin? m. cos = . The no of equations of waves before superpositions are (a2 (3 (os (@ infinite 38, The equation of stationary wave =~ 10008 2, sin yan teos F. sin S*. Now the equations of waves before superposition are aa You have either reached a page that is unavailable for viewing or reached your viewing limit for this book. aa You have either reached a page that is unavailable for viewing or reached your viewing limit for this book. aa You have either reached a page that is unavailable for viewing or reached your viewing limit for this book. ‘482 Pearson Guide to Objective Physics iL _V+VIS_OVIS_8& fV-ViAT IAS 48. ©) 20 v ori ee] aa You have either reached a page that is unavailable for viewing or reached your viewing limit for this book. aa You have either reached a page that is unavailable for viewing or reached your viewing limit for this book. aa You have either reached a page that is unavailable for viewing or reached your viewing limit for this book. aa You have either reached a page that is unavailable for viewing or reached your viewing limit for this book. aa You have either reached a page that is unavailable for viewing or reached your viewing limit for this book. aa You have either reached a page that is unavailable for viewing or reached your viewing limit for this book. aa You have either reached a page that is unavailable for viewing or reached your viewing limit for this book. aa You have either reached a page that is unavailable for viewing or reached your viewing limit for this book. aa You have either reached a page that is unavailable for viewing or reached your viewing limit for this book. 492 Pearson Guide to Objective Physics Specific hest of saturated vapours is negative. In adiabatic process specific heat is zero and in isothermal process itis infinite, Considering that increase in length of the pendulum increases time period, and therefore, the clock becomes fast. => Astimeinterval increases, it will ake less oscillation. and hence itwill become slow. 9. Considering that boiling pointand freezing point of | ‘water are standard. = Melting point and boiling point vary with pressure and impurities, SOLVED PROBLEMS 1. A vessel contains 1 mole of ©, (molar mass 32 g) at a temperature 7, The pressure is P, An identical vessel coniaining | mole of He gas (molar mass 4) at a temperature 27 has a pressure P we @ os (©2P (asp (©) PV=nRT P,,, = 2P as temperature of Heis doubled. 2. TO calorie ofheatis required toraise the temperature of a diatomic gas at constant pressure from 30 to 35°C. The amount of heat required (in calorie) 0 raise the temperature of the same gas through the same range (30 to 35°C) at constant volume is (30 (60 (©)50 (70 & OG (40), _ nC,AT Cy yy (40), ~ nC,aT C, (40), (49), = 47 =50eal 3. A vessel contains | mole of O, and 1 mole of He. The value of 7 for the mixture is (aia (b) 1.50 (1.53 (d) none of these (B) Co. =2R 10, Confusing the values of Rand k = R = 8314JK' mot! or R = 2eal ees) ~ ‘Avogadro number 138% 102 JK" 11. Confusing SI and CGS units. = Itis advised todo questions on calorimetry inCGS) as calculations become simple. Ifthe final answer is in Joules, then convert calories into Joules by multiplying the result by 4.2. 4. Steam at 100°C is passed into a calorimeter of water equivalent 10 g containing 94 cc of H,O and 10 g of ice at 0° C, The temperature of the calorimeter and contents rise by 5°C. The amount of steam passed is @ls ©) 28 3s @4g (b) Let me, he the amount of steam in grams Then m,L+m,C, (100-5) =W,,(5~0) + 10 « 80 +(104) (5-0) 1m, (540) +m, (95) = 10(5) +800+420 635 m,= 1270 orm, = 2g, 5. 10 g of ice is added in 40 g of water at 15°C. The temperature of the mixture is ~@o (se (12 @sc (a) Heat required to mel the ice = mi =10%80 £00 cal. Maximum heat which can be supplied by hot water mCAT=40 1 * 15 = 600 cal ‘As heat supplied < heat required to melt ice, temperature of the mixture will beO°C (as whole of the ice will not melt). 6 A.and B are made up of an isotropic medium. Both A and B are of equal volume. Body B has cavity as shown, in Fig 15.2 (b). Which of the following statements is true? aa You have either reached a page that is unavailable for viewing or reached your viewing limit for this book. aa You have either reached a page that is unavailable for viewing or reached your viewing limit for this book. aa You have either reached a page that is unavailable for viewing or reached your viewing limit for this book. aa You have either reached a page that is unavailable for viewing or reached your viewing limit for this book. aa You have either reached a page that is unavailable for viewing or reached your viewing limit for this book. aa You have either reached a page that is unavailable for viewing or reached your viewing limit for this book. aa You have either reached a page that is unavailable for viewing or reached your viewing limit for this book. aa You have either reached a page that is unavailable for viewing or reached your viewing limit for this book. aa You have either reached a page that is unavailable for viewing or reached your viewing limit for this book. aa You have either reached a page that is unavailable for viewing or reached your viewing limit for this book. aa You have either reached a page that is unavailable for viewing or reached your viewing limit for this book. aa You have either reached a page that is unavailable for viewing or reached your viewing limit for this book. aa You have either reached a page that is unavailable for viewing or reached your viewing limit for this book. aa You have either reached a page that is unavailable for viewing or reached your viewing limit for this book. aa You have either reached a page that is unavailable for viewing or reached your viewing limit for this book. aa You have either reached a page that is unavailable for viewing or reached your viewing limit for this book. aa You have either reached a page that is unavailable for viewing or reached your viewing limit for this book. rS0n Gulde to Objective Physics 9. Theaverage momentum ofa molecule ina sample of an ideal gas depends on (a) temperature (b) number of moles (©)volume (€) none of these 10. Which of the following quantities is zero onan average for the molecules of an ideal gas in equilibrium? (a) kinetic energy (b) momentum (©) density (d) speed 1. Consider a mixture of oxygen and hydrogen kept at room termperature. As compared to a hydrogen molecule, anoxygen molecule hits the wall (a) with greater average speed (b) with smaller average speed (©) with greateraverage kinetic energy (with smaller averagekinetic energy 2. Keeping the number of moles, volume and temperature the same, which of the following are the same for all ideal gases? (a) rmsspeed of molecules (b) density (©) pressure (@) average magnitude of momentum 3. Consider acollision between an oxygen molecule anda hydrogen molecule in a mixture of oxygen and hydrogen kept at room temperature. Which of the following are possible? (a) The kinetic energy of both the molecules increase (b) The kinteic energy of both the molecules decrease (©) The kinetic energy of the oxygen moleule increases and that of the hydrogen molecule decreases. (@)_ The kinetic energy of the hydrogen molecule increases and that of the oxygen molecule decreases. 24, Which of the following. quantities is the same for all ideal gases at the same temperature? (a) thekinetic energy of 1 molecule (b)_ the kinetic energy of 1 g (©) thenumber ofthe molecules in 1 mole (@ thenumber of moleculesin 1 g ad the following passage and answer the questions given ttheend. Vhile running, a 70 kg student generates thermal energy at a ate of 1200W. To maintain a constant body temperature of T C, this energy must be removed by perspiration or by ther mechanisms. If these mechanisms fail and heat cannot ‘ow out of the student's body, irreversible body damage ccurs. Protein structures in the body are irreversibly damaged if body temperature rises to 44°C or above. Specific hheat of human body is 3480 J kg" k and heat of vaporisation of water is 242 % 10° kg, 1, How much water has he to consume in 30 min running if the heat is eliminated through perspiration? (a)0.6L (b)0.7L, ()08L @osL (e) none of these 2. The specific heat capacity of human body is less than that of water because (a) continuous perspiration lowers the specific heat capacity (b)_ the presence of protein, fats and minerals which have lower specific heat cause the specific heat ‘of the body to be lower than that of water (©) specific heat of protein, fats is high and they consume heat lowering the specific heat of the body (@)__ the food stuff which we eat requires to be burnt ‘and takes up heat lowering the specific heat of the body 3. For what maximum time can a student run before imeversible damage occurs? (a) 15 min (b) 18.5 min (23 min (@ 27.4 min 1.(@) Pt= mb, 1200x30x60 or mT page = 09E 2.(b) 3.(¢)Px1=MCAT 70x3480x7 5 ort pg 7421s (see) orabout 23 min, Read the following passage and answer the questions given atthe end. ‘A man has an antique pendulum clock of 1832 which bears the signature of the purchaser. He does not want to replace it inthe fond memory of his great grand parents. Itticks offone second in each side to side swing. Itkeeps correct time at 20" CC. The pendulum shaft is made of steel and its mass can be ignored as compared to mass of the bob. Linear expansion coefficient of steel is 1.2 « 10°°C* 1, What is the fractional change in length if the shaft is cooled to 10° C? (a) .01% (b) 1.2 10°% (©)12%10°% (€)12* 104% 2. How many seconds the clock will gain or loosein aday at10°C? aa You have either reached a page that is unavailable for viewing or reached your viewing limit for this book. aa You have either reached a page that is unavailable for viewing or reached your viewing limit for this book. aa You have either reached a page that is unavailable for viewing or reached your viewing limit for this book. aa You have either reached a page that is unavailable for viewing or reached your viewing limit for this book. aa You have either reached a page that is unavailable for viewing or reached your viewing limit for this book. 516 Pearson Guide to Objective Physics the performance of an equivalent amoutof work. (Kelvin Planck statement) (b) Heat cannot flow itself from a colder to a hotter body. (©) It is impossible to have a process in which the entropy of an isolated system is decreased. Adiabatic —> Thermally insulating Diathermic — Thermally conducting Heat Engine A heat engine takes a heat Q, from the furnace and rejects Q to the heat sink and does a work = 2-9, ‘Thus efficiency of an engine 1) Col body or heatsink Fumace Tr Carnatengine « do Entropy d= = or $,-S,= [= Note that Tis not differentiable. Entropy is ameasure of randomness or disorder in a system. (Clausius inequality g2 <0 or ase [2 ordQ= TaS > dU + pd¥ Relation between entropy and statistical weight © (thermodynamic property) S=KlogQ where k is Boltzmann's constant. Amount of heat required to form a unit area of the liquid surface layer during the isothermal increase of its surface H= +e where ois surface tension, Carnot Engine _ The french scientist (auto engineer) NL ‘Sadi Camot in 1824 suggested an idealised engine called Cemnot engine. It has a cylinder piston system. The walls and the piston are completely adiabatic (insuleting) and the base isdiathermic (thermally conducting). Itcontains an ideal ‘gas, Itundergoes isothermal expansion, adiabatic expansion, isothermal compression and adiabatic compression to complete the cycle. PY and ST plots for a Carnot cycle are shown in Fig 16.9. Carot's engine isa reversibleengine. Camot’s theorem All reversible engines operating between the same two temperatures have equal efficiency and no engine operating between the same two temperatures can have an efficiency geeater than this. 5 & Cara a ‘According to Camot’s theorem, maximum efficiency q a wei g Ong Since T, cannot be zero (as 0 K cannot be obtained), therefore, efficiency cannot be 1. Refrigerator or heat pump A heat engine takes heat froma hot body, converts part of it into work and rejects to cold body. The reverse operation is done by a refrigerator (or heat pump). It takes an amount Q, of heat from cold body. an amount of work Wis done on itby the surrounding and 2 total heat Q, = Q, + Wis supplied to hot body as illustrated in Fig. 16.10. igh temp, body Low temp. body GREEN ®etrigrator based on carnot cycle - reall) This leads to another statement of second law: aa You have either reached a page that is unavailable for viewing or reached your viewing limit for this book. aa You have either reached a page that is unavailable for viewing or reached your viewing limit for this book. aa You have either reached a page that is unavailable for viewing or reached your viewing limit for this book. ‘520 Pearson Guide to Objective Physics @t ) + - NY a 3 - z 3 ‘OT Temperature (7) OF Temperature (7) Os @ + F i = 5 Fa Z Wh eapeae OF Tepes Fig. 16.16 (d) For 7-0, that is, at low temperature, molar specific heat ~ 7° and at high temperature it becomes constant = 3R. 3. 1g of H,0 changes from liquid to vapour phase at constant pressure at | atm. The volume increases from 1 ce to 1671 cc. Theheatof vaporisation atthis pressure 540 call, The increase in intemal energy of water is, (=)20991 (b) 3000 (9925 (2122) BST (2) = Peat) = 1.01 « 10°(1671 - 1) « 10° 1675 AQ =au+aW or AU =AQ-aW = mL - 167 = 540 «4.2 167= 2099 J 4, Agasmixtare consists of2 moles of oxygen and moles of Arat temperature 7. Neglectingall vibrational modes, the total internal energy of the system is (arr (15 RT (ooRT (uRT {IT 1999} 5. Abrepumetiapesne 75x27 Canny bursts, What isthe final temperature (y= 1.5)? @27rc {b)-27°C woe (a-73°C BAM (e) Wh =P A 300 Gaz)” 6 A sound wave patsing through ar at NTP produces « pressure of 0.001 dyne/cm: during acompression. The corresponding change in temperature (given y= 1.5 and assume gas to be ideal) is (0) 897 « 104K (b)8.97* 10° K ()897 «10° K (a) none of these (c) 77 PYY= constant. Differentiating, Wear Ps Pp Pear or r 00K =-73°C a yP (3 }{ 273 on!) 13.) | 76x13.5x981 = 8.97 <10*K When a system is taken from state Ito 2along the path a2 it absorbs 50 cal of heat and work done is 20 cal, ‘Along the path 162, = 36 cal. What is the work done or along 152? a » 2 1 - a Fig. 1617 (a) 56 cal (0) 66 cal (6) 16 cal (4)6 cal (@) dQ= du + dW 0 Q= (uu) + W W = Qy.~( t=) 0F Q,.~ We u,— us, 6 30= 6 cal or uj-u, = 50-20=30cal 8 I gmole of anideal gasat STP is subjected to areversible adiabatic expansion to double its volume. Find the change in internal energy ( 7= 1.4), (116985 (byT69.55 (©) 136953 (36955 ESM (6) Use 7,17" = 7,7" 23 = = GF 2207 0 r, @yr 7207K Change in internal energy aa You have either reached a page that is unavailable for viewing or reached your viewing limit for this book. aa You have either reached a page that is unavailable for viewing or reached your viewing limit for this book. aa You have either reached a page that is unavailable for viewing or reached your viewing limit for this book. 524 Pearson Guide to Objective Physics = [108 5x 10° +5 * 10] (0.1) = 553 28, 3 moles of an ideal monoaiomic gas perform a cycle showa in Fig. 16.19. The gas temperatures 7,= 400 K, 7, = 800K, T= 2400K, T,, = 1200 K. Find the work done by the gas. [Olympiad 1996] Fig. 16:19 Wee 3RIT Ty) Wy = Wo because the processes are isochoric Wy = RET) ‘Total work done Wy Wop BRT + Te Ty-T) = 3R(400+2400-800- 1200) = 2400 8=2010 29, One mole of Argon is heated using P/*° = const. Find the amount of heat obtained by the process when the temperature changes by AT=-26 K. (Olympiad 1998] Let p be the number of moles here p= 1 then © = 30. An ideal gas with adiabatic exponent 7, is expanded according to the law Pav where ais a constant. The initial volume of the gis is V,, As a result volume increases 7 times. Find the increment in internal energy and work done. Let kbe number of moles P = aVorPV'=a ‘The process is polytropic with index n = Yous = Vo Veas= 1%, and Praag = OMG Pos = 2M FR MU = =] Tart Foi Pat ~ Pats Min Work done, Posts Faas Vana _ Fe [1-1] nt a 3. Onemole of oxygen is expanded fromavolume {/105 Jat a constant temperature 7=280K. Find the change in internal energy and the amount of heat absorbed. ‘Assume te gas wo be a van der Waals Bas. 5: ( OP av-f (F) w From second aw of thermodynamics ou as. ar av > Thar}, PT lar} -? P OP) RT wu) a o Tor) =r lor, 1d wegen From the firstlaw of thermodynamics 32. Caleulate the heat absorbed by the system in going ‘through the process shown in Fig, 16.20. (314d (b)3.4S (31410 (4) none ay 0 (Wehumsy 400 Fig. 16.20 (a) Heat absorbed = m7 mP)V) =3.14(100 * 10°)(100 10%) 3145 aa You have either reached a page that is unavailable for viewing or reached your viewing limit for this book. aa You have either reached a page that is unavailable for viewing or reached your viewing limit for this book. aa You have either reached a page that is unavailable for viewing or reached your viewing limit for this book. aa You have either reached a page that is unavailable for viewing or reached your viewing limit for this book. aa You have either reached a page that is unavailable for viewing or reached your viewing limit for this book. aa You have either reached a page that is unavailable for viewing or reached your viewing limit for this book. aa You have either reached a page that is unavailable for viewing or reached your viewing limit for this book. aa You have either reached a page that is unavailable for viewing or reached your viewing limit for this book. aa You have either reached a page that is unavailable for viewing or reached your viewing limit for this book. aa You have either reached a page that is unavailable for viewing or reached your viewing limit for this book. 122, Two systems are in thermal equilibrium. The quantity ‘which is common for them is (@heat (@)momentum (© specific heat (@) temperature 123, The amount of heat necessary to raise the temperature of 0.2 mol of N, at constant pressure fiom 37° C to 337°C will be (@) 764 Joule (b) 1764 erg, (© 1764 calorie (@) 1764 Joule 124, Air is filled in a motor car tube at 27° C temperature ‘and 2 atmosphere pressure. Ifthe tube suddenly bursts ‘then the final temperature will be [given (1/2)°"=0.82] (0) 642K (b) 563K (300K, (246K 125, A reversible engine takes heat ftom a reservoir at 527" Cand gives out to perform useful mechanical work at the rate of 750 watt. The efficiency of the engine is 10% (b) 50% ©30% @10% 126, The temperature ys entropy diggram is shown in Fig 16.34. ts PV equivalent diagram is PA @ \* +y Pt ay © @ oy oy Fig, 10:34 127, Pure water, cooled to ~15° C, is kept in an insulated flask. Some ice is dropped into the flask. The fraction of water frozen into ice is (specific heat of ice = 0.5 ealg-1°C"') (@ 629 (b)3/35 (625 (OE 128, A block ofice of mass 50 kgis pushedoat on a horizontal plane ith a velocity of S mis. Dur to friction itcomes to Thermodynamics 935 restafter covering. distance of 25 m. How much ice will melt? (@) 100g (b) 1008, ©) 1.868 oss, 129, The height of a water spring is 50m. The difference of ‘temperature at the top and bottom of the spring will be (@o.1i7ec () LIC (04377 C @iurc 130. Theradiator ofa car contains 20 litre water. Ifthe motor supplies 2 * 10° calorie heat to it, then rise in its temperature will be (@) 100°C (b) 100°C (20°C (10°C Read the following passage and answer the questions given, attheend, The maximum power that can be extracted by a wind turbine froman air stream is approximately P =k’? where dis the blade diameter, vis the wind speed and constant k~ 0.5 17— m*. The cylinder shown in Fig. 16.35 has length L = v1, diameter d and density p. The Mod — 5B wind turbine at ‘Kahaku on the Hawaian island of Oahu has blade diameter of | 97 m slightly longer than a football field, andsits atop a SR m tower Itcan produce 3.2 MW ofelectric power, Assume 25% 1} ee eee! Fig. 16.35 1, What wind speed is required to produce amount of power mentioed in the passage? (@) 10 ms (b) 12 ms" © 4 ns! (@) 16 ms" 2. Commercial wind turbines are commonly located in or down wins of mountain passes because (@) wind speed is very high (b)__ wind is directional (©) wind isabundantly available (@) wind blows the whole year 1.()3.2*10°=.5 “ont ~e v=162ms" 20) aa You have either reached a page that is unavailable for viewing or reached your viewing limit for this book. aa You have either reached a page that is unavailable for viewing or reached your viewing limit for this book. aa You have either reached a page that is unavailable for viewing or reached your viewing limit for this book. Heat Transfer Processes 539 Growth oficein apond X= constant oT So Ingen-Hous2’s experiment Ingen Housz showed that if @ number of identical rods of different metals are coated with g rea wax and one of their ends is put in boiling water, then in gp = 2 = ==x— steady state, the square of length of the bar over which wax a phy melts is directly proportional to the thermal conductivity of iis the metal. Thatis, or eng t X= constant Z Thermal resistances in series oo Growth ofies ina The ratio of times for thickness Oto y: y to 2y: 2y to Byrrdsds5 Ina shell of radius rand, ao Ann 7 KGa) 2-4) [See Fig. 175] we eat Ka Ky ris Thickness ofthe shell (rn) = 2.89% 10° m-K, bait, = AgaT if temperature orwavelength ofa body i to be determined and that of the other is ‘known; for example, for one star 2, and 7, isknown and 4, oF T, is to be determined. 13, Planck’s Law is based on quantum nature of radiations. It assumes that discrete energy packets called photons are emitted or absorbed, E=hv=h> where c > speed of light 1 14, Intensity Je Sr fora point source and amplitude 1 Ase = fora cylindrical source CAUTION |. When you forget o take total length in case of series and total area in case of parallel while finding effective thermal conductivity and Heat Transfer Process: Fig. 17.13 (0) Inparallel ‘Confusing how to find temperature of the junction (or at any other point) like the one shown in Fig 174, = Apply Kirchhoff"s law, that is, = j, + i, and so on 3. Considering freezing of lake asa convection process. => Itisa conduction process and time taken to freeze a depth y is given by ee K6 4. Considering that conduction does not occur in liquids or gases = Itdoes occur but it is quite small as compared to solids 5. Considering any black colour object asa black body. => Black body is one which is capable of emitting or absorbing all possible radiations. Thuseven the sun isa black body. aa You have either reached a page that is unavailable for viewing or reached your viewing limit for this book. aa You have either reached a page that is unavailable for viewing or reached your viewing limit for this book. aa You have either reached a page that is unavailable for viewing or reached your viewing limit for this book. 20, Find the heat radiated per second by a body of surface area 12 cm® kept in thermal equilibrium in a room at temperature 20° C. The emissivity of the surface is 0 and o= 6 * 10+ WK“. (a) 423 (0.423 (©) 0.0425 @a2t () P= ctor" = 08 x 12 « 104% 6% 104 = (293)! 42 J Heat Transfer Processes 547 21, Two copper spheres, one of large size and the other small, are heated to the same temperature. Which will cool frst? (a)bigger ©) smaller (© both in equal time (@) insufficient data to reply (b)Smallersphere (cooling « —-) 22, Thetop of aninsulated cylindrical containers covered by a disc having emissivity 0.6, conductivity 0.167 WK" nv! and thickness 1 em, The temperature is maintained by circulating oil. (@) Find the radiation loss to the surroundings in Jmm~ 2¢"if temperature of the upper suriace ofthe disc is 127° C and temperature of the surroundings is 2PC (©) Also find the temperature of the circulating oil ‘Neglect the heat loss due to convection, a 3 *lo*wm?K* (2) The rate of heat loss per unit area due to Given: radistion=eo(7*~ 7;')=06 ((400)*-(300)*] a x10* = 595 Jars (b) Suppose temperature of oil is 8. The rate of heat flow through conduction = rate of heat loss due to radiation. 0.167xA (8-127) 10 =595A, 0=162.3°C 23, A point source of heat of power P isplaced atthe centre of a spherical shell of mean radius R. The material of the shell has thermal conductivity K. Calculate the thickness ofthe shell if temperature difference between the outer and inner surface of the shell in steady state is. BEM Considera concentric spherical shell of radius and thickness dx as shown in Fig. 17.20. The radial rate of flow ne or [s is s&s fiw do Kénab(9-6.) oF a ba In steady state no heat is absorbed, «HH =P 4-8 and a 4nRkT an Rer P = haay rb-a)= 24, A solid copper sphere (density p and specific heat C) ‘of radius rat an intial temperature 200 K is suspended inside a chamber whose walls are at almost 0 K. What is the time required for temperature of the sphere to drop to 100K? or 1991] According to Stefun’s law, P= edoT* dQ _ -medT _ a. t, where AG change in temperature, IL isclear from the equation that the gain orlossin time is, independent of time period Ar and depends on the time interval r. Also when temperature rises, the time per increases, therefore the clock loses time. ‘Assertion is true but reasonis false because liguid starts boiling when its vapour pressures equal to atmospheric pressure, So to boil the water without heating wehave to decrease the external pressure. Expansion of solids does not depend on its mass. ‘Work done during isothermal expansion is more than the work done during adiabatic expansion between the same values of initial and final volumes. Heat at constant pressure (C,) is supplied for doing, workin expansion and rise of temperature by 1° C, while heat at constant volume is supplied only for rise of temperature. Therefore C, is greaterthan C,.. ‘When gas is heated at constant volume (C,_), heat sup- plied to the gas is spent to increase the internal energy of the gas. Bu! same ammount of heat given to the gas at constant pressure (C,.) is spent in doing external work and to inerease the intemal energy of the gas. For the same valves of initial and final volumes, the ‘work done in adiabatic change is more than isothermal change. To produce equal changes in the volume of = a8, more pressure difference is required in the adia- boatic change. Specific heat is zero during an adiabatic change and ‘becomes infinite during an isothermal change. Also spe- cific heatat constant pressure C, is elways greater than ‘specific heat at constant volume C, end they are related 28 C,— C, = (Mayer's relation). Thus assertion and reason are correct. But these aretwo independent state- ments. Entropy is a measure of the disorder or randomness of 33. 34. 35. 36. 37. 38. 39. 40. ‘Assertion Reasoning Type Questions §75 the system, Greater is the randomness, greater is its entropy. During an adiabatic change the system performs work on the basis of its intemal energy. Therefore work done during an adiabatic change is, ue Wah initial temperature, constant. There is no change in internal energy of the system during an isothermal change, hence the energy taken by the gas is used up in doing work against external pressure thatis dQ = dW = pd’. The internal energy of a gas having f degrees of free- T,) where u = number of moles, T, = r= final temperature and R= gas y= domis U= sR For monoatomic gas, U= >. ‘During adiabatic expansion, the temperature of the sys- tem falls while it increases during adisbatic compres- sion. In this work is done by the gas at the expense of its internal energy and hence cooling takes place. The root mean square speed is inversely proportional to the square root of molecular weight ofthe gas thats L Cou Jpg Hence heavier the gas Both assertion and reason are correct and the reason is the correct explanation of assertion. Atoonstant temperature and pressure, the rate of diffu- sion of a gas is inversely proportional to the square 1 root of its density, therefore r= 75 Meltingof ice is an example of an isothermal change. aa You have either reached a page that is unavailable for viewing or reached your viewing limit for this book. aa You have either reached a page that is unavailable for viewing or reached your viewing limit for this book. aa You have either reached a page that is unavailable for viewing or reached your viewing limit for this book. Electrostatics 581 x 10m. If the charge is distributed, make a point charge by ‘considering a small element and linear charge density J (ifcharge is linear), surface charge density (if charge is spread on area) and volume charge density p (if charge is distributed throughout the volume). 2. Normally force is mutual ie. F,,=~ F,,. In certain cases Newton's 3rd law may not be valid. For example, ifa charge g, is placed in the shell and q, lies outside at adistance r from g, as shown in Fig 18.16, istration of Newton's third law faure then force due to g, on g, is non zero while force due tog, and g, is zero, 3. Inamedium = Fie,fe where e= Ais dielectric constant. Electrostatics 585 If there is more thati @né medium as shown in Fig. 18.17 where a dielectric slab of thickness ¢ and dielectric constant k has been added in between wo charges q, and q, separated by r. To solve such problems, find equivalent distance in vacuum. In the given problem equivalent distance in vacuum is 1 Jie - Thus, net distance between the charges will be 1k or ‘ _ a force P= rer ttt) e pen Finding force between charges in more than one medium ‘Note thateffectivedistance in vacuum fora dielectric of thickness t and dielectric constant k is INE he ty (SE 8. Theelectrc field intensity orelectric force is avector quantity. Therefore exploit vector algebra to solve the problems. 6 Eleciric field intensity due to a point charge Q ata o distance r form itis E ae 7. Electric field intensity inside a hollow conducting body is zero irrespective of its shape. 4 Gva'sLaw § Ede ek ae B Gauss Law in diferent form SE = 2 Electric field intensity duie to a shell (spherical) is and 9. Electric field intensity dueto a dipole Eee — 2px Fass = Ares and due to a short dipole. aa You have either reached a page that is unavailable for viewing or reached your viewing limit for this book. aa You have either reached a page that is unavailable for viewing or reached your viewing limit for this book. aa You have either reached a page that is unavailable for viewing or reached your viewing limit for this book. aa You have either reached a page that is unavailable for viewing or reached your viewing limit for this book. aa You have either reached a page that is unavailable for viewing or reached your viewing limit for this book. aa You have either reached a page that is unavailable for viewing or reached your viewing limit for this book. aa You have either reached a page that is unavailable for viewing or reached your viewing limit for this book. aa You have either reached a page that is unavailable for viewing or reached your viewing limit for this book. aa You have either reached a page that is unavailable for viewing or reached your viewing limit for this book. aa You have either reached a page that is unavailable for viewing or reached your viewing limit for this book. aa You have either reached a page that is unavailable for viewing or reached your viewing limit for this book. 3ax?, Using Gauss's law in oa) 38. In.aMillikan’s oil drop method, a charged drop of oil has mass m and charge ¢. It is made stationary by applying an electric field E. Find the strength of the electric field. Ifthe result of Millikans experiment with different drops is 6.48 x 10-"C, 12.82 « 10-PC, 19,3» 10-°C, 802 x 10-"C, 25.62 10 °C, 16.04 x 10°C, find the elementary charge. differential form 2 ce = 6 ax or p(x) = baer bam gE=mg E me Fig. 18.44 The HCF of the data is 1.6 « 10°C Hence elementary charge is 1.6% 10-"C, 39. A long wire carrying a uniform charge density 2 per unit length has the configuration as shown in Fig, 18.45, Findthe electric field at 0. 22 The field strength at O is is Fer. R due toeach straight part and net electric field ink to Sich straight wires is val 24 a Fre,R ) Trek in forward direction, E,,, Gue to semicircular par (as found in problem 20) is inthe backward direction “. E,,~ zi 2ne,R Fig. 18.45 Electrostatics 597 40. A space is filled up with volume charge density P =P, e®" where p, and care positive constants. r is the distance from the centre of this system, Find the magnitude of electric field strength as a function ofr ae Differentiating fiom Gaus's Law is 5 = 2 or faz E Read the following passage and answer the questions given atthe end. The earth has a net electric charge that causes a field at points near its surface. The charge on the earth is supposed to be a result of an atmospheric battery created between ionosphere and the earth. The eleciric field near the carth’s surface is believed to be 150 NC" and directed towards the centre of the earth. A man suggested that this electric field may be used in flying, 1, What magnitude and sign of charge would 2 60 ke human have to acquire to overcome his or her weight? (asc wac 6c @-4c (e)none. 2. What would be the force of repulsion between two people with the said charge when they are 100 m apart? (@) 144d 10°C (b) 144 < 108C (14d 1060 () Lad = 107€ 3. Can electric field of the earth be practically used as a ‘means of flight? (a) yes (b) no (c)only on solar flare days 1. (4) 60 * 10= gE 60x10 or a> sae 4xc4x<9310" Coy = 14x 10-N EEIMEEM 3. (b) Though theoretically it appears as lying ‘may be achieved with electric field of earth but the electric fields sosmall that avery big charge is required on the other body. The streamlining condition would demand making edges or sharp corners from where charge will lak by means of earona discharge. aa You have either reached a page that is unavailable for viewing or reached your viewing limit for this book. aa You have either reached a page that is unavailable for viewing or reached your viewing limit for this book. aa You have either reached a page that is unavailable for viewing or reached your viewing limit for this book. aa You have either reached a page that is unavailable for viewing or reached your viewing limit for this book. aa You have either reached a page that is unavailable for viewing or reached your viewing limit for this book. aa You have either reached a page that is unavailable for viewing or reached your viewing limit for this book. aa You have either reached a page that is unavailable for viewing or reached your viewing limit for this book. aa You have either reached a page that is unavailable for viewing or reached your viewing limit for this book. aa You have either reached a page that is unavailable for viewing or reached your viewing limit for this book. aa You have either reached a page that is unavailable for viewing or reached your viewing limit for this book. aa You have either reached a page that is unavailable for viewing or reached your viewing limit for this book. aa You have either reached a page that is unavailable for viewing or reached your viewing limit for this book. aa You have either reached a page that is unavailable for viewing or reached your viewing limit for this book. aa You have either reached a page that is unavailable for viewing or reached your viewing limit for this book. aa You have either reached a page that is unavailable for viewing or reached your viewing limit for this book. aa You have either reached a page that is unavailable for viewing or reached your viewing limit for this book. aa You have either reached a page that is unavailable for viewing or reached your viewing limit for this book. aa You have either reached a page that is unavailable for viewing or reached your viewing limit for this book. 616 Pearson Guideto Objective Physics o (OE Fre, 5. Anelliptcal cavity is carved out ina perfect conductor. ‘A positive charge qs placed at the centre ofthe cavity The points 4 and B are shown in Fig. 19.20. Then Fig. 19.20 (a) electric field near 4 in the cavity = electric field near B in the cavity. (b) charge density at 4 ~ charge density at B (©) potential at 4 = potential at B (@) total electric field flux through the surface of the cavity = [IT 1999] (©) and (d) Because A and B lie on the same conductor, potential at each poi is equal. For (d) Gauss law. 6 A solid conducting sphere having a charge Q is surrounded by an uncharged concentric conducting hollow spherical shell. Let the potential difference between the surface of the solid sphere and that ofthe ‘outer surface of the hollow shell be V. Ifthe shell is now given a charge ~30, the new potential difference between the same two surfaces is (ay ww (av @2v0 INIT 1989, DCE 1995] (a) Because the potential difference between solid sphere and hollow shell depends on the radii of two spheres and charge on the inner sphere. Since the two values have not changed. potential difference does not change. 7. Ametallic solid sphere is placed in a uniform electric field. The lines of force follow the path(s) shown in Fig. 19.21 as L 1 2 3 3 Fig. 19.21 @l 2 3 a {017 1999] (@ £,, = 0 and electric field lines are perpendicular to the equipotential surface. 8. Two conducting plates 4 and B are parallel. 4 is given acharge Q, and B is given a charge Q,, The charge on inner side of B is Fig. 19.22 (a) -2) wee 2AO+O) 2 (6) Electric field inside the conductor at point © P=0 8 to Oa Fig. 19.22(b) _O-) 4 , 4 2 “ Qde, 2de, 24e, 2Aey 2-2 or Q-4-(0.+4)=00rg= SO 9. Ahemisphere of radius’ is placed ina uniform electric field of strength E. The electric Mux through the hemisphere is ()2Em? (6) Em? (0)-2Em (@)2e10 x sta s * T fe Fig, 19.23 o= Glas = fleas |" case f‘ eas = Ene + Em? - En? =- En? Shortcut: AXB is symmetrical surface <. Electric flux duc to this partis zero, However, electric flux dus to AB part is— ExR? aa You have either reached a page that is unavailable for viewing or reached your viewing limit for this book. aa You have either reached a page that is unavailable for viewing or reached your viewing limit for this book. aa You have either reached a page that is unavailable for viewing or reached your viewing limit for this book. 620 son Guide to Obj 2. Appoint charge g isplaced asymmetrically in ametal cavity as shown in Figure 19.35 (a). Which one of the diagrams correctly represents the field lines? Fig. 19.35(a) Fig. 19.35 (b) 3. Two point charges q and q, are placed distance r apart. A dielectric sheet of ielectricconstantk and thickness 1 is placed in between, Find the force between the two charges a Fig. 19.36 4. Mee (8) Gekr () ane,[ (rt) Ar | 9, © ane,(r—reviny Drone 4. A-ring has charge Q, radius R and a charge q is placed at the centre, The increase in tension in the fing is ee @ Fig. 19.37 4 ree (d)zero = 2 wpe 2 BEE 1-60) g= 3 = 55 Fig. 19.38 -£ he ~2|-2 = 37 [Wore Tee hg" Dix lone x12 BSED 2.() 2. (c) The equivalent distance between the charges q and q, is (r—1) + fig. Therefore a F = Gre, (r09 Siny 4(e27: aa You have either reached a page that is unavailable for viewing or reached your viewing limit for this book. aa You have either reached a page that is unavailable for viewing or reached your viewing limit for this book. aa You have either reached a page that is unavailable for viewing or reached your viewing limit for this book. aa You have either reached a page that is unavailable for viewing or reached your viewing limit for this book. aa You have either reached a page that is unavailable for viewing or reached your viewing limit for this book. aa You have either reached a page that is unavailable for viewing or reached your viewing limit for this book. at, GEE] Paratte! plate capacitor fod then C = ~5~ d completely filled in the gap. if a dielectric of strength k is c aA aint k thickness (¢ +||- Hl +||- 4{/- +[- +f > =i GJ Polarization illustration If the charge is uniformly distributed throughout the volume then energy stored is U = 5 J¥_pdv where dv is volume element and ¥ is potential difference . Volume donsity of electric field energy ED aft u in free space volume density of electric field energy in a medium KE _ 66K ais Tg 4 ‘The maximum cepacitance of tuner capacitor (used eA(n=1) d each plate, 7 is total no. of plates and dis separation beween two successive plates, Normally a 11-plate tuner capacitor is available whose ratioof maximum to minimum capacity is os Capacitorin series the charge on each plate is equal but voltage across each for tuning in radio) is C= where A is the area of SceFig 20.7. In series, magnitude of c 1m equal capacitors ar in series then C., = aa You have either reached a page that is unavailable for viewing or reached your viewing limit for this book. aa You have either reached a page that is unavailable for viewing or reached your viewing limit for this book. aa You have either reached a page that is unavailable for viewing or reached your viewing limit for this book. finegative terminal of one battery is connected to the positive terminal of other battery then V.~V,. + Vi. as batteries are in series. It nezative terminal of one battery is connected to negative of the other, then Fae = Vi = Vin 9F Vn Vo, depending upon which is greater. The net emf has direction of greater emf | battery. From Fig 20.20 (b) V, = WaMu) G+G ie 7. It n identical plates each of area 4 are connect=4 alternately and separation between two consecutive plates is ¢, then C,,~(n~ 1) a For example C, ~ 34, + in Fig 20.21 >. 8 Ifa metal plate of thickness ‘is introduced in betwer the platcs of a capacitor separated by d then C,, = Ay Gan (ee fig 20.22) : | 9. A large number of identical capacitors of rating C/V are available and capacitor C'/ nV’ is to be designed then m capacitors are to be connected in series, Each Capacitors 631 c row of capacitors has C,,= =. To meke C', then m= Gig 10¥S ofr capacitors in series will be required. 10. Forthe network shown in Fig 20.23 IGO#EC,+C.G, a G#E+2C, 11. If capacitor Cis connected along each side ofa skeleton cube then equivalent capacitance along the longest 6c diagonal is =. Equivalent capacitance along face RD and along one side is = C. 4 diagonal is 12, If one side of skeleton cube is open as shown in Fig 20.24 then Cy, 13, [fone or more shells of concentric shell system is/are ‘grounded then net potential corresponding to grounded shells is zero, For example in Fig 20.25 V,=0 aa You have either reached a page that is unavailable for viewing or reached your viewing limit for this book. aa You have either reached a page that is unavailable for viewing or reached your viewing limit for this book. aa You have either reached a page that is unavailable for viewing or reached your viewing limit for this book. (b) The equivalent capacitance circuit is where —_ fie c= faba ang Deka? 1 joke ad G d a Ba PR)” eo 2Degat (kh, +h )ky Co” (hth + 2k )d Fig. 20.33 10. Each capacitor has capacitance C in the Fig 20.34 (a). Find Cys. a ® Aon Fig. 20.34 fc @)2C wry (c)Note thet doted put inthe circuitisa Wheat- stone bridge With C,=C. Cy 5 ftom Fig 20.35(0) Capacitors 635 LL. Find, c a ne Fig. 20.36 (a) ac (b)-2C (2c (ac (a) Let Xbe the equivalent capacitance. If one morenetwork is added capacitance remains unchanged. ‘Thus from equivalent circuit of Fig 20.36(b) C+ARC X= ie 1h +30) 2C =O or (+20) (X-C)=0 X- in : Fig. 20. (b) 12, Find C,y ifeach capacitor is Cin "AY VAY Fig. 20.37 (a) (3c Oe oc WY (6) Theeauivalentcreitis shown in Fig 20.37 (b) Fig. 20.37 (b) AA Fig. 20.37 (c) aa You have either reached a page that is unavailable for viewing or reached your viewing limit for this book. aa You have either reached a page that is unavailable for viewing or reached your viewing limit for this book. aa You have either reached a page that is unavailable for viewing or reached your viewing limit for this book. aa You have either reached a page that is unavailable for viewing or reached your viewing limit for this book. aa You have either reached a page that is unavailable for viewing or reached your viewing limit for this book. aa You have either reached a page that is unavailable for viewing or reached your viewing limit for this book. aa You have either reached a page that is unavailable for viewing or reached your viewing limit for this book. 17, Teecapeciance fa pull patecapectiorls Cute ‘the region between the plates has air. This region is now filled with a dielectric slab of dielectric constant K. The capacitoris connected to acell of emf €, and the slabs taken out. {@) Charge €C,(K ~1) flows through the cel (b)_ Energy €°C,(K--1) is absorbed by the cell. (©) The energy stored in the capacitor is reduced by 2G(K-1). 1 2, (2) The external agent has to do >€°C,(K-1) amount of work to take the slab out. 18. A parallel-plate air capacitor of capacitance C,, is connected to a cell of emf ¢ and then disconnected from it. A diclectric slab of diclectric constant K, which can just fill the air gap of the capacitor, is now inserted in it, (a) The potential difference between the plates decreases K times. (b) The energy stored in the capacitor decreases K times. hone (©) The change in energy is 5 Cue*(K 1) 1 1 tog(i 1 (@)_ The change in energy is #( ra 19. The gold leaf electroscope is charged so that its leaves somewhat diverge. If X-rays are incident on the electroscope then (a) the divergence will decrease (b) the divergence of leaves will remain unchanged (©) the goldleaves will melt (d) the divergence will increase 20. On removing the dielectric from a charged condenses, its energy (@) increases (b) remains unchanged (©) decreases (@)none of he sbove 21. A radioactive material isin the form ofa sphere whose radius is 9 « 10° m. If 6.25 x 10 B - particles are emitted per second by it then in how much time will a potential of Ivolt be produced on the sphere if it is isolated? (a) LL ms (b) 10" (c) LOus @ips 22, 64 water drops combine to form a bigger drop. If the charge and potential of a small drop are q and 7 respectively, then the charge on bigger drop will be Capacitors 645 (@)2q (4g (Ol6q (64q 23. Acondenser ofcapeity (241 F is charged toa potential of 600 ¥. The battery is now disconnected and the condenser of capacity 11 F isconnected acrossit. The potential of the condenser will reduce to (a0 (b)3007 (©1007 120" 24, Two condensers each of capacitance 2y1 F are connected in parallel and this combination is connected in series with a [24 F capacitor. The resultant capctity ofthe system will be (@) 16 F 13 F (6 nF ()3uF 25, The distance between the pletes of a parallel plate air condenser is d. If a copper plate of same atea but thickness d/2 is placed between the plates then the new capacitance will become (a) doubted (b) half (©)one fourth (@) remain unchanged 26, The energy stored in a condenser is in the from of (a) potential energy (b) magnetic energy (elastic energy () kinetic energy 27. A condenser of capeity 500 4 F is charged at the rate of 50 yt F Cis, The time taken for charging the condenser 10 ¥ will be (alos (25s (50s (@) 100s 28, Two condensers of capacity 41 F and 6 4 F are connected in series. A potential difference of 500 Vis applied between the outer plates of the compound capacitor. The numerical value of charge on each condenser will be (120006 (b) 1200 (©)6000. C (66000 29, ‘Two parallel wires are suspended in vacuum. When the potential difference between the wires is 30 then. the charge on the wires is 104 1 C. The capacitance of the system of wires will be (@3.48uF (spr (10.2 F (50uF 30. Two capacitors C, and C, are connected in parallel. Ifa charge @ is given to the combination, the charge gets charged. Then the ratio of charge on C, to charge on. Cis 1 @aq wc, wesc, WCC, aa You have either reached a page that is unavailable for viewing or reached your viewing limit for this book. aa You have either reached a page that is unavailable for viewing or reached your viewing limit for this book. aa You have either reached a page that is unavailable for viewing or reached your viewing limit for this book. aa You have either reached a page that is unavailable for viewing or reached your viewing limit for this book. aa You have either reached a page that is unavailable for viewing or reached your viewing limit for this book. aa You have either reached a page that is unavailable for viewing or reached your viewing limit for this book. aa You have either reached a page that is unavailable for viewing or reached your viewing limit for this book. aa You have either reached a page that is unavailable for viewing or reached your viewing limit for this book. aa You have either reached a page that is unavailable for viewing or reached your viewing limit for this book. aa You have either reached a page that is unavailable for viewing or reached your viewing limit for this book. aa You have either reached a page that is unavailable for viewing or reached your viewing limit for this book. aa You have either reached a page that is unavailable for viewing or reached your viewing limit for this book. aa You have either reached a page that is unavailable for viewing or reached your viewing limit for this book. aa You have either reached a page that is unavailable for viewing or reached your viewing limit for this book. aa You have either reached a page that is unavailable for viewing or reached your viewing limit for this book. aa You have either reached a page that is unavailable for viewing or reached your viewing limit for this book. aa You have either reached a page that is unavailable for viewing or reached your viewing limit for this book. aa You have either reached a page that is unavailable for viewing or reached your viewing limit for this book. aa You have either reached a page that is unavailable for viewing or reached your viewing limit for this book. aa You have either reached a page that is unavailable for viewing or reached your viewing limit for this book. aa You have either reached a page that is unavailable for viewing or reached your viewing limit for this book. * } 050 ne # 20, D Fig. 241.42 |___» 2A (2.54, © r =o? @x (d) zero. Fig. 21.44 (@) Ideal voltmeter has infinite resistance. CO) ‘Therefore current will be zero. 19. In Fig. 21.43 4B is 300cm long wire having resistance 1002 per meter Rheostat is se at 200. Thebalance point will be attained at Ls9 Fig. 21.43 @ 1.6m (125m ©1sm (4) cannot be determined Oy, 6V. Terminal voltage of 2x1. cell= =15v Using Y=) 1.5 25 tor 125 sing V=RI= 1.5 556, dor = 125em 20. Ohm's law can be applied to (@okmicdevices (©) non-ohmic devices (©) both (a) and (b) (4) none © 21. Which of the /~ graph obeys Ohm's law? 1 | 1 7 v @ (b) 22, The V- 1 graph of a conductor at two different temperatuses is shown in Fig. 21.45. The ratio of q temperature 7- is Fig. 21.45 (a) an? 8 (b)cor @ fe) eee 0 (A) cosse? @ tb) 23. Twocells of emf s 1.25V and 0.75V having equal internal resistance are connected in parallel. The effective emf (@o7sv @12sv ()20V @I10Vv (wosv 125xr4.75xr go r 24, A.250.cm long wice has diemacter Imm. tisconnected at the right gap of a slide wire bridge. When a 3 resistance is connected to left gap, the null point is obtained at 60cm. The specific resistance ofthe wire is (a) 6.28 10° Om (6) 6.28 «108 Om (c)6.28x 10° Om (d)6.28 x 107 Qm_ R13 6 owing = Ga =R R, = Rear? _ 2x3.14x( 5x10)? 7 25 = 628x107Qm aa You have either reached a page that is unavailable for viewing or reached your viewing limit for this book. aa You have either reached a page that is unavailable for viewing or reached your viewing limit for this book. aa You have either reached a page that is unavailable for viewing or reached your viewing limit for this book. 684 Poarson Guide to Object LR=(-1)R, AR U-DRAK, UA) R UHL ARAM, solving these equations we get 202028 =| alot SiS ee Fig. 21.57 (a) (ay 1A (D)L.25A (c)20A (LISA (c) Draw the equivalent circuit as shown in Fig.21.57 (b)and(c), Fig. 21.55 (b) PR +R) + ARR) RRRRIF RRR RD 42. An ideal voltmeterand an ideal ammeter are connected in Fig 21.56. The reading ofthe voltmeter is EN Fig. 21.56 @ov (Dev ow (@rer0 (@ Since ideal ammeter has zero resistance and we are measuring potential drop across it, V=0. 43. A voltmeter has a 25Q coil and S75Q in series. The coil takes 10mA for full scale deflection. The maximum potential difference which ean be measured is, ()250mV @s7sv (ssv (@6ov (4) Ving =f, (R, + R)= 10 10 (600) = 6 ‘44, Find the current J, Fig. 21.57 (a) Al the resistors shown have values in ohms. Vovre2a Fig. 21.57 (b) 140 49. Fig. 21.57 (c) 114742 i428 orR,= 289 45. al Fig. 21.58 (a) aa You have either reached a page that is unavailable for viewing or reached your viewing limit for this book. aa You have either reached a page that is unavailable for viewing or reached your viewing limit for this book. aa You have either reached a page that is unavailable for viewing or reached your viewing limit for this book. 688 Pearson Guideto ObjectivePhysics AQ = ngvddr, ‘The current carried by the positively charged particles is therefore Ag p= Oo na, ao oo os | of - Fig. 21.67 1. The total resistivity of the material (conductor) is sum of thermal based, impurity based and plastic deformation based resistivities ie., p= p,+* p,+ P How does resistivity vary at low temperatures? increase in resistivity by 0.012 w ohm—mand 0.16 4 ohm — m respectively. Compute resistivity of Cu— Ni-Agalloy at 300 K when 0.25% Ni and 0.4atomic Ag isadded toCu (@)0.0244uohm—m (6)0.006 ohm —m. 5. Constant current is flowing through a metal piece shown in Fig. 21.68, (b)0.018 chm m (@)none (@) Electron density at Ys less than that aX. (©) Electron mobi y at Yis less than that at X. @px t Ope Fs (©) Resistivity of the material varies along the length. par @p= rT (Resistivity ofthematerialis Pop 2. Name the mati in which no Thomson fet i S (Po ()Fe Emm +0) on lan 3. (@)J=nev, and J= oF (enone og = Mle = neal 3. Carrier concentration in a material of conductivity po? 0.018 (ohm mj" is 10!electrons m”. A voltage of 4, oe LEXIS. 38. 0.16 V is applied across 0.29 mm thick material, Find net ~ TO" X1.6x10 2 deft velocity. = 6.19ms" (6.19 ms! )4.19 ms" EMM 40 ()6.19ems" (6)4.19 mms" Pan, = (0.015+0012 «.25+0.16% 04) yohmm (€) none (015-+.003 +0.0064= 0.0244 ohm—m. 4, The resistivity of Cu is 0.015 1 ohm mat 300 K. Addition of each aiomic % of Ni and Ag eauses an SOL [ ‘QUESTIONS FOR PRACTICE 1. A straight conductor 4B lies along the axis of a hollow metal cylinder L, which is connected to earth ‘through a conductor C. A quantity of charge will flow through C (0) _ifa current begins to flow through 4B (b) ifthe current through 4B is reversed (©) if ABis removed, and a beam of electrons flows in its place (@ if ABis removed, and a beam of protons flows in its place aa You have either reached a page that is unavailable for viewing or reached your viewing limit for this book. aa You have either reached a page that is unavailable for viewing or reached your viewing limit for this book. aa You have either reached a page that is unavailable for viewing or reached your viewing limit for this book. 30, Current is being driven through a cell of emf £ and internai resistance r, as shown, Fig. 21.89 (a) The cell absorbs energy at the rate of Zi. (b)Thecellstores chemical energy at the rate of (Ei Pr). (©) The potential difference across the cell is £ + ir (d) Some heat is produced in the cell 31, In ahousehold electric circuit, (@)_allclecwie applianeesdrawing power are joined in parallel (b) switch may be either in series or in parallel with the applliance which it controls (©) iaswiteh isin paratel with an appliance, it will <éraw power when the switch isin the “of?” position (open) (@)_ifaswitch isin parattel with an appliance, the fase will blow (burn out) whea the switeh is put on” (closed) 32. Two heaters designed for the same voltage V have different power ratings. When connected individually across a source of voltage V, they produce #7 amount of heat each in times J, and ¢, respectively. When used ‘together across the same source, they produce amount of heat i time ¢ (a) If they are in series, #= 1,4 4 (b)_ Iethey are in series, #= 21, +). tu. (©) they ae inparallel.e= bs (@)_lebey ae in parallel. = F747) 33. Thecharge flowing through a resistance R varies with time # as Q = at ~ br. The total heat produced in & is oR yak (a) ob (b) 3b ; oat (c) Ub id) b 34. Twoelectric bulls rated at 25 W, 220 V and 100 W, 220 V are connected in series across 220 V voltage source. “The25 W and 100 W bulbs now draw P, and P, powers respectively. Electricity 693 (@P,=16W (yr =4w ()P=10W WPa4aw 35. Two identical fuses are rated at 10. I they are joined (2) imparallel, the combination acts as a fase of rating 20A (b)_ inparallel, the combination acts as a fuse of rating SA (6) inseries, the combination acts as a fuse of rating 10a (a) _inseries, the combination acts as fuse of rating 20 A 36. A uniform wire of resistance 50 2 is cut into 5 equal parts. These parts are now connected in parallel, The ‘equivalent resistance of the combination is (22 (wa (©2502 (a) 02500 37. Consider a capacitor-charging circuit, Let Q, be the charge given to the capacitor in a time interval of 10 ms and Q, be the charge given in the next time interval of 10 ms. Let 10 fC charge be deposited in a time interval 1, and the next 10 41C charge be deposited in the next time interval, (O,> Ont,> 4 (H2.> 0.44, by (9).Q,< Oye tye 38. Two non-ideal batteries are connected in series. Consider the following statements: (A) The equivalent emis larger than eitherof the two ems, (B) The equivalent internal resistance is smaller than either of the two intemal resistances, (a) Bach of A and B is correct (b) Ais correct but B is wrong. (©) Bis correct but A is wrong. (d)_ Each of A and B is wrong. 39. The net resistance of an ammeter should be small to censure that (a) itdoes not get overheated (b)_ itdoes not draw excessive current (©) itcan measure large currents @ locs not appreciably change the current to be measured 40. Electrons are emitted by a hot filament and are accelerated by an electric field as shown in fig. 21.88, The two stops atthe lett ensure that the electron beam, has a uniform cross-section, aa You have either reached a page that is unavailable for viewing or reached your viewing limit for this book. aa You have either reached a page that is unavailable for viewing or reached your viewing limit for this book. aa You have either reached a page that is unavailable for viewing or reached your viewing limit for this book. I. Let, Ey Current flows in the cell of emf e, from the positive plate to thenegative plate inside the cell and hence it absorbs energy. 12, The resistance of each side = Rin. For resistance between opposite comers, we have two resistance of R/2 in parallel For resistance between adjacent corners, we have two resistnceot © and (o=i)k in parallel. 13, ‘Disregard the capacitors and find the current through G. The potential difference across each capacitoris then found fiom the potential differences across the resistances in parallel with them. 14, A fully charged capacitor draws no current. Ifthe ‘eapacitor is removed from the circuit, we can distribute current and find the potential difference across each resistance. 18, Treatall voltmeters as resistances. Draw the circuit and find the currents and potential differences for each section. The voltmeter reading is the potential difference across its terminals when itis connected inthe circuit ‘The ammeterreading isthe current passing through = 10mA=0.01 A. = (Hi) 01 =i, 99. = 101, 1 = 10x0.01 ad “Or , AB 09a oa ay yi be Fig.21.133 29. Let P= potential at D. 0-D= 10 V0 = 20i, y-10 = 300-1) Solve fori, i,and V. Electricity 707 Fig. 24.134 32. Let R, and R, be the resistances of the two heaters. Let H be the heat produced. ae When used in parallel, = (= 33. QO = al-br. 1 = 0t0 ‘The heat produced = [Rat 34, Let ¥=220V; R, and R, and 100 W bulbs. P,=25= VIR, oR, = ¥2725 and R,= ¥'/100 a R —— wis» is ——_ + Fig. 24.436 resistances of the 25 W ‘When the bulbs are joined in series, the current ¥ RAR, Power in the 25 W bulb = fae 53. When Xis joined to ¥ fora long time (charging), the ‘energy stored in the eapacitor~ heat produced in R =H. When X is joined to Z (discharging), the energy stored in C= H,) reappears as heat (#1) in R. Thus, HoH, 54, A and B are effectively in parallel and hence give thesame reading at all times, 56. When the slab is removed, the potential of the ‘capacitor increases ktimes, i., itbecomeskV, For ‘the potential to fall to F, and in the 100 W bulb=

También podría gustarte